Sunteți pe pagina 1din 327

Item: 1 of 99 ~ 1 • Mark -<] C> Jill ~· ~J

QID: 23299 ..1 Previous Next Lab'V!I!ues Notes Calculator

•1 •
A 14-year-old girl is brought to the emergency department (ED) by paramedics while seizing . Her parents, who report she
•2
has a history of seizures, found her in her bedroom and were unsure of how long she had been seizing . On arrival to the
•3 ED the patient's pulse is 125/min, blood pressure is 160/100 mm Hg, temperature is 38.9°C { 102°F), and respiratory rate
•4 is 22/min . On examination the patient has her arms extended rigidly at her sides and she is arching her back rhythmically and
•5
appears to be aspirating . After several failed attempts at nasopharyngeal intubation, rapid sequence induction allowed
placement of a nasopharyngeal airway. Several infusions of intravenous benzodiazepines, phenytoin, and phenobarbital produce
•6 little effect .
•7
•8 Which of the following statements regard ing this patient's condition is true"
•9
• 10 A. Irreversible neuronal injury usually occurs after 10 minutes

·~
• 12
B. Management options are strongly influenced by the cause of the seizure

• 13 C. Mortality following status epilepticus is >50%


• 14 D. Patients with minimal myoclonic activity are not at risk for serious complications
• 15
E. Rhabdomyolysis is a potential life-threaten ing complication
• 16
• 17
• 18
• 19
• 20
• 21
• 22
• 23
• 24

Lock
s
Suspend
0
End Block
Item: 1 of 99 ~ 1 • Mark -<] C> Jill ~· ~J
QID: 23299 ..1 Previous Next Lab'V!I!ues Notes Calculator

1 •
The correct answer is E. 61% chose this.
•2
Status epilepticus (SE) refers to a continuous state of seizure activity or a series of seizures during which there is no return to
•3 consciousness in the inter-ictal period . The minimal duration of seizure activity in SE has traditionally been cited as 15-30
•4 minutes. Practically speaking, anyone who is brought to an ED who has been seizing for > 5 minutes will be treated as having
•5 SE. SE is a medical emergency with a wide range of potentially lethal complications. Prolonged intractable muscle contraction
secondary to seizures can lead to respiratory compromise, rhabdomyolysis, hyperthermia, and lactic acidosis .
•6 Rhabdomyolysis Status epilepticus Hyperthermia Lactic acidosis Medical emergency Acidosis Epileptic seizure Muscle contraction Muscle
•7
A is not correct. 130/o chose this .
•8
Irreversible neuronal injury (seen as cortical laminar necrosis) is thought to occur after as few as 30-60 minutes of
•9
continuous seizure activity.
• 10 Necrosis Laminar flow Cortex (anatomy) Epileptic seizure Cerebral cortex

·~
• 12
B is not correct. 120/o chose this .
The in itia l management of patients in SE is aimed at securing the patient's airway and administering agents to cause
• 13 cessation of seizure activity and is the same regardless of the underlying cause.
• 14 Respiratory tract Airway (aviation) Epileptic seizure

• 15 C is not correct. 100/o chose this .


• 16 The mortality of patients in SE is approximately 20% .
• 17 D is not correct. 4% chose this.
• 18 After 45 minutes of continuous seizure activity, symptoms may become subtle as myoclonic activity decreases . Such patients
• 19 are often still seizing within the central nervous system and remain at risk for serious neuronal damage .
Central nervous system Myoclonus Nervous system Epileptic seizure
• 20
• 21
• 22 Bottom Line:
• 23
Status epilepticus refers to a continuous state of seizure activity or a series of seizures during which there is no return to
• 24 -----:- · ·----- =- .a..L.- : ....... _ ... : -A..- 1 - - -:-..1 ""'--•-----' : ... .a.. ... - -&. -l-1- - - · · - - · - ---.r..---•:-- _____ .... __,. ·- --=-··--- --- ,__ _. ,.,_
Lock
s
Suspend
0
End Block
Item: 1 of 99 ~ 1 • Mark -<] C> Jill ~· ~J
QID: 23299 ..1 Previous Next Lab'V!I!ues Notes Calculator

1 • A i s not correct. 130/o chose this.


•2 Irreversible neuronal injury (seen as cortical laminar necrosis) is thought to occur after as few as 30-60 minutes of
•3 continuous seizure activity.
Necrosis Laminar flow Cortex (anatomy) Epileptic seizure Cerebral cortex
•4
•5 B is not correct. 120/o chose th is .
•6 The initial management of patients in SE is aimed at securing the patient's airway and administering agents to cause
cessation of seizure activity and is the same regardless of the underlying cause .
•7
Respiratory tract Airway (aviation) Epileptic seizure
•8
•9
C i s not correct. 100/o chose this .
The mortality of patients in SE is approximately 20% .
• 10
D i s not correct. 4 % chose this.
·[111 After 45 minutes of continuous seizure activ ity, symptoms may become subtle as myoclonic activity decreases . Such patients
·~ are often still seizing within the central nervous system and remain at risk for serious neuronal damage .
• 13 Central nervous system Myoclonus Nervous system Epileptic seizure
• 14

• 15
Bottom Line :
• 16
• 17
Status epilepticus refers to a continuous state of seizure activity or a series of seizures during which there is no return to
consciousness in the inter-ictal period . Prolonged intract able muscle contraction secondary to seizures can lead to
• 18
respiratory compromise, rhabdomyolysis, hypertherm ia, and lactic acidosis .
• 19 Rhabdomyolysis Status epilepticus Hyperthermia Lactic acidosis Acidosis Epileptic seizure Muscle contraction Muscle

• 20
• 21
• 22 Ref er en ces:
FA Step 2 CK 9th ed pp 270- 271
• 23
FA Step 2 CK 8th ed p 257
• 24

Lock
s
Suspend
0
End Block
Item: 2 of 99 ~ 1 • Mark -<] C> Jill ~· ~J
QID: 23910 ..1 Previous Next Lab'V!I!ues Notes Calculator

1 •
A 40-year-old woman with a history of alcohol dependence presents to the emergency department (ED) with 2 days of
•2
nausea, vomiting, and diarrhea. Despite her symptoms, she was able to drink a beer a few hours ago . She is diagnosed
•3 w ith gastroenteritis and is given intravenous fluids and an anti-emetic. Although her nausea subsides, a ha lf hour later the
•4 patient complains of blurred v ision and is slurring her words slightly. One hour later the ED resident finds the patient thrash ing
•5
on the bed and unaware of her surround ings. Her temperature is 35 .8°C (96.5°F), blood pressure is 90/70 mm Hg, pulse is
120/min, and respiratory rate is 24/min .
•6
•7
Administration of which of the following, most likely precipitated th is patient's condition?
•8
•9
A. Dextrose
• 10
B. Flumazenil
·~
• 12 C. Naloxone
• 13
D. Thiamine
• 14
• 15 E. Vitam in 612

• 16
• 17
• 18
• 19
• 20
• 21
• 22
• 23
• 24

Lock
s
Suspend
0
End Block
Item: 2 of 99 ~ 1 • Mark -<] C> Jill ~· ~J
QID: 23910 ..1 Previous Next Lab'V!I!ues Notes Calculator

2
•3
•4 The correct answer is A. 600/o chose this.
•5 In the acute setting, Wernicke's encephalopathy is characterized by autonomic instability, oculomotor and gait disturbances,
•6 and delirium . Malnutrit ion , commonly seen in the setting of alcoholism, results in depletion of thiamine stores and ultimately
•7 causes the findings. This patient was given intravenous fluids that likely contained glucose, which precipitated her condit ion .
Because glucose oxidation is a thiamine-intensive process, administration of dext rose in a patient w ho is thiamine deficient
•8
may further deplete thiamine levels and precipitate neurologic injury. Without rapid thiamine infusion, she could die.
•9 Glucose Thiamine Dysautonomia Wernicke's encephalopathy Encephalopathy Intravenous therapy Acute liver failure Alcoholism Malnutrition Redox Gait Delirium

• 10 Autonomic nervous system Neurology

·[111 B is not correct. 140/o chose this.


·~ Flumazenil is used to reverse the effects of sedative intoxication . Signs of hypnotic or benzod iazepine overdose include
• 13 slurred speech and nystagmus .
• 14 Benzodiazepine Flumazenil Nystagmus Benzodiazepine overdose Sedative Hypnotic Relaxed pronunciation Dysarthria Alcohol intoxication

• 15 C is not correct. 8% chose this.


• 16 Naloxone is a short-acting opioid antagonist given to pati ents in heroin overdose . These patients tend to suffer from
• 17 respiratory distress and exhibit constricted pupils.
Naloxone Opioid antagonist Opioid Heroin Receptor antagonist Miosis Dyspnea Opioid overdose
• 18
• 19 D is not correct. 150/o chose this .
• 20 This patient's rapid deteri oration was precipitated by r eceiv ing glucose before her thiamine stores were repleted . Thiamine
• 21 deficiency damages the mammillary bodies, thalamus, and peri-ductal gray matter. In a chronic setting, patients exhibit
confusion, ataxia, and diplopia .
• 22
Diplopia Mammillary body Thalamus Thiamine Ataxia Grey matter
• 23
• 24
E is not correct. 30/o chose this .

Lock
s
Suspend
0
End Block
Item: 2 of 99 ~ 1 • Mark -<] C> Jill ~· ~J
QID: 23910 ..1 Previous Next Lab'V!I!ues Notes Calculator

1 • Benzodiazepine Flumazenil Nystagmus Benzodiazepine overdose Sedative Hypnotic Relaxed pronunciation Dysarthria Alcohol intoxication

2 C i s not correct. SOfo chose this .


•3 Naloxone is a short-acting opioid antagon ist given to pati ents in heroin overdose . These patients tend to suffer from
•4 respiratory distress and exhibit constricted pupils .
Naloxone Opioid antagonist Opioid Heroin Receptor antagonist Miosis Dyspnea Opioid overdose
•5
•6 D i s not correct. 150/o chose this .
•7 This patient's rapid deterioration was precipitated by receiv ing glucose before her thiamine stores were repleted . Thiamine
•8 deficiency damages the mammillary bodies, thalamus, and peri-ductal gray matter. In a ch ron ic setting, patients exh ibit
confusion, ataxia, and diplopia .
•9
Diplopia Mammillary body Thalamus Thiamine Ataxia Grey matter
• 10
E i s not corr ect. 30/o c hose th is.
·[111 Vitamin 6 1 2 is necessary to treat pernicious anem ia, an autoimmune cond it ion that results in dementia and loss of
·~ sensorimotor function. Patients exhibit cogn it ive changes as well as loss of proprioception and vibratory sense, and
• 13 hyperreflexia .
• 14 Pernicious anemia Proprioception Hyperreflexia Vitamin 812 Anemia B vitamins Dementia Vitamin Autoimmunity Autoimmune disease Sensory-motor coupling

• 15
• 16
Bottom line :
• 17
Wern icke's encephalopathy is characterized by autonom ic instability, oculomotor and gait distu rbances, and delirium, which
• 18
can be precipitated by glucose infusion prior to thiamine infusion .
• 19 Thiamine Wernicke's encephalopathy Dysautonomia Encephalopathy Autonomic nervous system Glucose Delirium Oculomotor nerve Gait

• 20
• 21
• 22 References:
FA Step 2 CK 9th ed p 440
• 23
FA Step 2 CK 8th ed p 276
• 24

Lock
s
Suspend
0
End Block
Item: 3 of 99 ~ 1 • Mark -<] C> Jill ~· ~J
QID: 22200 ..1 Previous Next Lab'V!I!ues Notes Calculator

1 •
A 30-year-old man presents to the emergency department with a severe headache behind his right eye. He has a history ~~ A I
2 of similar headaches, and they occur in periodic cycles that last several months . On average he has one headache attack
•3 per day. He describes the headaches as located behind the right eye, severe, and associated with a red, tearing, right eye
•4 and nasal stuffiness on the same side .
•5
•6 Which of the following is the best init ial treatment?
•7
•8 A. Lithium
•9 B. Nonsteroidal anti-inflammatory drugs
• 10
C. Oxygen
·~
• 12 D. Propanolol
• 13 E. Verapamil
• 14
• 15
• 16
• 17
• 18
• 19
• 20
• 21
• 22
• 23
• 24

Lock
s
Suspend
0
End Block
Item: 3 of 99 ~ 1 • Mark -<] C> Jill ~· ~J
QID: 22200 ..1 Previous Next Lab'V!I!ues Notes Calculator

2
The correct answer is C. 840/o chose this.
3
This patient is experiencing typical symptoms of cluster headaches . Cluster headache is a relatively uncommon diagnosis with
•4
a prevalence of less than< 1% . Men are affected more commonly than women , with a peak age of onset of 25-50 years.
•5 Involvement of the trigeminovascular system is theorized to be responsible in cluster headaches. Acute cluster headaches can
•6 be aborted by inhalation of high-flow 100% oxygen in the majority of patients. Other acute therapy measures include ergots,
•7 sumatriptan, intranasal lidocaine, and corticosteroids .
Cluster headache Lidocaine Trigeminovascular system Sumatriptan Headache Corticosteroid Oxygen Ergoline
•8
•9 A is not correct. OOfo chose this.
• 10 Lithium appears to be particularly effective for the chronic form of cluster headaches and is used as prophylactic therapy for
cluster headaches. However, lithium is not used in the acute management of these headaches .
·[111 Cluster headache Lithium Preventive healthcare

·~ B is not correct. 9% chose this .


• 13
Nonsteroidal anti-inflammatory drugs are first-line abortive therapy for tension headaches as well as migraine headaches .
• 14
They are not effective for the treatment of cluster headache .
• 15 Cluster headache Migraine Tension headache Anti-inflammatory Nonsteroidal anti-inflammatory drug Headache
• 16
D is not correct. 3% chose this.
• 17
~-Blockers, such as propanolol, are excellent for the prophylactic treatment of migraine headaches . They play no role in the
• 18 acute treatment of cluster headache .
• 19 Cluster headache Propranolol Migraine Headache Preventive healthcare

• 20 E is not correct. 40/o chose this .


• 21 Calcium channel blockers, such as verapamil, are used for the prophylactic treatment of migraine, cluster, and tension-type
• 22 headaches. However, they are not used in the acute management of cluster headache .
Cluster headache Verapamil Migraine Calcium channel blocker Calcium channel Headache Calcium Preventive healthcare
• 23
• 24

Lock
s
Suspend
0
End Block
Item: 3 of 99 ~ 1 • Mark -<] C> Jill ~· ~J
..1
'.
QID: 22200 Previous Next Lab'V!I!ues Notes Calculator

1
.
2
Lithium appears to be particularly effective for the chronic form of cluster headaches and is used as prophylactic therapy for
cluster headaches. However, lithium is not used in the acute management of these headaches .
3 Cluster headache Lithium Preventive healthcare
•4
B is not correct. 90/o chose this .
•5
Nonsteroidal anti-inflammatory drugs are first-line abortive therapy for tension headaches as well as migraine headaches .
•6
They are not effective for the treatment of cluster headache .
•7 Cluster headache Migraine Tension headache Anti-inflammatory Nonsteroidal anti-inflammatory drug Headache
•8
D is not correct. 3% chose this.
•9
~-B l ockers,
such as propanolol, are excellent for the prophylactic treatment of m igraine headaches . They play no role in the
• 10 acute treatment of cluster headache .
·[111 Cluster headache Propranolol Migraine Headache Preventive healthcare

·~ E is not correct. 40/o chose this .


• 13 Calcium channel blockers, such as verapamil, are used for the prophylactic treatment of migraine, cluster, and tension-type
• 14 headaches. However, they are not used in the acute management of cluster headache .
Cluster headache Verapamil Migraine Calcium channel blocker Calcium channel Headache Calcium Preventive healthcare
• 15
• 16
• 17 Bottom line:
• 18
Acute cluster headaches can be aborted by inhalation of high-flow 100% oxygen in the majority of patients.
• 19 Cluster headache Oxygen

• 20
• 21
• 22 Ref er en ces:
FA Step 2 CK 9th ed pp 267- 268
• 23
FA Step 2 CK 8th ed p 253
• 24

Lock
s
Suspend
0
End Block
Item: 4 of 99 ~ 1 • Mark -<] C> Jill ~· ~J
QID: 23501 ..1 Previous Next Lab'V!I!ues Notes Calculator

1 •
A 69-year-old white man with type 2 diabetes and hypertension presents with acute onset of right-sided facial, arm, and ~~AI
2 leg weakness . On examination he is alert w ith norma l mental status and intact sensation . No visua l or language deficits
3 are detected .
•4
•5 This patient most likely suffered from a lacunar infarct at wh ich site?
•6
•7 A. Cerebellum
•8
B. Interna l capsule
•9
• 10 C. Pons

·~
• 12
D. Thalamus

E. Thalamus and internal capsu le


• 13
• 14
• 15
• 16
• 17
• 18
• 19
• 20
• 21
• 22
• 23
• 24

Lock
s
Suspend
0
End Block
Item: 4 of 99 ~ 1 • Mark -<] C> Jill ~· ~J
QID: 23501 ..1 Previous Next Lab'V!I!ues Notes Calculator

1 •
2 The correct answer i s B. 710/o chose th is.
3 Hypertension and diabetes mellitus are two risk factors for the development of lacunar infarcts or small vessel strokes . This
patient has suffered a pure motor stroke, the most common type of lacunar stroke . The typical site of inju ry is the posterior
4
li mb of the internal capsule, which carries the descending corticospinal and corticobulbar tracts, leading to deficits in the
•5 pattern seen in this patient.
•6 lacunar stroke Internal capsule Diabetes mellitus Hypertension Stroke Pyramidal tracts Infarction

•7 A i s not correct. 4 % chose this .


•8 A patient with a lesion in the cerebell um would present w ith "cerebellar" signs (ie, dysmetria, ataxia, intention tremor).
•9 Dysmetria can be elicited in the finger-to-nose test. Ataxia can be elicited by asking the patient to wa lk in a straight line .
Dysmetria Intention tremor Ataxia Cerebellum Tremor Lesion
• 10

·~
C i s not correct. 6 % chose this .
• 12 The main symptoms of a lacunar infarct in the pons are dysarthria and clumsiness of the hand that is most prominent when
• 13 the patient is writing (dysarthria-clumsy hand lacuna r syndrome) .
Dysarthria lacunar stroke Pons Infarction
• 14
• 15 D is not correct. 100/o chose this .
• 16 A patient with a lesion in the thalamus would likely present with only sensory losses because a small vessel stroke in the
thalamus is not associated w ith motor deficits . The patient might complain of numbness and/or tingling on one side of the
• 17
body (face, arm, leg, trunk) .
• 18 Thalamus Sensory system lesion Stroke Paresthesia Hypoesthesia
• 19
E i s n ot corr ect. 9 0/o c hose th is .
• 20
Lesions in the thalamus and adj acent posterior internal capsu le cause a mixed sensorimotor stroke. This type of lacunar
• 21 syndrome is characterized by contralateral hemiparesis with an ipsilateral sensory deficit.
• 22 Hemiparesis Internal capsule Thalamus lacunar stroke Contralateral Anatomical terms of location Stroke Ipsilateral

• 23
• 24

Lock
s
Suspend
0
End Block
Item: 4 of 99 ~ 1 • Mark -<] C> Jill ~· ~J
QID: 23501 ..1 Previous Next Lab'V!I!ues Notes Calculator

1 • Dysmetria Intention tremor Ataxia Cerebellum Tremor Lesion

2 C i s not correct. 6 % chose this.


3 The main symptoms of a lacunar infarct in the pons are dysarthria and clumsiness of the hand that is most prominent when
4 the patient is writing (dysarthria-clumsy hand lacunar syndrome) .
Dysarthria Lacunar stroke Pons Infarction
•5
•6 D i s not correct. 100/o chose this .
•7 A patient with a lesion in the thalamus would likely present with only sensory losses because a small vessel stroke in the
•8 thalamus is not associated w ith motor deficits . The patient might complain of numbness and/or tingling on one side of the
body (face, arm, leg, trunk) .
•9
Thalamus Sensory system Lesion Stroke Paresthesia Hypoesthesia
• 10
E i s n ot correct. 9 0/o c hose th is.
·[111 Lesions in the thalamus and adjacent posterior internal capsu le cause a mixed sensorimotor stroke. This type of lacunar
·~ syndrome is characterized by contra lateral hemiparesis with an ipsilateral sensory deficit.
• 13 Hemiparesis Internal capsule Thalamus Lacunar stroke Contralateral Anatomical tenns of location Stroke Ipsilateral
• 14

• 15
Bottom Line :
• 16
• 17
Hypertension and diabetes mellitus are two risk factors for the development of lacunar infarcts or small vessel strokes . The
most common type of lacunar stroke is a pure motor str oke . The typical site of injury is the posterior limb of the internal
• 18
capsule.
• 19 Lacunar stroke Diabetes mellitus Internal capsule Hypertension Stroke Infarction

• 20
• 21
• 22 References:
FA Step 2 CK 9th ed p 258
• 23
FA Step 2 CK 8th ed pp 244-245
• 24

Lock
s
Suspend
0
End Block
Item: 5 of 99 ~ 1 • Mark -<] C> Jill ~· ~J
QID: 23458 ..1 Previous Next Lab'V!I!ues Notes Calculator

1 •
A 75-year-old man with a 50-pack-year smoking history presents to his primary care doctor with loss of vision in his left ~~AI
2 eye . Over the past few weeks he has noticed increased difficu lty reading and driving . He reports stra ight lines appear
3 distorted and a loss of centra l vision . He has no recent trauma, medication changes, or any other symptoms. On
4 funduscopic examination a large area of subretinal hemorrhage is present.
•5
•6 Which of the following is the most appropriate initial management?
•7
•8 A. Azithromycin
•9 B. Cataract removal
• 10
C. Diet modification
·~
• 12 D. Urgent ophthalmologic evaluation
• 13 E. Zinc oxide
• 14
• 15
• 16
• 17
• 18
• 19
• 20
• 21
• 22
• 23
• 24

Lock
s
Suspend
0
End Block
Item: 5 of 99 ~ 1 • Mark -<] C> Jill ~· ~J
QID: 23458 ..1 Previous Next Lab'V!I!ues Notes Calculator

2
The correct answer i s D. 820/o chose this .
3
This patient likely has wet type age-related macu lar degeneration (ARMD). Any patient presenting with a recent history of
4
v isual loss who is suspected of having wet ARMD shou ld be urgently referred to an ophthalmologist to prevent further vision
5 loss. Effective med ica l therapies (VEGF inhibitors such as ran ibizumab and bevacizumab ), and surgical therapies (including
•6 thermal laser photocoagulation , photodynamic therapy, intravit reous inj ection of a vascular endothelial growth factor inhibitor,
•7 and macu lar translocation surgery) are ava ilable for the treatment of ARMD .
Bevacizumab Ranibizumab Macular degeneration Vascular endothelial growth factor Photodynamic therapy Ophthalmology Laser coagulation Growth factor Endothelium
•8
Visual impairment Blood vessel Laser
•9
• 10 A i s not correct. 1 Ofo chose this .
Chlamydia pneumoniae infection may be associated with an increased risk of development of ARMD. Treatment with
·[111 azithromycin would treat th is type of infection, but has not been shown to have an effect on preventing acute visual loss in
·~ t he setting of wet ARMD •
• 13 Azithromycin Chlamydia infection Chlamydophila pneumoniae Infection Visual impairment
• 14
B is not correct. 6 0/o chose this .
• 15
Cataracts are a common cause of visual loss in the elderly; however, the clin ical presentation is more consistent w ith wet
• 16 ARMD. Although cataracts and ARMD can affect the same eye, visua l loss is more gradua l w ith cataracts and patients
• 17 complain of blurry vision rather than central vision loss .
Cataract Visual impairment Blurred vision
• 18
• 19 C i s not correct. 5 % chose this .
• 20 Diet modifications including increasing foods rich in carotenoids and decreasing fat intake may decrease the risk of
• 21 progression of ARMD; however, given the recent onset of visual loss th is patient needs urgent evaluation and treatment by an
ophtha lmologist to prevent further acute visual loss .
• 22
Ophthalmology Carotenoid Visual impairment
• 23
• 24
E i s not correct. 6 0/o c hose th is .

Lock
s
Suspend
0
End Block
Item: 5 of 99 ~ 1 • Mark -<] C> Jill ~· ~J
QID: 23458 ..1 Previous Next Lab'V!I!ues Notes Calculator

1 • Azithromycin Chlamydia infection Chlamydophila pneumoniae Infection Visual impairment

2 B is not correct. 60/o chose this.


3 Cataracts ar e a common cause of visual loss in the elderly; however, the clinical pr esentation is more consistent w ith wet
4 ARMD. Although cataracts and ARMD can affect the same eye, visual loss is more gradual w ith cataracts and patients
5
complain of blurry vision rather than central vision loss .
Cataract Visual impairment Blurred vision
•6
•7 C is not correct. SOfo chose this.
•8 Diet modifications including increasing foods rich in carotenoids and decreasing fat intake may decrease the risk of
progression of ARMD ; however, given the recent onset of v isual loss this patient needs urgent evaluation and treatment by an
•9
ophthalmologist to pr event further acute visual loss .
• 10 Ophthalmology Carotenoid Visual impairment

·[111 E is not correct. 60/o chose this.


·~ Zinc oxide is used in long-term therapy for patients w ith wet ARMD that may decrease progr ession of the disease in the other
• 13 eye .
• 14 Zinc oxide Zinc Oxide

• 15
• 16
Bottom line:
• 17
Any patient presenting with a recent history of v isual loss who is suspected of having wet-type age-related macular
• 18
degeneration should be urgently referred to an ophthalmologist to pr event further vision loss.
• 19 Ophthalmology Macular degeneration Visual impairment

• 20
• 21
• 22 References:
FA Step 2 CK 9th ed pp 293-294
• 23
FA Step 2 CK 8th ed pp 278-279
• 24

Lock
s
Suspend
0
End Block
Item: 6 of 99 ~ 1 • Mark -<] C> Jill ~· ~J
QID: 23470 ..1 Previous Next Lab'V!I!ues Notes Calculator

1 •
A 65-year-old retired accountant presents to the clinic w ith his daughter. He has noticed a progressive slowing of his ~~AI
2 movement over the past year. He takes longer to get dressed in the morning and to perform his household chores . He also
3 compla ins of stiffness and difficu lty writing checks due to a change in his handwriting . On physical exam, he has li mited
4 vertical gaze.
5
•6 Which of these is this patient's most likely diagnosis?
•7
•8 A. Corticobasa l degeneration
•9 B. Lewy body dementia
• 10
C. Multiple system atrophy
·~
• 12 D. Parkinson disease
• 13 E. Progressive supranuclear palsy
• 14
• 15
• 16
• 17
• 18
• 19
• 20
• 21
• 22
• 23
• 24

Lock
s
Suspend
0
End Block
Item: 6 of 99 ~ 1 • Mark -<] C> Jill ~· ~J
QID: 23470 ..1 Previous Next Lab'V!I!ues Notes Calculator

2
The correct answer i s E. 40% chose this.
3
Progressive supranuclear palsy (PSP) is a Parkinson disease (PD)-plus syndrome that has onset at age 40 years or later, and
4
more common in patients in their 60s and 70s . Vertical ocular gaze paresis (wh ich is tested by asking the patient to look from
5 downgaze to midgaze, since midgaze to upgaze is restricted in normal individua ls > 75 years old) is more suggestive of PSP
6 than PD, although PSP may also have bradykinesia, rigidity, and in some cases tremor.
Progressive supranuclear palsy Bradykinesia Parkinson's disease Tremor Paresis
•7
•8 A i s not correct. 100/o chose this .
•9 Corticobasal degeneration (CBD) has an age of onset in the 60s and most commonly presents with a "useless" limb, ataxia,
• 10 and sensory or behavioral disturbances. Patients may also have rigidity, bradykinesia, and tremor, making CBD difficult to
distinguish from Parkinson disease (PD) and other PD-plus syndromes . Prominent motor apraxia and dystonia are features
·[111 that make CBD more likely.
·~ Corticobasal degeneration Bradykinesia Dystonia Ataxia Tremor Parkinson 's disease Apraxia

• 13
B is not correct. 6 0/o chose this .
• 14
Lewy body dementia, or diffuse Lewy body disease (DLBD), is another Parkinson disease (PD)-plus syndrome that can be
• 15 difficult to distinguish from other PD- plus syndromes. Features that make DLBD more likely include dementia (which can be
• 16 screened for with a Min i-Mental State Examination score <20), v isual hallucinations, and a fluctuating level of cogn it ion .
Dementia with lewy bodies Lewy body Dementia Parkinson's disease ~1ini-~1ental State Examination Cognition Hallucination
• 17
• 18 C i s not correct. 9 0/o chose this .
• 19 Multiple system atrophy commonly presents with early autonomic symptoms such as orthostatic hypotension marked by a
• 20 history of falls with in the first 6 months of onset of symptoms and urinary incontinence, although most patients rapidly
progress to parkinsonian features such as akinesia and rigidity. Average age of onset is in the 50s .
• 21 Orthostatic hypotension Multiple system atrophy Urinary incontinence Hypokinesia Hypotension Parkinsonism Atrophy Autonomic nervous system Fecal incontinence
• 22
D i s not correct. 3 5 0/o chose this .
• 23
This patient's presentation is consistent with Parkinson d isease (PD); however, these symptoms may also be associated with
• 24

Lock
s
Suspend
0
End Block
Item: 6 of 99 ~ 1 • Mark -<] C> Jill ~· ~J
QID: 23470 ..1 Previous Next Lab'V!I!ues Notes Calculator

1
B is not correct. 60/o chose this.
2
Lewy body dementia, or diffuse Lewy body disease (DLBD), is another Parkinson disease (PD)-plus syndrome that can be
3 difficult to distinguish from other PD-plus syndromes. Features that make DLBD more likely include dementia (which can be
4 screened for with a Mini-Mental State Examination score <20), v isual hallucinations, and a fluctuating level of cognition .
Dementia with lewy bodies lewy body Dementia Parkinson's disease Mini-Mental State Examination Cognition Hallucination
5
6 C is not correct. 9% chose this.
•7 Multiple system atrophy commonly presents with early autonomic symptoms such as orthostatic hypotension marked by a
•8 history of falls with in the first 6 months of onset of symptoms and urinary incontinence, although most patients rapid ly
progress to parkinsonian features such as akinesia and rigidity. Average age of onset is in the 50s .
•9
Orthostatic hypotension ~1ultiple system atrophy Urinary incontinence Hypokinesia Hypotension Parkinsonism Atrophy Autonomic nervous system Fecal incontinence
• 10
D is not correct. 350/o chose this.
·[111 This patient's presentation is consistent with Parkinson disease (PD); however, these symptoms may also be associated with
·~ several other diseases with parkinsonian features . Findings that make PD more likely include a unilateral pill-rolling resting
• 13 tremor in the hand and loss of smell.
• 14 Parkinson's disease Parkinsonism Tremor Anosmia

• 15
• 16
Bottom line:
• 17
Symptoms such as rigid ity and bradykinesia are consistent with Parkinson disease, but also a number of other Parkinson-
• 18
plus syndromes . Limited vertical gaze is suggestive of progressive supranuclear palsy.
• 19 Progressive supranudear palsy Bradykinesia Parkinson's disease

• 20
• 21
• 22 References:
FA Step 2 CK 9th ed p 283
• 23
FA Step 2 CK 8th ed pp 269-270
• 24

Lock
s
Suspend
0
End Block
Item: 7 of 99 ~ 1 • Mark -<] C> Jill ~· ~J
QID: 23466 ..1 Previous Next Lab'V!I!ues Notes Calculator

1
2

A 50-year-old man is brought to the clinic by his niece, who has noticed a slowly progressive change in his persona lity. The 1•AI
patient has noticed increasing difficulty with balance, bladder incontinence, and occasional "lightning-like" pains. On A
3 exam ination, he has small, irregularly shaped pupi ls that constrict with accommodation but not to light. He has a loss of
4 sensation and reflexes in his lower extrem it ies, a positive Romberg sign, and an ataxic ga it. A medical evaluation confirmed the
suspected diagnosis.
5
6
What is the treatment of choice for the diagnosis suggested by the vignette?
•7
•8
•9
A. CT of the head

• 10 B. Penici llin

·~
• 12
C. Thiam ine

D. Vitam in B12
• 13
• 14
• 15
• 16
• 17
• 18
• 19
• 20
• 21
• 22
• 23
• 24

Lock
s
Suspend
0
End Block
Item: 7 of 99 ~ 1 • Mark -<] C> Jill ~· ~J
QID: 23466 ..1 Previous Next Lab'V!I!ues Notes Calculator

2 The correct answer is B. 670/o chose th is.


3 This patient has tabetic neurosyph il is (also known as tabes dorsalis) with the slow progression of ataxia, bladder
4 incontinence, and lightning-li ke pains as well as the examination findings of Argyll Robertson pupils and loss of sensation and
deep tendon reflexes in the lower extremities. A serum fluorescent treponema! antibody absorbed assay (a lso known as FTA-
5
A6S assay) is the most sensit ive and specific test for later stages of syphilis. Treatment is with penicil lin .
6 Tabes dorsalis Syphilis Neurosyphilis Penicillin Ataxia Argyll Robertson pupil Antibody Fluorescent treponema! antibody absorption test Urinary incontinence Urinary bladder Reflex
7 Tendon
•8
A is not correct. 100/o chose this .
•9
Normal pressure hydrocephalus can present with dementia, incontinence, and ataxia . However, this patient has complaints
• 10 more consistent with neurosyphilis.
·[111 Neurosyphilis Normal pressure hydrocephalus Hydrocephalus Syphilis Ataxia Dementia Urinary incontinence

·~ C is not corre ct. 9 0/o chose this.


• 13
Thiamine deficiency is seen in alcoholics, who can have a periphera l neuropathy, loss of deep tendon reflexes, ataxia, and
• 14 change in personality if they develop Wern icke- Korsakoff syndrome . The combination of symptoms and signs in this patient
• 15 including changes in the pupils, the " lightn ing" pains, and the spinal cord pathology are ind icative of neurosyphilis .
Neurosyphilis Syphilis Wemicke-Korsakoff syndrome Thiamine Peripheral neuropathy Ataxia Spinal cord Alcoholism Pathology Tendon
• 16
• 17 D is not correct. 140/o chose this .
• 18 Cobalamin (vitamin 6 1 2) deficiency is a common cause of paresthesias and loss of sensation in the lower extremit ies of
• 19
elderly people; however, vitamin 6 12 deficiency alone would not explain the bladder incontinence, Argyll Robertson pu pils, or
loss of deep tendon reflexes, nor the damage seen on the pathologic specimen .
• 20 Cobalamin Vitamin 812 Paresthesia Urinary incontinence Fecal incontinence B vitamins Vitamin Urinary bladder
• 21
• 22
• 23
Bottom line :
• 24 Tabetic neurosvohilis oresents with the slow oroaression of ataxia. bladder incontinence. and liahtnina-like cains as well as

Lock
s
Suspend
0
End Block
Item: 7 of 99 ~ 1 • Mark -<] C> Jill ~· ~J
QID: 23466 ..1 Previous Next Lab'V!I!ues Notes Calculator

1
A i s not correct. 100/o chose this .
2
Normal pressur e hydrocephalus can present with dementia, incontinence, and ataxia . However, this patient has complaints
3 more consistent with neurosyphilis.
4 Neurosyphilis Normal pressure hydrocephalus Hydrocephalus Syphilis Ataxia Dementia Urinary incontinence

5 C i s not correct. 9 0/o chose this.


6 Thiamine deficiency is seen in alcoholics, who can have a periphera l neuropathy, loss of deep tendon reflexes, ataxia, and
7 change in personality if they develop Wern icke- Korsakoff syndrome . The combination of symptoms and signs in this patient
•8 including changes in the pupils, the " lightning" pains, and the spinal cord pathology ar e indicative of neur osyphilis .
Neurosyphilis Syphilis Wemicke-Korsakoff syndrome Thiamine Peripheral neuropathy Ataxia Spinal cord Alcoholism Pathology Tendon
•9
• 10 D i s not correct. 140/o chose this .
·[111 Cobalamin ( vitamin 6 1 2) deficiency is a common cause of par esthesias and loss of sensation in the lower extremit ies of
elderl y peoplei however, vitamin 6 12 deficiency alone would not explain the bladder incontinence, Argyll Robertson pupils, or
·~ loss of deep tendon reflexes, nor the damage seen on the pathologic specimen .
• 13 Cobalamin Vitamin 812 Paresthesia Urinary incontinence Fecal incontinence B vitamins Vitamin Urinary bladder
• 14

• 15
Bottom Line :
• 16
• 17
Tabetic neu rosyphilis presents with the slow progression of ataxia, bladder incontinence, and lightning-like pains as well as
Argyll Robertson pupils and loss of sensation and deep tendon reflexes in the lower extremit ies . Penicillin is the appropriate
• 18
t reatment.
• 19 Neurosyphilis Penicillin Syphilis Ataxia Argyll Robertson pupil Urinary incontinence Urinary bladder Tendon

• 20
• 21
• 22 References:
FA Step 2 CK 9th ed pp 218-219
• 23
FA Step 2 CK 8th ed pp 200-202
• 24

Lock
s
Suspend
0
End Block
Item: 8 of 99 ~ 1 • Mark -<] C> Jill ~· ~J
QID: 23286 ..1 Previous Next Lab'V!I!ues Notes Calculator

1 •
A homeless man is brought to the emergency department in a confused state following a seizure in public. Witnesses
2 report the seizure as occurring suddenly and lasting at least 2-3 minutes . A first responder on the scene says it was
3 characterized by rhythmic coordinated flailing of the upper extremities and arching of the back. On examination the
4 patient's tongue is unharmed, although his pants are covered in fresh urine. He is oriented to person but not place or time and
has no recollection of what happened, but he is aware he has "some sort" of seizure disorder for which he "occasionally" takes
5
an unnamed medication that he is not carrying . More in-depth questioning reveals that he has been using increasing amounts of
6 cocaine and heroin since he was kicked out of a group home and rarely takes his seizure medication as a result.
7
•8 Which type of seizure has this man most likely suffered?
•9
• 10 A. Absence

·~
• 12
B. Alcohol withdrawal seizure

• 13 C. Complex partial
• 14 D. Simple partial
• 15
E. Tonic-clonic (grand mal) seizure
• 16
• 17
• 18
• 19
• 20
• 21
• 22
• 23
• 24

Lock
s
Suspend
0
End Block
Item: 8 of 99 ~ 1 • Mark -<] C> Jill ~· ~J
QID: 23286 ..1 Previous Next Lab'V!I!ues Notes Calculator

1
The correct answer is E. 74% chose this.
2
This patient with an underlying seizure disorder has most likely suffered a grand mal (tonic clonic) seizure involving both
3 cerebral hemispheres and resulting in complete loss of consciousness. These types of seizures begin suddenly with tonic
4 extension of the back and extremities and typically last for several minutes . Patients often experience incontinence and/or
5
tongue biting during the seizure . The post-ictal period is marked by confusion as the patient slowly returns to consciousness .
In this patient the most likely underlying cause is low serum antiepileptic levels secondary to medication noncompliance.
6 Anticonvulsant Unconsciousness Epilepsy Postictal state Epileptic seizure Generalised tonic-clonic seizure Urinary incontinence Syncope (medicine) Cerebral hemisphere
7 Pharmaceutical drug Clonus
8
A is not correct. 1% chose this.
•9
Absence seizures also involve both hemispheres and cause sudden loss of consciousness . However, these are typically very
• 10
brief, lasting only a fe w seconds, and occur many times per day in affected children . It appears as if the patient is staring
·[111 blankly; the patient is sometimes blinking, rolling his or her eyes, or chewing .
·~ Absence seizure Epileptic seizure Unconsciousness Syncope (medicine) Cerebral hemisphere

• 13 B is not correct. SO/o chose this .


• 14 Although seizures can occur in the context of alcohol w ithdrawa l, there is no evidence in the question stem that documents
• 15 that this patient is { 1) addicted to alcohol, or {2) has abstained long enough to experience withdrawal symptoms . Although
• 16 he is admittedly a drug addict and may be li kely to abuse alcohol as well, his known seizure disorder and acknowledged lack
of medication compliance constitute a more likely cause .
• 17 Alcohol withdrawal syndrome Substance dependence Alcoholic beverage Alcohol Compliance (medicine) Epilepsy Addiction Epileptic seizure Benzodiazepine withdrawal syndrome
• 18
Pharmaceutical drug
• 19
Cis not correct. 170/o chose this .
• 20
Focal, or "partial," seizures, whether simple or complex, by definition involve only part of the brain, as opposed to the whole-
• 21
brain involvement of general seizures. Thus they typically present with localized neurologic symptoms suggestive of the
• 22 region of seizure activity. While it is possible for a focal seizure to generalize into a grand mal seizure, this patient's seizure
• 23 seems to have had a sudden onset.
Partial seizure Generalised tonic-clonic seizure Epileptic seizure Neurology Brain Human brain
• 24

Lock
s
Suspend
0
End Block
Item: 8 of 99 ~ 1 • Mark -<] C> Jill ~· ~J
QID: 23286

1
..1
. . . . . ..
Previous
. .
Next
..
Lab'V!I!ues
. ..
Notes
.
Calculator

Alcohol withdrawal syndrome Substance dependence Alcoholic beverage Alcohol Compliance (medicine) Epilepsy Addiction Epileptic seizure Benzodiazepine withdrawal syndrome
2
Pharmaceutical drug
3
4
Cis not correct. 170/o chose this.
Focal, or "partial," seizur es, whether simple or complex, by definit ion involve only part of the brain, as opposed to the whole-
5
brain involvement of general seizures. Thus they typically present with localized neurologic symptoms suggestive of the
6 region of seizure activity. While it is possible for a focal seizure to generalize into a grand mal seizur e, this patient's seizur e
7 seems to have had a sudden onset.
Partial seizure Generalised tonic-donie seizure Epileptic seizure Neurology Brain Human brain
8
•9 D is not correct. 4 % chose this.
• 10 Focal, or "partial," seizur es, whether simple or complex, by definit ion involve only part of the brain, as opposed to the whole-
·[111 brain involvement of general seizures. Thus they typically present with localized neurologic symptoms suggestive of the
region of seizure activity. While it is possible for a focal seizure to generalize into a grand mal seizur e, this patient's seizur e
·~ seems to have had a sudden onset.
• 13 Partial seizure Generalised tonic-donie seizure Epileptic seizure Neurology Brain Human brain
• 14

• 15
Bottom Line :
• 16
• 17
Grand mal seizur es begin suddenly with tonic extension of the back and ext r emit ies and last for several minutes. Patients
may experience incontinence and/ or tongue bit ing during the seizure . The postictal period is marked by confusion as the
• 18
patient slowly r eturns to consciousness.
• 19 Postictal state Urinary incontinence Epileptic seizure Generalised tonic-clonic seizure Fecal incontinence

• 20
• 21
• 22 References:
FA Step 2 CK 9th ed pp 269-270
• 23
FA Step 2 CK 8th ed pp 256-257
• 24

Lock
s
Suspend
0
End Block
Item: 9 of 99 ~ 1 • Mark -<] C> Jill ~· ~J
QID: 24444 ..1 Previous Next Lab'V!I!ues Notes Calculator

1 •
A 35-year-old man presents to his primary care physician complain ing of worsening horizontal double vision over the past ~~AI
2 week. He notices it mostly wh ile driving/ especia lly when looking to the left. He denies dizziness 1 falls 1 headaches1 or foca l
3 loss of strength or sensation . He is otherwise hea lthy/ takes no med ications 1 and has no pertinent med ical history. Physica l
4 exam ination findings are shown in the video.
5 OPEN MEDIA

6
7
These eye movements are most closely associated with which of the following?
8
•9
A. Brain abscess
• 10

·~
B. Cerebrovascu lar occlusion
• 12
C. Migraine
• 13
D. Multiple sclerosis
• 14
• 15 E. Seizure
• 16
• 17
• 18
• 19
• 20
• 21
• 22
• 23
• 24

Lock
s
Suspend
0
End Block
Item: 9 of 99 ~ 1 • Mark -<] C> Jill ~· ~J
QID: 24444 ..1 Previous Next Lab'V!I!ues Notes Calculator

1 •
A 35-year-old man presents to his primary care physician complain ing of worsening horizontal double vision over the past ~ ~A I
2 week. He notices it mostly wh ile driving/ especially when looking to the left. He denies dizziness1 falls 1 headaches1 or foca l
3 loss of strength or sensation. He is otherwise healthy/ takes no med ications1 and has no pertinent medica l history. Physical
4 examination findings are shown in the video.
5
I~
6
7
8
•9
• 10

·~
• 12

• 13
• 14
• 15
• 16
• 17
• 18
• 19
• 20
• 21
• 22
• 23
• 24

Lock
s
Suspend
0
End Block
2
3
4

5
6
7
8
•9
• 10

·[111
·~
• 13
• 14
These eye movements are most closely associated with which of the following?
• 15
• 16
A. Brain abscess
• 17
• 18 B . Cerebrovascu lar occlusion
• 19
C. Migraine
• 20
D . Multiple sclerosis
• 21
• 22 E. Seizure
• 23

• 24

a
Lock
s
Suspend
8
End Block
Item: 9 of 99 ~ 1 • Mark -<] C> Jill ~· ~J
QID: 24444 ..1 Previous Next Lab'V!I!ues Notes Calculator

2 The correct answer is D. 680/o chose this.


3 The examination demonstrates internuclear ophthalmoplegia (INO) . In this example, the left eye is able to fully abduct (move
4
laterally), whereas the right eye is unable to adduct (move medially). This causes diplopia when gazing left. Either eye can be
affected . INO is caused by a lesion in the medial long itudina l fasciculus, where fibers from the abducens nucleus cross and
5 ascend to the contralateral medial rectus subnucleus of the oculomotor nucleus complex, thus coordinating gaze. Because the
6 medial rectus nucleus is still intact, the eye is still able to adduct on convergence because convergence uses different
7 circuitry. In a healthy young person, this finding is common ly related to multiple sclerosis and is often the presenting
complaint.
8
~1ultiple sclerosis Internuclear ophthalmoplegia Diplopia Medial longitudinal fasciculus Oculomotor nucleus Abducens nucleus Ophthalmoparesis Medial rectus muscle
9
Anatomical terms of location Abducens nerve lesion Contralateral
• 10
A is not correct. 8 % chose this.
·[111
A brain abscess is rare without associated symptoms, including fevers, headache, and an infection source .
·~ Brain abscess Abscess Headache Human brain Brain Infection
• 13
• 14 B is not correct. 160/o chose th is .
• 15
A stroke may present in this fashion; however, in this otherwise healthy young patient, INO is much more likely to represent
mu lt iple sclerosis than a stroke .
• 16 Multiple sclerosis Stroke
• 17
C is not correct. 4 % chose this.
• 18
A classic migraine may have focal neurologic findings but wou ld not last for 1 week without the development of a headache .
• 19
Migraines are not typically associated with INO .
• 20 Migraine Neurology Headache

• 21
E is not correct. 4 0/o chose th is .
• 22
A focal seizure producing these symptoms would not be continuous for 1 week without associated neurologic find ings .
• 23 Partial seizure Neurology Epileptic seizure
• 24

Lock
s
Suspend
0
End Block
Item: 9 of 99 ~ 1 • Mark -<] C> Jill ~· ~J
QID: 24444 ..1 Previous Next Lab'V!I!ues Notes Calculator

1 Anatomical terms of location Abducens nerve lesion Contralateral

2
A is not correct. 8 % chose this.
3 A brain abscess is rare without associated symptoms, including fever s, headache, and an infection source .
4 Brain abscess Abscess Headache Human brain Brain Infection

5
B is not correct. 160/o chose this.
6
A stroke may pr esent in this fashion; however, in t his ot herwise healthy young pati ent, INO is much more likely to repr esent
7 mu lt iple scler osis than a stroke.
~1ultiple sclerosis Stroke
8
9 C is not correct. 4 % chose this.
• 10 A classic migraine may have focal neurologic findings but would not last for 1 week without the development of a headache .
·[111 Migraines are not typically associated with I NO.
Migraine Neurology Headache
·~
• 13 E is not correct. 4 0/o chose th is .
• 14 A focal seizure producing these symptoms would not be continuous for 1 week without associat ed neu rologic findings .
Partial seizure Neurology Epileptic seizure
• 15
• 16
• 17 Bottom line :
• 18
Internuclear ophtha lmoplegia is often t he presenting symptom in multiple sclerosis.
• 19 Multiple sclerosis Internuclear ophthalmoplegia Ophthalmoparesis Symptom

• 20
• 21
• 22 References:
FA Step 2 CK 9th ed p 275
• 23
FA Step 2 CK 8th ed p 263
• 24

Lock
s
Suspend
0
End Block
Item: 10 of 99 ~ 1 • Mark <:] (:>- Jill ~· ~J
QID: 21069 ..1 Previous Next Lab'V!I!ues Notes Calculator

1 •
A 47-year-old woman is brought to the emergency department by the police after being found wandering around a loca l ~~AI
2 park in a nightgown . Her brother is ca lled and he reports a worsening in impu lse control in his sister over the past 2 years .
3 During this time, she has become more sexua lly aggressive, fondl ing strangers in subways and disrobing in public.
4 Occasionally, she laughs at unknown stimul i and cannot be redirected during these emotional outbursts . Over the past month
she has become a voracious eater and has ga ined 4. 5 kg ( 10 lb) . She was fired from her job as a postal worker 6 months ago
5
for inappropriate behavior. Her blood pressure is 138/80 mm Hg; she appears older than her stated age, and has a disheveled
6 appearance. She is alert and oriented, but cannot recall why she was in the park. Physical examination reveals a mi ld intention
7 tremor. Her Min i-Mental State Examination score is 27, with deficits in short-term recall.
8
9 Which of the following is the most likely diagnosis?
• 10

·~
A. Attention deficit disorder
• 12
B. Diffuse Lewy body dementia
• 13
• 14
C. Histrion ic persona lity disorder

• 15 D. Huntington disease
• 16
E. Kluver-Bucy syndrome
• 17
• 18
• 19
• 20
• 21
• 22
• 23
• 24

Lock
s
Suspend
0
End Block
Item: 10 of 99 ~ 1 • Mark <:] (:>- Jill ~· ~J
QID: 21069 ..1 Previous Next Lab'V!I!ues Notes Calculator

1 •
2 The correct answer i s E. 7 3% chose this .
3 KIUver- Bucy syndrome is classified as a synd rome w ithin the temporal lobe dementias . Classic symptoms include hyperorality,
4 hypersexuality, and perseverative speech or behavior. Other symptoms include apathy, personality changes, and amotivation .
Hypersexuality Temporal lobe
5
6 A i s not correct. 1% chose this .
7 Attention deficit disorder may present in adu lts, but it is characterized by inability to focus on tasks, and difficu lty attending
8 to details.
Attention deficit hyperactivity disorder
9
10 B is not correct. 140/o chose th is.
Diffuse Lewy body disease has prominent psychiatric signs such as personality changes, depression, and hallucinations.
·~
• 12
Patients often have weight loss, rather than weight gain . Fluctuating mental status and cognitive impa irments are common .
This patient, however, fits the classic presentation of Kluver-Bucy syndrome .
• 13 Dementia with lewy bodies Lewy body Hallucination Major depressive disorder Depression (mood)
• 14
C i s not correct. 4 % chose this .
• 15
Histrionic persona lity disorder is a persistent pattern of behaviors present throughout a patient's life. Prominent signs include
• 16 inappropriate sexua l behavior and unstable relationsh ips .
• 17 Histrionic personality disorder Personality disorder Human sexual activity

• 18
D is not correct. SOfo chose this .
• 19 Huntington's disease is a familial dementia with anticipatory expression . It is caused by a triplet repeat and successive
• 20 generations tend to express the mutation earlier in age. Symptoms of the disease include difficulty in speaking and
• 21 swallowing, involuntary movements, cogn it ive impairment, and depression . Uncommonly, patients can present with
delusions, hallucinations, and obsessive compulsive disorders. It is uniform ly fatal.
• 22
Huntington's disease Dementia Mutation Major depressive disorder Cognitive deficit Depression (mood) Hallucination
• 23
• 24

Lock
s
Suspend
0
End Block
Item: 10 of 99 ~ 1 • Mark <:] (:>- Jill ~· ~J
QID: 21069 ..1 Previous Next Lab'V!I!ues Notes Calculator

1 • Attention deficit hyperactivity disorder

2 B is not correct. 140/o chose this.


3 Diffuse Lewy body disease has prominent psychiatric signs such as personality changes, depression, and hallucinations.
4 Patients often have weight loss, rather than weight gain . Fluctuating mental status and cognitive impa irments are common .
5
This patient, however, fits the classic presentation of Kluver-Bucy syndrome .
Dementia with lewy bodies Lewy body Hallucination Major depressive disorder Depression (mood)
6
7 C is not correct. 4% chose this.
8 Histrionic personality disorder is a persistent pattern of behaviors present throughout a patient's life. Prominent signs include
inappropriate sexual behavior and unstable relationships.
9 Histrionic personality disorder Personality disorder Human sexual activity
10
D is not correct. 8% chose this.
·[111 Huntington's disease is a familial dementia with anticipatory expression . It is caused by a triplet repeat and successive
·~ generations tend to express the mutation earlier in age . .Symptoms of the disease include difficulty in speaking and
• 13 swallowing, involuntary movements, cognitive im pairment, and depression. Uncommonly, patients can present with
• 14 delusions, hallucinations, and obsessive compulsive disorders. It is uniformly fatal.
Huntington's disease Dementia Mutation Major depressive disorder Cognitive deficit Depression (mood) Hallucination
• 15
• 16
• 17 Bottom line:
• 18
KIUver-Bucy syndrome is a bilateral temporal lobe dementia . Symptoms include hyperorality, hypersexuality, perseverative
• 19 speech or behav ior, apathy, personality changes, and lack of motivation .
Hypersexuality Temporal lobe Dementia
• 20
• 21
• 22
References:
• 23
FA Step 2 CK 9th ed p 532
• 24

Lock
s
Suspend
0
End Block
Item: 11 of 99 ~ 1 • Mark <:] (:>- Jill ~· ~J
QID: 28005 ..1 Previous Next Lab'V!I!ues Notes Calculator

1 •
The following vignette applies to the next 2 items. The items in the set must be answered in sequential order. Once you click Proceed to Next Item, you wil l not be
2 able to add or change an answer.
3
4
A 19-year-old obese college student visits the university health center complaining of excessive daytime sleepiness. She 1•Aj
states she has put on 70 pounds over the past year. She i s so tired that she easily falls asleep during class for brief periods A
5 and often writes off the page when taking notes. She admits that her sleepiness temporarily improves after brief naps. She
6
states that even when she was on summer break 2 months ago she experienced the same same symptoms despite sleeping for
10 hours each night. Both of her prior roommates moved out but neglected to tell her why.She is 5 feet 2 inches tall and weighs
7
214 pounds, physica l exam reveals normal dentition, a Mallampati class 3 oropharynx, a short thick neck, sonorous breath
8 sounds, and normal 51 52 on cardiac auscultation.
9
10 What is the best next step in management?

·~
• 12 A. Obtain polysomnography
• 13
B. Perform trial of acetazolamide
• 14
• 15 C. Perform trial of continuous positive airway pressure
• 16 D. Provide reassurance
• 17
E. Recommend weight loss
• 18
• 19
PROCEED TO NEXT ITEM
• 20
• 21
• 22
• 23
• 24

Lock
s
Suspend
0
End Block
Item: 11 of 99 ~ 1 • Mark <:] (:>- Jill ~· ~J
QID: 28 005 ..1 Previous Next Lab'V!I!ues Notes Calculator

1 The correct answer is A. 690/o chose this.


2 Based on this patient's symptoms and physical exam , she has a sleep disorder that should be evaluated by overnight
3 polysomnography (PSG) for proper diagnosis. She likely has obstructive sleep apnea (OSA) based on her body habitus and
the fact she experi enced the same symptoms despite 10 hours of sleep during the summer. Her r oommates likely moved out
4
due to her snoring . PSG is considered the gold standard d iagnostic test for sleep disorders . During PSG the patient sleeps
5 w hile connected to a variety of monitoring dev ices that r ecord physiologic vari ables.
6 Polysomnography Sleep disorder Obstructive sleep apnea Sleep apnea Gold standard {test) Apnea Physical examination Snoring

7 B is not correct. 20/o chose this.


8 Acetazolamide is a carbonic anhydrase inhibitor used to treat central sleep apnea . A diagnosis needs to be established before
9 this therapy is initiated .
Acetazolamide Carbonic anhydrase inhibitor Carbonic anhydrase Sleep apnea Central sleep apnea Apnea
10

~ C is not correct. 9% chose this.


·~ Continuous posit ive airway pr essure is used to treat obstructive sleep apnea . Before this therapy is initiated, however, a
• 13 diagnosis should first be established . The fact that her sleepiness improves after brief naps points the diagnosis away from
• 14
sleep apnea because these patients do not find naps refr eshing . Further evaluation is needed .
Continuous positive airway pressure Obstructive sleep apnea Sleep apnea Respiratory tract Apnea Somnolence Positive airway pressure
• 15
• 16
Dis not correct. 1% chose this.
• 17
Reassurance is not appropriate in this scenario because this patient's sleepiness is clearl y interfering with her everyday
activities .
• 18 Somnolence
• 19
E is not correct. 19% chose this.
• 20
Weight loss would be beneficial in a patient with obstructive sleep apnea . However, the fact that her sleepiness improves after
• 21
brief na ps points the diagnosis away from obstructive sleep apnea because these patients do not find naps ref reshing . Further
• 22 eval uation is needed .
• 23 Sleep apnea Obstructive sleep apnea Apnea Weight loss Somnolence

• 24

Lock
s
Suspend
0
End Block
Item: 11 of 99 ~ 1 • Mark <:] (:>- Jill ~· ~J
QID: 28 005 ..1 Previous Next Lab'V!I!ues Notes Calculator

1 • Acetazolamide carbonic anhydrase inhibitor carbonic anhydrase Sleep apnea Central sleep apnea Apnea

2 C is not correct. 9 % chose this.


3 Continuous positive airway pressure is used to treat obstructive sleep apnea. Before th is therapy is initiated, however, a
4 diagnosis should first be established . The fact that her sleepiness improves after brief naps points the diagnosis away from
5
sleep apnea because these patients do not find naps ref reshing . Further evaluation is needed .
Continuous positive airway pressure Obstructive sleep apnea Sleep apnea Respiratory tract Apnea Somnolence Positive airway pressure
6
7 Dis not correct. 1% chose this.
8 Reassurance is not appropriate in th is scenario because this patient's sleepiness is clearly interfering with her everyday
activities .
9 Somnolence
10
E is not correct. 19% chose this.
~ Weight loss would be beneficial in a patient with obstructive sleep apnea. However, the fact that her sleepiness improves after
·~ brief naps points the diagnosis away from obstructive sleep apnea because these patients do not find naps refreshing . Further
• 13 evaluation is needed .
• 14 Sleep apnea Obstructive sleep apnea Apnea Weight loss Somnolence

• 15
• 16
Bottom line :
• 17
Polysomnography is recommended for eva luating excessive daytime sleepiness when there is suspicion for obtructive sleep
• 18
apnea, other sleep-related breathing disorders, periodic limb movement disorder, narcolepsy, and central hypersomnias .
• 19 Polysomnography Narcolepsy Periodic limb movement disorder Sleep apnea Excessive daytime sleepiness Movement disorder Hypersomnia Somnolence Apnea Sleep and breathing

• 20
• 21
• 22 References:
FA Step 2 CK 9th ed pp 443-445
• 23
FA Step 2 CK 8th ed pp 414-415
• 24

Lock
s
Suspend
0
End Block
Item: 12 of 99 ~ 1 • Mark <:] (:>- Jill ~· ~J
QID: 28006 ..1 Previous Next Lab'V!I!ues Notes Calculator

1
Overnight polysomnography shows an average of 8 apnek events per hour of sleep.
2
3
What is the most appropriate treatment for this condition?
4

5
A. Acetazolamide
6
7 B. Continuous positive airway pressure ventilation
8 C. Diphenhydramine
9
D. Methylphenidate
10
E. Modafinil
.~ F. Pramipexole
• 13
• 14

• 15
• 16
• 17
• 18
• 19
• 20
• 21
• 22
• 23
• 24

Lock
s
Suspend
0
End Block
Item: 12 of 99 ~ 1 • Mark <:] (:>- Jill ~· ~J
QID: 28006 ..1 Previous Next Lab'V!I!ues Notes Calculator

1
The correct answer i s B. 9 2 0/o chose th is.
2
Weight loss and continuous positive airway pressure (CPAP) ventilation are the cornerstones of therapy for obstructive sleep
3
apnea (OSA) . The diagnosis requires 5 or more apneic events per hour of sleep with one or more of the following : 1)
4 sleepiness, nonrestorative sleep, fatigue, or insomnia symptoms; 2) waking from sleep choking, gasping, or breath holding;
5 3)habitual snoring, breathing interruptions, or both noted by bed partner or observer; 4) hypertension, mood disorder, type 2
diabetes mellitus, atrial fibril lation, congestive heart failurre, stroke, cognitive dysfunction, or coronary artery disease. If 15 or
6
greater apneic episodes occur per hour, OSA is diagnosed regardless of the other findings listed above.
7 ~1ood disorder Atrial fibrillation Continuous positive airway pressure Obstructive sleep apnea Coronary artery disease Heart failure Sleep apnea Insomnia Congestive heart failure
8 Diabetes mellitus Diabetes mellitus type 2 Hypertension Weight loss Positive airway pressure Apnea Fatigue (medical) Snoring Somnolence Stroke Respiratory tract
9
A i s not correct. 1 Ofo chose this .
10
Acetazolamide is listed as an alternative therapy w ith a proposed mechanism to increase respiratory drive however it has not
proven effective in obstructive sleep apnea .
Acetazolamide Sleep apnea Obstructive sleep apnea Apnea

• 13
C i s not correct. 1% chose this .
• 14
Diphenhydramine is used to treat insomn ia, not obstructi ve sleep apnea .
• 15 Diphenhydramine Sleep apnea Obstructive sleep apnea Insomnia Apnea
• 16
Dis not correct. 1% chose this .
• 17
Although methylphenidate can promote wakefulness, it is not used in the treatment of obstructive sleep apnea .
• 18 Methylphenidate Sleep apnea Obstructive sleep apnea Wakefulness Apnea
• 19
E i s n ot corr ect. 4 0/o c hose th is .
• 20
Modafinil may be beneficia l as an adjunctive therapy for excessive daytime sleepiness that persists despite documentation of
• 21 adequate successful therapy with continuous positive airway pressure; however, it is not the best initial therapy.
• 22 Modafinil Continuous positive airway pressure Excessive daytime sleepiness Somnolence Adjuvant therapy

• 23
F is not correct. 1 0/o chose this .
• 24

Lock
s
Suspend
0
End Block
Item: 12 of 99 ~ 1 • Mark <:] (:>- Jill ~· ~J
QID: 28006 ..1 Previous Next Lab'V!I!ues Notes Calculator

1

2
Diphenhydramine is used to treat insomnia, not obstructi ve sleep apnea .
Diphenhydramine Sleep apnea Obstructive sleep apnea Insomnia Apnea
3
4 Dis not correct. 1% chose this.
5
Although methylphenidate can promote wakefulness, it is not used in the treatment of obstructive sleep apnea .
Methylphenidate Sleep apnea Obstructive sleep apnea Wakefulness Apnea
6
7 E is not correct. 4 0/o chose th is.
8 Modafinil may be beneficial as an adjunctive therapy for excessive daytime sleepiness that persists despite documentation of
adequate successful therapy with continuous posit ive airway pressure; however, it is not the best init ial therapy.
9 Modafinil Continuous positive airway pressure Excessive daytime sleepiness Somnolence Adjuvant therapy
10
F is not correct. 1 0/o chose this.
Pramipexole, a dopamine agonist, is used to treat restless legs syndrome. I t does not have a r ole in the treatment obstructive
sleep apnea .
• 13 Restless legs syndrome Dopamine agonist Pramipexole Dopamine Sleep apnea Obstructive sleep apnea Agonist Apnea
• 14

• 15
Bottom Line :
• 16
• 17
Weight loss and continuous posit ive airway pressure ( CIPAP) ventilation are the cornerstones of therapy for obstructive sleep
apnea . Modafinil may be added w hen patients continue to experience excessive daytime sleepiness despite adequate
• 18
conventional therapy w ith CPAP.
• 19 Modafinil Continuous positive airway pressure Sleep apnea Obstructive sleep apnea Excessive daytime sleepiness Weight loss Positive airway pressure Apnea Somnolence

• 20
• 21
• 22 References:
FA Step 2 CK 9th ed pp 443-445
• 23
FA Step 2 CK 8th ed pp 414-415
• 24

Lock
s
Suspend
0
End Block
Item: 13 of 99 ~ 1 • Mark <:] (:>- Jill ~· ~J
QID: 23708 ..1 Previous Next Lab'V!I!ues Notes Calculator

1 • •
A 53-year-old man in the neurosurgical intensive care unit because of glioblastoma mu lt iforme suddenly develops ~~AI
2 headache, altered mental status, dilation of the ipsilatera l pupil, ptosis, and loss of media l gaze . His pu lse is 49/min, blood
3 pressure is 170/90 mm Hg, and r espiratory rate is 9/min . Concern for increased intracranial pressure is raised .
4

5 The ocular find ings seen in th is patient are most concern ing for pathology involving which cranial nerve?
6
7 A. Cranial nerve I II
8
B. Cranial nerve VI
9
10
C. Cranial nerve VII

D. Cranial nerve X

E. Cranial nerve XII


• 13
• 14

• 15
• 16
• 17
• 18
• 19
• 20
• 21
• 22
• 23
• 24

Lock
s
Suspend
0
End Block
Item: 13 of 99 ~ 1 • Mark <:] (:>- Jill ~· ~J
QID: 23708 ..1 Previous Next Lab'V!I!ues Notes Calculator

2 The correct answer i s A. 900/o chose this.


3 Crania l nerve {CN) III is affected in transtentorial herniation syndromes, wh ich can be caused by increased intracranial
pressure . Increased intracranial pressure is suspected given the presence of a brain tumor and Cushing t riad: bradycardia,
4
hypertension, and decreased respiratory rate . Although the most accurate way to measure intracranial pressure is directly,
5 this patient has all the ind irect signs of transtentorial herniation . Transtentorial herniation can affect the th ird cranial nerve
6 but is can also cause compression of the upper bra instem, and the cerebra l peduncles. CN III defects may also cause eye
7
deviation downward and outward .
Brain herniation Bradycardia Intracranial pressure Brainstem Cerebral peduncle Cranial nerves Hypertension Brain tumor Respiratory rate Oculomotor nerve Neoplasm Human brain
8
Brain Skull Hernia
9
10 B is not correct. ] Ofo chose this.
Patients with a CN VI nerve palsy often complain of horizontal nystagmus, not ptosis, or loss of medial gaze. CN nerve VI
palsies can occur with increased intracranial pressure but are not seen in this patient. Increased intracranial pressure that
result s in downward displacement of the brainstem can cause stretching of CN VI , which is tet hered as it exits the pons and
13 inside Dorello canal.
• 14 Nystagmus Pons Ptosis (eyelid) Brainstem Intracranial pressure Bell's palsy

• 15 Cis not correct. 1% chose this .


• 16 CN VII is responsible for taste sensation from the anterio r two-th irds of the tongue . It also supplies motor function to the
• 17 muscles that allow facial expression . It is not responsible for the ocular findings described above .
Facial expression Facial nerve
• 18
• 19 D i s not correct. 1 Ofo chose this .
• 20 CN X is the vagus nerve. Th is supplies the body w ith parasympathetic tone, and can cause bradycard ia; however, vagus nerve
• 21 pat hology would not explain the ocu lar findings in th is patient. Cushing triad resu lts due to both sympathetic and
parasympathetic responses. The sympathetic system causes hypertension via constriction of arteries. Baroreceptors then
• 22
detect this increase in blood pressu re and activate the parasympathetic system, wh ich leads to bradycardia via vagal nerve
• 23 stimulation .
,.. R~rlvr::arrli::a \/::.nuc: n~ru,c. P::ar::ac:vmn::ath110tir nPrvnuc: c:vc:tcm c;vmn::athPtir n10rvnuc: c:v~,c.m ~vnPr+Pnc:inn Rlnnrf nriOC:C:IIrP R::arnrprc.ntnr V:~~nuc: n,c.ntiO c:tim11l::atinn P::athnlnnv 6rt,c.rv
• 24

Lock
s
Suspend
0
End Block
Item: 13 of 99 ~ 1 • Mark <:] (:>- Jill ~· ~J
QID: 23708 ..1 Previous Next Lab'V!I!ues Notes Calculator

1
Cis not correct. 10/o chose this.
2
CN VII is responsible for taste sensation from the anterior two-th irds of the tongue . It also supplies motor function to the
3 muscles that allow facial expression . I t is not responsible for the ocular findings described above .
4 Facial expression Facial nerve

5 Dis not correct. 1% chose this.


6 CN X is the vagus nerve. Th is supplies the body w ith parasympathetic tone, and can cause bradycardia; however, vagus nerve
7 pathology would not explain the ocu lar findings in th is patient. Cushing triad resu lts due to both sympathetic and
8 parasympathetic responses. The sympathetic system causes hypertension via constriction of arteries. Baroreceptors then
detect this increase in blood pressu re and activate the parasympathetic system, wh ich leads to bradycardia via vagal nerve
9
stimulation .
10 Bradycardia Vagus nerve Parasympathetic nervous system Sympathetic nervous system Hypertension Blood pressure Baroreceptor Vagus nerve stimulation Pathology Artery

E is not correct. 1 0/o chose th is.


CN XII, the hypoglossa l nerve, innervates the muscles of the tongue . It does not produce any ocu lar findings.
13 Hypoglossal nerve
• 14

• 15
Bottom Line :
• 16
• 17
CN III is affected in transtentorial herniation syndromes, which can be caused by increased intracran ial pressure . Brain
tumor and Cushing triad (bradycard ia, hypertension, and decreased respiratory rate) indicate increased intracranial
• 18
pressure .
• 19 Brain herniation Bradycardia Intracranial pressure Respiratory rate Hypertension Brain tumor Neoplasm Oculomotor nerve Human brain Hernia Brain

• 20
• 21
• 22 References:
FA Step 2 CK 9th ed p 266
• 23
FA Step 2 CK 8th ed p 250
• 24

Lock
s
Suspend
0
End Block
Item: 14 of 99 ~ 1 • Mark <:] (:>- Jill ~· ~J
QID: 22196 ..1 Previous Next Lab'V!I!ues Notes Calculator

1 • •
A 40-year-old woman experiences sporadic headaches throughout the month . She describes them as bilateral and usually ~~AI
2 occipital, radiating in a band-like pattern around the head . They usually last several hours and are not associated with any
3 other symptoms. There is no nausea, vomiting, photo- or phonophobia, or any type of aura. She notices that her
4 headaches are more frequent during t imes when she feels exceptionally stressed. Her medical history is otherwise on ly
sign ificant for chronic low back pain, for wh ich she takes hydrocodone. Her physical examination is unremarkable.
5
6
Which of the following is the most appropriate management?
7
8
A. CT scan of the head to rule out subarachnoid hemorrhage
9
10 B. Discontinuation of hydrocodone

C. Oxygen therapy
D. Prescription of a serotonin agonist
13
• 14 E. Relaxation techniques
• 15
• 16
• 17
• 18
• 19
• 20
• 21
• 22
• 23
• 24

Lock
s
Suspend
0
End Block
Item: 14 of 99 ~ 1 • Mark <:] (:>- Jill ~· ~J
QID: 22196 ..1 Previous Next Lab'V!I!ues Notes Calculator

2
3
The correct answer is E. 69% chose this.
4
The description is typical for tension-type headaches. The pain is usually bilateral, occipital, and described as being like a
5 compressive band around the head . There are usually no other associated symptoms. Stress management and relaxation
6 therapies can be effective treatment modalit ies, as can other more conventional pharmacotherapies such as nonsteroidal
7
anti-inflammatory drugs .
Pharmacotherapy Nonsteroidal anti-inflammatory drug Stress management Anti-inflammatory
8
9 A is not correct. 2 % chose this.
10
The headaches associated with su barachnoid hemorrhage are sudden and severe, often being described as being "the worst
headache of one's life. " They can also be associated with any number of neurologic symptoms, and do not recur th roughout
the month in this pattern . Subarachnoid hemorrhages should be considered medical emergencies .
Subarachnoid hemorrhage Headache Medical emergency Bleeding Neurology

13
B is not correct. 150/o chose th is.
14
Medication overuse or rebound headaches have been reported in patients who frequently use analgesic medications. Usually
• 15 these occur when patients discontinue the medications, and so th is patient would potentially be at risk for th is if she
• 16 discontinued her hydrocodone .
Hydrocodone Analgesic Rebound effect Pharmaceutical drug
• 17
• 18 C is not correct. 60/o chose this.
• 19 Oxygen therapy has been shown to be useful in the treatment of cluster headaches . Cluster headaches are more likely to
• 20 affect men, and tend to occur in groups or "clusters" w ith a well -defined pattern of cycl ical recurrence and remission . The
pain is sometimes associated with lacrimation, conjunctival injection, nasal congestion, and Horner's syndrome, and is usually
• 21
located unilaterally behind the nose or one eye .
• 22 Homer's syndrome Tears Cluster headache Nasal congestion Conjunctiva Conjunctivitis Oxygen Oxygen therapy Cure

• 23
D is not correct. SOfo chose this.
• 24

Lock
s
Suspend
0
End Block
Item: 14 of 99 ~ 1 • Mark <:] (:>- Jill ~· ~J
QID: 22196 ..1 Previous Next Lab'V!I!ues Notes Calculator
• I I • • I I ..
1
Medication overuse or rebound headaches have been reported in patients who frequently use analgesic medications. Usually
2
these occur when patients discontinue the medications, and so th is patient would potentially be at risk for th is if she
3 discontinued her hydrocodone.
4 Hydrocodone Analgesic Rebound effect Pharmaceutical drug

5 C is not correct. 6 0/o chos e this .


6 Oxygen therapy has been shown to be useful in the t r eatment of cluster headaches . Cluster headaches are more likely to
7 affect men, and tend to occur in groups or "clusters" w ith a well -defined pattern of cycl ical recu rrence and remission . The
8 pain is sometimes associated with lacrimation , conjunctival injection, nasal congestion, and Horner's syndrome, and is usually
located unilaterally behind the nose or one eye.
9 Homer's syndrome Tears Cluster headache Nasal congestion Conjunctiva Conjunctivitis Oxygen Oxygen therapy Cure
10
D is not correct. S Ofo chos e this .
Serotonin agonists such as sumatriptan and other triptan medications are used in the treatment of migraines. Migraine
headaches usually present earlier in life, and are associat ed with symptoms such as nausea, vomit ing, photophobia, and
13 certain types of auras . They are also mostly unilateral.
14 Triptan Sumatriptan Photophobia Serotonin Migraine Nausea Serotonin receptor agonist Vomiting Agonist

• 15
• 16
Bottom line :
• 17
Tension-type headaches present with bilateral, occipital pain, described as being like a band ar ound the head . There are
• 18
usually no other associated symptoms . These headaches often respond to relaxation and stress management techniques .
• 19 Stress management Occiput

• 20
• 21
• 22 References:
FA Step 2 CK 9th ed p 268
• 23
FA Step 2 CK 8th ed p 254
• 24

Lock
s
Suspend
0
End Block
Item: 15 of 99 ~ 1 • Mark <:] (:>- Jill ~· ~J
QID: 21070 ..1 Previous Next Lab'V!I!ues Notes Calculator

1
A 75-year-old man with a history of diabetes and hypertension is brought to the cl inic by his concerned daughter because
2 of worsening memory and behavior. Over the past year he has had a progressive deterioration in his ability to care for
3 himself. Occasionally she discovers that the stove is left o,n, and recently he has become lost on his way home from
4 running errands. He lacks attention to his personal hygiene and is irritable . On examination, he is responsive and pleasant but
w ith marked poverty of speech. Neurologic exam is nonfocal. Mini-Mental State Examination r eveals a scor e of 21, with deficits
5
in memory, object identification , and insight.
6
7
What is the next step in management?
8
9
10
A. CT of the brain

B. Electr oencephalogram
13
C. Genetic testing for APOE4
14
• 15 D. Laboratory testing for v itamin B12, folate, thyroid-stimulating hormone (TSH), and rapid plasma reagin ( RPR)
• 16 E. Start donepezil
• 17
• 18
• 19
• 20
• 21
• 22
• 23
• 24

Lock
s
Suspend
0
End Block
Item: 15 of 99 ~ 1 • Mark <:] (:>- Jill ~· ~J
QID: 21070 ..1 Previous Next Lab'V!I!ues Notes Calculator

Like Dislike

2
3
The correct answer is D. 690/o chose this.
4
This patient's symptoms of decreased memory, attention, language, and problem solving are suggestive of dementia . The
5
most common types of dementia include Alzheimer's, vascular, frontotemporal, and Lewy body dementia . However, the first
6 step in evaluating any patient w ith dementia is to check for reversible causes of dementia because these are treatable .
7 Dementia with lewy bodies Dementia Lewy body Blood vessel

8 A is not correct. 120/o chose this.


9 Imaging may be considered in the evaluation of a patient with mental status changes. However, there are no focal exam
10 findings to suggest a mass lesion .
Neoplasm Lesion Altered state of consciousness

~
13
2 B is not correct. 20/o chose this.
An electroencephalogram (EEG) can show diffuse slowing of brain activity in dementia . Some types of dementia also have
14 characteristic EEG findings, such as sharp-wave complexes in Creutzfelt-Jakob disease . However, this would not be the first
step in evaluation .
15
Electroencephalography Dementia Human brain Brain
• 16
• 17
C is not correct. JO/o chose this.
The most common type of dementia is Alzheimer's disease . Fam ilial forms are associated with the AP04E allele and the p-App
• 18
gene . The diagnosis of Alzheimer's is usually clinical, as definitive diagnosis can only be made at autopsy. The first step in
• 19 evaluating this patient for dementia is to evaluate for reversible causes .
• 20 Alzheimer's disease Allele Gene Dementia Autopsy

• 21
E is not correct. 14% chose this.
• 22 Donepezil is an acetylcholinesterase inhibitor used in the treatment of Alzheimer's dementia . It has been shown to have
• 23 modest benefits in cognition . However, it is important to first rule out treatable causes of dementia before considering
• 24 y startino this aoent .

Lock
s
Suspend
0
End Block
Item: 15 of 99 ~ 1 • Mark <:] (:>- Jill ~· ~J
QID: 21070 ..1 Previous Next Lab'V!I!ues Notes Calculator

1
B is not correct. 20/o chose this.
2
An electroencephalogram (EEG) can show diffuse slowing of brain activity in dementia . Some types of dementia also have
3 characteristic EEG findings, such as sharp-wave complexes in Creutzfelt-Jakob disease . However, this would not be the first
4 step in evaluation.
Electroencephalography Dementia Human brain Brain
5
6 C is not correct. 3% chose this.
7 The most common type of dementia is Alzheimer's disease . Fam ilial forms are associated with the AP04E allele and the p-App
8 gene . The diagnosis of Alzheimer's is usually clinical, as definitive diagnosis can only be made at autopsy. The first step in
evaluating this patient for dementia is to evaluate for reversible causes .
9 Alzheimer's disease Allele Gene Dementia Autopsy
10
E is not correct. 14% chose this.
Donepezil is an acetylcholinesterase inhibitor used in the treatment of Alzheimer's dementia . It has been shown to have
modest benefits in cognition . However, it is important to first rule out treatable causes of dementia before considering
13 starting this agent.
14 Acetylcholinesterase inhibitor Donepezil Acetylcholinesterase Dementia Alzheimer's disease Cognition Enzyme inhibitor

15
• 16
Bottom line:
• 17
The first step in evaluating a patient for dementia is to r ule out reversible causes . Laboratory testing for TSH, vitamin B1 2,
• 18
folate, and RPR is wa rran ted .
• 19 Folic acid Vitamin 812 B vitamins Dementia Thyroid-stimulating hormone Vitamin

• 20
• 21
• 22 References:
FA Step 2 CK 9th ed pp 430-431
• 23
FA Step 2 CK 8th ed pp 265-266
• 24

Lock
s
Suspend
0
End Block
Item: 16 of 99 ~ 1 • Mark <:] (:>- Jill ~· ~J
QID: 22415 ..1 Previous Next Lab'V!I!ues Notes Calculator

1
A 64-year-old man presents to the emergency department with what he describes as "the worst headache of my life" . He
2 also complains of nausea, vom it ing, photophobia, and nuchal rig idity. He denies a history of cerebra l aneurysms or
3 trauma . Non-contrast CT of the head shows no evidence of blood products or increased intracranial pressure .
4

5 Which of the following is the best next step in the work-up"


6
7 A. Contrast CT of the head
8
B. Discharge home and tell the patient to return if headache persists
9
10
C. Lumbar puncture

D. MRI

E. Repeat non-contrast CT of the head


13
14

15
• 16
• 17
• 18
• 19
• 20
• 21
• 22
• 23
• 24

Lock
s
Suspend
0
End Block
Item: 16 of 99 ~ 1 • Mark <:] (:>- Jill ~· ~J
QID: 22415 ..1 Previous Next Lab'V!I!ues Notes Calculator

2 The correct answer i s C. 680/o chose this .


3 This patient has many concern ing signs/symptoms of a subarachnoid hemorrhage (SAH) including the "worst headache of his
life," photophobia, and nuchal rigidity. The nuchal rigidity may not develop until hours after the bleed since it is caused by the
4
breakdown of blood products within the cerebrospinal fluid (CSF), wh ich leads to an aseptic meningitis. Many patients report
5 a history of a sudden and severe headache (the sentinel headache) that precedes a major SAH by 6- 20 days; this is beli eved
6 to represent a m inor hemorrhage . Most SAHs are caused by a ruptured saccular aneurysm . When there is a strong suspicion
7
of SAH yet non-contrast-enhanced CT of the head is negative, it is necessary to do a lumbar puncture to look for
xanthochromic CSF (a pink or yellow tint representing hemoglobin degradation products) . With more minor bleeds, CT scans
8 may be negative in up to 55% of patients, but xanthochromic CSF from lumbar puncture is seen in 90% of patients within 12
9 hours of rupture. When there is suspicion of SAH, lumba r puncture is mandatory even if CT findings are negative.
Subarachnoid hemorrhage Lumbar puncture Photophobia Cerebrospinal fluid Asepti c meningitis Meningitis Hemoglobin CT scan Meningism Headache Neck stiffness Aneurysm
10
Bleeding Neck Lumbar Symptom Lumbar vertebrae

A is not correct. 100/o chose this .


13 If one suspects subarachnoid hemorrhage or any intracranial bleed ing, contrast is not needed because fresh blood appears
14 bright wh ite on CT even without contrast. Contrast is used for head CT scans for suspected neoplasms or infectious etiologies.
Subarachnoid hemorrhage CT scan Etiology Bleeding Neoplasm Intracranial hemorrhage
15
16 B is not correct. 20/o chose this.
• 17 When signs and symptoms point to a subarachnoid hemorrhage (SAH), the patient cannot be sent home w ithout fu rther
• 18 workup to rule it out. SAH is associated with a high fatality rate (51%) .
Subarachnoid hemorrhage Bleeding
• 19
• 20 Dis not correct. 110/o chose this .
• 21 MRI is not the next best step in diagnosis . Li mited data suggest that specific sequences on MRI may be nearly as sensitive as
head CT for the acute detection of subarachnoid hemmor hage; however, this patient has already had a negative head CT and
• 22
therefore another study is indicated for the diagnosis .
• 23 Magnetic resonance imaging CT scan
• 24

Lock
s
Suspend
0
End Block
Item: 16 of 99 ~ 1 • Mark <:] (:>- Jill ~· ~J
QID: 22415 ..1 Previous Next Lab'V!I!ues Notes Calculator

1 • If one suspects subarachnoid hemorrhage or any intracranial bleeding, contrast is not needed because fresh blood appears
2
bright white on CT even without contrast. Contrast is used for head CT scans for suspected neoplasms or infectious etiolog ies.
Subarachnoid hemorrhage cr scan Etiology Bleeding Neoplasm Intracranial hemonrhage
3
4 B is not correct. 20/o chose this.
5
When signs and symptoms point to a subarachnoid hemorrhage (SAH), the patient cannot be sent home w ithout further
workup to rule it out. SAH is associated with a high fata lity rate (51%).
6 Subarachnoid hemorrhage Bleeding
7
Dis not correct. 110/o chose this.
8
MRI is not the next best step in diagnosis. Limited data suggest that specific sequences on MRI may be nearly as sensitive as
9
head CT for the acute detection of subarachnoid hemmorhage; however, this patient has already had a negative head CT and
10 therefore another study is indicated for the diagnosis.
Magnetic resonance imaging cr scan

E is not correct. 90/o chose this.


13 Repeat non-contrast CT of the head would not have any diagnostic utility in light of the previous negative findings.
14 Central Time Zone cr scan Connecticut

15
16
Bottom line:
• 17
When there is a strong suspicion of SAH but CT of the head is negative, it is necessary to do a lumbar puncture to look for
• 18
xanthochromic cerebrospina l fluid .
• 19 lumbar puncture Cerebrospinal fluid lumbar lumbar vertebrae

• 20
• 21
• 22 References:
FA Step 2 CK 9th ed p 263
• 23
FA Step 2 CK 8th ed pp 247-249
• 24

Lock
s
Suspend
0
End Block
Item: 17 of 99 ~ 1 • Mark <:] (:>- Jill ~· ~J
QID: 222 19 ..1 Previous Next Lab'V!I!ues Notes Calculator

1 • •
A 61-year-old right-handed man presents to the emergency department with new-onset speech difficulties. His wife says ~~AI
2 he has not been making any sense for the past hour or so. There was no trauma, loss of consciousness, convulsions, or
3 bowel/bladder incontinence. His past medical history is significant for coronary artery disease, poorly controlled diabetes,
4 and hyperlipidemia . He has a 30-pack year smoking history. On examination, his speech is noted to be fluent but is mostly
meaningless. He is unable to repeat back the phrase "no ifs, ands, or buts." When asked to name a watch, he replies "round
5
thing ." He was unable to comprehend your command to raise his arms. He does not seem frustrated about his ordeal and
6 appears quite cheerful.
7
8 Where is the lesion r esponsible for this kind of aphasia located?
9
10 A. Arcuate fasciculus

B. Frontal lobe region

13 C. Large left hemisphere lesion


14 D. Left posterior inferior frontal gyrus
15
E. Left posterior superior temporal lobe
16
• 17
• 18
• 19
• 20
• 21
• 22
• 23
• 24

Lock
s
Suspend
0
End Block
Item: 17 of 99 ~ 1 • Mark <:] (:>- Jill ~· ~J
QID: 222 19 ..1 Previous Next Lab'V!I!ues Notes Calculator

1
The correct answer is E. 60% chose this.
2
This patient presen ts with Wernicke aphasia, w here patients have fluent speech that is mostly meaningless and impaired
3 comprehension . Patients may exhibit paraphasic errors (i e, inappropriate substitutions of a word with one of similar meaning
4 or sound), neologisms (ie, "nonwords"), problems with naming, and inability to repeat back phrases or words . However, most
5
patients are unaware of their speech deficit and will behave as if they are carrying on a normal conversation despite evidence
to the contrary. Sometimes Wernicke aphasia is accompanied by a contralateral superior homonymous quadrantanopia, which
6 may be subtly tested w ith decreased blinking to threat on the right side . The lesion is located in Wern icke area, found in the
7 left posterior superior temporal lobe (perisylvian) . For the distinguishing featu res of aphasia syndromes commonly tested in
8 the USMLE, note the following table .
Receptive aphasia Aphasia Temporal lobe Wernicke's area Paraphasia Quadrantanopia Contralateral lesion Neologism Anatomical terms of location
9
10 Aphasia
Fluency Repetition Comprehension
syndrome

~ Impaired Impaired Intact


13
Wernicke Intact Impaired Impaired
14

15 Conduction Intact Impaired Intact


16
Transcortical
Impaired Intact Intact
17 motor

• 18 Transcortical
Intact Intact Impaired
sensory
• 19
Transcortical
• 20 Impaired Intact Impaired
mixed
• 21
Global Impaired Impaired Impaired
• 22
• 23 A is not correct. 100/o chose this .
• 24

Lock
s
Suspend
0
End Block
Item: 17 of 99 ~ Mark <:] (:>- Jill ~· ~J

----
1 •

QID: 222 19 ..1 Previous Next Lab'V!I!ues Notes Calculator

2 A i s not correct. 100/o chose this .


3 Conduction aphasia is characterized by problems w ith repeating what is sa id but preserved fluency and comprehension . The
lesion is most likely in the arcuate fasciculus .
4
Conduction aphasia Arcuate fasciculus Aphasia Lesion
5
B is not correct. 6 0/o chose this .
6
Frontal lobe atrophy is associated with progressive nonfluent aphasia . It is also characterized by word- finding difficulty and
7
effortful speech but preserved understanding of word meaning . I n the late stages, the patient becomes mute .
8 Frontal lobe Aphasia Atrophy Progressive nonfluent aphasia Speech disorder
9
C i s not correct. 9 % chose this.
10
The patient would suffer from a global aphasia, character ized by a globa l impa irment in language comprehension, as well as
fluency and word repetition . The lesion is most likely a large left hemisphere lesion .
Aphasia Global aphasia Lateralization of brain function Lesion Speech repetition Sentence processing

13 D i s not correct. 150/o chose this.


14 Broca aphasia is characterized by preserved comprehension but problems w ith language production . The lesion is most likely
15 in Broca area, left posterior inferior frontal gyrus, and is usually caused by a large stroke in the territory of the superior
16 division of the middle cerebral artery. It can present with associated right-sided hemiparesis.
Hemiparesis Inferior frontal gyrus Aphasia Middle cerebral artery Lesion Broca's area Stroke Primary progressive aphasia Cerebral arteries
17
• 18
• 19 Bottom Line :
• 20 Wernicke aphasia features problems with language comprehension as well as impairment of both language input and output .
• 21 Receptive aphasia Aphasia Sentence processing

• 22
• 23
References:
• 24

Lock
s
Suspend
0
End Block
Item: 17 of 99 ~ 1 • Mark <:] (:>- Jill ~· ~J
QID: 222 19 ..1 Previous Next Lab'V!I!ues Notes Calculator

1 • Conduction aphasia is characterized by problems w ith repeating what is sa id but preserved fluency and comprehension . The
2
lesion is most likely in the arcuate fasciculus .
Conduction aphasia Arcuate fasciculus Aphasia Lesion
3
4 B is not correct. 60/o chose this .
5
Frontal lobe atrophy is associated with progressive nonfluent aphasia . It is also characterized by word- finding difficulty and
effortful speech but preserved understanding of word meaning . I n the late stages, the patient becomes mute .
6 Frontal lobe Aphasia Atrophy Progressive nonfluent aphasia Speech disorder
7
C is not correct. 9 % chos e this .
8
The patient would suffer from a global aphasia, cha racter ized by a globa l impa irment in language comprehension, as well as
9
fluency and word repetition . The lesion is most likely a large left hemisphere lesion .
10 Aphasia Global aphasia Lateralization of brain function Lesion Speech repetition Sentence processing

D is not correct. 150/o chose this .


Broca aphasia is characterized by preserved comprehension but problems with language production . The lesion is most likely
13 in Broca area, left posterior inferior frontal gyrus, and is usually caused by a large stroke in the territory of the superior
14 division of the middle cerebral artery. It can present with associated right-sided hemiparesis.
Hemiparesis Inferior frontal gyrus Aphasia Middle cerebral artery Lesion Broca's area Stroke Primary progressive aphasia Cerebral arteries
15
16
17 Bottom line :
• 18
Wernicke aphasia features problems with language comprehension as well as impairment of both language input and output.
• 19 Receptive aphasia Aphasia Sentence processing

• 20
• 21
• 22 References:
FA Step 2 CK 9th ed p 289
• 23
FA Step 2 CK 8th ed p 274
• 24

Lock
s
Suspend
0
End Block
Item: 18 of 99 ~ 1 • Mark <:] (:>- Jill ~· ~J
QID: 2107 2 ..1 Previous Next Lab'V!I!ues Notes Calculator

1 • •
A 45-year-old veteran is seen in the clinic because of poo1r impu lse control. Over the past month he has spent all of his ~~AI
2 savings on business ventures and is currently estranged from his wife. He admits a decreased need for sleep, a new sense
3 of grandiosity, and increased energy spent on researching projects to make money. He has no prior psychiatric history, and
4 his only medica l history includes treatment for gonorrhea during his m ilitary service and an appendectomy at 36 years old .
Physical examination reveals a th in, midd le-age man with pressured speech. His pupillary examination shows the pupils are
5
reactive to convergence but not to light.
6
7
Infection w ith which of the following organisms is the most likely etiology of his symptoms?
8
9
A. Chlamydia trachomatis
10
B. Herpes simplex virus
C. Neisseria gonorrhoeae
13
D. Toxoplasma
14

15
E. Treponema pallidum
16
17
• 18
• 19
• 20
• 21
• 22
• 23
• 24

Lock
s
Suspend
0
End Block
Item: 18 of 99 ~ 1 • Mark <:] (:>- Jill ~· ~J
QID: 2107 2 ..1 Previous Next Lab'V!I!ues Notes Calculator

2
3
4 The correct answer i s E. 80% chose this .
5 Argyll Robertson pupil in the presence of acute mania is suggestive of neurosyphilis. Other central nervous system
6 manifestations, such as gait abnormalities (ie, tabes dorsalis) are now rare. A remote history of sexually transmitted disease
7
and a solitary presentation of a manic episode without a prior psychiatric history are also important in this case . Often
patients may have been infected years earlier yet do not present to the physician until this stage of infection . FTA-ABS is used
8 to confirm the diagnosis. It is a treponema! test that is both sensitive and specific in latent, secondary, tertiary, and
9 quaternary syphilis. These tests are typically used as confirmatory tests when the nontreponemal tests such as VDRL and RPR
10 are reactive. VDRL and RPR can be used in an office setting for initial screening and serial follow-up.
Tabes dorsalis Sexually transmitted infection Syphilis Argyll Robertson pupil Neurosyphilis Central nervous system Fluorescent treponema I antibody absorption test

Nontreponemal tests for syphilis Mania Nervous system Gait abnormality Gait Venereal Disease Research laboratory test

13 A is not correct. 3% chose this.


14 Patients with Chlamydia trachomatis infections usua lly pr esent with urethritis, epididymitis, and urethral discharge.
Chlamydia trachomatis Epididymitis Urethritis Chlamydia infection Urethra Chlamydia (genus)
15
16 B is not correct. 100/o chose th is.
17 Herpes simplex virus can cause altered mental status, seizures, and meningitis in young children but is unlikely in an
18
immune-competent adult host.
Herpes simplex virus Meningitis Virus Herpes simplex Epileptic seizure Immunocompetence Altered level of consciousness
• 19
• 20 C i s not correct. 30/o chose this .
• 21 Neisseria gonorrhoeae may cause monoarticular arth ritis and genital symptoms, but is unlikely to result in acute psychiatric
symptoms such as in this patient .
• 22
Neisseria gonorrhoeae Arthritis Neisseria
• 23
• 24
D is not correct. 4 % chose this .

Lock
s
Suspend
0
End Block
Item: 18 of 99 ~ 1 • Mark <:] (:>- Jill ~· ~J
QID: 2107 2 ..1 Previous Next Lab'V!I!ues Notes Calculator
• •
1

2
Patients with Chlamydia trachomatis infections usual ly present with urethritis, epididymit is, and urethral discharge.
Chlamydia trachomatis Epididymitis Urethritis Chlamydia infection Urethra Chlamydia (genus)
3
4 B is not correct. 100/o chose this.
5
Herpes simplex virus can cause altered mental status, seizures, and meningit is in young children but is un li kely in an
immune-competent adult host.
6 Herpes simplex virus Meningitis Virus Herpes simplex Epileptic seizure Immunocompetence Altered level of consciousness
7
C is not correct. 3% chose this.
8
Neisseria gonorrhoeae may cause monoarticular arthrit is and genital symptoms, but is unlikely to result in acute psych iatric
9
symptoms such as in this patient.
10 Neisseria gonorrhoeae Arthritis Neisseria

D is not correct. 4% chose this.


Toxoplasmosis is usua lly transmitted by contact w ith cat feces and is also unusual in an immunocompetent host. It is unlikely
13 to result in this patient's cli nical findings.
14 Toxoplasmosis Immunocompetence Feces Cat

15
16
Bottom line:
17
Argyll Robertson pupils (pupils reactive to convergence but not to light) in the presence of acute mania are suggestive of
18 neur osyph il is .
• 19 Neurosyphilis Syphilis Argyll Robertson pupil ~1ania Douglas Argyll Robertson Bipolar disorder

• 20
• 21
• 22 References:
FA Step 2 CK 9th ed pp 220; 349
• 23
FA Step 2 CK 8th ed pp 200-202
• 24

Lock
s
Suspend
0
End Block
Item: 19 of 99 ~ 1 • Mark <:] (:>- Jill ~· ~J
QID: 23610 ..1 Previous Next Lab'V!I!ues Notes Calculator

1 • •
A 33-year-old fema le office secretary comes to the physician complain ing of waking up in the m iddle of the night with ~~AI
2 numbness and tingling in her hands. She says that the numbness is mostly in her thumb, index, and midd le fingers. She
3 says that her workload has recently increased . Some of her other friends at the office have been having simila r symptoms.
4

5 What step should the physician do next to make the diagnosis?


6
7 A. Do nothing ; no imaging or testing is necessary to make the diagnosis
8
B. Have the patient move her hand in a circular motion
9
10
C. Tap on the patient's med ian nerve to try to elicit symptoms

D. Tap on the patient's ulna r nerve to try to elicit sym ptoms

E. X-ray the patient's wrist


13
14

15
16
17
18
• 19
• 20
• 21
• 22
• 23
• 24

Lock
s
Suspend
0
End Block
Item: 19 of 99 ~ 1 • Mark <:] (:>- Jill ~· ~J
QID: 23610 ..1 Previous Next Lab'V!I!ues Notes Calculator

1 •
2 The correct answer is C. 900/o chose this .
3 This patient has carpal tunnel syndrome, which most often occurs in individuals who do repetitive hand work. These
individuals usually have symptoms of numbness and tingling in their hands, especially at night or after a certain number of
4
hours of continuous work, in the distribution of the median nerve distribution (radial 3. 5 fingers) . The symptoms can be
5 reproduced by acute wr ist flexion maintained for 30-60 seconds (the Pha len maneuver) or by pressing on the median nerve
6 over t he carpal tunnel (the Hoffman-Tine! test) . First-line treatment involves the use of spli nts and anti-inflammatory agents,
7
w ith surgery used for more refractory cases.
Carpal tunnel syndrome Phalen maneuver Median nerve Flexion Paresthesia Carpal tunnel Carpal bones Splint (medicine) Hypoesthesia
8
9 A is not correct. 6 % chos e this .
10
Some testing should be done to elicit symptoms so that nothing more serious is overlooked .
B is not correct. 2 0/o chos e this .

~
13
2 Moving the patient's hand in a circular motion would not elicit any symptoms because it is not directed toward the median
nerve .
Median nerve
14
D is not correct. OOfo chos e this .
15
Carpal tunnel syndrome is a result of inflammation causing constriction of the median nerve, not the ulnar nerve.
16 Carpal tunnel syndrome Ulnar nerve Median nerve Carpal tunnel Carpal bones Inflammation
17
E is not correct. 2 0/o c hose th is.
18
Although an x-ray of the patient's wrist should be done eventually to rule out other pathology, th is wou ld not be the first step.
19
Pathology X-ray
• 20
• 21
• 22 Bottom line :
• 23 Patients with carpal tunnels syndrome often present with numbness and tingling in their hands, especially at night or after
• 24 hours of continuous work usinq their hands. Symptoms can be reproduced by acute wrist flexion maintained for 30-60 •

Lock
s
Suspend
0
End Block
Item: 19 of 99 ~ 1 • Mark <:] (:>- Jill ~· ~J
QID: 23610 ..1 Previous Next Lab'V!I!ues Notes Calculator

1
-- --- ......... -
Carpal tunnel syndrome Phalen maneuver Median nerve Flexion Paresthesia Carpal tunnel Carpal bones Splint (medicine) Hypoesthesia
2
A i s not correct. 6 % chose this .
3
Some testing should be done to elicit symptoms so that nothing more serious is overlooked.
4
B is not correct. 2 0/o chose this.
5
Moving the patient's hand in a circu lar motion would not elicit any symptoms because it is not directed toward the median
6
nerve .
7 Median nerve

8
D i s not correct. OOfo chose this.
9
Carpal tunnel syndrome is a result of inflammation causing constriction of the median nerve, not the ulnar nerve.
10 Carpal tunnel syndrome Ulnar nerve Median nerve Carpal tunnel Carpal bones Inflammation

E i s not corr ect. 2 0/o c hose th is.


Although an x-ray of the patient's wrist should be done eventually to rule out other pathology, th is would not be the first step.
13 Pathology X-ray
14

15
Bottom Line :
16
17
Patients with carpal tunnels syndrome often present with numbness and t ingling in their hands, especially at night or after
hours of continuous work using their hands. Symptoms can be reproduced by acute wrist flexion maintained for 30- 60
18 seconds (the Phalen maneuver) or by pressing on the median nerve over the carpal tunnel (the Hoffman-Tine! test) .
19 Median nerve Phalen maneuver Flexion Carpal tunnel Paresthesia Carpal bones Hypoesthesia

• 20
• 21
• 22 References:
FA Step 2 CK 9th ed pp 239-240
• 23
FA Step 2 CK 8th ed pp 221-222
• 24

Lock
s
Suspend
0
End Block
Item: 20 of 99 ~ 1 • Mark <:] (:>- Jill ~· ~J
QID: 22073 ..1 Previous Next Lab'V!I!ues Notes Calculator

1
A 65-year-old Asian man with a history of hypertension and diabetes presents to the emergency department w ith
2 decreased vision. The patient was sitting at home when he suddenly began having severe eye pa in, headache, and
3 nausea. Exam ination reveals bilateral conjunctival injection and corneal edema. Visual acu ity is decreased in both eyes,
4 and the patient states that he is seeing "ha los" surrounding the lights in the bu ilding . Gon ioscopy revea ls closed angles, and slit-
lamp grading revea ls shallow anterior chambers.
5
6
Which of the following would best treat this patient's cond it ion?
7
8
A. Atropine
9
10 B. Methylprednisolone sodium succinate
C. Pilocarpine

D. Scopolam ine
13
14 E. Triamcinolone
15
16
17
18
19
• 20
• 21
• 22
• 23
• 24

Lock
s
Suspend
0
End Block
Item: 20 of 99 ~ 1 • Mark <:] (:>- Jill ~· ~J
QID: 22073 ..1 Previous Next Lab'V!I!ues Notes Calculator

2
3
The correct answer is C. 780/o chose this.
4
Pilocarpine is a muscarinic agonist that produces rapid m iosis and contraction of the ciliary muscles. This can be used in the
5 acute treatment of closed- or open-angle glaucoma to open the trabecular meshwork around Schlemm's canal, increasing
6 drainage of aqueous humor and decreasing intraocular pr essure (lOP) . The gonioscopic exam, wh ich measures the angle
7
between the cornea and iris, shows that this patient's glaucoma is closed-angle glaucoma . Acute closed-angle glaucoma is
described as a sudden rise in l OP due to narrowing of the anterior chamber angle. Glaucoma, in general, is defined as an
8 optic neuropathy involving characteristic atrophy of the optic nerve head . The exact etiology of the optic neuropathy is not
9 known, although established risk factors include l OP, family history, race, age older than 60 years, and hyperopia . There is at
10 present no consistent threshold for lOP that has been determined to lead to glaucoma . The treatment options include a-
agonists, ~-blockers, carbonic anhydrase inhibitors, miot'ic agents, prostaglandin analogs, and surgical correction . Prognosis is
generally good for patients; w ith careful follow-up care and compliance w ith therapy, the vast majority reta ins useful vision
throughout its lifetime.
13 Pilocarpine Glaucoma Hyperopia Miosis Trabecular meshwork Muscarinic agonist Cornea Schlemm's canal Aqueous humour Intraocular pressure Optic nerve

14 Anterior chamber of eyeball Carbonic anhydrase inhibitor Iris (anatomy) Prostaglandin Ciliary muscle Agonist Muscarinic acetylcholine receptor Ootic disc Peripheral neuropathy

15 Prostaglandin analogue Etiology Carbonic anhydrase Atrophy Optic neuropathy

16
A is not correct. 9% chose this.
17
Atropine is a derivative of belladonna that acts as an antficholinergic agent and is used to produce long- lasting mydriasis for
18 diagnostic purposes. It is contraindicated in patients with increased intraocula r pressure .
19 Anticholinergic Mydriasis Atropine Intraocular pressure Atropa belladonna Derivative (chemistry) Contraindication

20 B is not correct. SO/o chose this.


• 21 Methylprednisolone sodium succinate (Solu-Medrol), as well as mannitol, can be administered intravenously in an attempt to
• 22 decrease intraocular pressu re (l OP) in cases of severe sulfonamide-induced angle closure (l OP >45 mm Hg) . However,
• 23
intravenous methylprednisolone sodium succinate is not indicated for this patient with closed-angle glaucoma .
Glaucoma Methylprednisolone Mannitol Intraocular pressure Intravenous therapy Sodium
• 24

Lock
s
Suspend
0
End Block
Item: 20 of 99 ~ 1 • Mark <:] (:>- Jill ~· ~J
QID: 22073 ..1 Previous Next Lab'V!I!ues Notes Calculator

1 • Anticholinergic Mydriasis Atropine Intraocular pressure Atropa belladonna Derivative (chemistry) Contraindication

2
B is not correct. SO/o chose this.
3
Methylprednisolone sodium succinate (Solu-Medrol), as well as mannitol, can be administered intravenously in an attempt to
4 decrease intraocular pressure (lOP) in cases of severe Sllllfonamide-induced angle closu re (lOP >45 mm Hg) . However,
5 intravenous methylprednisolone sodium succinate is not indicated for this patient with closed-angle glaucoma .
Glaucoma ~1ethylprednisolone Mannitol Intraocular pressure Intravenous therapy Sodium
6
7 D is not correct. 6% chose this.
8 Scopolamine acts as a competitive antagonist at muscarinic acetylcholine receptors; it is used in ophthalmology to induce
9
cycloplegia and mydriasis so certain diagnostic procedures may be performed . However, it is contraindicated in patients with
increased intraocu lar pressu re because of its potential to induce acute angle-closure glaucoma .
10 Cycloplegia Hyoscine hydrobromide Mydriasis Glaucoma Intraocular pressure Acetylcholine Ophthalmology Competitive antagonist Receptor antagonist

~
Muscarinic acetylcholine receptor Contraindication Acetylcholine receptor Receptor (biochemistry)
2
E is not correct. 20/o chose this.
13
Triamcinolone is a steroid that has been shown to increase intraocular pressure (lOP) in normal eyes (steroid-induced
14
glaucoma) . One study showed that following intravitreal injection of triamcinolone, more than 50% of nonglaucomatous eyes
15 w il l have an increase in lOP, wh ich can persist for as long as 6 months after the injection . Steroids should be avoided in
16 glaucomatous eyes.
Glaucoma Intraocular pressure Triamcinolone Vitreous body Steroid lntravitreal administration Anabolic steroid
17
18
19 Bottom line:
20 Acute-angle-closure glaucoma is described as a multifactorial optic neuropathy that is acute with a characteristic increase in
• 21 lOP. Treatment options include a-agonists, ~-blockers, carbonic anhydrase inh ibitors, miotic agents, prostaglandin analogs,
• 22 and surgical correction .
Glaucoma Prostaglandin Carbonic anhydrase Carbonic anhydrase inhibitor ~1iosis Peripheral neuropathy Prostaglandin analogue Optic neuropathy
• 23
• 24

Lock
s
Suspend
0
End Block
Item: 20 of 99 ~ 1 • Mark <:] (:>- Jill ~· ~J
QID: 22073

1
..1
. .. . . . ..
Previous
. Next
. - Lab'V!I!ues
.. . Notes
.
Calculator
.. . .. . . . . .. . . . ... . ... .
intravenous methylprednisolone sodium succinate is not indicated for th is patient with closed-angle glaucoma .
2 Glaucoma Methylprednisolone Mannitol Intraocular pressure Intravenous therapy Sodium

3
D is not correct. 6% chose this.
4
Scopolamine acts as a competitive antagonist at muscarinic acetylcholine receptors; it is used in ophthalmology to induce
5 cycloplegia and mydriasis so certain diagnostic procedures may be performed . However, it is contraindicated in patients with
6 increased intraocu lar pressure because of its potential to induce acute ang le-closure glaucoma .
Cycloplegia Hyoscine hydrobromide Mydriasis Glaucoma Intraocular pressure Acetylcholine Ophthalmology Competitive antagonist Receptor antagonist
7
Muscarinic acetylcholine receptor Contraindication Acetylcholine receptor Receptor (biochemistry)
8
9 E is not correct. 20/o chose this.
10 Triamcinolone is a steroid that has been shown to increase intraocular pressure (IOP) in normal eyes (steroid-induced
glaucoma) . One study showed that following intravitreal injection of triamcinolone, more than 50% of nonglaucomatous eyes
w ill have an increase in IOP, wh ich can persist for as long as 6 months after the injection . Steroids should be avoided in
glaucomatous eyes.
13 Glaucoma Intraocular pressure Triamcinolone Vitreous body Steroid Intravitreal administration Anabolic steroid
14

15
Bottom Line:
16
17
Acute-angle-closure glaucoma is described as a multifactorial optic neuropathy that is acute with a characteristic increase in
IOP. Treatment options include a-agon ists, ~-blockers, carbon ic anhydrase inh ibitors, miotic agents, prostaglandin analogs,
18 and surgical correction .
19 Glaucoma Prostaglandin Carbonic anhydrase Carbonic anhydrase inhibitor Miosis Peripheral neuropathy Prostaglandin analogue Optic neuropathy

20
• 21
• 22 References:
FA Step 2 CK 9th ed p 293
• 23
FA Step 2 CK 8th ed p 278
• 24

Lock
s
Suspend
0
End Block
Item: 21 of 99 ~ 1 • Mark <:] (:>- Jill ~· ~J
QID: 24440 ..1 Previous Next Lab'V!I!ues Notes Calculator

1 • •
A 70-year-old right-handed man presents to the emerg ency department with right-sided weakness of the extremities ~~ A I
2
lasting 5 hours. His temperature is 37.5°C {99 .5°F), blood pressure is 175/1 10 mm Hg, pulse is 100/min, and respiratory
3 rate is 16/min. Physical examination reveals word-finding difficulties, poor comprehension, and right-arm weakness . There
4 is right-sided hemisensory loss . There are no cranial nerv·e deficits, and left-sided strength and sensation are intact.
5
6 Which of the following is the most likely diagnosis?
7
8 A. Basilar artery occlusion
9 B. Carotid artery dissection
10
C. Lacunar infarction
D. Left middle cerebral artery occlusion
13 E. Posterior cerebral artery occlusion
14
F. Right middle cerebral artery occlusion
15
16
17
18
19
20
• 21
• 22
• 23
• 24

Lock
s
Suspend
0
End Block
Item: 21 of 99 ~ 1 • Mark <:] (:>- Jill ~· ~J
QID: 24440 ..1 Previous Next Lab'V!I!ues Notes Calculator

2
3
4 The correct answer is D. 8 30/o chose this.
5 This patient is presenting with an acute stroke. The clinica l presentation is most consistent with middle cerebral artery
infarction in the dominant hemisphere, wh ich is nearly always the left in a right-handed patient. Contralateral hemiparesis
6
and sensory loss are typical, and the face and arms are usually affected more severely than the legs. Aphasia can be present
7 if the dominant hemisphere is involved . The aphasia can be expressive, receptive, or both.
8 Hemiparesis Aphasia Middle cerebral artery Contralateral Stroke Anatomical terms of location Cerebral arteries Lateralization of brain function

9
A is not correct. 2 % chose this.
10 Basilar artery occlusion can cause severe quadriplegia, coma, or " locked- in " syndrome . In the latter situation, the patient has

~
complete muscu lar paralysis except that upward gaze is preserved .
2 Basilar artery Tetraplegia Locked-in syndrome Paralysis Coma

13 B is not correct. 20/o chose this.


14 Carotid artery dissection is typically associated with neck pain, headache, and, variably, Horner's syndrome and dysgeusia
15 (change in taste) .
Carotid artery dissection Dysgeusia Horner's syndrome Headache Neck pain Carotid artery Common carotid artery
16
17 C i s not correct. 80/o chose this.
18 Lacunar infarcts cause pure motor or pure sensory deficits. They can also cause dysarthria-clumsy hand syndrome or ataxic
19
hemiparesis.
Hemiparesis Lacunar stroke Ataxia Infarction Stroke
20
21 E i s not correct. 2 0/o c hose th is.
• 22 Posterior cerebral artery infa rction predominantly presents with a contralateral homonymous hemianopia . It can present with
memory deficits such as dyslexia or alexia . Patients might not be aware of their deficits until formally assessed .
• 23 Posterior cerebral artery Homonymous hemianopsia Dyslexia Contralateral Hemianopsia Anatomical terms of location Infarction
• 24 y

Lock
s
Suspend
0
End Block
Item: 21 of 99 ~ 1 • Mark <:] (:>- Jill ~· ~J
QID: 24440 ..1 Previous Next Lab'V!I!ues Notes Calculator

1 • Carotid artery dissection is typically associated with neck pain, headache, and, variably, Horner's syndrome and dysgeusia
2
(change in taste) .
Carotid artery dissection Dysgeusia Horner's syndrome Headache Neck pain Carotid artery Common carotid artery
3
4 C is not correct. SOfo chose this.
5
Lacuna r infarcts cause pure motor or pur e sensory deficit s. They can also cause dysarthria-clumsy hand syndrome or ataxic
hemipar esis.
6 Hemiparesis lacunar stroke Ataxia Infarction Stroke
7
E is not correct. 20/o chose this.
8
Posteri or cerebral artery infarction predominantly pr esents with a contralateral homonymous hemianopia . It can pr esent with
9
memory deficits such as dyslexia or alexia . Patients might not be aware of their deficits until formally assessed .
10 Posterior cerebral artery Homonymous hemianopsia Dyslexia Contralateral Hemianopsia Anatomical terms of location Infarction

F is not correct. 30/o chose this.


Right middle cer ebral artery ( MCA) stroke would present with hemiparesis and sensory loss on the left side of the body,
13 Contralateral neglect may be pr esent if the nondominant hemispher e is affected by an MCA stroke . Patients with MCA
14 infarctions might also demonstrate homonymous hemia nopsia and a gaze prefer ence toward the side of the lesion .
Homonymous hemianopsia Hemiparesis Middle cerebral artery Stroke Contralateral Hemianopsia lesion Anatomical terms of location Cerebral arteries
15
16
17 Bottom line:
18 The clinical presentation can help in identifying the vascular territory affected by a stroke.
19 Stroke Vascular plant Blood vessel

20
21
• 22 References:
FA Step 2 CK 9th ed p 258
• 23
FA Step 2 CK 8th ed pp 243-247
• 24

Lock
s
Suspend
0
End Block
Item: 22 of 99 ~ 1 • Mark <:] (:>- Jill ~· ~J
QID: 22201 ..1 Previous Next Lab'V!I!ues Notes Calculator

1 • •
A 50-year-old man suffers from headaches. They occur in periodic cycles, only severa l months out of the year, with 1-2 ~~ A I
2 headache attacks per day. The headaches are severe; located behind the righ t eye ; and are associated w ith a r ed, tearing,
3 right eye and nasal stuffiness on the same side .
4

5 Which of the following is the best first step in management?


6
7 A. ~-B l ockers
8
B. Anticonvulsants
9
10
C. Avoiding factors that precipitate these headaches ( eg, alcohol or physical stress)

D. Nonsteroidal anti-inflammatory drugs

E. Tricyclic antidepressants
13
14

15
16
17
18
19
20
21
• 22
• 23
• 24

Lock
s
Suspend
0
End Block
Item: 22 of 99 ~ 1 • Mark <:] (:>- Jill ~· ~J
QID: 22201 ..1 Previous Next Lab'V!I!ues Notes Calculator

2 The correct answer i s C. 510/o chose this .


3 This patient is suffering from cluster headaches. Treatment is primarily focused on avoiding factors that exacerbate these
headaches, particularly alcohol and/or strenuous physica l exercise . This should be the first step in management before
4
pharmacotherapy is initiated . If therapy is indicated, patuents will typically require both prophylactic and abortive therapy for
5 relief of their symptoms . Abortive therapy includes oxygen administration and the use of triptans . Prophylactic therapy
6 includes the use of prednisone, lithium, and calcium channel blockers. Verapamil is the Ca channel blocker most often used
7
but nimod ipine and diltiazem have also been reported to be effective.
Nimodipine Verapamil Diltiazem Prednisone Calcium channel blocker Cluster headache Pharmacotherapy Triptan Calcium channel Alcohol Physical exercise Calcium
8
Preventive healthcare Oxygen Lithium
9
10 A i s not correct. 1 20/o chose this .
~ - Blockers are good prophylactic medications for migraine headaches. Their role in cluster headaches is less well defined .
Migraine Cluster headache Preventive healthcare

13 B is not correct. 110/o chose th is.


14 Anticonvulsants such as valproate are good prophylactic medications against migraine headaches, less so for cluster
headaches.
15
Migraine Valproate Cluster headache Anticonvulsant Preventive healthcare
16
D i s not correct. 160/o chose this.
17
Nonsteroidal anti-inflammatory drugs are helpful in stopping a headache once it has begun . However, it has a greater
18
abortive role in m igraine and tension-type headaches .
19 Migraine Anti-inflammatory Nonsteroidal anti-inflammatory drug Headache
20
E i s not corr ect. 10% chose this.
21
Tricyclic antidepressants are good prophylactic medications against migraine headaches and have been used in tension
22 headaches, but their role in cluster headaches is less well! defined .
• 23 Migraine Tension headache Tricyclic antidepressant Cluster headache Antidepressant Preventive healthcare Tricyclic

• 24

Lock
s
Suspend
0
End Block
Item: 22 of 99 ~ 1 • Mark <:] (:>- Jill ~· ~J
QID: 22201 ..1 Previous Next Lab'V!I!ues Notes Calculator
• •
1
~ - B l ockers are good prophylactic medications for mig raine headaches. Their role in cluster headaches is less well defined .
2
Migraine Cluster headache Preventive healthcare
3
4 B is not correct. 110/o chose th is.
5
Anticonvulsants such as valproate are good prophylactic medications against m igraine headaches, less so for cluster
headaches.
6 Migraine Valproate Cluster headache Anticonvulsant Preventive healthcare
7
D is not correct. 160/o chose this.
8
Nonsteroidal anti -inflammatory drugs are helpfu l in stopping a headache once it has begun . However, it has a greater
9
abortive role in m igraine and tension-type headaches .
10 Migraine Anti-inflammatory Nonsteroidal anti-inflammatory drug Headache

E is not correct. 10% chose this.


Tricyclic antidepressants are good prophylactic med ications against migraine headaches and have been used in tension
13 headaches, but their role in cluster headaches is less welll defined .
14 Migraine Tension headache Tricyclic antidepressant Cluster headache Antidepressant Preventive healthcare Tricyclic

15
16
Bottom line :
17
Treatment of cluster headaches is primarily focused on avoiding factor s t hat exacerbate these headaches, particu larly
18
alcohol and/ or strenuous physical exer cise.
19 Cluster headache Alcohol Alcoholic beverage Physical exercise

20
21
22
References:
FA Step 2 CK 9th ed pp 267-268
• 23
FA Step 2 CK 8th ed p 253
• 24

Lock
s
Suspend
0
End Block
Item: 23 of 99 ~ 1 • Mark <:] (:>- Jill ~· ~J
QID: 24184 ..1 Previous Next Lab'V!I!ues Notes Calculator

1 • •
A 39-year-old man presents to his physician for follow-up for Crohn disease. He reports good symptom control w ith no ~~AI
2 abdominal pain since his small bowel resection 3 years ago . He den ied diarrhea, constipation, or hematochezia. He does
3 note, however, increasing fatigue, tingling sensations in his fingers, and increasing forgetfulness. Neurological examination
4 is notable for spasticity in the lower extremities and dista I symmetric sensory loss. Resu lts of a peripheral blood smear are
shown in the image.
5
6
7
8
9
10

13
14

15
16
17
18
19
20
21
22
What is the most likely underlying etiology of the neurologic symptoms and abnormal smear?
• 23

• 24

Lock
s
Suspend
0
End Block
Item: 23 of 99 ~ 1 • Mark <:] (:>- Jill ~· ~J
QID: 24184 ..1 Previous Next Lab'V!I!ues Notes Calculator

2
3
4

5
6
7
8
9
10

13
14
What is the most li kely underlying etiology of the neurolog ic symptoms and abnormal smear"
15
16
A. Bone marrow failing to produce adequate RBCs
17
18 B. Decreased absorption of vitamin B12
19 C. Decreased dietary intake of folate
20
D. Decreased intake of thiamine
21
22 E. Dumping syndrome
• 23
• 24

Lock
s
Suspend
0
End Block
Item: 23 of 99 ~ 1 • Mark <:] (:>- Jill ~· ~J
QID: 24184 ..1 Previous Next Lab'V!I!ues Notes Calculator

2 The correct answer is B. 910/o chose th is.


3 The clinical picture is consistent with combined system disease caused by vitamin 8 1 2 deficiency. Vitamin 8 1 2 deficiency may
4
occur following ileal resection due to decreased absorption of vitamin 8 1 2. Symptoms include megaloblastic anemia, stocking-
glove sensory neuropathy, leg stiffness, paraplegia, dementia, and bladder and bowel dysfunction . Hypersegmented
5 neutrophils (as pictured above) are characteristic in vitamin 8 12 deficiency. The posterior and lateral columns of the spinal
6 cord degenerate. It is treated with high doses of vitam in 8 1 2 supplementation .
Megaloblastic anemia Vitamin B12 Anemia B vitamins Dementia Neutrophil Spinal cord Ileum Vitamin Peripheral neuropathy Urinary bladder Gastrointestinal tract Malabsorption
7
8 A i s not correct. 2% chose this.
9 Aplastic anemia may cause marrow failure result ing in profound anemia, but w ill not affect the spinal cord . Aplastic anem ia
10 has normal R8C morphology on a peripheral blood smear. On bone marrow biopsy the marrow appears hypocellular with fatty
replacement.
Aplastic anemia Bone marrow examination Blood film Biopsy Anemia Bone marrow Spinal cord Morphology (biology) Bone Red blood cell

13
C i s not correct. 3 % chose this.
14
Folate deficiency may cause megaloblastic anemia and gi ve a sim ilar appearing peripheral smear. However, no neurologic
deficits are associated with folate deficiency.
15 Megaloblastic anemia Folate deficiency Folic acid Anemia Neurology
16
D is not correct. 2% chose this.
17
Thiamine deficiency is commonly seen in alcoholics, starving patients, dialysis patients, and in persons with AIDS. It presents
18 as Wernicke encephalopathy (ataxia, mental status change, and opthalmoplegia). The absorption of thiamine does not
19 depend on the small intestine.
Thiamine Small intestine Wernicke's encephalopathy Dialysis Encephalopathy Gastrointestinal tract HIV/AIDS Alcoholism
20
21 E i s not correct. 2 0/o c hose th is.
22 Patients with dumping syndrome have prominent symptoms of nausea, vomit ing, early satiety, flushing, and palpitations .
23 Over t ime the symptoms may lead to malnutrition, but the above clinical scenario is more consistent with decreased vitam in
• 24 812 absorption at the level of the ileum .

Lock
s
Suspend
0
End Block
Item: 23 of 99 ~ 1 • Mark <:] (:>- Jill ~· ~J
QID: 24184 ..1 Previous Next Lab'V!I!ues Notes Calculator

1 • Aplastic anemia Bone marrow examination Blood film Biopsy Anemia Bone marrow Spinal cord Morphology (biology) Bone Red blood cell

2 C is not correct. 3% chos e this .


3 Folate deficiency may cause megaloblastic anemia and gi ve a similar appearing peripheral smear. However, no neurologic
4 deficits are associated with folate deficiency.
Megaloblastic anemia Folate deficiency Folic acid Anemia Neurology
5
6 D is not correct. 2% chos e this .
7 Thiamine deficiency is commonly seen in alcoholics, starving patients, dialysis patients, and in persons w ith AIDS. It presents
8 as Wernicke encephalopathy (ataxia, mental status change, and opthalmoplegia). The absorption of thiamine does not
depend on the small intestine.
9 Thiamine Small intestine Wernicke's encephalopathy Dialysis Encephalopathy Gastrointestinal tract HIV/AIDS Alcoholism
10
E is n ot correct. 2 0/o c hose th is.
Patients with dumping syndrome have prominent symptoms of nausea, vomiting, early satiety, flushing, and palpitations .
Over time the symptoms may lead to malnutrition, but the above clinical scenario is more consistent with decreased vitam in
13 B12 absorption at the level of the ileum .
14 Ileum Vitamin B12 Gastric dumping syndrome Palpitations Nausea B vitamins Vomiting Malnutrition Hunger (motivational state) Vitamin Flushing (physiology)

15
16
Bottom line :
17
Vitamin B12 deficiency may occur following il eal resection due to decreased absorption . Symptoms include megaloblastic
18 anemia, stocking-glove sensory neuropathy, leg stiffness, paraplegia, dementia, and bladder and bowel dysfunction .
19 Megaloblastic anemia Vitamin B12 Anemia B vitamins Vitamin Dementia Ileum Urinary bladder Peripheral neuropathy Malabsorption Gastrointestinal tract

20
21
22
References:
FA Step 2 CK 9th ed pp 175-176
23
FA Step 2 CK 8th ed pp 276; 486
• 24

Lock
s
Suspend
0
End Block
Item: 24 of 99 ~ 1 • Mark <:] (:>- Jill ~· ~J
QID: 22204 ..1 Previous Next Lab'V!I!ues Notes Calculator

2 A 50-year-old woman with a newly diagnosed brain tumor presents with aphasia . Her naming ability, word repetition
ability, and language comprehension are preserved . However, her fluency is impa ire d. She has no other symptoms, and
3
the remainder of her neu rologic examination is unremarkable.
4

5
From what kind of aphasia is she suffering?
6
7
A. Broca's aphasia
8
9
B. Conduction aphasia
10 C. Transcortical motor aphasia
~ D. Transcortical sensory aphasia
~ E. Wern icke's aphasia
13
14
15
16
17
18
19
20
21
22
23
• 24

Lock
s
Suspend
0
End Block
Item: 24 of 99 ~ 1 • Mark <:] (:>- Jill ~· ~J
QID: 22204 ..1 Previous Next Lab'V!I!ues Notes Calculator

2 The correct answer i s C. 3 6 0/o chose this .


3 The patient suffers from a transcortical motor aphasia, ch aracterized by a nonfluent aphasia in which repetit ion is preserved .
4 Aphasia Expressive aphasia Transcortical motor aphasia

5 A i s not correct. 3 7 0/o chose this .


6 Broca's aphasia is characterized by preserved comprehension, but problems with language production, nam ing, and
7 repetition. These patients speak nonfluently, ha lt ingly, and w ithout much intonation. They are aware of their problem and
8
may often become depressed . Interesting ly, some patients can ma intain fluency when singing . Broca's area includes the
posterior part of the inferior frontal gyrus and a surrounding rim of prefronta l heteromodal cortex .
9 Broca's area Inferior frontal gyrus Aphasia Expressive aphasia Intonation (linguistics) Cortex (anatomy) Prefrontal cortex Primary progressive aphasia
10
B is not correct. 120/o chose t h is.
~ Conduction aphasia is characterized by problems w ith repeating what is sa id but preserved fluency and comprehension .
~ Conduction aphasia Aphasia
13
D i s not correct. 6% chose this.
14
Transcortical sensory aphasia is characterized by impaired comprehension of language but preserved repetition and fluency.
15 Transcortical sensory aphasia Aphasia Receptive aphasia
16
E i s n ot cor rect. 90/o c hose th is.
17
Wern icke's aphasia represents a problem with language comprehension, which affects both language output and input.
18
Language output is paraphasic with circumlocution . Repetit ion, naming, reading, and writing are also impa ired although
19 certain axial tasks, such as opening one's eyes, may be preserved . Wernicke's area includes the posterior third of the superior
20 temporal gyrus, as well as a surround ing rim of inferior parietal and midtemporal cortex .
Wernicke's area Superior temporal gyrus Aphasia Receptive aphasia Sentence processing Paraphasia Parietal lobe Cortex (anatomy) Cerebral cortex Circumlocution
21
22
23 Bottom line :
24

Lock
s
Suspend
0
End Block
Item: 24 of 99 ~ 1 • Mark <:] (:>- Jill ~· ~J
QID: 22204 ..1 Previous Next Lab'V!I!ues Notes Calculator

• repet1t1on . These pat1ents speak nonfluently, haltmgly, and Without much mtonat1on . They are awar e of the1r problem and
2 may often become depressed . I nterestingly, some patients can maintain fluency w hen singing . Broca's area includes the
3 posterior part of the inferi or f rontal gyrus and a surr ounding rim of prefrontal heteromodal cortex .
Broca's area Inferiorfrontal gyrus Aphasia Expressive aphasia Intonation {linguistics) Cortex (anatomy) Prefrontal cortex Primary progressive aphasia
4

5 B is not correct. 120/o chose this.


6 Conduction aphasia is characterized by problems w ith r epeating what is said but preserved fluency and compr ehension .
Conduction aphasia Aphasia
7
8 D is not correct. 60/o chose this.
9 Transcort ical sensory aphasia is characterized by impaired comprehension of language but preserved repetit ion and fluency.
Transcortical sensory aphasia Aphasia Receptive aphasia
10

~ E is not correct. 90/o chose this.


~ Wernicke's aphasia r epresents a problem with language comprehension, which affects both language output and input.
13
Language output is paraphasic with circumlocution , Repetition, naming, read ing, and writ ing are also impaired although
certain axial tasks, such as open ing one's eyes, may be preserved. Wernicke's area includes the posterior third of the superior
14
temporal gyrus, as well as a surrounding rim of inferior parietal and midtemporal cortex .
15 Wernicke's area Superior temporal gyrus Aphasia Receptive aphasia Sentence processing Paraphasia Parietal lobe Cortex (anatomy) Cerebral cortex Circumlocution

16
17
Bottom line:
18
Transcort ical motor aphasia is characterized by a nonfluent aphasia in which r epetition is preserved .
19 Transcortical motor aphasia Aphasia Expressive aphasia Transcortical sensory aphasia
20
21
22 References:
23 FA Step 2 CK 9th ed pp 288-289
FA Step 2 CK 8th ed p 274
24

Lock
s
Suspend
0
End Block
Item: 25 of 99 ~ 1 • Mark <:] (:>- Jill ~· ~J
QID: 23267 ..1 Previous Next Lab'V!I!ues Notes Calculator

3 A 67-year-old fema le secretary presents to the clin ic complaining of several years of numbness in the palmar aspect of her ~~AI
left hand that rad iates to her forearm . She cla ims that the pain occasiona lly wakes her up at night and that shaking her
4
arm often al leviates her symptoms . She is mildly obese and qu it smoking 3 years ago after a 30-pack-year history. On
5 exam ination her blood pressure is 130/90 mm Hg . There is no atrophy of the thenar or hypothenar muscles.
6
7 Which of the following would confirm the diagnosis?
8
9 A. Cardiac enzymes
10
B. Fasting glucose level

~
13
C. MRI of the head
D. Nerve biopsy
14
E. Nerve conduction studies
15
16
17
18
19
20
21
22
23
24
• 25

Lock
s
Suspend
0
End Block
Item: 25 of 99 ~ 1 • Mark <:] (:>- Jill ~· ~J
QID: 23267 ..1 Previous Next Lab'V!I!ues Notes Calculator

3
4 The correct answer is E. 84% chose this.
5 The patient has signs and symptoms of carpa l tunnel syndrome, wh ich is entrapment of the med ian nerve characterized by
pain and paresthesias in the ra dial side of the palm that may rad iate pro ximally down the forearm . Carpal tunnel syndrome
6
often presents as a work- related illness in persons whose jobs require prolonged use of the hand and wrist . The diag nosis can
7 be made clinica lly, but nerve conduction studies are a frequently used method to confirm the diagnosis.
8 Carpal tunnel syndrome Median nerve Nerve conduction study Paresthesia Carpal bones Carpal tunnel Forearm

9
A is not correct. 1% chose this.
10 Myocardial infarction (MI) can present with a variety of symptoms, the most common of which is acute, localized

~
squeezing/burning chest pain that may radiate to the arm, neck, jaw, or abdomen . This pain typically lasts from 30 m inutes
2 to several hours and may be accompan ied by shortness of breath, diaphore sis, anxiety, or a feeling of impending doom .
Several characteristics imply that this patient's pain is not due to Ml: it is long standing, localized only to the distal portion of
13
her extremity, and responds to shaking of the arm .
14 Myocardial infarction Perspiration Dyspnea Chest pain Anxiety Anatomical terms of location Abdomen Jaw Infarction
15
B is not correct. 90/o chose this.
16
Type 2 diabetes mellitus (DM) can result in peripheral neuropathy that classically involves the upper and lower extremities in
17 a "stocking glove" distribution . However, this woman's symptoms are confined to her left hand and fore arm and are transient,
18 whereas the loss of sensation associated with type 2 DM is symmetric, slowly progressive, and permanent.
Diabetes mellitus Diabetes mellitus type 2 Peripheral neuropathy
19
20 C is not correct. 3% chose this.
21 The patient is at an increased risk for stroke given her age and smoking history, but no evidence noted here suggests stroke
22 or vascular disease . Given that her symptoms are contained to a sing le nerve distribution, a peripheral cause is more likely.
Stroke Vascular disease Blood vessel
23
24 D is not correct. 3% chose this.
25 y
Nerve biopsy is not necessary to diagnose carpal tunnel syndrome . Nerve biopsy is often used in the diag nosis of

Lock
s
Suspend
0
End Block
Item: 25 of 99 ~ 1 • Mark <:] (:>- Jill ~· ~J
QID: 23267 ..1
. . ... ~ . .. .
Previous Next
. .. .Lab'V!I!ues
..
'
Notes
. . ..
Calculator
. . .. . .. . . . . .. .
whereas the loss of sensation associated with type 2 DM is symmetric, slowly progressive, and permanent.
3
Diabetes mellitus Diabetes mellitus type 2 Peripheral neuropathy
4
5
C i s not corr ect. 30/o ch ose this.
6
The patient is at an increased risk for stroke given her age and smoking history, but no evidence noted here suggests stroke
or vascular disease . Given that her symptoms are contained to a single nerve distribution, a peripheral cause is more likely.
7 Stroke Vascular disease Blood vessel
8
D is not correct. 3% chose this.
9
Nerve biopsy is not necessary to diagnose carpal tunnel syndrome . Nerve biopsy is often used in the diagnosis of
10
mononeurit is multiplex, a vasculitic neuropathy. Mononeuritis multiplex is a migratory isolated neuropathy that typically

~
involves one nerve at a time and is seen only in diabetes, trauma, HIV infection, or vasculitides such as polyarterit is nodosa .
This patient has had years of symptoms in one nerve but does not report other symptoms of diabetes, trauma, or vasculitis.
The first step in diagnosis would be rheumatologic titers such as an anti-nuclear antibody level.
13
Mononeuritis multiplex Carpal tunnel syndrome Polyarteritis nodosa Anti-nuclear antibody Vasculitis Peripheral neuropathy H!V Diabetes mellitus Biopsy Rheumatology
14
Mononeuropathy Antibody
15
16
17 Bottom Li ne:
18 Carpal tunnel syndrome (entrapment of the median nerve) is characterized by pain and paresthesias in the radial side of the
19 palm that may rad iate proximally down the forearm . Although carpa l tunnel syndrome is a clin ical diagnosis, nerve
conduction studies and electromyography can be used to confirm the diagnosis.
20 Carpal tunnel syndrome Median nerve Electromyography Nerve conduction study Paresthesia Carpal tunnel Carpal bones Forearm
21
22
23 References:
24 FA Step 2 CK 9th ed pp 239-240
FA Step 2 CK 8th ed pp 221-222
25

Lock
s
Suspend
0
End Block
Item: 26 of 99 ~ 1 • Mark <:] (:>- Jill ~· ~J
QID: 22076 ..1 Previous Next Lab'V!I!ues Notes Calculator

4 A 36-year-old man with sarcoidosis is seen in the ophthalmology clinic for follow-up for chronic anterior uveitis. He has ~~AI
been taking steroid eye drops for treatment of the anterior uveitis for several months. His visua l acuity is 20/20 in both
5
eyes, and intraocular pressure is 32 mm Hg in the right eye and 34 mm Hg in the left eye (normal range : 10-21 mm Hg) .
6 Gonioscopic evaluation shows open anterior-chamber ang le and no peripheral anterior synechiae. Funduscopic exam shows
7 cupping of the optic nerve .
8
9 What is the most appropriate next step in management?
10

~
A. Taper off steroids
B. Trabeculectomy
13
C. Treat w ith atropine
14

15 D. Treat with pilocarpine


16 E. Treat w ith timolol
17
18
19
20
21
22
23
24
25
• 26

Lock
s
Suspend
0
End Block
Item: 26 of 99 ~ 1 • Mark <:] (:>- Jill ~· ~J
QID: 22076 ..1 Previous Next Lab'V!I!ues Notes Calculator

5
6
The correct answer is A. 360/o chose this.
The patient has secondary open-angle glaucoma {OAG) due to topical steroids. Ster oid-induced glaucoma is a form of OAG
7
that is associated with topical steroid use { it can also rarely occur w ith systemic steroid use) . Treatment requires the
8 discontinuation of corticosteroids. I f intraocular pressures {lOPs) do not normalize in several days, patients should be treated
9 medically with agents typically used in the treatment of OAG . Th is patient should be tapered off the steroids because it is the
10 likely etiology for the increase in intraocular pre ssure.
Glaucoma Topical steroid Intraocular pressure Steroid Corticosteroid Etiology Anabolic steroid

~
13
B is not correct. 110/o chose this.
Although trabeculectomy can provide curative treatment of glaucoma, th is patient's open-ang le glaucoma is secondary to
steroids. Trabeculectomy is not indicated in this situation . Underlying disease that increases intraocular pre ssure can cause
14
secondary glaucoma, including uveitis, old trauma, vasoproliferati ve retinopathy, and steroid therapy.
15 Glaucoma Trabeculectomy Uveitis Intraocular pressure Retinopathy Diabetic retinopathy Steroid Anabolic steroid
16
C is not correct. 6% chose this.
17
Atropine is an anti muscarinic agent that results in mydriasis and is not an acceptable form of treatment for glaucoma .
18 Muscarinic antagonist Mydriasis Atropine Glaucoma
19
D is not correct. 200/o chose this.
20
Pilocarpine is a muscarinic agonist that produces rapid m iosis and contraction of the ciliary muscles. This can be used in the
21 acute treatment of open-angle glaucoma or closed-angle glaucoma to open the trabecular meshwork around Schlemm's
22 canal, increasing drainage of aqueous humor and decreasing intraocula r pressure {l OP) . However, in the in itial management
23 of this patient, the steroids must be tapered first because it is the likely etiology for the elevated l OP.
Pilocarpine Miosis Trabecular meshwork Glaucoma Muscarinic agonist Schlemm's canal AQueous humour Intraocular pressure Muscarinic acetylcholine receptor Agonist Ciliary muscle
24
Etiology Anabolic steroid
25
26 E is not correct. 27% chose this.

Lock
s
Suspend
0
End Block
Item: 26 of 99 ~ 1 • Mark <:] (:>- Jill ~· ~J
QID: 22076 ..1
- .. . ..
Previous
. . Next
. . Lab'V!I!ues
. . Notes
.. - . .
Calculator
. . . . ... . . . .. . . .. . .
Muscarinic antagonist Mydriasis Atropine Glaucoma
4

5 D is not correct. 200/o chose this.


6 Pilocarpine is a muscarinic agonist t hat produces rapid miosis and contraction of the ciliary muscles. Th is can be used in the
7
acute treatment of open-angle glaucoma or closed-angle glaucoma to open the trabecular meshwork around Schlemm's
canal, increasing drainage of aqueous humor and decrea.sing intraocu lar pressure (lOP) . However, in the in itial management
8 of this patient, the steroids must be tapered first because it is the li kely etiology for the elevated lOP.
9 Pilocarpine Miosis Trabecular meshwork Glaucoma Muscarinic agonist Schlemm's canal AQueous humour Intraocular pressure Muscarinic acetylcholine receptor Agonist Ciliary muscle

10 Etiology Anabolic steroid

~
13
E is not correct. 2 7 % chose this.
This patient has secondary open-angle glaucoma (OAG) w ith visible cupping of the optic nerve, retinal nerve fiber layer
defects, and increased intraocular pressure (l OP) . Timolol is a nonselective ~- antagonist that reduces the production of
14 aqueous humor in the eye. It can be used topically in the treatment of OAG . Treatment w ith timolol may be ind icated if lOP
15
does not normalize on cessation of topical steroid use. However, at this point, the patient must first be tapered off the
steroids because it is the likely etiology for the increased lOP.
16 Timolol Glaucoma Intraocular pressure Optic nerve AQueous humour Nerve fiber Retinal Steroid Etiology
17
18
19
Bottom li ne:
20 Steroid-induced glaucoma is a form of open-angle glaucoma that is associated with topical steroid use . Treatment requires
the discontinuation of corticosteroids.
21 Glaucoma Topical steroid Corticosteroid Steroid
22
23
24 References:
25 FA Step 2 CK 9th ed p 293
FA Step 2 CK 8th ed p 278
26

Lock
s
Suspend
0
End Block
Item: 27 of 99 ~ 1 • Mark <:] (:>- Jill ~· ~J
QID: 22190 ..1 Previous Next Lab'V!I!ues Notes Calculator
.
5 A 58-year-old woman has been suffering from daily headaches for months . They are not associated with aura, nausea, ~~AI
vomiting, or photophobia . They last for 6 hours per day and occur about ha lf the month . Her med ica l history is significant
6
only for arthritis, for which she has been taking high doses of nonsteroidal anti-inflammatory drugs and acetaminophen-
7 based ana lgesics daily for the past several months . She has been gradua lly esca lating her analgesic dose to manage her
8 condition . Her physical examination is within normal limits.
9
10 Which of the following is most likely to be effective in preventing her daily headaches?

~
13
A. Behaviora l modification therapies

B. Prophylact ic treatment with ~-blockers


14
C. Reduce her analgesic use
15
16 D. Treatment with selective serotonin reuptake inh i b ~tors

17 E. Treatment with tricyclic antidepressants


18
19
20
21
22
23
24
25
26
• 27

Lock
s
Suspend
0
End Block
Item: 27 of 99 ~ 1 • Mark <:] (:>- Jill ~· ~J
QID: 22190 ..1 Previous Next Lab'V!I!ues Notes Calculator
.
5
6
7
The correct answer is C. 550/o chose this.
The patient most likely suffers from chronic daily headaches induced by analgesic medication overuse ( eg, acetaminophen),
8
also known as analgesic rebound headaches . They typically last at least 4 hours per day and occur 15 days per month and
9 are seen most commonly in women in their fifties . The fir st step in prevention would be to withd ra w the patient from the
10 offending medication . If there is another underlying headache syndrome, this will then reveal itself. Another analgesic can be
used instead, or the same one can be later reintroduced, but w ith the instruction to consume it less often and in lower doses.

~
13
2
Analgesic Paracetamol Rebound effect Headache Pharmaceutical drug

A is not correct. 130/o chose this.


14 Although behaviora l therapies such as relaxation therapy and biofeedback can be successful in the management of
headaches, it would not be the first-line or sole preventive measure for medication-induced headaches .
15
Biofeedback Autism therapies Behaviour therapy
16
17
B is not correct. 190/o chose this.
~-Blockers have a greater impact for prophylactic treatment of migraine headaches than for headaches due to chronic
18
analgesic use.
19 Analgesic Migraine Preventive healthcare
20
D is not correct. 5% chose this.
21
Selective serotonin reuptake inhibitors are a newer addition to the prophylactic treatment of headaches. Although useful, the
22 first step in management of medication overuse headaches is to control the offending medication rather than start a new drug
23 treatment.
Serotonin Selective serotonin reuptake inhibitor Reuptake Preventive healthcare Pharmaceutical drug
24
25 E is not correct. SO/o chose this.
26 Tricyclic antidepressants such as amitriptyline and nortriptyline are useful prophylactic agents in the long-term treatment and
27 y
prevention of migraines and tension headaches. However, the first step in management of medication overuse headaches is y

Lock
s
Suspend
0
End Block
Item: 27 of 99 ~ 1 • Mark <:] (:>- Jill ~· ~J
QID: 22190 ..1 Previous Next Lab'V!I!ues Notes Calculator
. • headaches, 1t would not be the f1rst-lme or sole prevent1ve measure for med1cat1on-mduced headaches .
5 Biofeedback Autism therapies Behaviour therapy

6
B is not correct. 190/o chose this.
7
~-B l ockers have a greater im pact for prophylactic treatment of migraine headaches than for headaches due to ch ronic
8 analgesic use.
Analgesic Migraine Preventive healthcare
9
10 D is not correct. 5% chose this.

~
Selective seroton in reuptake inhibitors are a newer addition to the prophylactic treatment of headaches. Although usefu l, the
first step in management of med ication overuse headaches is to control the offending medication rather than start a new drug
treatment.
13
Serotonin Selective serotonin reuptake inhibitor Reuptake Preventive healthcare Pharmaceutical drug
14
15
E is not correct. SO/o chose this.

16
Tricyclic antidepressants such as amitriptyline and nortriptyline are useful prophylactic agents in the long-term treatment and
prevention of migraines and tension headaches. However, the f irst step in management of medication overuse headaches is
17 to control the offending med ication, rather than start a new drug .
18 Amitriptyline Nortriptyline Tricyclic antidepressant Tension headache Migraine Preventive healthcare Antidepressant Pharmaceutical drug Prevention of migraines Tricyclic

19
20
Bottom line:
21
The patient most likely suffers from chronic daily headaches induced by ana lgesic medication overuse ( eg, acetam inophen),
22
also known as analgesic rebound headaches . The first step in prevention is to withd raw the offending med ication .
23 Analgesic Paracetamol Rebound effect Phanmaceutical drug

24
25
26 References:
FA Step 2 CK 9th ed p 266
27

Lock
s
Suspend
0
End Block
Item: 28 of 99 ~ 1 • Mark <:] (:>- Jill ~· ~J
QID: 23280 ..1 Previous Next Lab'V!I!ues Notes Calculator

6 An 85-year-old wh ite man w ith hypertension and cerebrovascular disease presents to his physician's office for a routine
check-up. Review of systems is positive only for periodic "ringing in the ears ." Physical examination reveals left facial
7
paralysis and poor hearing on the left side .
8
9
A stroke in which vascular territory is most likely?
10

~
13
A. Anterior cerebral artery

B. Anterior inferior cerebellar artery


14 C. Middle cerebral artery
15
D. Posterior cerebral artery
16
17
E. Posterior inferior cerebellar artery
18
19
20
21
22
23
24
25
26
27
• 28

Lock
s
Suspend
0
End Block
Item: 28 of 99 ~ 1 • Mark <:] (:>- Jill ~· ~J
QID: 23280 ..1 Previous Next Lab'V!I!ues Notes Calculator

6 The correct answer is B. 380/o chose this.


7 Although there are other possible causes for th is man's symptoms, an ischemic stroke of the anterior inferior cerebellar artery
8 (AICA) is most likely. Strokes in this distribution typically present with gaze pa lsy, deafness, tinnitus, ipsilateral facial
weakness, facial sensory loss, gait and limb ataxia, and Horner syndrome. Unlike strokes in the posterior inferior cerebellar
9
artery, there is no dysphagia or dysarthria. The AICA is a branch of the basilar artery.
10 Anterior inferior cerebellar artery Dysarthria Tinnitus Dysphagia Basilar artery Horner's syndrome Posterior inferior cerebellar artery Ischemic stroke Ataxia Stroke Ischemia

~
Cerebellum Hearing loss Ipsilateral Anatomical terms of location

A is not correct. 6% chose this.


13
Anterior cerebral artery strokes present with leg paresis, amnesia, personality changes, foot drop, gait dysfunction, or
14
cognitive changes.
15 Anterior cerebral artery Amnesia Gait (human) Anatomical terms of location

16
Cis not correct. 210/o chose this.
17
Middle cerebral artery strokes typica lly present w ith aphasia (dominant hemisphere), neglect (nondominant hemisphere)
18 contralatera l hemiparesis, gaze preference, or homonymous hemianopsia.
19 Homonymous hemianopsia Hemiparesis Middle cerebral artery Aphasia Contralateral Anatomical terms of location Hemianopsia Lateralization of brain function Cerebral arteries

20 D is not correct. 130/o chose this.


21 Posterior cerebral artery strokes present with homonymous hemianopsia, memory deficits, or dyslexia/a lexia .
22 Homonymous hemianopsia Posterior cerebral artery Hemianopsia Cerebral arteries

23 E is not correct. 220/o chose this.


24 Posterior inferior cerebellar artery strokes present with sudden onset nausea/vomiting, vertigo, hoarseness, ataxia, ipsilateral
25 palate and tongue weakness, contralateral disturbance of pain/temperature, dysphagia/dysarthria/hiccup, and ipsilateral
26 Horner syndrome.
Posterior inferior cerebellar artery Horner's syndrome Vertigo Ataxia Dysphonia Anatomical terms of location Contralateral Cerebellum Ipsilateral Palate
27
28

Lock
s
Suspend
0
End Block
Item: 28 of 99 ~ 1 • Mark <:] (:>- Jill ~· ~J
QID: 23280 ..1 Previous Next Lab'V!I!ues Notes Calculator

• Anten or cerebral artery strokes present w1th leg par es1s, amnes1a, personal1 ty changes, foot drop, ga1t dysfunction , or
6 cogn it ive changes.
Anterior cerebral artery Amnesia Gait (human) Anatomical terms of location
7
8 C is not correct. 2 10/o chose this .
9 Middle cerebral artery strokes typically present w ith aphasia ( dominant hemisphere), neg lect ( nondom inant hemisphere)
10 contralateral hemipar esis, gaze preference, or homony mous hem ianopsia.
Homonymous hemianopsia Hemiparesis Middle cerebral artery Aphasia Contralateral Anatomical terms of location Hemianopsia Lateralization of brain function Cerebral arteries

~
13
D is not correct. 1 30/o chose t h is.
Posteri or cerebral artery strokes present with homonymous hemianopsia, memory deficits, or dyslexia/ alexia.
Homonymous hemianopsia Posterior cerebral artery Hemianopsia Cerebral arteries
14
15 E is n ot correct. 220/o ch ose this .
16 Posteri or inferior cerebellar artery strokes present wit h sudden onset nausea/vomit ing , vertigo, hoar seness, ataxia, ipsilateral
17
palate and tongue weakness, contralateral disturbance of pain/ temperature, dysphagia/ dysarthria/ hiccup, and ipsilateral
Horner syndrome.
18 Posterior inferior cerebellar artery Horner's syndrome Vertigo Ataxia Dysphonia Anatomical terms of location Contralateral Cerebellum Ipsilateral Palate
19
20
21
Bottom li ne:
22 Ischemic stroke in the anterior inferior cerebellar artery ty pical ly presents wit h gaze palsy, deafness, tinnitus, ipsilateral
facial weakness, facial sensory loss, gait and limb ataxia, and Horner syndrome .
23 Anterior inferior cerebellar artery Tinnitus Horner's syndrome Stroke Ataxia Anatomical terms of location Ischemic stroke Ischemia Cerebellum Hearing loss Ipsilateral
24
25
26 References:
27 FA Step 2 CK 9th ed p 260
FA Step 2 CK 8th ed pp 243-247
28

Lock
s
Suspend
0
End Block
Item: 29 of 99 ~ 1 • Mark <:] (:>- Jill ~· ~J
QID: 22416 ..1 Previous Next Lab'V!I!ues Notes Calculator
.
7 A 54-year-old homeless wh ite man presents to the loca l emergency department with altered menta l status and largely
un intelligible speech . On further examination, he has considerable gait difficu lt ies and eye movement abnormalit ies.
8
During the few moments that he is somewhat lucid, he cannot recall where he lives, where he was born, or what he does
9 for a living. His current glucose level is 50 mg/ dL.
10

~
Which of the following is the most appropriate immed iate treatment?

13 A. Glucose, then th iamine


14
B. Thiam ine, then glucose
15
16 C. Thiam ine, then glucose and folate
17 D. Vitamin 6 1 2, then thiam ine and glucose
18
E. Vitam in 66, then glucose and th iamine
19
20
21
22
23
24
25
26
27
28
• 29

Lock
s
Suspend
0
End Block
Item: 29 of 99 ~ 1 • Mark <:] (:>- Jill ~· ~J
QID: 22416 ..1 Previous Next Lab'V!I!ues Notes Calculator
.
7
8 The correct answer is B. 570/o chose this.
9 The patient's presentation corresponds to the classic presentation of thiam ine deficiency, w ith the patient perhaps suffering
from permanent Korsakoff's syndrome as evidenced by amnesia . However, thiamine should be administered in the hope that
10
the patient simply has reversible Wernicke's encephalopathy. Both disorders are caused by thiamine deficiency. Wernicke's

~
encepha lopathy refers to an acute presentation of the symptoms described in the question . Korsakoff's or Wernicke-Korsakoff
2 syndrome refers to the persistence of these symptoms, particu larly amnesia, and may be warded off by proper treatment of
13 Wern icke's encephalopathy. Patients with this presentation should be treated first with thiamine, and then their hypoglycemia
can be corrected . Giving glucose before thiamine has been reported to exacerbate thiamine deficiency.
14
Korsakoff's syndrome Wernicke-Korsakoff syndrome Wernicke's encephalopathy Hypoglycemia Thiamine Amnesia Encephalopathy Glucose Thiamine deficiency Receptive aphasia
15
A is not correct. 100/o chose this.
16
Glucose should not be given first in a patient with suspected thiam ine deficiency, because high-dose glucose administration
17
can precipitate symptoms of thiamine deficiency. This is because even more substrate is given for biochem ical pathways that
18 do not have proper substrates.
19 Substrate (chemistry) Thiamine Glucose Biochemistry Precipitation (chemistry) Thiamine deficiency

20 C is not correct. 230/o chose this.


21 Folate deficiency does not produce neurologic deficits, with the exception of some memory loss and personality changes. Most
22 common ly, folate deficiency presents with gastrointestina l symptoms, such as a swollen and red tongue, nausea, emesis,
23 diarrhea, and abdomina l pain after mea ls.
Folic add Folate deficiency Vomiting Diarrhea Nausea Abdominal pain Neurology Human gastrointestinal tract Amnesia
24
25 D is not correct. 7% chose this.
26 The patient does not have sensory and motor deficits consistent w ith vitamin 612 deficiency. The classic triad of vitamin 6 1 2
deficiency includes weakness, sore tongue, and paresthesias . However, the chief symptoms are usually weight loss and
27
anorexia, malaise, and neurologic symptoms such as paresthesias and gait abnormalities .
28 Vitamin 812 Gait abnormality B vitamins Paresthesia Vitamin Weight loss Malaise Anorexia (symptom) Anorexia nervosa Gait Neurology
29 y

Lock
s
Suspend
0
End Block
Item: 29 of 99 ~ 1 • Mark <:] (:>- Jill ~· ~J
QID: 22416 ..1 Previous Next Lab'V!I!ues Notes Calculator
• I I - • I I '"

7 Folate deficiency does not produce neurologic deficits 1 with the exception of some memory loss and personality changes. Most
8
common ly1 folate deficiency presents with gastrointestina l symptoms/ such as a swollen and red tongue 1 nausea 1 emesis1
diarrhea 1 and abdomina l pain after mea ls.
9 Folic acid Folate deficiency Vomiting Diarrhea Nausea Abdominal pain Neurology Human gastrointestinal tract Amnesia
10
D i s not correct. 7 % chose this.

~
13
The patient does not have sensory and motor deficits consistent w ith vitamin 612 deficiency. The classic triad of vitamin
deficiency includes weakness 1 sore tongue 1 and paresthesias . However1 the chief symptoms are usually weight loss and
anorexia 1 malaise 1 and neurolog ic symptoms such as paresthesias and gait abnormalit ies .
6 12

14 Vitamin 812 Gait abnormality 8 vitamins Paresthesia Vitamin Weight loss Malaise Anorexia (symptom) Anorexia nervosa Gait Neurology

15 E i s n ot correct. 30/o c hose t his.


16 Vitam in 6 6 deficiency would not present in this manner. Such deficiency usua lly presents with bilatera l distal extremity
17 numbness1 weakness/ sideroblastic anemia1 and skin changes 1 such as erythematous itching and burn ing 1 blisters1 and
18
hyperpigmentation with thicken ing . Vitam in B supplementation would not aid in this patient's condit ion acutely.
Vitamin 812 Anemia 8 vitamins Hyperpigmentation Itch Sideroblastic anemia Ervtl1ema Vitamin Anatomical terms of location Paresthesia
19
20
21 Bottom Li ne:
22 Thiam ine should be administered to a patient w ith Wernicke's encepha lopathy in an effort to reverse the condit ion . Glucose
23 should be avoided as the first step1 because adm inistration of high-dose glucose can precipitate symptoms of th iamine
deficiency.
24 Wernicke's encephalopathy Thiamine Glucose Encephalopathy Thiamine deficiency Receptive aphasia Wernicke's area
25
26
27 Refer ences:
28 FA Step 2 CK 9th ed p 291
FA Step 2 CK 8th ed p 276
29

Lock
s
Suspend
0
End Block
Item: 30 of 99 ~ 1 • Mark <:] (:>- Jill ~· ~J
QID: 23275 ..1 Previous Next Lab'V!I!ues Notes Calculator

8 A 6-year-old boy is referred to a pediatrician's office by hi s kindergarten teacher, who complains that he will not stop
daydreaming . She describes instances too numerous to count of the child "staring off into space" while fluttering his eyes.
9
His attention returns after 5-10 seconds, and he is never aware of these lapses. His parents state that they are aware of
10 their ch ild's daydream ing but were not troubled by it at home. The pediatrician performs an electroencephalogram, which shows

~
a bilateral, symmetric, synchronous 3- Hz spike-and-wave pattern .

13 Which of the following statements about this patient's diagnosis is correct?


14
15
16
A. Ch ildren with this diagnosis have absolutely no adverse effects and recover by early adulthood
17
18
B. Ethosuximide can be used to manage his symptoms, but eventually the child w ill requ ire hemispherectomy

19 C. It can be managed medically during childhood and normally disappears before adulthood
20
D. It is a manifestation of a progressive degenerative disorder that can be slowed but not cured
21
E. No treatment is ever indicated; the condit ion resolves quickly on its own
22
23
24
25
26
27
28
29
• 30

Lock
s
Suspend
0
End Block
Item: 30 of 99 ~ 1 • Mark <:] (:>- Jill ~· ~J
QID: 23275 ..1 Previous Next Lab'V!I!ues Notes Calculator

8
9 The correct answ e r i s C. 8 40/o chose t h is.
10 Absence seizures, also called "petit mal" seizures, ar e a condit ion of childhood characterized by brief (5- 10 seconds) peri ods

~
of impaired consciousness that can occur several hundred t imes per day. They often present when a teacher observes the
2
child "daydreaming" or "staring" in class. There is no loss of muscle tone, although there may be motor activity in the form of
li p-smacking and/ or eye fluttering . The classic EEG find irngs are 3- Hz spike-and- wave discharges . The condit ion is often
13 familial and disappear s before the child reaches adulthood . Prior to r esolution, symptoms can be managed medically with
14 ethosuxim ide or other antiepileptics .
Ethosuximide Spike-and-wave Electroencephalography Absence seizure Anticonvulsant Epileptic seizure
15
16 A i s not correct. 4 % ch ose this.
17 Although the medical prognosis of patients with absence seizur es is excellent, they often suffer from poor school performance
18 and behavioral problems in early childhood as a resu lt oftheir cond it ion . Medical management is intended to help minimize
t hese consequences.
19
Absence seizure Epileptic seizure Prognosis
20
21
B is not correct. 90/o chose this.
22
Ethosuximide is useful in treating absence seizures, but the seizures w ill eventually resolve as the child matur es.
Hemispher ectomy is an emerging t r eatment for refractor y malignant epilepsy and would never be ind icated for absence
23 seizures.
24 Ethosuximide Epilepsy Hemispherectomy Absence seizure Epileptic seizure Cancer ~1alignancy

25
D i s not correct. 20/o chose this.
26
Absence seizures resolve on their own before the child reaches adulthood. They are idiopathic and are not associated with
27 any know n brain lesion of degenerative disorder.
28 Absence seizure Lesion Epileptic seizure Degenerative disease Idiopathy Human brain Brain Stroke

29 E i s n ot correct. 1 0/o c hose th is.


30 y Althouah absence seizures are self-li mit ina condition and eventuallv r esolve soontaneouslv. treatment wit h antieoileotics is

Lock
s
Suspend
0
End Block
Item: 30 of 99 ~ 1 • Mark <:] (:>- Jill ~· ~J
QID: 23275 ..1 Previous Next Lab'V!I!ues Notes Calculator

• these consequences.
8 Absence seizure Epileptic seizure Prognosis

9
B is not correct. 90/o chose t h is.
10 Ethosuximide is useful in treating absence seizures, but the seizures will eventually resolve as the child matures.

~
Hemispherectomy is an emerging treatment for refractory malignant epilepsy and would never be ind icated for absence
seizures.
Ethosuximide Epilepsy Hemispherectomy Absence seizure Epileptic seizure Cancer ~1alignancy
13
14 D is not correct. 20/o chose this .
15 Absence seizures resolve on their own before the child reaches adulthood. They are idiopath ic and are not associated with
any known brain lesion of degenerative disorder.
16
Absence seizure Lesion Epileptic seizure Degenerative disease Idiopathy Human brain Brain Stroke
17
18
E is n ot correct. 1 0/o chose t his.

19
Although absence seizures are self-lim it ing condition and eventually resolve spontaneously, treatment with antiepileptics is
recommended to minim ize seizures prior to complete resolution.
20 Absence seizure Anticonvulsant Epileptic seizure
21
22
23
Bottom li ne:
24 Absence seizures are a cond it ion of ch ildhood characterized by brief (5-10 seconds) periods of impaired consciousness that
can occur several hundred t imes per day. This condition generally resolves before adulthood .
25 Absence seizure Epileptic seizure
26
27
28 Re fe re nces:
29 FA Step 2 CK 9th ed p 270
FA Step 2 CK 8th ed p 257
30

Lock
s
Suspend
0
End Block
Item: 31 of 99 ~ 1 • Mark <:] (:>- Jill ~· ~J
QID: 22203 ..1 Previous Next Lab'V!I!ues Notes Calculator
.
9 A 45-year-old patient presents w ith the chief complaint of new-onset headaches. He states they are worse at night and ~~AI
lying supine or flat. He states that he is unable to "speak well" over the last week. Physical exam reveals preservation of
10
cranial nerve function; however, dilated fundoscopic exam reveals mild papilledema bilaterally. Testing for his speech

~
reveals that he is not able to repeat anything that is said to him or to name objects . However, he has preserved fluency of
speech and good compr ehension of spoken and written Ia nguage.
13
14 Which of the following is the most likely diagnosis of his speech deficit?
15
16 A. Broca aphasia
17
B. Conduction aphasia
18
C. Global aphasia
19
20 D. Transcortical sensory aphasia
21 E. Wernicke aphasia
22
23
24
25
26
27
28
29
30
• 31

Lock
s
Suspend
0
End Block
Item: 31 of 99 ~ 1 • Mark <:] (:>- Jill ~· ~J
QID: 22203 ..1 Previous Next Lab'V!I!ues Notes Calculator
.
9
10
The correct answer is B. 630/o chose this.

~
13
2
The patient suffers from conduction aphasia, which is characterized by problems w ith r epeating what is sa id and nam ing, but
preserved fluency and comprehension. Anatomica lly the lesion occurs at the arcuate fascicu lus between Broca and Wernicke
area . In th is vignette, it is due to a tumor compressing t h is area, which is also responsible for his headaches and
14 papilledema .
Arcuate fasciculus Aphasia Papilledema Conduction aphasia Wernicke's area lesion Neoplasm
15
16 A is not correct. 130/o chose this.
17 Broca aphasia is characterized by preserved comprehension, but problems with language production. These patients speak
18 nonfluently, haltingly, and without much intonation . Naming and repetition are also impaired . They are aware of their problem
and may often become depressed . I nteresting ly, some patients can ma intain fluency when singing . Broca area includes the
19
posterior part of the inferior f rontal gyrus and a surrounding rim of prefrontal heteromodal cortex .
20 Inferior frontal gyrus Aphasia Broca's area Intonation (linguistics) Cortex (anatomy) Prefrontal cortex
21
C is not correct. 4% chose this.
22
This is caused by a large lesion in the dominant hem isphere, lead ing to problems with language production, comprehension,
23 and repetition. It can be conceptua lized as a combination of Broca and Wernicke aphasia.
24 Receptive aphasia Aphasia lateralization of brain function lesion

25
D is not correct. 120/o chose this.
26 This type of aphasia is characterized by impaired comprehension of language, but preserved repetition and fluency.
27 Aphasia

28
E is not correct. SO/o chose this.
29
Wern icke aphasia represents a problem with language compr ehension, which affects both language output and input.
30 Language output is paraphasic and circum locutious . Repetition, naming, reading, and writing are also impaired although
31 y
certa in axial tasks, such as opening one's eyes, may be preserved. Wernicke area includes the posterior third of the superior y

Lock
s
Suspend
0
End Block
Item: 31 of 99 ~ 1 • Mark <:] (:>- Jill ~· ~J
QID: 22203 ..1 Previous Next Lab'V!I!ues Notes Calculator
. • postenor part of the mfenor frontal gyrus and a su rr oundmg n m of prefrontal heteromodal cortex .
9 Inferior frontal gyrus Aphasia Broca's area Intonation (linguistics) Cortex (anatomy) Prefrontal cortex

10
C is not correct. 4% chose this.

~
This is caused by a large lesion in the dominant hemisphere, lead ing to problems with language production, comprehension,
and repetit ion . It can be conceptua lized as a combination of Broca and Wernicke aphasia.
Receptive aphasia Aphasia lateralization of brain function lesion
13
14 D is not correct. 120/o chose this.
15 This type of aphasia is characterized by im pa ired comprehension of language, but preserved repetit ion and fluency.
16 Aphasia

17 E is not correct. SO/o chose this.


18 Wern icke aphasia represents a problem with language compr ehension, which affects both language output and in put.
19 Language output is paraphasic and circum locutious . Repetit ion, naming, reading, and writing are also impaired although
20
certa in axial tasks, such as open ing one's eyes, may be preserved . Wernicke area includes the posterior thi rd of the superior
tempora l gyrus and a surrounding rim of inferior parietal and midtemporal cortex.
21 Receptive aphasia Superior temporal gyrus Aphasia Wernicke's area Sentence processing Paraphasia Parietal lobe Cortex (anatomy) Parietal bone
22
23
24
Bottom line:
25 Conduction aphasia is characterized by problems w ith r epeating what is said and nam ing, but preserved fluency and
comprehension . Anatomically the lesion occu rs at the arcuate fascicu lus between Broca and Wernicke area .
26 Conduction aphasia Arcuate fasciculus Aphasia Wernicke's area lesion
27
28
29 References:
30 FA Step 2 CK 9th ed pp 288-289
FA Step 2 CK 8th ed p 274
31

Lock
s
Suspend
0
End Block
Item: 32 of 99 ~ 1 • Mark <:] (:>- Jill ~· ~J
QID: 22195 ..1 Previous Next Lab'V!I!ues Notes Calculator
.
10 A 58-year-old woman complains of throbbing right-sided headaches that began sudden ly 4 days ago. She describes a 5-
minute visual aura preceding the headache . She has never experienced a headache like th is previously and has no past

~
13
medical history of migra ines . Fam ily history is also negative for migra ines. Her neurologic examination is unremarkable .

Which of the following is the most logical next step?


14
15
A. Advise patient to avoid headache triggers
16
17
B. Neurologic imaging with MRI or CT scan

18 C. Nonsteroidal anti-inflammatory drug therapy


19
D. Start ~-b l ockers
20
21
E. Start triptan pharmacotherapy
22
23
24
25
26
27
28
29
30
31
• 32

Lock
s
Suspend
0
End Block
Item: 32 of 99 ~ 1 • Mark <:] (:>- Jill ~· ~J
QID: 22195 ..1 Previous Next Lab'V!I!ues Notes Calculator
.
10 The correct answer is B. 460/o chose this.

~
The patient is older than 50 and has a sudden development of new headaches . The most common primary headache
2 disorders are migraine, tension headaches, and cluster headaches . This patient most li kely presents with migraine, which is a
13
disorder of recurrent attacks with headaches that are oft·en unilateral, tend to have a throbbing qua lity, and may be
accompanied by nausea, vom it ing, photophobia, or phonophobia. However, there are several 'red flags' that need to be
14 considered when evaluating a headache . The mnemonic S NOO P can help to identify warning signs of possible serious
15 underlying disease . S NOOP stands for :
16 Systemic symptoms, illness, or condition ( eg, fever, weight loss, cancer, pregnancy, immunocomprom ised state includ ing
HIV) .
17
Neurolog ic symptoms or abnormal signs ( eg, confusion, impaired alertness or consciousness, papilledema, focal neurological
18 symptoms or signs, meningismus, or seizures) .
19 New Onset, particularly for age > 40 years, or sudden Onset.
20
Other associated conditions of reatures ( eg, head trauma, illicit drug use, toxic exposure, headache awakens from sleep, is
worse with Valsalva maneuvers 1 is precipitated by cough, exertion 1 or sexual activity) .
21
Previous headache history with headache progression or change in attack frequency, severity, or clinical features .
22 Given our patient's age and new onset of symptoms, neurologic imaging should be performed .
23 Papilledema Photophobia Phonophobia Migraine Immunodeficiency Meningism Cluster headache Nausea Headache Tension headache Vomiting HIV Epileptic seizure Head injury

24 Cough Mnemonic Weight loss Pregnancy Cancer Neurology Fever

25 A is not correct. 170/o chose this.


26 Migraine trigger factors may include stress, menstruation, visual stimiuli, weather changes, sleep disturbances, or certain
27 food items like red w ine, cheese, or chocolate. Avoiding these triggers is an effective method for some patients to prevent
28
migraine headaches . However, before recommending this approach to this patient, a thorough evaluation should be
undertaken in order to rule out more serious causes of her headache .
29 Migraine Menstruation Headache Red wine Sleep disorder Chocolate Stress (biology) Wine Cheese
30
C is not correct. 190/o chose this.
31
Nonsteroidal anti-inflammatory drugs (NSAIDs) are often useful as abortive therapy for migra ine headaches; ie, they stop a
32 y - : - .. -.: ... - L-.--....1-,...J,...,... ............... :• L...-.- .... ,...,... , ,,...,...,...,...1 1\lt="/\TI""'\.,... •• , :.I... ,...,..._,...,....,,.......! .... ~&:,..._,...,, &,...,... - :- .... -: ... ,...,... •• ,;.I,..,...,... .•. ;~1,..,..., ,'- _, , ,..._ : ....... ! , ,....1,... _,... ... :,...; ...

Lock Suspend
s 0
End Block
Item: 32 of 99 ~ 1 • Mark <:] (:>- Jill ~· ~J
QID: 22195 ..1 Previous Next Lab'V!I!ues Notes Calculator

• • • I g gg • • • p p
10 migraine headaches . However, before recommend ing this approach to this patient, a thorough evaluation should be
undertaken in order to rule out more serious causes of her headache .

~
13
Migraine Menstruation Headache Red wine Sleep disorder Chocolate Stress (biology) Wine Cheese

C i s not correct. 190/o chose this.


14
Nonsteroidal anti-inflammatory drugs (NSAIDs) are often useful as abortive therapy for migra ine headaches; ie, they stop a
migraine headache once it has occurred. NSAIDs with reported efficacy for m igraines w ith or without aura include aspirin,
15 ibuprofen, naproxen, diclofenac, and ketorolac. Although NSAIDs may prov ide acute, symptomatic relief of her headaches,
16 they would not address the potentia lly serious underlying cause of her headaches . Therefore, further work-up is necessary.
Ibuprofen Naproxen Diclofenac Migraine Aspirin Anti-inflammatory Nonsteroidal anti-inflammatory drug Headache
17
18 D i s not correct. SOfo chose this.
19 ~ - Blockers are good prophylactic therapy for migraine headaches. However, before starting this patient on any
20 pharmacotherapy, a thorough neurologic evaluation should be undertaken in order to rule out more serious causes of her
headaches.
21
Migraine Neurological examination Pharmacotherapy Preventive healthcare Neurology
22
23
E i s n ot correct. 1 3% chose this.
24
Triptans are serotonin l b/ld agonists that are effective for the acute, symptomatic treatment of migraine headache because
they inh ibit the release of vasoactive peptides and thus promote vasoconstriction . Based on limited evidence, triptans should
25 be avoided in patients with hemiplegic migraine, basilar migra ine, ischemic stroke, ischemic heart disease, Prinzmetal's
26 angina, uncontrolled hypertension, and pregnancy. Combination with MAO inhibitors is contraindicated with most triptans .
27 Also, responsiveness to treatment w ith triptans should not be considered diagnostic for migraine, as secondary headaches
may also improve with triptan therapy. This patient needs further work-up of her headaches to address the underlying cause .
28
Triptan Serotonin Prinzmetal's angina Coronary artery disease Vasoconstriction Ischemic stroke Migraine Hypertension Stroke Angina pectoris Monoamine oxidase inhibitor Headache
29
Basilar artery Hemiparesis Cardiovascular disease Peptide Contraindication Ischemia Pregnancy Agonist
30
31
32 Bottom line : I.
Lock
s
Suspend
0
End Block
Item: 32 of 99 ~ 1 • Mark <:] (:>- Jill ~· ~J
QID: 22195
. ..1
. .. . . . . ...
Previous
..
Next
. . .. . .
Lab'V!I!ues Notes
. . .. .
Calculator
.. . . . .. . .. . .
Ibuprofen Naproxen Diclofenac Migraine Aspirin Anti-inflammatory Nonsteroidal anti-inflammatory drug Headache
10

~
D is not correct. SOfo chose this.
~ - Blockers are good prophylactic therapy for migraine headaches. However, before starting this patient on any

13
pharmacotherapy, a thorough neurologic evaluation should be undertaken in order to rule out more serious causes of her
headaches.
14 Migraine Neurological examination Pharmacotherapy Preventive healthcare Neurology
15
E is not correct. 1 3% chose this.
16
Triptans are serotonin l b/ld agonists that are effective for the acute, symptomatic treatment of migraine headache because
17
they inh ibit the release of vasoactive peptides and thus promote vasoconstriction . Based on limited evidence, triptans should
18 be avoided in patients with hemiplegic migraine, basilar migra ine, ischemic stroke, ischemic heart disease, Prinzmetal's
19 angina, uncontrolled hypertension, and pregnancy. Combination with MAO inhibitors is contraindicated with most triptans .
Also, responsiveness to treatment w ith triptans should not be considered diagnostic for migraine, as secondary headaches
20
may also improve with triptan therapy. This patient needs further work-up of her headaches to address the underlying cause .
21 Triptan Serotonin Prinzmetal's angina Coronary artery disease Vasoconstriction Ischemic stroke Migraine Hypertension Stroke Angina pectoris Monoamine oxidase inhibitor Headache
22 Basilar artery Hemiparesis Cardiovascular disease Peptide Contraindication Ischemia Pregnancy Agonist
23
24
25
Bottom li ne:
26 Patients over SO years of age presenting with acute onset of migraine-like headaches shou ld undergo neurologic imaging to
rule out more serious pathology.
27 Pathology Neurology
28
29
30 References:
31 FA Step 2 CK 9th ed p 267
FA Step 2 CK 8th ed p 253
32

Lock
s
Suspend
0
End Block
Item: 33 of 99 ~ 1 • Mark <:] (:>- Jill ~· ~J
QID: 23612 ..1 Previous Next Lab'V!I!ues Notes Calculator
I

A 6-year- old boy is referred to the ped iatrician by his kindergarten teacher due to poor performance and inattentiveness

~
13
during class . She reports that the child often seems to be staring into space and then will return to his work after 5-10
seconds. These staring spells are often accompanied by excessive blinking . His parents have also noticed that their child
often daydreams wh ile at home.
14
15 Which next step in management will provide the most specific information for the ch ild's likely diagnosis?
16
17 A. CT of the head
18
B. Electroencepha lography
19
20 C. MRI of the head
21 D. Start ethosuximide
22
E. Visual field test
23
24
25
26
27
28
29
30
31
32
• 33

Lock
s
Suspend
0
End Block
Item: 33 of 99 ~ 1 • Mark <:] (:>- Jill ~· ~J
QID: 23612 ..1 Previous Next Lab'V!I!ues Notes Calculator
I

~2
13
The correct answer is B. 790/o chose this.
This patient's staring spells are absence seizures/ also called "petit mal" seizures1 a condition of childhood characterized by
brief (5-10 seconds) periods of im paired consciousness tlhat can occur several hundred times per day. They often present
14
when a teacher observes the child "daydreaming" or "staring" in class . There is no loss of muscle tone 1 although there may
15 be motor activity in the form of lip smacking or eye fluttering . The condition is often familia l and disappears before the child
16 reaches adu lthood . Absence seizures are diagnosed with electroencephalography/ which classically shows a 3-Hz symmetric
17 spike-and-wave pattern . Absence seizures are typically treated with the ca lcium channel blocker ethosuximide or valproic acid
(in cases that do not respond to ethosuximide) .
18
Calcium channel blocker Ethosuximide Valproate Electroencephalography Spike-and-wave Absence seizure Calcium channel Epileptic seizure Calcium ~1uscle tone Consciousness
19
A is not correct. 2% chose this.
20
Imaging is not indicated if a chi ld presents typically with symptoms of absence seizures/ such as brief loss of consciousness
21
w ithout posticta l lethargy/ eye fluttering/ and triggered by hyperventilation .
22 Hyperventilation Postictal state Epileptic seizure Absence seizure Syncope (medicine) Unconsciousness Lethargy
23
C is not correct. 2% chose this.
24
Imaging modalities such as CT and MRI would likely be performed in th is patient to rule out other neurologic disorders 1 such
25 as childhood neoplasms. However1 these modalities wou ld not be specigic to diagnose his current cond iton .
26 Magnetic resonance imaging Neoplasm Neurology CT scan

27
D is not correct. 160/o chose this.
28 A history of staring spells and repetitive movements in an otherwise healthy child necessitates a work-up for an undiagnosed
29 seizure disorder. It is inappropriate to start medications without further investigation .
Epilepsy Epileptic seizure
30
31 E is not correct. 10/o chose this.
32 A visua l field test cou ld show evidence of certain intracran ial neoplasms/ but this ch ild has no history of headaches1 endocrine
33 y
abnormalities/ or changes in vision . Alternate testing wou ld be more appropriate in this case. y

Lock
s
Suspend
0
End Block
Item: 33 of 99 ~ 1 • Mark <:] (:>- Jill ~· ~J
QID: 23612 ..1 Previous Next Lab'V!I!ues Notes Calculator
I
Without post1ctal lethargy, eye flutten ng, and t nggered by hypervent1lat1on .

~
Hyperventilation Postictal state Epileptic seizure Absence seizure Syncope (medicine) Unconsciousness Lethargy

C is not correct. 2 % chose this.


13 Imaging modalit ies such as CT and MRI would li kely be performed in this patient to rule out other neurologic disor ders, such
14 as childhood neoplasms. However, these modalit ies wou ld not be specigic to diagnose his current conditon .
Magnetic resonance imaging Neoplasm Neurology CT scan
15
16 D is not correct. 1 60/o chose this.
17 A history of staring spells and repetit ive movements in an otherw ise healthy ch ild necessitates a work-up for an undiagnosed
18 seizure disorder. It is inappropriate to start medications w ithout further investigation .
Epilepsy Epileptic seizure
19
20 E is not correct. 1 0/o chose th is.
21 A visual field test could show evidence of certain intracranial neoplasms, but this child has no history of headaches, endocrine
22
abnormalit ies, or changes in vision . Alternate testing would be more appropriate in this case.
Visual field test Endocrine system Visual field Neoplasm
23
24
25 Bottom Li ne:
26 Absence seizures are a cond it ion of ch ildhood characterized by brief periods of impa ired consciousness that can occur
27 several hundred t imes per day. Absence seizures are diagnosed with electroencephalog raphy, w hich classically shows a 3-Hz
symmetric spike-and- wave pattern .
28 Electroencephalography Spike-and-wave Absence seizure Epileptic seizure
29
30
31 References:
32 FA Step 2 CK 9th ed pp 269-270
FA Step 2 CK 8th ed p 257
33

Lock
s
Suspend
0
End Block
Item: 34 of 99 ~ 1 • Mark <:] (:>- Jill ~· ~J
QID: 23290 ..1 Previous Next Lab'V!I!ues Notes Calculator

A 70-year-old man is brought to the emergency department in a confused state following a seizure in public. Witnesses ~~AI
report the seizure as occurring sudden ly and lasting at least 2-3 m inutes . A call to his fam ily reveals that he has recently
13
had a stroke and is not known to have an underlying seizure disorder. On examination the patient's tongue is unharmed,
14 although his pants are covered in fresh urine . He is oriented to person but not place or time and has no recoll ection of what
15 happened . Physical examination is otherwise unremarkab le.
16
17 Which of the following is the most likely cause of this patient's seizure?
18
19 A. Alcohol withdrawal
20 B. Brain tumor
21
C. Cerebrovascular disease
22
23 D. Electrolyte imbalance
24 E. Primary seizure disorder
25
26
27
28
29
30
31
32
33
• 34

Lock
s
Suspend
0
End Block
Item: 34 of 99 ~ 1 • Mark <:] (:>- Jill ~· ~J
QID: 23290 ..1 Previous Next Lab'V!I!ues Notes Calculator
---------------

13
14
The correct answer is C. 680/o chose this.
This patient has most likely suffered a grand mal (tonic-clonic) seizure involving both cerebral hemispheres and resulting in
15
complete loss of consciousness. Patients often experience incontinence and/or tongue biting during the seizure . The post-ictal
16 period is marked by confusion as the patient slowly returns to consciousness. The most common cause of seizures in patients
17 >35 years old is cerebrovascular disease and as many as SO% of seizures in patients >65 years old are due to it. Seizures
18 secondary to strokes can occur from hours to years after the initial ischemic event. Hemorrhagic strokes, strokes involving
cortical structures, and strokes covering large areas are t he most significant risk factors for the development of post-stroke
19
epilepsy.
20 Cerebrovascular disease Epilepsy Cerebral hemisphere Postictal state Epileptic seizure Generalised tonic-clonic seizure Urinary incontinence Syncope (medicine) Unconsciousness
21 Seizure types Ischemia Stroke Cerebral cortex
22
A is not correct. 4 % chose this.
23
Alcohol withdrawal often causes seizures, but in this patient without a known history of severe alcohol abuse, but a known
24 history of a stroke, a stroke is by far the most likely cause of the seizure .
25 Alcohol withdrawal syndrome Alcohol abuse Alcoholic beverage Alcohol Epileptic seizure Stroke

26
B is not correct. ] Ofo chose t his.
27 A brain tumor (either a primary tumor or metastasis) can cause a seizure in the elderly. The patient in the stem is not known
28 to have a history of cancer, nor any symptoms suggesting malignancy (weight loss, chills, and night sweats), nor any
29 symptoms localizing a tumor to the cranium (morn ing headaches) . In any person with a history of a stroke, the
cerebrovascular accident itself is the most likely cause of a seizure.
30
Stroke Metastasis Brain tumor Malignancy Neoplasm Primary tumor Skull Cancer Brain Weight loss Cerebrovascular disease Night sweats Human brain Epileptic seizure
31
32
D is not correct. 1 50/o chose t his.
A number of electrolyte disorders can present with seizures, and a basic metabolic panel wou ld be indicated in the work-up of
33
this patient. However, given that he has a known history of a stroke, the seizure is much more likely attributable to the bra in
34 i ne•dt

Lock
s
Suspend
0
End Block
Item: 34 of 99 ~ 1 • Mark <:] (:>- Jill ~· ~J
QID: 23290 ..1 Previous Next Lab'V!I!ues Notes Calculator

B is not correct. ]Ofo chose this.


13
A brain tumor (either a primary tumor or metastasis) can cause a seizure in the elderly. The patient in the stem is not known
to have a history of cancer, nor any symptoms suggesting malignancy (weight loss, chills, and night sweats), nor any
14
symptoms localizing a tumor to the cranium (morn ing headaches) . In any person with a history of a stroke, the
15 cerebrovascular accident itself is the most likely cause of a seizure.
Stroke Metastasis Brain tumor Malignancy Neoplasm Primary tumor Skull Cancer Brain Weight loss Cerebrovascular disease Night sweats Human brain Epileptic seizure
16
17 D is not correct. 150/o chose this.
18 A number of electrolyte disorders can present with seizu r es, and a basic metabolic panel would be indicated in the work-up of
19 this patient. However, given that he has a known history of a stroke, the seizure is much more likely attributable to the bra in
insult .
20
Electrolyte Basic metabolic panel Brain Metabolism Stroke Epileptic seizure
21
22
E i s n ot cor rect. 6 0/o c hose th is.

23
Undiagnosed primary epilepsy in the elderly is rare, and it is typica lly attributed to a stroke ( as in this patient) or dementia.
Epilepsy Dementia Stroke
24
25
26 Botto m Line :
27 Seizures secondary to strokes can occur from hours to years after the init ial ischemic event. Hemorrhagic strokes, strokes
28 involving cortical structures, and strokes covering large areas are the most sign ificant risk factors for the development of
epilepsy after stroke.
29 Epilepsy Epileptic seizure Ischemia Stroke Cortex (anatomy) Cerebral cortex Bleeding
30
31
32 Ref er en ces :
33 FA Step 2 CK 9th ed pp 269- 270
FA Step 2 CK 8th ed pp 256- 257
34

Lock
s
Suspend
0
End Block
Item: 35 of 99 ~ 1 • Mark <:] (:>- Jill ~· ~J
QID: 23460 ..1 Previous Next Lab'V!I!ues Notes Calculator

1•AI
Lro1
13 A 23-year-old man presents to the clin ic with worsen ing headaches over the past year. The pain feels like his head is being
squeezed in a vise and radiates from the occiput to the parietal/fronta l region . The headaches occur a couple of t imes a A
14
week and can last from 30 minutes to a few hours. They are not worse with physical activity and are occasionally relieved
15 w ith nonprescription pain relievers; he has noticed, however, that the frequency of the headaches has increased since he has
16 become busier at work. The pain is nonpulsitile and there is no associated photophobia, visual changes, tearing, or nasal
17
congestion . Physical examination is unremarkable.
18
What is the most likely underlying mechanism of his headache?
19
20
21
A. Contraction of the scalp and neck muscles
22 B. Increased intracranial pressure
23
C. Inflammation of the medium and large arteries
24
25
D. Release of vasoactive neuropeptides

26 E. Vasodilation
27
28
29
30
31
32
33
34
• 35

Lock
s
Suspend
0
End Block
Item: 35 of 99 ~ 1 • Mark <:] (:>- Jill ~· ~J
QID: 23460 ..1 Previous Next Lab'V!I!ues Notes Calculator

Lro1
13
14
15
The correct answer is A. 610/o chose this.
16
This case is typical for episodic tension headaches, which require two of the following signs or symptoms for diagnosis :
17 pressure or tightening pain, frontal-occiptal location, bilateral with moderate intensity, and not aggravated by physical
18 activity. The pain in tension headaches is thought to be due to a combination of muscle contractions and psychogenic factors.
Psychogenic pain Tension headache Musde Psychogenic disease
19
20 B is not correct. 50/o chose this.
21 Headaches due to increased intracranial pre ssure are usually worse in the morn ing (because cerebrospina l fluid absorption is
22 poorer when one lies flat) and relieved by sitting up. Patients w ith increased intracranial pre ssure often have papilledema or,
in chronic cases, pa le discs due to optic atrophy.
23
Papilledema Cerebrospinal fluid Intracranial pressure Optic neuropathy Atrophy
24
25
C is not correct. 4% chose this.
Inflammation of the medium and large arteries is the mechanism for headache in temporal arteritis {TA) . Although the pain is
26
typically un ilateral in TA, it can be bilateral and diffuse; however, TA is rare in individuals <50 yea rs old . In addition, the pain
27 in TA is worsened by activity or by putting pressure on the head .
28 Giant-cell arteritis Headache Inflammation Artery Arteritis

29
D is not correct. 140/o chose this.
30 Release of vasoactive neuropeptides such as serotonin and dopamine has been proposed as a mechanism for migraine
31 headaches. Migraine headaches are typically uni lateral with a pulsing or th robbing pain that lasts 4-72 hours and are
32 accompanied by nausea, vomiting, photophobia, and phonophobia. Attacks may be preceded by an au ra, usua lly visua l. Pain
is usually worse with activity (as opposed to this patient) and relieved by lying down in a dark, qu iet room .
33
Photophobia Phonophobia Serotonin ~1igraine Dopamine Nausea Vomiting Vasoactive Neuropeptide
34
35
E is not correct. 16% chose this.

Lock
s
Suspend
0
End Block
Item: 35 of 99 ~ 1 • Mark <:] (:>- Jill ~· ~J
QID: 23460 ..1 Previous Next Lab'V!I!ues Notes Calculator
11'11 I I - " • f I •

13 Inflammation of the medium and large arteries is the mechanism for headache in tempora l arteritis (TA). Although the pain is
14
typically unilateral in TA, it can be bilatera l and diffuse ; however, TA is rare in individuals < 50 years old. In addit ion , the pain
in TA is worsened by activity or by putting pressur e on the head.
15 Giant-cell arteritis Headache Inflammation Artery Arteritis
16
D is not correct. 140/o chose t his.
17
Release of vasoactive neuropeptides such as seroton in and dopam ine has been proposed as a mechanism for m igraine
18
headaches. Migraine headaches ar e typically un ilateral w ith a pulsing or throbbing pain that lasts 4-72 hour s and ar e
19 accompanied by nausea, vomit ing, photophobia, and phonophobia . Attacks may be preceded by an aura, usually visual. Pain
20 is usually worse with activity ( as opposed to th is patient) and relieved by lying down in a dark, quiet room.
Photophobia Phonophobia Serotonin ~1igraine Dopamine Nausea Vomiting Vasoactive Neuropeptide
21
22 E is not correct. 16% chose this.
23 Vasodilation has been hypothesized as a mechanism of cluster headaches, which are thought to be associated with histamine
24
release . Episod ic cluster headaches are usually sharp, unilateral in the temple and periorbital regions, and may be associated
with ipslateral lacrimation and rhinorrhea . Cluster headaches typically occur during a 4- to 16-week period at the same t ime
25 each year. The case presented her e is more consistent with tension headache than cluster headaches .
26 Tension headache Rhinorrhea Tears Vasodilation Histamine Cluster headache Headache

27
28
Bottom line :
29
Pain in tension headaches is thought to be due to a combination of muscle contractions and psychogenic factors.
30 Psychogenic pain Tension headache Psychogenic disease
31
32
33 References:
34 FA Step 2 CK 9th ed p 268
FA Step 2 CK 8th ed p 254
35

Lock
s
Suspend
0
End Block
Item: 36 of 99 ~ 1 • Mark <:] (:>- Jill ~· ~J
QID: 23310 ..1 Previous Next Lab'V!I!ues Notes Calculator

14 A 39-year-old obese African-American man presents with several episodes of weakness and tingling in his right hand. Th is ~~AI
morning he became extremely concerned when he was unable to grip his coffee cup and cou ld not open the car door. He is
15
an electrician and has been unable to go to work for the past week as a r esu lt of these episodes. He notes that he has also
16 been awakened at night w ith numbness and tingling in th,e same arm . He has a history of diabetes and has smoked for 25
17 years . He r eports that his father also has diabetes and recently had coronary artery bypass surgery. Physical examination
18
reveals weak grip of the right hand compared to the left with signs of thenar atrophy.
19
20
Which of the following is the most appropriate next diagnostic test?
21
22
A. Cerebral ang iogram
23 B. CT scan of the head
24
C. Electroencepha logram
25
26
D. Electromyog raphy
27 E. MRI of the head
28
29
30
31
32
33
34
35
• 36

Lock
s
Suspend
0
End Block
Item: 36 of 99 ~ 1 • Mark <:] (:>- Jill ~· ~J
QID: 23310 ..1 Previous Next Lab'V!I!ues Notes Calculator

14
15
16
The correct answer is D. 770/o chose this.
17
The man has signs and symptoms of carpal tunnel syndrome, entrapment of the median nerve characterized by pain and
18 paresthesias in the medial portion of the palm . Carpal tunnel is often a cl inical diagnosis, although several objective
19 examination findings and studies can be of use, including nerve conduction studies and electromyography.
Carpal tunnel syndrome Electromyography Median nerve Nerve conduction study Paresthesia Carpal tunnel Carpal bones
20
21 A is not correct. 5 % chose this.
22 Angiography is used in the setting of an acute ischem ic stroke, when identifying the vascu lar anatomy would change medical
23 or surgical management. This invasive maneuver is far too risky in a patient whose symptoms are not very li kely to be
attributable to stroke; in such a patient if stroke were more likely than carpal tunnel syndrome (the correct diagnosis), then a
24
CT scan of the head wou ld be a better in it ial study.
25 Carpal tunnel syndrome cr scan Ischemic stroke Stroke Angiography Ischemia Anatomy Blood vessel
26
B is not correct. 110/o chose t his.
27
CT scan of the head wou ld be usefu l to determine if the patient's symptoms had a cerebrovascular etiology. However, the fact
28 that he reports chron ic interm ittent numbness and t ingling that occasionally wake him at night and the fact that he has
29 weakness and atrophy in the distribution of the median nerve distal to the carpal tunnel are highly suggestive of carpal tunnel
30 syndrome. While th is man clearly has risk factors for stroke, the more li kely diagnosis is carpal tunnel syndrome .
Carpal tunnel syndrome cr scan Median nerve Paresthesia Carpal bones Carpal tunnel Etiology Atrophy Stroke Hypoesthesia Cerebrovascular disease Anatomical terms of location
31
32 C is not correct. 4 % chose this.
33 An electroencephalogram would be useful if there were reason to suspect that the man's symptoms were due to a seizure
disorder. However, there is no history of uncontrollable movements, loss of consciousness, or sensory disturbance other than
34
the paresthesias indicative of carpal tunnel syndrome . It would be very unusual for a focal simple seizure to present in this
35 manner, moving it far lower on the differential than carpal tunnel or stroke.
36 y
Electroencephalography Carpal tunnel syndrome Paresthesia Syncope (medicine) Ep ilepsy Stroke Unconsciousness Epileptic seizure Carpal tunnel

Lock
s
Suspend
0
End Block
Item: 36 of 99 ~ 1 • Mark <:] (:>- Jill ~· ~J
QID: 23310 ..1 Previous Next Lab'V!I!ues Notes Calculator

• • • p • y p • • gy • I

14 that he reports chronic intermittent numbness and ting ling that occasionally wake him at night and the fact that he has
15
weakness and atrophy in the distribution of the median nerve distal to the carpal tunnel are highly suggestive of carpal tunnel
syndrome. While this man clearly has risk factors for stroke, the more likely diagnosis is carpal tunnel syndrome .
16 Carpal tunnel syndrome cr scan Median nerve Paresthesia Carpal bones Carpal tunnel Etiology Atrophy Stroke Hypoesthesia Cerebrovascular disease Anatomical terms of location
17
C is not correct. 40/o chose this.
18
An electroencephalogram would be useful if there wer e reason to suspect that the man's symptoms were due to a seizure
19
disorder. However, there is no history of uncontrollable movements, loss of consciousness, or sensory disturbance other than
20 the paresthesias indicative of carpal tunnel syndrome . It would be very unusual for a focal simple seizure to present in this
21 manner, moving it far lower on the differentia l than carpal tunnel or stroke.
Electroencephalography Carpal tunnel syndrome Paresthesia Syncope (medicine) Epilepsy Stroke Unconsciousness Epileptic seizure Carpal tunnel
22
23 E is not correct. 30/o chose th is.
24 MRI of the head would be one modality to look for a stroke and is more sensit ive than CT in identifying early ischemic
25
strokes. However, there is nothing in the stem to suggest an acute pathology, and, in this case, the test would be a waste of
resources. More importantly, the clinica l diagnosis of carpal tunnel is most likely, and an MRI of the head wou ld not be
26 indicated .
27 Magnetic resonance imaging Pathology Stroke Ischemia

28
29
Bottom line:
30
Carpal tunnel syndrome is often a clinica l diagnosis, although nerve conduction studies and electromyography can be usefu l.
31 Carpal tunnel syndrome Electromyography Nerve conduction study Carpal tunnel
32
33
34 Refere n ces:
35 FA Step 2 CK 9th ed pp 239-240
FA Step 2 CK 8th ed pp 221-222
36

Lock
s
Suspend
0
End Block
Item: 37 of 99 ~ 1 • Mark <:] (:>- Jill ~· ~J
QID: 22419 ..1 Previous Next Lab'V!I!ues Notes Calculator
.
15 A 23-year-old man with epilepsy presents to the emergency department (ED) following an episode lasting roughly 15 ~~AI
seconds in which he lost consciousness. He said that while jogging on a hot, sunny day he began to feel faint and
16
progressively lightheaded, and then was lim p for roughly 15 seconds during which he gradually lost consciousness . He
17 then returned to consciousness, but felt confused for rough ly 10 minutes thereafter. The doctor in the ED tries to figure out
18 whether the episode was syncope or a seizure.
19
20 Which aspect of the vignette is more consistent with a seizure?
21
22 A. Confusion following the event
23
B. Duration of 15 seconds
24
C. Faintness
25
26 D. Jogging on a hot, sunny day
27 E. Progressive nature of onset
28
29
30
31
32
33
34
35
36
• 37

Lock
s
Suspend
0
End Block
Item: 37 of 99 ~ 1 • Mark <:] (:>- Jill ~· ~J
QID: 22419 ..1 Previous Next Lab'V!I!ues Notes Calculator
.
15
16
17
The correct answer is A. 870/o chose this.
Patients with syncope typica lly immediately return to norma l mentation, wh ile this patient's confusion is consistent with a
18
post-icta l state .
19 Syncope (medicine) Syncope (phonology) Postictal state
20
B is not correct. 50/o chose this.
21
Seizures typical ly last 1-2 minutes, while syncopa l episodes rarely last > 15 seconds.
22 Epileptic seizure
23
Cis not correct. 1% chose this.
24
Faintness is a hal lmark of syncope .
25 Syncope (medicine) Syncope (phonology)
26
D is not correct. 2% chose this.
27
Dehydration, as well as increased physiolog ic demands placed on the body throu gh runnin g, wou ld increase the likelihood of
28
syncope.
29 Syncope (medicine) Dehydration Syncope (phonology)

30
E is not correct. 50/o chose this.
31
A progressive onset is consistent with syncope, in which patients often feel progressively light-headed .
32 Syncope (phonology) Syncope (medicine) Progressive rock
33
34
Bottom line:
35
36
Patients with syncope typica lly immediately return to norma l mentation . When the patient continues to be confused in the
posticta l state, a seizure is more likely.
37

Lock
s
Suspend
0
End Block
Item: 37 of 99 ~ 1 • Mark <:] (:>- Jill ~· ~J
QID: 22419 ..1 Previous Next Lab'V!I!ues Notes Calculator
. Syncope (med1cme) Syncope (phonology) Postictal state
15
B is not correct. 50/o chose this.
16
Seizures typica lly last 1-2 minutes, while syncopal episodes rarely last > 15 seconds.
17
Epileptic seizure
18
19
Cis not correct. 1 % chos e this .
Faintness is a ha llmark of syncope .
20
Syncope (medicine) Syncope (phonology)
21
22
D is not correct. 2% chos e this .
Dehydration, as well as increased physiologic demands placed on the body thr ough running, would increase the li kelihood of
23
syncope .
24 Syncope (medicine) Dehydration Syncope (phonology)
25
E is not correct. 50/o chose th is.
26
A progressive onset is consistent with syncope, in which patients often feel progr essively light-headed .
27 Syncope (phonology) Syncope (medicine) Progressive rock
28
29
30
Botto m line :
31 Patients with syncope typical ly immediately return to normal mentation . When the patient continues to be confused in the
postictal state, a seizure is more likely.
32 Postictal state Syncope (medicine) Epileptic seizure
33
34
35 References:
36 FA Step 2 CK 9th ed pp 268-271
FA Step 2 CK 8th ed p 255
37

Lock
s
Suspend
0
End Block
Item: 38 of 99 ~ 1 • Mark <:] (:>- Jill ~· ~J
QID: 22185 ..1 Previous Next Lab'V!I!ues Notes Calculator

16 A 40-year-old woman presenting with sensorineu ral hearing loss on ly for low-frequency sounds in the right ear confirmed
by audiometry, severa l episodes of vertigo lasting 20-30 m inutes, tinnitus, and associated nausea was diagnosed with
17
Meniere disease. To manage her symptoms, she was put on dietary restrictions . She has maintained a daily intake of
18 sodium of < 1000 mg for the past 6 weeks. She does not d rink or smoke tobacco. However, her symptoms of vertigo, t inn itus,
19 and associated nausea have not been alleviated .
20
21 Which of the following is the best next step in management?
22
23 A. Intracochlear gentamicin injection
24
B. Labyrinthectomy
25
C. Metoclopramide
26
27 D. Sacculotomy
28 E. Tria l of diuretics
29
30
31
32
33
34
35
36
37
• 38

Lock
s
Suspend
0
End Block
Item: 38 of 99 ~ 1 • Mark <:] (:>- Jill ~· ~J
QID: 22185 ..1 Previous Next Lab'V!I!ues Notes Calculator

16
17
18
The correct answer is E. 64% chose this.
Meniere disease is believed to be derived from an abnorma l homeostasis of inner ear f luids, with excess fluid build-up in
19
endolymphatic spaces. The most common clin ical features include sensorineural hearing loss, vertigo, tinnitus, and pressure
20 sensation . In a patient w ith newly diagnosed Meniere disease, the fi rst step in management is dietary restriction on the use
21 of sa lt, ca ffeine, and tobacco. If this does not provide symptomatic relief, the next step is to try pharmacotherapy w ith
22 diuretics before more invasive procedu re s are performed . More specifically, hydrochlorothiazide with or without ~-histin e is
typically the first- line agent in management of Meniere.
23
Tinnitus Vertigo Caffeine Hydrochlorothiazide Sensorineural hearing loss Homeostasis Diuretic Inner ear Edema Hearing loss Pharmacotherapy Endolymph Tobacco
24
A is not correct. 6% chose this.
25
Am inoglycosides are ototoxic; therefore, admin istration to the ear permanently destroys hair cells and decreases labyrinthine
26
function without adverse systemic effects. This is undertaken only when first- line agents fail and the degree of sensorineu ral
27 hearing loss is great.
28 Ototoxicity Sensorineural hearing loss Aminoglycoside Hearing loss Hair cell

29 B is not correct. 120/o chose this.


30 Because the procedure causes permanent hearing loss in patients, it is only performed for patients with symptoms not
31 responsive to conventional treatment and who already have poor or complete hearing loss in the affected ear.
Hearing loss
32
33 Cis not correct. 110/o chose this.
34 Metoclopramide is not indicated for the treatment of Men iere disease . Metoclopramide is used primarily to treat nausea and
35 vomiting, and to facil itate gastric emptying in patients with gastroparesis. It is also commonly used to treat migraine
headaches.
36
Metodopramide Gastroparesis Migraine Nausea Vomiting
37
38
D is not correct. 7% chose this.

Lock
s
Suspend
0
End Block
Item: 38 of 99 ~ 1 • Mark <:] (:>- Jill ~· ~J
QID: 22185 ..1 Previous Next Lab'V!I!ues Notes Calculator

• OtotOXICity Sensonneural heanng loss AmmoglycosJde Heanng loss Ha.r cell


16
B is not correct. 120/o chose this.
17
Because the procedure causes permanent hearing loss in patients1 it is only performed for patients with symptoms not
18 responsive to conventiona l treatment and who already have poor or complete hearing loss in the affected ear.
19 Hearing loss

20
Cis not correct. 110/o chose this.
21
Metoclopramide is not indicated for the treatment of Men iere disease . Metoclopramide is used primarily to treat nausea and
22 vomiting 1 and to facil itate gastric emptying in patients with gastroparesis. It is also commonly used to treat migraine
23 headaches.
Metoclopramide Gastroparesis Migraine Nausea Vomiting
24
25 D is not correct. 7% chose this.
26 A sacculotomy is surgery involving implantation of a permanent1 tack- like device that allows endolymph to dra in when
27
pressu re builds up. Th is procedu re works well in patients with little or no sensorineura l hearing loss . Although it is common ly
performed 1 a trial of pha rmacotherapy shou ld be undertaken first.
28 Endolymph Sensorineural hearing loss Hearing loss
29
30
31
Bottom line:
32 If conservative management of Men iere disease with lifestyle mod ifications fails1 the next step is to try pharmacotherapy
w ith diuretics .
33 Pharmacotherapy
34
35
36 Refere n ces:
37 FA Step 2 CK 9th ed pp 272- 273
FA Step 2 CK 8th ed pp 259- 260
38

Lock
s
Suspend
0
End Block
Item: 39 of 99 ~ 1 • Mark <:] (:>- Jill ~· ~J
QID: 22403 ..1 Previous Next Lab'V!I!ues Notes Calculator
.
17 A 40-year-old caucasian man presents w ith sudden-onset dance-like movements and rapidly declining cognit ive abil ity. He
also has had some personality changes, being more "hot-tempered" than usual. Family history is significant for another
18
brother diagnosed with a genetic disorder when he presented with the same symptoms. His other two brothers have
19 obtained genetic testing which did not show any mutation of the implicated gene. The patient's partner denies any family history
20 of genetic disorder s. His son asks what his odds of developing sim ilar symptoms ar e.
21
22
What is the son's probability of developing this disorder ?
23
24
25
A. O%
26 B. 12.5%
27
c. 25%
28
29
0 . 50%
30 E. 100%
31
32
33
34
35
36
37
38
• 39

Lock
s
Suspend
0
End Block
Item: 39 of 99 ~ 1 • Mark <:] (:>- Jill ~· ~J
QID: 22403 ..1 Previous Next Lab'V!I!ues Notes Calculator
.
17
18
19
The correct answer is D. 770/o chose this.
20
The signs and symptoms described by this patient are consistent w ith Huntington disease, a rapid-onset dementia of m iddle
21 age associated w ith chorea (dance- like) movements. The disease is inherited in an autosoma l dom inant manner. Two
22 (including the patient) out of four brothers developed the disease; therefor e, the patient is most li kely to have heterozygous
23 disease . Since the mother's genotype is most likely to be normal, the probabi lity of the son inheriting the disease will be 1 in
2, or SO%. Life expectancy from the time of diagnosis is approximately 20 years and there is no treatment to cure or halt the
24
disease . Treatment is for symptoms only and can involve ha loperidol for psychosis and res erpine to minim ize unwanted
25 movements.
26 Reserpine Haloperidol Huntington's disease Heterozygous Dominance (genetics) Chorea Psychosis Genotype Dementia life expectancy Autosome

27 A is not correct. 5% chose this.


28 A zero percent chance would not be expected for any type of disease unless it is gender-specific.
29 B is not correct. 20/o chose this.
30 A one in eight ( 12.5%) chance is not characteristic of Huntington disease, which is inherited in an autosomal dominant
31 pattern .
Huntington's disease Dominance (genetics) Autosome
32
33 C is not correct. 7% chose this.
34 A one in four (25%) chance wou ld be consistent with autosoma l recessive inheritance patterns, in which both parents are
35 carriers of the disease .
Autosomal recessive Recessive Autosome Dominance (genetics)
36
37 E is not correct. 90/o chose this.
38 A 100% chance of inheritance would only be expected in autosomal dominant diseases if one parent has two "bad" copies of
the gene.
39 y

Lock
s
Suspend
0
End Block
Item: 39 of 99 ~ 1 • Mark <:] (:>- Jill ~· ~J
QID: 22403 ..1 Previous Next Lab'V!I!ues Notes Calculator
. - -- -- - -- - - - -- -- - - - - - - - - - - - - -- -

17 A is not correct. 5 % chos e this .


18 A zero percent chance would not be expected for any type of disease unless it is gender·specific.
19 B is not correct. 20/o chose this .
20 A one in eight ( 12.5%) chance is not character istic of Huntington disease, which is inherited in an autosomal dominant
21 pattern .
Huntington's disease Dominance (genetics) Autosome
22
23 C is not correct. 7 % chos e this .
24 A one in four (25%) chance would be consistent with autosomal recessive inheritance patterns, in which both parents are
carriers of the disease .
25
Autosomal recessive Recessive Autosome Dominance (genetics)
26
27
E is not correct. 9 0/o c hose th is.

28
A 100% chance of inher itance would only be expected in autosomal dom inant diseases if one parent has two "bad" copies of
the gene.
29 Gene Dominance (genetics) Autosome
30
31
32
Bottom line :
33 Huntington disease is inherited in an autosomal dominant manner. Assuming one parent is genotypically normal, the odds of
a ch ild inheriting the disease from a heterozygous parent is 1 in 2, or 50% .
34 Huntington's disease Dominance (genetics) Heterozygous Autosome Genotype
35
36
37 References:
38 FA Step 2 CK 9th ed p 282
FA Step 2 CK 8th ed pp 268-269
39

Lock
s
Suspend
0
End Block
Item: 40 of 99 ~ 1 • Mark <:] (:>- Jill ~· ~J
QID: 22418 ..1 Previous Next Lab'V!I!ues Notes Calculator

18 A 32-year-old female presents with a 2-month history of progressive tingling sensation and numbness in her feet. She also ~~AI
reports difficulty with balance and falling more frequently for the past month. She also mentions that at work "I feel slow,
19
and I 'm not as efficient at doing tasks as I used to be ." Her past medical history is significant for a Roux-en-Y gastric
20 bypass being done 1 year ago due to morbid obesity, and she admits she hasn't been following up with her physicians. Her
21 temperature is 37°C (98 .6°F), heart rate is 88/min, blood pressure is 118/80 mm Hg, respiratory rate is 12/min, and BMI is 24
22
kg/m 2. She is alert and oriented to time, person, and place . Her cardiorespiratory examination is normal, and her abdom inal
examination reveals striae . Neurologic examination revea ls symmetric loss of vibration and proprioception in both lower
23
extremities along with a positive Romberg sign; lower limb reflexes are brisk at 3+. Her complete metabolic panel was w ithin
24 normal lim its. A complete blood cell count was done and showed a hemoglobin of 10, and a smear is shown below.
25
26
27
28
29
30
Which of the following is likely deficient in this patient?
31
32 A. Folate
33 B. Vitamin B12
34
C. Vitamin B1
35
36
D. Vitamin B6
37 E. Vitamin D
38
39
• 40

Lock
s
Suspend
0
End Block
Item: 40 of 99 ~ 1 • Mark <:] (:>- Jill ~· ~J
QID: 22418 ..1 Previous Next Lab'V!I!ues Notes Calculator

18 A 32-year-old female presents with a 2-month history of progressive tingling sensation and numbness in her feet. She also ~~AI
reports difficulty with balance and falling more frequently for the past month. She also mentions that at work "I feel slow,
19
and I 'm not as efficient at doing tasks as I used to be ." Her past medical history is significant for a Roux-en-Y gastric
20 bypass being done 1 year ago due to morbid obesity, and she admits she hasn't been following up with her physicians. Her
21 temperature is 37°C (98 .6°F), heart rate is 88/min, blood pressure is 118/80 mm Hg, respiratory rate is 12/min, and BMI is 24
22
kg/m 2 . She is alert and oriented to time, person, and place . Her cardiorespiratory examination is normal, and her abdom inal
examination reveals striae . Neurologic examination revea ls symmetric loss of vibration and proprioception in both lower
23
extremities along with a positive Romberg sign; lower limb reflexes are brisk at 3+. Her complete metabolic panel was w ithin
24 normal lim its. A complete blood cell count was done and showed a hemoglobin of 10, and a smear is shown below.
25
26
27
28
29
30
31
32
33
34
35
36
37
38
39
• 40

Lock
s
Suspend
0
End Block
18
19
20
21
22
23
24
25
26
27
28
29
30
31 Which of the following is likely deficient in this patient?
32
33 A. Folate
34 B . Vitamin B12
35
C. Vitamin B1
36
37 D . Vitamin B6
38 E. Vitamin D
39
• 40 •

a
Lock
s
Suspend
8
End Block
18
19
20
21
22
23
24
25
26
27
28
29
30
31 Which of the following is likely deficient in this patient?
32
33 A. Folate
34 B . Vitamin B12
35
C. Vitamin B1
36
37 D . Vitamin B6
38 E. Vitamin D
39
• 40 •

a
Lock
s
Suspend
8
End Block
Item: 40 of 99 ~ 1 • Mark <:] (:>- Jill ~· ~J
QID: 22418 ..1 Previous Next Lab'V!I!ues Notes Calculator

18 The correct a nswer i s B. 920/o chose t h is.


19 This patient likely has vitamin 6 12 deficiency secondary to the Roux-en-Y gastric bypass . Similar to folate deficiency, patients
20 w ith 6 1 2 deficiency may present w ith anem ia, stomatitis, and cheilitis . However, unique to 6 1 2 deficiency are neurologic
21 manifestations. Subacute combined degeneration, secondary to myelin dysfunction, impacts the dorsal columns and lateral
tracts of the spinal cord. Symptoms begin with paresthesias and ataxia, and can progress to severe weakness, spasticity,
22
clonus, and incontinence. Patients may also show signs of dementia. Treatment is with large-dose vitamin 6 12 injections.
23 Vitamin 6 1 2 is found only in animal products, and deficiency is usually due to malabsorption, lack or intrinsic factor (as with
24 pern icious anemia or gastrectomy), or absence of the terminal ileum, the site of vitamin 6 12 absorption .
Pernicious anemia Intrinsic factor Ileum Myelin Gastrectomy Spasticity Folate deficiency Folic acid Vitamin 812 Ataxia Malabsorption Paresthesia 8 vitamins Anemia
25
Fecal incontinence Dementia Spinal cord Urinary incontinence Vitamin Subacute combined degeneration of spinal cord Posterior column Neurology Stomatitis Gastric bypass surgery
26
Neurological disorder Roux-en-Y gastric bypass Anatomical terms of location
27
28 A i s not correct. 4 % ch ose this.
29 Folate deficiency can present with altered mental status and megaloblastic, macrocytic anemia as in vitamin 6 12 deficiency,
30 but not with the focal neurologic deficits. Folate is found in green leafy vegetables, and it is the most common vitamin
deficiency in the United States. Supplemental folic acid is especially important in early pregnancy to prevent neural tube
31
defects. Folic acid deficiency, although common in gastric bypass, is less common than 6 1 2 deficiency, as it is absorbed
32 throughout the small bowel.
33 Folate deficiency Folic acid Macrocytic anemia Vitamin 812 Anemia Neural tube defect 8 vitamins Vitamin Small intestine Avitaminosis Gastric bypass surgery Leaf vegetable

34 Neurology Pregnancy Macrocytosis Neural tube

35 C i s not corr ect. 1 0/o ch ose t his.


36 Vitamin 6 1 , or thiamine, deficiency manifests with beriberi (dry and wet) or Wern icke-Korsakoff syndrome . Dry beriberi is
37 characterized by a symmetrical polyneuropathy affecting both sensory and motor pathways. Wet beriberi includes neuropathy
38 w ith signs of cardiac damage (cardiomegaly, congestive heart failure, peripheral edema, tachycardia) . Wernicke- Korsakoff
syndrome is characterized by the Wernicke triad of nystagmus, ophthalmoplegia, and ataxia with Korsakoff short-term
39
memory dysfunction and confabu lation . Patients on total parenteral nutrition (TPN) and chronic alcoholics have a high risk of
40 y riPvPioninn R, riPfir:iPnr:v.

Lock
s
Suspend
0
End Block
Item: 40 of 99 ~ 1 • Mark <:] (:>- Jill ~· ~J
QID: 22418 ..1 Previous Next Lab'V!I!ues Notes Calculator

CIS not correct. 1 0/o ch ose th1s.


18
Vitamin 6 1 , or thiamine, deficiency manifests with beriberi (dry and wet) or Wern icke- Korsakoff syndrome . Dry beriberi is
19 characterized by a symmetrical polyneuropathy affecting both sensory and motor pathways. Wet beriberi includes neuropathy
20 with signs of cardiac damage (cardiomegaly, congestive heart failure, peripheral edema, tachycardia) . Wernicke- Korsakoff
21
syndrome is characterized by the Wernicke triad of nystagmus, ophthalmoplegia, and ataxia with Korsakoff short-term
memory dysfunction and confabu lation . Patients on total parenteral nutrition {TPN) and chronic alcoholics have a high risk of
22
developing 6 1 deficiency.
23 Wemicke-Korsakoff syndrome Beriberi Parenteral nutrition Nystagmus Cardiomegaly Thiamine Heart failure Confabulation Congestive heart failure Tachycardia Ophthalmoparesis

24 Ataxia Peripheral neuropathy B vitamins Vitamin Polyneuropathy Vitamin 812 Peripheral edema Edema Alcoholism Nutrition Parenteral Short-term memory

25
D is not correct. 2% chose this.
26
Vitamin 6 6 is found in many foods, including meats, whole grains, nuts, and vegetables. Deficiency may manifest as
27 stomatitis, glossitis, cheilitis, irritability, confusion, and depression . 6 6 deficiency may be induced by ison iazid, oral
28 contraceptives, alcohol, and pregnancy. Low levels are also seen in diabetes, asthma, heart disease, breast cancer, and
29
Hodgkin lymphoma .
Isoniazid Asthma Glossitis Vitamin B12 Diabetes mellitus B vitamins Vitamin Lymphoma Breast cancer Major depressive disorder Cancer Depression (mood) Cardiovascular disease
30
Alcohol Oral contraceptive pill Pregnancy Hodgkin's lymphoma Stomatitis Irritability Combined oral contraceptive pill
31
32 E i s n ot correct. 1 0/o chose t his.
33 Patients with fat malabsorption or decreased sun exposure may develop vitamin D deficiency. Deficiency can result in rickets
in children {bending of the bones), osteomalacia in adults (weak bones), and hypocalcemic tetany. Roux-en-Y can lead to
34
v itam in D malabsorption but it would not lead to the cli ni cal picture described. Vitamin D deficiency has been shown to
35 correlate with an increased risk of multiple sclerosis {MS) in certain HLA subtypes; however, this progressive picture does not
36 go with MS, and vitamin D deficiency wou ld not cause megaloblastic anemia .
Megaloblastic anemia Multiple sclerosis Osteomalacia Rickets Vitamin D Tetany Malabsorption Anemia Hypovitaminosis D Hypocalcaemia Vitamin Human leukocyte antigen Fat
37
Osteoporosis
38
39
40 Rnttnm I i nA: I.
Lock
s
Suspend
0
End Block
Item: 40 of 99 ~ 1 • Mark <:] (:>- Jill ~· ~J
QID: 22418 ..1 Previous Next Lab'V!I!ues Notes Calculator

V1tam 1n 8 6 IS found 1n many foods, mcludmg meats, whole grams, nuts, and vegetables. Def1c1ency may man 1fest as
18 stomatit is, glossitis, cheilit is, irritability, confusion, and depression . 8 6 deficiency may be induced by ison iazid, oral
19 contraceptives, alcohol, and pregnancy. Low levels are also seen in diabetes, asthma, heart disease, breast cancer, and
Hodgkin lymphoma .
20
Isoniazid Asthma Glossitis Vitamin 812 Diabetes mellitus B vitamins Vitamin Lymphoma Breast cancer Major depressive disorder Cancer Depression (mood) Cardiovascular disease
21
Alcohol Oral contraceptive pill Pregnancy Hodgkin's lymphoma Stomatitis Irritability Combined oral contraceptive pill
22
23 E is not correct. 1 0/o chose th is.
24 Patients with fat malabsorption or decreased sun exposure may develop vitam in D deficiency. Deficiency can result in rickets
in children (bending of the bones), osteomalacia in adu lts ( weak bones), and hypocalcemic tetany. Roux-en-Y can lead to
25
v itam in D malabsorption but it would not lead to the cli ni cal picture described. Vitamin D deficiency has been shown to
26 correlate with an increased risk of multiple sclerosis ( MS) in certain HLA subtypes; however, th is progressive picture does not
27 go with MS, and vitam in D deficiency wou ld not cause megaloblastic anem ia.
Megaloblastic anemia Multiple sclerosis Osteomalacia Rickets Vitamin D Tetany ~1alabsorption Anemia Hypovitaminosis D Hypocalcaemia Vitamin Human leukocyte antigen Fat
28
Osteoporosis
29
30
31 Bottom line :
32 Patients with vitam in 8 12 deficiency can develop subacute combined degeneration, which is characterized by paresthesias,
33 weakness, spasticity, confusion, and incontinence . Neurologic symptoms separate 8 12 deficiency from folate deficiency.
34 Nutrient deficiency is a common complication of gastric bypass and shou ld be monitored and treated .
Spasticity Folic acid Vitamin 812 Folate deficiency B vitamins Paresthesia Vitamin Fecal incontinence Urinary incontinence Subacute combined degeneration of spinal cord
35 Gastric bypass surgery Nutrient Acute (medicine) Neurology
36
37
38 References:
39 FA Step 2 CK 9th ed p 291
FA Step 2 CK 8th ed p 276
40

Lock
s
Suspend
0
End Block
Item: 41 of 99 ~ 1 • Mark <:] (:>- Jill ~· ~J
QID: 23909 ..1 Previous Next Lab'V!I!ues Notes Calculator
.
19 A 53-year-old man with an IQ of 60 is evaluated for progressive memory loss . His mother states that although her son ~~AI
used to be quite independent, over the past 2 years he has gotten lost walking in his neighborhood and has difficulty
20
performing his job stocking shelves at the grocery store. She has had to increasingly assist him with his activities of daily
21 living . On physical examination the patient exhibits large epicanthal folds, hyperflexible joints, and a II/VI systolic murmur.
22
23 Which of the following is the most likely cause of this patient's cognitive changes?
24
25 A. Alzheimer's disease
26
B. HIV encephalopathy
27
28 C. Normal-pressure hydrocephalus
29 D. Progressive supranuclear palsy
30
E. Vascular dementia
31
32
33
34
35
36
37
38
39
40
• 41

Lock
s
Suspend
0
End Block
Item: 41 of 99 ~ 1 • Mark <:] (:>- Jill ~· ~J
QID: 23909 ..1 Previous Next Lab'V!I!ues Notes Calculator
.
19
20 The correct answer is A. 780/o chose this.
21 This patient's dementia is characteristic of Alzheimer's disease . His IQ and physical examination findings are consistent with
Down 's syndrome . Adu lts with Down's syndrome have a higher prevalence of Alzheimer's disease than the genera l
22
popu lation . The onset of the disease is also earlier than in patients without the syndrome . It is believed that mutations on a
23 gene found on chromosome 21, the APP gene, plays a role in some cases of the dementia.
24 Down syndrome Alzheimer's disease Gene Dementia Chromosome 21 (human) Chromosome Intelligence quotient Physical examination Mutation

25
B is not correct. 20/o chose this.
26 HIV encephalopathy is seen in patients with AIDS. Patients display psychomotor retardation, apathy, and memory loss. It is
27 important to distinguish whether these manifestations ar e due to the HIV virus itself or are due to opportunistic infections.
HIV Psychomotor retardation HIV-associated neurocognitive disorder HIV/AIDS Encephalopathy Opportunistic infection Virus Psychomotor teaming Amnesia Apathy
28
29 C is not correct. 40/o chose this.
30 Normal-pressure hydrocephalus is characterized by abnormal ga it, urinary incontinence, and mild or moderate dementia .
31 These patients have a communicating hydrocepha lus that results in enlarged ventricles but normal cerebrospina l fluid
pressure on lum bar puncture .
32
lumbar puncture Hydrocephalus Cerebrospinal fluid Urinary incontinence Dementia Ventricular system lumbar Fecal incontinence Ventricle (heart) Gait (human) Gait
33
lumbar vertebrae
34
35 D is not correct. 5% chose this.
36
Progressive supranuclear palsy is a degenerative disorder characterized by a loss of voluntary eye movements, slow unsteady
gait, and rigid posture. Because the patient's gaze is fixe·d upwards, he or she is at great risk for fall injuries . Cognitive
37
decline includes loss of executive function, impaired fluency, and slowed thought processes.
38 Progressive supranuclear palsy Gait (human) Degenerative disease Executive functions Eye movement Gait Dementia

39
E is not correct. 11% chose this.
40
This form of dementia results from progressive cerebrovascular damage from hypertension and atherosclerosis . Patients
41 y usuallv have a historv of r ecurrent strokes. and svmotoms mav include oersonalitv chances and emotional labilitv in addition

Lock
s
Suspend
0
End Block
Item: 41 of 99 ~ 1 • Mark <:] (:>- Jill ~· ~J
QID: 23909 ..1 Previous Next Lab'V!I!ues Notes Calculator
. C is not correct. 4% chose this.
19
Normal-pressure hydrocephalus is characterized by abnormal ga it, urinary incontinence, and mild or moderate dementia.
20 These patients have a communicating hydrocepha lus that results in enlarged ventricles but normal cerebrospina l fluid
21 pressure on lumbar puncture .
lumbar puncture Hydrocephalus Cerebrospinal fluid Urinary incontinence Dementia Ventricular system lumbar Fecal incontinence Ventricle (heart) Gait (human) Gait
22
lumbar vertebrae
23
24 D is not correct. 5% chose this.
25 Progressive supranuclear palsy is a degenerative disorder characterized by a loss of voluntary eye movements, slow unsteady
26 gait, and rigid posture. Because the patient's gaze is fixe·d upwards, he or she is at great risk for fall injuries . Cognitive
decline includes loss of executive function, impaired fluency, and slowed thought processes.
27
Progressive supranuclear palsy Gait (human) Degenerative disease Executive functions Eye movement Gait Dementia
28
29
E is not correct. 11% chose this.

30
This form of dementia results from progressive cerebrovascular damage from hypertension and atherosclerosis. Patients
usually have a history of recurrent strokes, and symptoms may include personality changes and emotional lability in addition
31 to the cognitive deterioration .
32 Atherosclerosis Pseudobulbar affect Hypertension Dementia Cerebrovascular disease

33
34
Bottom line:
35
Adults with Down's syndrome have a higher prevalence of Alzheimer's disease than the general population .
36 Alzheimer's disease Down syndrome
37
38
39 References:
40 FA Step 2 CK 9th ed pp 277- 279
FA Step 2 CK 8th ed pp 265- 266
41

Lock
s
Suspend
0
End Block
Item: 42 of 99 ~ 1 • Mark <:] (:>- Jill ~· ~J
QID: 23601 ..1 Previous Next Lab'V!I!ues Notes Calculator

20 A 6-year-old girl has generalized tonic-clonic seizures. She has failed three reg imens of monotherapy for her condition. ~~AI
The patient currently takes valproic acid, and her neurologist wants to add a second agent to achieve better control of the
21
girl's condition.
22
23
Which of the following has the least sign ificant drug interactions with valproic acid?
24

25
A. Carbamazepine
26
27
B. Felbamate

28 C. Lamotrigine
29
D. Levetiracetam
30
31
E. Phenytoin
32
33
34
35
36
37
38
39
40
41
• 42

Lock
s
Suspend
0
End Block
Item: 42 of 99 ~ 1 • Mark <:] (:>- Jill ~· ~J
QID: 23601 ..1 Previous Next Lab'V!I!ues Notes Calculator

20 The correct answer is D. 450/o chose this.
21 The mechan ism for levetiracetam is unknown but bel ieved to play a role in synaptic vesicle release and GABA modulation . It
22 is primarily used as an adjunct for focal seizures, primary genera lized ton ic-clon ic seizures, and myoclonic seizures.
Levetiracetam is not metabolized by the hepatic CYP450 system and is mostly excreted by the kidneys . For this reason, it has
23
very few drug interactions and may be paired with valproate.
24 Synaptic vesicle Levetiracetam Gamma-Aminobutyric acid Partial seizure Valproate Cytochrome P450 Myoclonus Generalised tonic-clonic seizure Epileptic seizure
25 Vesicle (biology and chemistry) Synapse Kidney Liver Drug interaction Metabolism Myoclonic epilepsy Seizure types
26
A is not correct. 100/o chose this.
27
Carbamazepine inh ibits voltage-dependent sodium channels and is used for focal and genera lized seizures . It is mostly
28
protein-bound and metabolized by the CYP450 system. I t is a potent inducer of the CYP system so it has many drug
29 interactions. The risk of carbamazepine toxicity increases when paired with va lproate, as it increases the level of active
30 metabolite of carbamazepine, which warrants close monitoring . Side effects of carbamazepine might include Steven-Johnson
syndrome, aplastic anem ia, and hyponatremia .
31
Carbamazepine Hyponatremia Cytochrome P450 Valproate Metabolite Sodium channel Anemia Active metabolite Epileptic seizure Sodium Protein Drug metabolism Metabolism
32
Adverse drug reaction Toxicity Side effect
33
34 B is not correct. 120/o chose this.
35
Felbamate plays a role in inhibiting NMDA receptors and augmenting GABA signals. With associated side effects of aplastic
anemia and hepatic failure, it is only used for Lennox-Gastaut syndrome. It is known to increase the toxicity of valproic acid
36
and phenytoin .
37 Felbamate Valproate Phenytoin Aplastic anemia Gamma-Aminobutyric acid Anemia Liver failure NMDA receptor Liver Toxicity

38
Cis not correct. 210/o chose this.
39
Lamotrigine is believed to inhibit voltage-gated sodium channels specific to neurons that synthesize glutamate and aspartate.
40 It is commonly used as an adjunct for foca l seizures, genera lized tonic-clonic seizures, and Lennox-Gastaut syndrome. It is
41 primarily excreted by hepatic glucuronidation, wh ich is inhibited by va lproate so levels are increased when these medications
42
are paired . Thus close monitoring is mandatory when a combination of these medications is prescribed.
y

Lock
s
Suspend
0
End Block
Item: 42 of 99 ~ 1 • Mark <:] (:>- Jill ~· ~J
QID: 23601 ..1 Previous Next Lab'V!I!ues Notes Calculator
• •
20 Lamotrigine is believed to inhibit voltage-gated sodium channels specific to neurons that synthesize glutamate and aspartate.
21 It is commonly used as an adjunct for focal seizures, generalized tonic-clonic seizures, and Lennox-Gastaut syndrome. It is
primarily excreted by hepatic glucuronidation, which is in hibited by valproate so levels ar e increased when these medications
22
are paired . Thus close monitoring is mandatory when a combination of these medications is prescribed.
23 lennox-Gastaut syndrome lamotrigine Glucuronidation Partial seizure Valproate Glutamic acid Sodium channel Aspartic acid Epileptic seizure Neuron Generalised tonic-clonic seizure
24 Liver Sodium Seizure types
25
E is not correct. 12% chose this.
26
Phenytoin acts by inhibit ing voltage-gated dependent sodium channels. It is commonly used for focal and generalized
27
seizures and status epilepticus . It is largely metabolized by the liver ( glucuron idation ) and is a potent inducer of the CYP450
28 enzymes. Valproic acid competes with phenytoin for binding sites on plasma proteins and also inhibits the metabolism of
29 phenytoin by hepatic enzymes. These interactions can re·sult in toxic levels of phenytoin, so the use of va lpr oic acid is a
relative contraindication to the use of phenytoin. If the t w o drugs must be used together, plasma phenytoin levels must be
30
closely followed .
31 Valproate Phenytoin Status epilepticus Glucuronidation Cvtochrome P450 Contraindication Sodium channel Blood plasma Epileptic seizure Metabolism Liver Sodium Enzyme
32 Drug metabolism
33
34
35
Bottom li ne:
36 Valproic acid is t ightly protein- bound and inhibits CYP enzymes as well as glucur onidation, which leads to numerous drug-
drug interactions. Levetiracetam , with low levels of hepatic metabolism, can be paired w ith valproic acid for management of
37
generalized tonic-clonic seizur es.
38 levetiracetam Valproate Epileptic seizure Protein Metabolism Generalised tonic-clonic seizure Drug interaction Enzyme Liver Cytochrome P450 Seizure types

39
40
41 References:
FA Step 2 CK 8th ed pp 483-485
42

Lock
s
Suspend
0
End Block
Item: 43 of 99 ~ 1 • Mark <:] (:>- Jill ~· ~J
QID: 24100 ..1 Previous Next Lab'V!I!ues Notes Calculator
I

21 A 69-year-old Asian man complains of extreme eye pa in and blurry v ision shortly after phenylephrine was dropped into his ~ ~A I
right eye. The eye is hard and t he pupil is dilated and non reactive to light.
22
23
Which of the following is the most likely diagnosis?
24
25
A. Ang le-closure glaucoma
26
27 B. Open-angle glaucoma
28 C. Retinal detachment
29
D. Rupt ured globe
30
31 E. Strabismus
32
33
34
35
36
37
38
39
40
41

42
• 43

Lock
s
Suspend
0
End Block
Item: 43 of 99 ~ 1 • Mark <:] (:>- Jill ~· ~J
QID: 24100 ..1 Previous Next Lab'V!I!ues Notes Calculator
I
The correct answer IS A. 8 30/o chose th1s.
21
This is a classic presentation for ang le-closure glaucoma . It is a medical emergency that can be precipitat ed by papillary
22 dilation with drugs such as phenylephrine. Risk factors ar e older age and Asian descent. Signs and symptoms include a hard
23 eyeball , blurry vision, ext reme eye pain, and elevated intraocular pressure. Th is patient should be treated immediately w ith
acetazolamide to decrease intraocula r pressure .
24
Phenylephrine Glaucoma Acetazolamide Intraocular pressure Human eye Vasodilation
25
26
B is not correct. 1 30/o chose th is.
27
Open-angle glaucoma is the most common form of glaucoma . It is a chronic condit ion that develops slowly over time . It
typically leads to decreased periphera l v ision rather t han blurry vision, and it is pa inless.
28 Glaucoma Peripheral vision Blurred vision Chronic condition
29
C is not correct. 2 % chose this.
30
Retinal detachment is a common ophthalmic emergency t hat occurs most often after t rauma . It is not associat ed with dilating
31
eyedrops. Patients may experience flashes, floaters, and vision loss.
32 Retinal detachment Visual impairment Ophthalmology Retinal Floater

33
Dis not correct. 1 % chose this.
34
Rupt ured globe is a common ophthalm ic emergency in which the contents of the vitreous humor are exposed. It typically
35 follows trauma to the eye and can lead to sign ificant v ision loss if not promptly repaired surgically.
36 Vitreous body Visual impairment Ophthalmology Major trauma

37 E is not correct. 1 0/o c hose th is.


38 Strabismus is a condition in wh ich the eyes are not fused together (cross-eye) . It is most often seen in children and can lead
39 to significant vision loss if not corrected .
Strabismus Esotropia Visual impairment
40
41

42 Bottom line :
43

Lock
s
Suspend
0
End Block
Item: 43 of 99 ~ 1 • Mark <:] (:>- Jill ~· ~J
QID: 24100 ..1 Previous Next Lab'V!I!ues Notes Calculator
I
• typ1cally leads to decreased penpheral v 1s1on rather t han blurry v1s1on, and 1t IS pamless.
21 Glaucoma Peripheral vision Blurred vision Chronic condition

22
C is not correct. 2 % chose this.
23 Retinal detachment is a common ophthalm ic emer gency that occurs most often after t rauma . It is not associat ed with dilating
24 eyedrops. Patients may experience flashes, floaters, and vision loss.
Retinal detachment Visual impairment Ophthalmology Retinal Floater
25
26 Dis not correct. 1 % chose this.
27 Ruptur ed globe is a common ophthalmic emergency in w hich the contents of the vit reous humor are exposed . It typically
28 follows t rauma to the eye and can lead to sign ificant v ision loss if not promptly r epaired sur gica lly.
Vitreous body Visual impairment Ophthalmology ~1ajor trauma
29
30 E is not correct. 1 0/o chose th is.
31 Strabismus is a condit ion in which the eyes are not fused together (cross-eye) . It is most often seen in children and can lead
32
to significant vision loss if not corrected .
Strabismus Esotropia Visual impairment
33
34
35 Bottom Line :
36 Ang le-closure glaucoma is a med ical emergency that can be precipitated by papillary dilation with drugs such as
37 phenylephrine . It presents w ith a hard eyeball, blurry vi sion , extreme eye pain, and elevated intraocular pr essure. Treat
immediately wit h acetazolamide to decrease intraocu lar pressur e.
38 Phenylephrine Acetazolamide Glaucoma Intraocular pressure Human eye Vasodilation
39
40
41 References:
42 FA Step 2 CK 9th ed pp 291-292
FA Step 2 CK 8th ed pp 277-278
43

Lock
s
Suspend
0
End Block
Item: 44 of 99 ~ 1 • Mark <:] (:>- Jill ~· ~J
QID: 22182 ..1 Previous Next Lab'V!I!ues Notes Calculator

22 A 79-year-old white man with a 6-month history of Parki nson disease is brought to the clinic by his wife, who says her
husband has become more forgetfu l over the past 4 months. He gets lost in fami liar places and is no longer capable of
23
balancing his checkbook. I n addition, he sleeps more frequently during the daytime and stares into space for long periods.
24 He cannot express his ideas in an organized and flowing manner, and he has had episodes of visual hallucinations . Although his
25 cognitive symptoms wax and wane in weekly to monthly phases, they have progressively worsened over the past several
26
months. His dose of levodopajcarbidopa has not changed since beginn ing therapy upon diagnosis. His neurologic examination is
unchanged over the past 6 months . His score on the Mini-Mental State Examination is 20.
27
28
Which of the following is the most likely diagnosis?
29
30
A. Alzheimer disease
31
32 B. Delirium
33 C. Dementia with Lewy bodies
34
D. Depression
35
36 E. Vascular dementia
37
38
39
40
41
42
43
• 44

Lock
s
Suspend
0
End Block
Item: 44 of 99 ~ 1 • Mark <:] (:>- Jill ~· ~J
QID: 22182 ..1 Previous Next Lab'V!I!ues Notes Calculator

22
The correct answer is C. 750/o chose this.
Dementia with Lewy bod ies is classica lly seen in patients with Parkinson disease or parkinsonian featu res. The dementia is
23
progressive, presenting in it ially with episodes of waxing and waning of cognitive function, and is set apart from other
24 dementia syndromes by the following key symptoms: increased daytime drowsiness and sleep, episodes of staring into space,
25 disorganized flow of ideas, and visua l hallucinations. Note that in dementia with Lewy bodies, dementia comes on before or
26 w ithin 1 year of the onset of parkinsonian symptoms. Dementia in long-term Parkinson patients is called Parkinson disease
dementia. Nonpharmacologic treatment including behav iora l therapy is preferred over medications. If medications are
27
warranted , cholinesterase inh ibitors for cognitive symptoms and levodopa for parkinson ian symptoms are used. For disabling
28 psychotic symptoms, antipsychotics are used with caution, as they are associated with severe neuroleptic sensitivity
29 reactions.
Dementia with lewy bodies l-DOPA Parkinson's disease Lewy body Parkinsonism Acetylcholinesterase inhibitor Antipsychotic Dementia Somnolence Psychosis Cognition Cholinesterase
30
Hallucination
31
32 A is not correct. 110/o chose this.
33 Alzheimer disease is manifested by a progressive dementia, with memory loss, general cognitive dysfunction, and functional
34 impairments . However, although intensity of symptoms may fluctuate from day to day, they genera lly do not follow weekly or
monthly patterns, and there are no visual hallucinations associated with the dementia . Cholinesterase inhibitors and
35
memantine are used for the treatment of Alzheimer dementia .
36 Memantine Alzheimer's disease Acetylcholinesterase inhibitor Cholinesterase Dementia Cognitive disorder Amnesia Hallucination Cognition
37
B is not correct. 60/o chose this.
38
Delirium has a more acute onset of days to weeks, not months to years. Patients genera lly have a more clouded sensorium,
39 w ith fluctuations in level of consciousness . In delirium, treatment is targeted at the underlying cause. However, neuroleptics
40 may be indicated for treating severe agitation .
Delirium Sensorium Antipsychotic Altered level of consciousness
41
42 D is not correct. 4% chose this.
43 Patients with depression are likely to present with memory loss rather than general cognitive deficits . Affect changes are
44 y
more prominent, as well as more psychomotor slowing, while in dementia the intent to perform tasks is present but the

Lock
s
Suspend
0
End Block
Item: 44 of 99 ~ 1 • Mark <:] (:>- Jill ~· ~J
QID: 22182 ..1 Previous Next Lab'V!I!ues Notes Calculator

y • • • g g
22 Delirium Sensorium Antipsychotic Altered level of consciousness

23 D is not correct. 4 % chose this.


24 Patients with depression are li kely to present w ith memo ry loss ra t her than general cognitive deficits . Affect changes are
25 more prominent, as well as more psychomotor slowing, while in dementia the intent to perform tasks is present but the
26
cogn it ive ability is not, which can lead to frustration in t hese patients as opposed to the lack of in it iative noticed with
depression . Depressed patients are aware of their sympt,o ms and tend to present by themselves, whereas in dementia they
27
are brought to a physician more commonly by a family member.
28 Dementia Amnesia Major depressive disorder Depression (mood) Physician Psychomotor retardation

29
E is not correct. 4 0/o chose th is.
30
Vascular dementia may present with the development of cognitive symptoms immediately following a stroke, evidence of
31 infarcts on imaging, and findings on physical examination consistent with a prior stroke. The onset of cognit ive dysfunction
32 can be abrupt rather than progressive, followed by a continued progressive deterioration . In this patient w ith no history of
33
vascular disease, it is an unlikely diagnosis. Treatment of vascu lar dementia is directed at management and prevention of
stroke .
34 Vascular dementia Dementia Cognitive disorder Stroke Vascular disease Blood vessel Physical examination Infarction Cognition
35
36
37
Bottom line :
38 Lewy body dementia is classically seen in patients with Parkinson disease. It presents with increased daytime dro wsiness
and sleep, episodes of staring into space, disorganized f low of ideas, and visual ha llucinations.
39 Dementia with lewy bodies Somnolence lewy body Parkinson's disease Dementia Hallucination
40
41
42 References:
43 FA Step 2 CK 9th ed p 278
FA Step 2 CK 8th ed pp 398-399
44

Lock
s
Suspend
0
End Block
Item: 45 of 99 ~ 1 • Mark <:] (:>- Jill ~· ~J
QID: 24094 ..1 Previous Next Lab'V!I!ues Notes Calculator

23 A 56-year-old man with hypertension and diabetes is seen in the neurology clinic 4 weeks after a stroke . He has regained ~~AI
some function of the right side of his body. However, his speech is very fragmented and telegraphic, and he is visibly
24
frustrated by the amount of t ime it takes for him to express himself. He is able to follow simple and complex commands
25 but is unable to correctly name objects or repeat phrases.
26
27 What is the most likely location of the lesion causing this patient's impaired speech?
28
29 A. Arcuate fasciculus
30
B. Diffuse territory of the left middle cerebral artery
31
32 C. Posterior inferior frontal gyrus
33 D. Posterior superior temporal lobe
34
35
36
37
38
39
40
41

42
43
44
• 45

Lock
s
Suspend
0
End Block
Item: 45 of 99 ~ 1 • Mark <:] (:>- Jill ~· ~J
QID: 24094 ..1 Previous Next Lab'V!I!ues Notes Calculator

The correct answer is C. 570/o chose this.


23
This patient has Broca's aphasia secondary to damage to Broca 's area in the frontal lobe . Broca's area is involved in the
24
production of speech . Broca 's aphasia is an expressive aphasia in which those affected are able to comprehend normal
25 speech, but thei r speech is nonfluent. They also demonstrate impa ired naming and repetition and are typically aware and
26 thereby frustrated with their disability.
Broca's area Expressive aphasia Frontal lobe Aphasia
27
28 A is not correct. 140/o chose this.
29 The arcuate fasciculus acts as a bridge between Broca's and Wernicke's areas. When it is damaged, patients present with
30
conduction aphasia . The major deficit in th is type of aphasia is repetition, but production and comprehension remain intact.
Therefore, the aphasia can go unnoticed .
31 Arcuate fasciculus Aphasia Conduction aphasia Expressive aphasia Receptive aphasia Broca's area
32
B is not correct. 140/o chose this.
33
Damage to such a large area of the cortex would be detrimental to language as wel l as to other central nervous system
34
functions . These patients present with a global aphasia atnd have deficits in production, comprehension, naming, and
35 repetition .
36 Central nervous system Aphasia Global aphasia Nervous system Cortex (anatomy) Cerebral cortex

37 D is not correct. 150/o chose this.


38 Wern icke's area is in the posterior superior tempora l lobe, wh ich is involved in the comprehension of speech . Un like patients
39 w ith Broca's aphasia, patients with Wernicke's aphasia typically are able to produce fluent speech . However, their
40
comprehension is not preserved and their speech is nonsensica l. They also frequently use neologisms and are unaware of
their disability.
41 Wernicke's area Aphasia Temporal lobe Receptive aphasia Expressive aphasia Neologism
42
43
44
Bottom line:
45 Broca's aphasia occurs secondary to damage to Broca's area in the frontal lobe, which is involved in the production of

Lock
s
Suspend
0
End Block
Item: 45 of 99 ~ 1 • Mark <:] (:>- Jill ~· ~J
QID: 24094 ..1 Previous Next Lab'V!I!ues Notes Calculator

g • :• • • :f:J. J:J. • g •I p p
23 conduction aphasia. The major deficit in this type of aphasia is repetition, but production and comprehension remain intact.
24
Therefore, the aphasia can go unnoticed .
Arcuate fasciculus Aphasia Conduction aphasia Expressive aphasia Receptive aphasia Broca's area
25
26 B is not correct. 140/o chose this.
27
Damage to such a large area of the cortex would be detr·imental to language as well as to other central nervous system
functions . These patients present with a global aphasia a1nd have deficits in production, comprehension, naming, and
28
repetition .
29 Central nervous system Aphasia Global aphasia Nervous system Cortex (anatomy) Cerebral cortex

30
D is not correct. 150/o chose this.
31
Wernicke's area is in the posterior superior tempora l lobe, wh ich is involved in the comprehension of speech . Unlike patients
32 w ith Broca's aphasia, patients with Wernicke's aphasia typically are able to produce fluent speech . However, their
33 comprehension is not preserved and their speech is nonsensical. They also frequently use neologisms and are unaware of
34
their disability.
Wernicke's area Aphasia Temporal lobe Receptive aphasia Expressive aphasia Neologism
35
36
37 Bottom Line:
38 Broca's aphasia occurs secondary to damage to Broca's area in the frontal lobe, which is involved in the production of
39 speech. This is an expressive aphasia in wh ich those affected are able to comprehend normal speech, but their speech is
nonfluent.
40 Broca's area Frontal lobe Aphasia Expressive aphasia
41

42
43 References:
44 FA Step 2 CK 9th ed p 288
FA Step 2 CK 8th ed p 274
45

Lock
s
Suspend
0
End Block
Item: 46 of 99 ~ 1 • Mark <:] (:>- Jill ~· ~J
QID: 22421 ..1 Previous Next Lab'V!I!ues Notes Calculator

24 During a routine eye examination in an 18-year-old patient, an ophtha lmologist notices pigmented hamartomas of the iris. ~~AI
Not knowing what to make of this, the doctor does a fu ll physical examination, which is unremarkable except for the skin
25
findings shown in the image. The patient notes a family history of simi lar skin findings .
26
27
28
29
30
31
32
33
34
35
36
37
38
39
40
41
42
43
44
45
• 46

Lock
s
Suspend
0
End Block
24
25
26
27
28
29
30
31
32
33
34
35
36
37 Which of t he following conditions is most likely?
38
39 A. Multiple neuroendocrine neoplasia t y pe I
40 B . Neurofibromatosis type I
41
C. Neurofibromatosis type II
42
43 D . Tuberous sclerosis
44 E. Von Hlppei-Lindau syndrome
45
• 46 •

a
Lock
s
Suspend
8
End Block
Item: 46 of 99 ~ 1 • Mark <:] (:>- Jill ~· ~J
QID: 22421 ..1 Previous Next Lab'V!I!ues Notes Calculator

24 The correct answer is B. 660/o chose th is.


25 The skin findings in the photograph are neurofibromas, and the eye finding is an accurate description of a Lisch nodule, both
26 consistent with neu rofibromatosis (N F) type I (an autosoma l dominant disease caused by a gene found on chromosome 17).
Lisch nodule Neurofibromatosis Gene Dominance (genetics) Chromosome 17 (human) Neurofibroma Chromosome
27
28 A i s not correct. 3% chose this.
29 Neither Lisch nodules nor cafe-au-lait spots are found in multiple neuroendocrine neoplasia (M EN) type I. MEN type I is
30 typified by "the 3 P's: Pituitary (usually prolactinoma), Parathyroid (hypercalcemia), and Pancreatoma (gastrinoma) . MEN
type I is autosomal dominant and tends t o present in m iddle age .
31
Prolactinoma Gastrinoma Hypercalcaemia Dominance (genetics) Neoplasm Neuroendocrine cell Nodule (medicine)
32
C is not correct. 100/o chose this.
33
Lisch nodules are consistent with NF type I, not NF type II. NFII is associated w ith acoustic neuromas as well as multiple
34
intracranial meningiomas. The gene for NF type II is located on chromosome 22 .
35 Gene Chromosome 22 (human) Chromosome ~1eningioma Vestibular schwannoma
36
Dis not correct. 170/o chose this.
37
Neither Lisch nodules nor cafe-au-lait spots are found in tuberous sclerosis . Tuberous sclerosis is associated w ith
38 hypopigmented macules (ash leaf spots), shagreen spots (leathery cutaneous thickening of the skin), facial hamartomas,
39 seizu res, and mental retardation . Tuberous sclerosis is inherited in an autosomal dominant fashion, and it presents in
40 childhood as mental retardation, epilepsy, and facial hamartoma .
Tuberous sclerosis Epilepsy Hamartoma Dominance (genetics) Autosome Intellectual disability Cutaneous condition Epileptic seizure Nodule (medicine) Hypopigmentation
41
42 E i s not correct. 4 0/o c hose th is.
43 Neither Lisch nodules nor cafe-au-lait spots are found in von Hippei-Li ndau syndrome (VH L) . VH L presents with several
hemangiomas in many organs. It is associated w ith a renal cell cancer and polycythemia (due to increased eryth ropoietin) .
44
Von Hippei-Undau syndrome Erythropoietin Polycythemia Renal cell carcinoma Nodule (medicine) Cancer Kidney Von Hippei-Lindau disease
45
46

Lock
s
Suspend
0
End Block
Item: 46 of 99 ~ 1 • Mark <:] (:>- Jill ~· ~J
QID: 22421 ..1 Previous Next Lab'V!I!ues Notes Calculator

24 The correct answer is B. 660/o chose th is.


25 The skin findings in the photograph are neurofibromas, and the eye finding is an accurate description of a Lisch nodule, both
26 consistent with neu rofibromatosis (N F) type I (an autosoma l dominant disease caused by a gene found on chromosome 17).
Lisch nodule Neurofibromatosis Gene Dominance (genetics) Chromosome 17 (human) Neurofibroma Chromosome
27
28 A i s not correct. 3% chose this.
29 Neither Lisch nodules nor cafe-au-lait spots are found in multiple neuroendocrine neoplasia (M EN) type I. MEN type I is
30 typified by "the 3 P's: Pituitary (usually prolactinoma), Parathyroid (hypercalcemia), and Pancreatoma (gastrinoma) . MEN
type I is autosomal dominant and tends t o present in m iddle age .
31
Prolactinoma Gastrinoma Hypercalcaemia Dominance (genetics) Neoplasm Neuroendocrine cell Nodule (medicine)
32
C is not correct. 100/o chose this.
33
Lisch nodules are consistent with NF type I, not NF type II. NFII is associated w ith acoustic neuromas as well as multiple
34
intracranial meningiomas. The gene for NF type II is located on chromosome 22 .
35 Gene Chromosome 22 (human) Chromosome ~1eningioma Vestibular schwannoma
36
Dis not correct. 170/o chose this.
37
Neither Lisch nodules nor cafe-au-lait spots are found in tuberous sclerosis . Tuberous sclerosis is associated w ith
38 hypopigmented macules (ash leaf spots), shagreen spots (leathery cutaneous thickening of the skin), facial hamartomas,
39 seizu res, and mental retardation . Tuberous sclerosis is inherited in an autosomal dominant fashion, and it presents in
40 childhood as mental retardation, epilepsy, and facial hamartoma .
Tuberous sclerosis Epilepsy Hamartoma Dominance (genetics) Autosome Intellectual disability Cutaneous condition Epileptic seizure Nodule (medicine) Hypopigmentation
41
42 E i s not correct. 4 0/o c hose th is.
43 Neither Lisch nodules nor cafe-au-lait spots are found in von Hippei-Li ndau syndrome (VH L) . VH L presents with several
hemangiomas in many organs. It is associated w ith a renal cell cancer and polycythemia (due to increased eryth ropoietin) .
44
Von Hippei-Undau syndrome Erythropoietin Polycythemia Renal cell carcinoma Nodule (medicine) Cancer Kidney Von Hippei-Lindau disease
45
46

Lock
s
Suspend
0
End Block
Item: 46 of 99 ~ 1 • Mark <:] (:>- Jill ~· ~J
QID: 22421 ..1 Previous Next Lab'V!I!ues Notes Calculator

• type I IS autosomal dommant and tends to present m m1ddle age .


24 Prolactinoma Gastrinoma Hypercalcaemia Dominance (genetics) Neoplasm Neuroendocrine cell Nodule (medicine)

25
C is not correct. 100/o chose this.
26 Lisch nodules are consistent with NF type I, not NF type II. NFII is associated w ith acoustic neuromas as well as mu lt iple
27 intracranial meningiomas. The gene for NF type II is located on chromosome 22 .
Gene Chromosome 22 (human) Chromosome Meningioma Vestibular schwannoma
28
29 Dis not correct. 170/o chose this.
30 Neither Lisch nodu les nor cafe-au-la it spots are found in tuberous sclerosis. Tuberous sclerosis is associated w ith
31 hypopigmented macules (ash leaf spots), shagreen spots (leathery cutaneous th icken ing of the skin), facial hamartomas,
seizures, and menta l reta rdat ion . Tuberous sclerosis is inherited in an autosoma l dom inant fashion, and it presents in
32
chi ldhood as menta l retardation, epilepsy, and facial hamartoma .
33 Tuberous sclerosis Epilepsy Hamartoma Dominance (genetics) Autosome Intellectual disability Cutaneous condition Epileptic seizure Nodule (medicine) Hypopigmentation
34
E is not correct. 40/o chose this.
35
Neither Lisch nodu les nor cafe-au-la it spots are found in von Hippei-Lindau syndrome (VHL) . VHL presents with several
36 hemangiomas in many organs. It is associated w ith a renal cell cancer and polycythemia (due to increased erythropoietin) .
37 Von Hippei-Lindau syndrome Erythropoietin Polycythemia Renal cell carcinoma Nodule (medicine) Cancer Kidney Von Hippei-Lindau disease

38
39
Bottom line:
40
Neurofibromatosis type I presents with neurofibromas and Lisch nodules (pigmented hamartomas of the iris) .
41
Neurofibromatosis type I Neurofibromatosis Lisch nodule Neurofibroma Iris (anatomy) Hamartoma
42
43
44 References:
45 FA Step 2 CK 9th ed pp 286-288
FA Step 2 CK 8th ed pp 272-273
46

Lock
s
Suspend
0
End Block
Item: 47 of 99 ~ 1 • Mark <:] (:>- Jill ~· ~J
QID: 23278 ..1 Previous Next Lab'V!I!ues Notes Calculator
.
25 A 75-year-old Asian man with acute-onset hemiparesis and aphasia is brought to the emergency department from his ~~ A I
nursing home by an ambulance, which was held up in traffic for at least half an hour. A thorough neurologic examination is
26
strong ly suggestive of deficits in the right anterior cerebra l artery/middle cerebral artery distribution . Bedside carotid
27 duplex ultrasound shows complete occlusion of the right internal carotid just dista l to the bifu rcation . He r emains somewhat
28 responsive with stable vita l signs but his neurolog ic deficits are unchanged . Past medica l history is significant for diverticular
29
bleed ing six months ago.
30
Which of the following would prevent administration of alteplase to th is patient?
31
32
33
A. Fam ily history of hemoph il ia A
34 B. History of a left-sided ischemic cerebrovascular accident 2 months ago
35
C. Myocardial infarction three years ago treated with card iac catheterization
36
37
D. Platelet count of 120,000/mm 3
38 E. Sigmoidectomy 6 months ago for bleeding diverticulosis
39
40
41

42
43
44
45
46
• 47

Lock
s
Suspend
0
End Block
Item: 47 of 99 ~ 1 • Mark <:] (:>- Jill ~· ~J
QID: 23278 ..1 Previous Next Lab'V!I!ues Notes Calculator
.
25
26
27
The correct answer is B. 590/o chose th is.
28
Historica l exclusion criteria for thrombolytic therapy includes any history of stroke or head t rauma within the past three
29 months. Other exclusion criteria includes a history of intracran ial hemorrhage, neoplasm, arteriovenous malformation, or
30 aneurysm; recent intracran ial or intraspinal surgery; or an arterial punctu re at a noncompressible site in the previous seven
31 days . Clin ica l exclusion criteria include improving or on ly minor stroke symptoms, symptoms of subarachnoid hemorrhage,
pregnancy, refractory hypertension (systolic ::2:185 mm Hg or diastolic ::2:1 10 mm Hg), or active bleeding. Laboratory exclusion
32
criteria include a platelet count <100,000/mm 3 , current anticoagulant use w ith an INR > 1.7 or PT > 15 seconds, heparin use
33 w ithin 48 hours and an abnorma lly elevat ed aPTT or current use of a direct th rom bin inh ibitor or direct factor Xa inhibitor with
34 evidence of anticoagulant effect by laboratory t ests, and hypo- or hyperglycemia .
Subarachnoid hemorrhage Arteriovenous malformation Intracranial hemorrhage Anticoagulant Thrombolysis Direct thrombin inhibitor Heparin Stroke Factor X Thrombin Platelet
35
Neoplasm Hyperglycemia Hypertension Thrombolytic drug Partial thromboplastin time Bleeding Aneurysm Direct Xa inhibitor Diastole Major trauma Systole Pregnancy
36
37 A is not correct. 150/o chose this.
38 A family history of a bleed ing disorder w ill not prevent throm bolysis as there is no indicat ion the patient has any persona l
39 history of t his affliction .
Thrombolysis Coagulopathy Family history (medicine)
40
41 C is not correct. 4 % chose this.
42 Myocard ial infarction in the preceding t hree months is a contraindication for alteplase. Arterial puncture at a noncompre ssible
site within the past 7 days precludes thrombolysis. Neither of these have occurred within the time lim its; therefore, this does
43
negate throm bolysis.
44 Tissue plasminogen activator Myocardial infarction Thrombolysis Contraindication Infarction
45
D is not correct. 70/o chose this.
46
Thrombocytopenia below 100,000 is a cont raindication to alteplase.
47 y Thrnmhnrvtnngni::. Tic:c:u~ nl::ac:minnn,c.n ;:artill:::.tnr lnntr;:~~inrlir::.tinn

Lock
s
Suspend
0
End Block
Item: 47 of 99 ~ 1 • Mark <:] (:>- Jill ~· ~J
QID: 23278 ..1 Previous Next Lab'V!I!ues Notes Calculator
. • h1story of t h1s affhct1on .
25 Thrombolysis Coagulopathy Family history (medicine)

26
C i s not correct. 4 % chose this.
27 Myocardial infarction in the preceding three months is a contraindication for alteplase. Arterial puncture at a noncompressible
28 site within the past 7 days precludes thrombolysis. Neither of these have occurred within the time limits; therefor e, this does
29 negate throm bolysis.
Tissue plasminogen activator Myocardial infarction Thrombolysis Contraindication Infarction
30
31 D i s not correct. 70/o chose this.
32 Thr ombocytopenia below 100, 000 is a contraindication to alteplase.
Thrombocytopenia Tissue plasminogen activator Contraindication
33
34 E i s not corr ect. 15% chose this .
35 The patient is not currently bleeding from his diverticula. Within the eligibility criteria, r ecent gastrointestinal or urinary tract
36
bleeding within the previous 21 days is considered a relative exclusion criteria. Th is patient experienced diverticular bleeding
six months ago, so th is exclusion criterion is not being met in this vignette .
37 Diverticulum Urinary system
38
39
40
Bottom line :
41 A recent stoke (w ithin 3 months) precludes thrombolysi s. Any other active issues that may put the patient at risk of life-
thr eaten ing bleeding contraindicate thrombolysis.
42 Thrombolysis Contraindication
43
44
45 References:
46 FA Step 2 CK 9th ed p 260
FA Step 2 CK 8th ed p 246
47

Lock
s
Suspend
0
End Block
Item: 48 of 99 ~ 1 • Mark <:] (:>- Jill ~· ~J
QID: 23386 ..1 Previous Next Lab'V!I!ues Notes Calculator

26 A 66-year-old man is accompanied by his daughter on a visit to a primary care specialist. The daughter is concerned that
her father is suffering from Alzheimer's disease . She said that he is ret ired and appeared to be enjoying his r etirement
27
life, yet over the last two years he has demonstrated forgetfulness, wh ich appears to be getting worse, as well as
28 progressively worsening grooming habits and flattening of his emotional responses to his family. On examination, he scores
29 19/30 on a Mini-Mental State Examination and is noted to have difficulty formulating the words for his responses. He appea rs
30
otherwise healthy, w ith no physica l examination finding; a brain MRI shows diffuse cerebral atrophy.
31
Which of the following is the most appropriate initial therapy to help slow the patient's cognitive decline?
32
33
34 A. Catechol 0-methyltransferase inhibitor
35 B. Cholinesterase inhibitor
36
C. GABA agonist
37
38
D. N-methyi-D-aspartate receptor agonist
39 E. Serotonin reuptake inh ibitor
40
41
42
43
44
45
46
47
• 48

Lock
s
Suspend
0
End Block
Item: 48 of 99 ~ 1 • Mark <:] (:>- Jill ~· ~J
QID: 23386 ..1 Previous Next Lab'V!I!ues Notes Calculator

26
27
28
The correct answer is B. 790/o chose this.
29
Patients with Alzheimer's disease (AD) have reduced cerebral production of choline acetyl transferase, wh ich leads to a
30 decrease in acetylcholi ne synthesis and im paired cortical cholinergic function . Cholinesterase inh ibitors increase cholinergic
31 transmission by inh ibiting cholinesterase at the synaptic cleft. Four cholinesterase inh ibitors, tacrine, donepezil, rivastigmine,
32 and ga lantamine are currently approved for use in AD.
Rivastigmine Donepezil Galantamine Alzheimer's disease Tacrine Acetylcholine Choline Cholinesterase Cholinergic Synaptic cleft Chemical synapse Transferase
33
Acetylcholinesterase inhibitor Acetyl
34
35 A is not correct. 4% chose this.
36 Catechol 0-methyltransferase inhibitors inhibit the peripheral metabol ism of levodopa to 3-0-methyldopa, thereby prolong ing
the levodopa half-life and making more levodopa ava ilable for transport across the blood-brain barrier. They are used to treat
37
Parkinson's disease .
38 Parkinson's disease Catechol-O-m ethyl transferase Catechol l-DOPA Blood-brain barrier Half-life 3-D-Methyldopa Metabolism
39
C is not correct. 4% chose this.
40
GABA is a major inhibitory neurotransmitter. It has not been shown to have a role in the treatment of Alzheimer's disease .
41 Alzheimer's disease Gamma-Aminobutyric acid Neurotransmitter
42
Dis not correct. 110/o chose this.
43
Memantine is an antagonist of the N-methyi-D-aspartate (NMDA) receptor, one that is involved in learning and memory. The
44
blockage of pathologic stimulation of NMDA receptors may protect against progressive neuronal damage, slowing cognitive
45 decline, and potentially, leading to symptomatic improvement. Nevertheless it is considered to be second li ne to
46 cholinesterase inhibitors for the treatment of Alzheimer's disease, considered for patient with moderate to advanced dementia
47
(with accompanying mini mental status exam scores < 17), either combined with a cholinesterase inhibitor, or for use as
monotherapy in patients who either do not tolerate or benefit from a cholinesterase inhibitor. There is little, if any, evidence
48 y

Lock
s
Suspend
0
End Block
Item: 48 of 99 ~ 1 • Mark <:] (:>- Jill ~· ~J
QID: 23386 ..1 Previous Next Lab'V!I!ues Notes Calculator
- . .. . . - . .. . . . . ..
26 Dis not correct. 110/o chose this.
27 Memantine is an antagon ist of the N-methyi- D-aspartate { NMDA) receptor, one that is involved in learn ing and memory. The
28 blockage of pathologic stimulation of NMDA r eceptors may protect aga inst progressive neuronal damage, slowing cognit ive
29
decline, and potentially, leading to symptomatic improvement. Nevertheless it is considered to be second line to
cholinesterase inhibitors for the t r eatment of Alzheimer's disease, considered for patient with moderate to advanced dementia
30 (with accompanying m ini mental status exam scores < 17), either combined with a choli nesterase inhibitor, or for use as
31 monotherapy in patients who either do not tolerate or benefit from a cholinesterase inhibitor. There is little, if any, evidence
32 that patients with milder Alzheimer's disease benefit from memantine.
Alzheimer's disease Memantine Acetylcholinesterase inhibitor Cholinesterase Receptor antagonist Mental status examination Dementia N~1DA receptor N-Methyi-D-aspartic acid
33
Antagonist Pathology Enzyme inhibitor Receptor (biochemistry) Combination therapy
34
35 E i s n ot correct. 2 0/o c hose th is.
36 Serotonin reuptake inhibitors can be used to treatment of major depressive disorder, if diagnosed in the setting of Alzheimer's
37
disease ( AD) . This vignette does not provide the symptom profile of major depressive disorder. Further, this medication class
does not improve cogn it ive decline associated with AD.
38 Major depressive disorder Alzheimer's disease Serotonin Reuptake Symptom Dementia
39
40
41
Bottom line :
42 Treat patients with Alzheimer's disease with cholinesterase inhibitors including tacrine, donepezil , rivastigmine, and
galantamine.
43 Rivastigmine Donepezil Galantamine Alzheimer's disease Tacrine Cholinesterase Acetylcholinesterase inhibitor
44
45
46 References:
47 FA Step 2 CK 9th ed pp 277-279
FA Step 2 CK 8th ed pp 265-266
48

Lock
s
Suspend
0
End Block
Item: 49 of 99 ~ 1 • Mark <:] (:>- Jill ~· ~J
QID: 22424 ..1 Previous Next Lab'V!I!ues Notes Calculator
.
27 Following prenatal diagnosis of a cardiac mass on ultrasound, an infant undergoes MRI imag ing, which revea ls a cardiac ~~AI
rhabdomyoma and an astrocytoma in the right foramen of Monroe . Over the course of time, the patient is revea led to have
28
global developmental delay as wel l as a facial ang iofibroma .
29
30
Which of the following complications is this ch ild most likely to experience in the future?
31
32
A. Colonic adenocarcinoma
33
34 B. Lymphoma
35 C. Renal failure
36
D. Seizures
37
38
39
40
41

42
43
44
45
46
47
48
• 49

Lock
s
Suspend
0
End Block
Item: 49 of 99 ~ 1 • Mark <:] (:>- Jill ~· ~J
QID: 22424 ..1 Previous Next Lab'V!I!ues Notes Calculator
.
27
28
29
The correct answer is D. 720/o chose this.
30
Based on the location of the cardiac and centra l nervous system neoplastic lesions and the presence of the facia l lesion, th is
31 patient most li kely has tuberous sclerosis. Such patients typically begin to develop seizures with in the first year of life, in
32 addition to other neoplastic lesions in such places as the kidneys, liver, brain, and heart.
Tuberous sderosis Central nervous system liver Neoplasm Lesion Kidney Nervous system Epileptic seizure Human brain Brain
33
34 A is not correct. 4% chose this.
35 Colon cancer is not among the cancers associated w ith tuberous sclerosis . It can be associated with ulcerative colitis and
36
inherited polyposis syndromes including Gardner's disease (adenomatous polyps and osteomas), familial adenomatous
polyposis, and Turcot's syndrome (colon polyps and centr al nervous system tumors) . All these syndromes would be expected
37
to present later in life and none would expla in the cardiac and skin lesions in this case .
38 Familial adenomatous polyposis Ulcerative colitis Tuberous sclerosis Colorectal cancer Central nervous system Colon (anatomy) Cancer Colitis Adenoma Osteoma Nervous system

39 Polyp

40
B is not correct. 40/o chose this.
41
Lymphoma is not among the cancers associated with tuberous sclerosis. It can be associated with hereditary or induced
42 immunodeficiencies.
43 Tuberous sderosis Lymphoma Immunodeficiency

44 C is not correct. 200/o chose this.


45 Renal failure is not associated with tuberous sclerosis, although r enal tumors have been r eported. In early life renal failure
46 can be caused by the autosomal recessive form of polycystic kidney disease, which is characterized by bilaterally enlarged
47 kidneys filled with multiple cysts (detectable at birth). Cysts can also be found in the liver, heart, and central nervous system,
but this disorder is not typically associated with skin lesions.
48 Tuberous sderosis Central nervous system Polycystic kidney disease Autosomal recessive Liver Dominance (genetics) Kidney Autosome Nervous system Neoplasm Kidney cancer
49

Lock
s
Suspend
0
End Block
Item: 49 of 99 ~ 1 • Mark <:] (:>- Jill ~· ~J
QID: 22424 ..1 Previous Next Lab'V!I!ues Notes Calculator
. polypos1s, and Turcot's syndrome (colon polyps and centr al nervous system tumors) . All these syndromes would be expected
27 to present later in life and none would expla in the cardiac and skin lesions in this case .
28 Familial adenomatous polyposis Ulcerative colitis Tuberous sclerosis Colorectal cancer Central nervous system Colon (anatomy) Cancer Colitis Adenoma Osteoma Nervous system

29 Polyp

30
B is not correct. 40/o chose this.
31
Lymphoma is not among the cancers associated with tuberous sclerosis. It can be associated with hereditary or induced
32 immunodeficiencies.
Tuberous sclerosis Lymphoma Immunodeficiency
33
34 C is not correct. 200/o chose this.
35 Renal failure is not associated with tuberous sclerosis, although renal tumors have been reported. In early life renal failure
36 can be caused by the autosomal recessive form of polycystic kidney disease, which is characterized by bilaterally enlarged
kidneys filled with multiple cysts (detectable at birth ). Cysts can also be found in the liver, heart, and central nervous system,
37
but this disorder is not typically associated with skin lesions.
38 Tuberous sclerosis Central nervous system Polycystic kidney disease Autosomal recessive Liver Dominance (genetics) Kidney Autosome Nervous system Neoplasm Kidney cancer
39 Kidney disease Cyst Recessive
40
41

42
Bottom line:
43 Patients with tuberous sclerosis typically begin to develop seizures within the first year of life, in addition to other neoplastic
lesions in such places as the kidneys, liver, brain, and heart.
44 Tuberous sclerosis Liver Epileptic seizure Kidney Neoplasm Human brain Brain
45
46
47 References:
48 FA Step 2 CK 9th ed pp 287-288
FA Step 2 CK 8th ed p 273
49

Lock
s
Suspend
0
End Block
Item: 50 of 99 ~ 1 • Mark <:] (:>- Jill ~· ~J
QID: 23754 ..1 Previous Next Lab'V!I!ues Notes Calculator

28 A 26-year-old man visits his fami ly practitioner complain ing of 1 week of headaches. The patient describes the pain as a
pressure in the frontal reg ion, 5/10 in intensity. He says the pa in gets worse gradua lly throughout the day. When asked
29
about stressful events in his life, the patient reports that he broke up with his girlfriend of 2 years approximately 1 week
30 ago. The patient reports no photophobia, nausea, or other symptoms. Physical examination shows some paraspinal neck
31 tenderness but is otherwise unremarkable .
32
33 Which of the following is the most appropriate next step in management?
34
35 A. 100% Inhaled oxygen
36 B. Antibiotics
37
C. MRI of brain
38

39 D. Over-the-counter analgesics
40 E. Triptans
41
42
43
44
45
46
47
48
49
• 50

Lock
s
Suspend
0
End Block
Item: 50 of 99 ~ 1 • Mark <:] (:>- Jill ~· ~J
QID: 23754 ..1 Previous Next Lab'V!I!ues Notes Calculator

28
29
30
The correct answer is D. 8 30/o chose this.
The patient has a classic presentation of a tension headache. Tension headaches respond well to over-the-counter ana lgesics,
31
so these are the first step in management. The patient reports a moderate-intensity headache that is worse in the evening,
32 which is typica l of tension headaches. Tension headaches are also associated with stress and bilatera l, fronta l-occipita l
33 distributions
Tension headache Over-the-counter drug Headache Analgesic
34
35 A is not correct. 7 % chose this.
36 One hundred percent inhaled oxygen is used as t reatment for cluster headaches, which usually appear in brief episodes
37 lasting minutes to hou rs with a very sudden onset. These episodes can occur severa l t imes a day for many months at a t ime,
and are known as "clusters." Patients then typically have cluster- free periods lasting a few weeks to years . Pain is usually
38
described as intense and lancinating . Pain is usua lly unilateral in t he distribution of cranial nerves V 1 and V2 and is associated
39 w ith lacrimation and nasal stuffiness .
40 Tears Cranial nerves Cluster headache Oxygen

41
B is not correct. 1 0/o chose this.
42 Antibiotics are part of the treatment for sinusit is. The type of pain the patient describes is also typical of frontal sinus pain.
43 However, a patient with sinusit is would typically, although not necessarily, be expected to present with upper respiratory
44 infection symptoms such as sinus congestion or puru lent discharge in addit ion to the sinus pa in . Tenderness to palpation of
the sinuses would also be expected .
45
Frontal sinus Sinusitis Upper respiratory tract infection Antibiotics Paranasal sinuses Sinus (anatomy) Pus Palpation Respiratory tract infection
46
47
C is not correct. 4 % chose this.
MRI of t he brain can used to identify the cause of headaches associated with bra in tumors. These headaches are typically
48
associated w ith pain t hat is worst in the morning . This is beca use cerebrospinal fluid builds up in the head after a patient lies
49 down overn ight and has trouble draining t he next morning due to physica l obstruction or pressure caused by a mass lesion .
50 y
This pain is most likely due to a tension headache, so a brain MRI is not ind icated at th is t ime. y

Lock
s
Suspend
0
End Block
Item: 50 of 99 ~ 1 • Mark <:] (:>- Jill ~· ~J
QID: 23754 ..1
. .. . . .. Previous
...
' .
Next
. . . .. .Lab'V!I!ues
. . . . . .. .
Notes Calculator
.. . .
. . .. . .. . ..
infection symptoms such as sinus congestion or purulent discharge in addition to the sinus pain . Tenderness to palpation of
28
the sinuses would also be expected .
29 Frontal sinus Sinusitis Upper respiratory tract infection Antibiotics Paranasal sinuses Sinus (anatomy) Pus Palpation Respiratory tract infection
30
C is not correct. 4% chose this.
31
MRI of the brain can used to identify the cause of headaches associated with bra in tumors. These headaches are typically
32 associated w ith pain that is worst in the morning . This is because cerebrospinal fluid builds up in the head after a patient lies
33 down overn ight and has trouble draining the next morning due to physica l obstruction or pressure caused by a mass lesion .
34 This pain is most likely due to a tension headache, so a brain MRI is not ind icated at this time.
Tension headache Cerebrospinal fluid ~1agnetic resonance imaging Neoplasm Headache Lesion Human brain Brain tumor Brain Mass effect (medicine)
35
Magnetic resonance imaging of the brain
36
37 E is not correct. SOlo chose this.
38 Triptans are abortive treatment for migraine headaches as well as cluster headaches. Cluster headache is not the most likely
39
diagnosis. As the patient is not experiencing photophobia, any prodrome, nausea, or un ilateral or throbbing pain, a migraine
headache, which typically involves these symptoms, is also un likely.
40 Cluster headache Photophobia Migraine Prodrome Triptan Headache Nausea
41
42
43
Bottom line:
44 Tension headaches are associated w ith stress, bilateral, fronta l-occipital distributions, and worsen ing in the evening. The
first step in management of tension headaches is over-the-counter analgesics .
45
Over-the-counter drug Tension headache Analgesic Stress (psychological) Stress (biology)
46
47
48 References:
49 FA Step 2 CK 9th ed p 268
FA Step 2 CK 8th ed p 254
50

Lock
s
Suspend
0
End Block
Item: 51 of 99 ~ 1 • Mark <:] (:>- Jill ~· ~J
QID: 24092 ..1 Previous Next Lab'V!I!ues Notes Calculator
.
29 A 16-year-old girl is brought to the emergency department by her friends after collapsing at a party. She was standing at
the party when she suddenly fel l to the floor and began convulsing and was unresponsive. Due to snowy weather
30
conditions, it took the ambulance 32 minutes to reach the hospita l. Lorazepam was administered twice en route but failed
31 to resolve the seizure. The patient is still convulsing on presentation and a mixture of vomitus and blood is running out of her
32 mouth .
33
34 What is the best f irst step in management?
35
36 A. Check v ital signs and blood glucose
37 B. Give intravenous glucose
38
C. Give intravenous phenytoin
39
40 D. Intubate the patient
41 E. Perform a urine drug screen
42
F. Perform CT of the head
43
44
45
46
47
48
49
so
• 51

Lock
s
Suspend
0
End Block
Item: 51 of 99 ~ 1 • Mark <:] (:>- Jill ~· ~J
QID: 24092 ..1 Previous Next Lab'V!I!ues Notes Calculator
. The correct answer IS D. 710/o chose th1s.
29
This patient is in status epilepticus, defined as either 30 minutes of continued seizure activity or sequential seizures without
30 regain ing full consciousness in the interim periods. The pediatric advanced life support gu idelines recommend the following
31 algorithm for status epilepticus : Lorazepam (may be repeated once if seizure activity persists) , followed by phenytoin,
32 followed by phenobarbital. However, in a critically ill patient such as this girl the circulation, airway, and breathing (note : C-A-
B rather than the old A-B-C) must always be eva luated before administering advanced treatment. A seizing patient with blood
33
and vom itus in the oral cavity cannot protect her airway and warrants immediate intubation .
34 Phenobarbital Phenytoin Status epilepticus Vomiting Intubation Lorazepam Advanced life support Pediatric advanced life support Pediatrics Epileptic seizure Respiratory tract

35 Consciousness Mouth
36
A is not correct. 7% chose this.
37
Checking vital signs and blood glucose levels can exclude nonneurologic causes of impaired consciousness such as
38 hypoglycemia, hypertensive encephalopathy, shock, hyperthermia, and cardiac arrhythmia . The first step, however, is to
39 secure the airway.
Hypertensive encephalopathy Cardiac arrhythmia Hypoglycemia Hyperthermia Blood sugar Respiratory tract Glucose Encephalopathy Vital signs Consciousness
40
41 B is not correct. 30/o chose this.
42 Although severe hypoglycemia may induce seizures, it is important to remember to adm inister glucose with thiam ine in
43 patients in whom alcohol withdrawal may be expected.
Hypoglycemia Thiamine Alcohol withdrawal syndrome Epileptic seizure Glucose Alcoholic beverage Alcohol
44
45 Cis not correct. 170/o chose this.
46 Phenytoin is a fast-acting anticonvulsant given to patients in status epi lepticus . It does not cause respiratory depression.
However, it has been shown to cause hypotension and arrhythmia and is typica lly given immediately after lorazepam . It
47
would be the appropriate next anticonvulsive, but the patient must be stabi lized f irst.
48 Anticonvulsant Lorazepam Phenytoin Hypoventilation Status epilepticus Hypotension Cardiac arrhythmia Major depressive disorder Depression (mood)
49
E is not correct. 10/o chose this.
so
Urine drug screen provides an important diagnostic clue in patients w ith prolonged seizures. Intoxication w ith cocaine,
51 ;::.n-t.nhct:::.~ino nhont"'\/r lirlinc I"H" inh:::. l;:.ntc: ;:lC: \Moll ;:ac: \Mithrlr::.\AI::II I fri'H''r"' c:orl::.tiHoc: 1"\t" ;::. 11"'1"\hnl r::.n t"";::II IIC: O c:ci'7 • •rcc: l-lr"''\AtC\/Qr' .:::.

Lock
s
Suspend
0
End Block
Item: 51 of 99 ~ 1 • Mark <:] (:>- Jill ~· ~J
QID: 24092 ..1 Previous Next Lab'V!I!ues Notes Calculator
.
Phenytoin is a fast-acting anticonvulsant given to patients in status epi lepticus . It does not cause respiratory depression .
29
However, it has been shown to cause hypotension and arrhythmia and is typica lly given immediately after lorazepam . I t
30 would be the appropriate next anticonvulsive, but the patient must be stabi lized f irst.
31 Anticonvulsant Lorazepam Phenytoin Hypoventilation Status epilepticus Hypotension Cardiac arrhythmia Major depressive disorder Depression (mood)

32
E is not correct. 10/o chose this.
33 Urine drug screen provides an important diagnostic clue in patients w ith prolonged seizures. Intoxication w ith cocaine,
34 amphetamine, phencyclidine, or inhalants as well as w ithdrawal from sedatives or alcohol can cause seizures . However, a
35 drug screen should be performed later in the emergency assessment of th is patient.
Amphetamine Phencyclidine Substituted amphetamine Cocaine Drug test Urine Alcoholic beverage Alcohol Status epilepticus Intoxicative inhalant Epileptic seizure Sedative
36
Substance intoxication Alcohol intoxication
37
38 F is not correct. 10/o chose this.
39 CT of the head is especially useful if the patient has foca l neurologic signs that point to a tumor or trauma-induced seizure. It
40
is also standard of care to perform a CT scan before performing a lumbar puncture if increased intracran ial pressure is
suspected . However, this wou ld not be the best first step in management.
41 Lumbar puncture CT scan Intracranial pressure Focal neurologic signs Neurology Neoplasm Lumbar Epileptic seizure Standard of care Lumbar vertebrae
42
43
44
Bottom line:
45 Any critica lly ill patient must have stable circu lation, be able to protect his or her airway, and ventilate/oxygenate sufficiently
before advanced therapy can be started.
46 Respiratory tract Airway (aviation)
47
48
49 References:
so FA Step 2 CK 9th ed p 271
FA Step 2 CK 8th ed p 257
51

Lock Suspend
s 0
End Block
Item: 52 of 99 ~ 1 • Mark <:] (:>- Jill ~· ~J
QID: 24441 ..1 Previous Next Lab'V!I!ues Notes Calculator

30 A 70-year-old man with hypertension and type 2 diabetes mellitus presents to the emergency department with left-sided
extremity weakness of 7 hours' duration. His temperature is 37 .5°C (99.5°F), blood pressure is 175/100 mm Hg, pulse is
31
95/min, and respiratory rate is 16/min. His blood glucose level is 110 mg/dL. Physical examination reveals left arm
32 weakness and sensory loss . There are no cran ial nerve deficits, and right-sided strength and sensation are intact. CT of the head
33 w ithout contrast is shown .
34
35
36
37
38
39
40
41
42
43
44
45
46
47
48
49
50
51 Which of the following is the most appropriate treatment?
• 52 y y

Lock
s
Suspend
0
End Block
30
31
32
33
34
35
36
37
38
39
40
41
Which of the following is the most appropriate treatment?
42
43
A . Aspirin
44
45 B . Insu lin
46 C. Labeta lol
47
D. Lisinopril
48
49 E. Mannitol
so F. Tissue plasminogen activator
51
• 52 •

a
Lock
s
Suspend
8
End Block
Item: 52 of 99 ~ 1 • Mark <:] (:>- Jill ~· ~J
QID: 24441 ..1 Previous Next Lab'V!I!ues Notes Calculator

30 The correct answer is A. 47% chose this.
31 This patient is presenting with an acute ischemic stroke. As shown in the CT, the hypodense area coincides with the right
32 middle cerebra l artery territory, which is consistent with the left-sided deficits . Aspirin is associated with decreased morbidity
33 and morta lity in acute ischemic stroke when patients present with in 48 hours from onset; this effect is mostly due to the
prevention of further strokes. However, the lack of parenchymal hemorrhage must be confirmed before giving antiplatelet
34
therapy because this may lead to further bleeding and damage.
35 Stroke Ischemic stroke Middle cerebral artery Aspirin Antiplatelet drug Ischemia Bl eeding Parenchyma Cerebral arteries cr scan

36
B is not correct. 40/o chose this.
37
Hyperglycemia is associated with worse prognoses in the setting of acute stroke . However, this patient is not hyperglycemic.
38 Hyperglycemia Stroke Prognosis
39
Cis not correct. 170/o chose this.
40
Labeta lol is not ind icated for this patient. I n patients w ith acute ischemic stroke who are not treated w ith thrombolytics, such
41 as the patient in th is vignette, only treat severe hypertension (systolic blood pressure >220 mm Hg or diastolic blood
42 pressu re >120 mm Hg) . First-line agents are intravenous labeta lol or intravenous nicardipine. Allow perm issive hypertension
43 and hypoxemia to maintain perfusion of ischemic cerebral tissue for the first 24 hours following an ischemic stroke. For
patients who are elig ible for thrombolytic therapy, blood pressure shou ld be treated to the goal of systolic blood pressure
44
~185 mm Hg and diastol ic blood pressu re < 110 mm Hg prior to beginning the therapy. First-line t reatments are intravenous
45 labetalol or intravenous nicardipine. During thrombolytic therapy and for 24 hours following it, ma intain blood pressure
46 at <180/105 mm Hg .
Thrombolysis Hypertension Hypoxemia Blood pressure Labetalol Thrombolytic drug Nicardipine Stroke Intravenous therapy Ischemia Ischemic stroke Perfusion Diastole
47
48 D is not correct. 5% chose this.
49 Although lisinopril may be a good agent for t reating the patient's hypertension and therefore decreasing his risk for future
50
strokes, it is not appropriate in the acute setting .
Lisinopril Hypertension
51
52 E is not correct. 18% chose this.

Lock
s
Suspend
0
End Block
Item: 52 of 99 ~ 1 • Mark <:] (:>- Jill ~· ~J
QID: 24441 ..1 Previous Next Lab'V!I!ues Notes Calculator

• labetalol or Intravenous mcard1p1ne. Dunng thrombolytiC thera py and for 24 hours followmg 1t, mamtam blood pressure
30 at <180/105 mm Hg .
Thrombolysis Hypertension Hypoxemia Blood pressure Labetalol Thrombolytic drug Nicardipine Stroke Intravenous therapy Ischemia Ischemic stroke Petfusion Diastole
31
32 D is not correct. SOfo chose this.
33 Although lisinopril may be a good agent for treating the patient's hypertension and therefore decreasing his risk for future
34 strokes, it is not appropriate in the acute setting .
Lisinopril Hypertension
35
36 E is not correct. 18% chose this.
37 Mannitol is part of the treatment for increased intracran ial pressure and herniation .
Mannitol Intracranial pressure
38
39 F is not correct. 90/o chose this.
40 Current American College of Chest Physicians guidelines suggest intravenous tissue plasminogen activator w ithin 3 hours {4 .5
41
hours in select patients), and intraarterial tissue plasminogen activator within 6 hours in patients with proximal cerebral
artery occlusion . Unfortunately, this patient does not meet these criteria .
42 Tissue plasminogen activator American College of Chest Physicians Plasmin Intravenous therapy
43
44
45
Bottom line:
46 The appropriate treatment for a patient with an acute ischem ic stroke who is not a candidate for tissue plasminogen
activator is aspirin .
47
Tissue plasminogen activator Stroke Aspirin Ischemic stroke Ischemia Plasmin
48
49
50 References:
51 FA Step 2 CK 9th ed pp 258-262
FA Step 2 CK 8th ed pp 243-247
52

Lock
s
Suspend
0
End Block
Item: 53 of 99 ~ 1 • Mark <:] (:>- Jill ~· ~J
QID: 23387 ..1 Previous Next Lab'V!I!ues Notes Calculator
I

31 A 39-year-old man presents to a neurologist for "muscular problems" that he noted approximately 4 months ago . Around
that time, he noted that his arms, especially his hands, would sudden ly feel weak as he was working on the job as a
32
construction worker. He reports that it began as trouble operating drills and grasping tools tightly, but that it has worsened
33 to the point where he can barely lift a hammer above his head . His primary care physician referred him for a full neurologic
34 work-up. Exam ination shows 3/5 strength in the upper extremities and atrophy of hand muscles . Also noted are 4/5 strength in
35
both legs and bilateral foot drop . Fasciculations are noted when he flexes his biceps or outstretches his tongue . Babsinski is
present bilaterally.
36
37
This patient most li kely has wh ich of the following?
38
39
40
41 A. A chronic demyelinating condition involving the central nervous system, characterized by lesions separated in space
42
and time
43 B. A degenerative disorder of myelin, leading to increasing disability and inherited in a sex- linked manner on the X
chromosome
44
45
C. A progressive disorder of unknown etiology involv ing anterior horn cells in the spinal cord and cranial motor nerves

46 D. An autoimmune disorder characterized by antibod ies directed toward the acetylcholi ne receptors in the
47 neuromuscular junction
48 E. An autosomal recessive disorder characterized by progressive weakness of the lower motor neurons
49
so
51
52
• 53

Lock
s
Suspend
0
End Block
Item: 53 of 99 ~.

i. Mark -<l I>


LabfJues
..

~J

·-
J.
.
QID: 23387 Previous Next Notes Calculator
I
Like Dislike
31
32
33 The correct a nswer i s C. 720/o chose t his .
34 Amyotrophic lateral sclerosis (ALS) primarily involves anterior horn cells in the spinal cord and cran ial motor nerves.
35 Involvement of both upper and lower motor neurons is characteristic. Patients develop variable hyperreflexia, clonus,
36 spasticity, extensor plantar responses, and limb or tongue fasciculations. Prognosis for patients with ALS is poor, and
treatment options remain li mited . Progressive deterioration and death occur over the course of 5- 10 years in adult-onset
37
forms. Treatment with riluzole can delay the need for tracheostomy and provide a three-month survival advantage on
38 average .
39 Amyotrophic lateral sclerosis Clonus Hyperreflexia Riluzole Spasticity Tracheotomy Lower motor neuron Spinal cord Anterior grey column Fasciculation Motor neuron Neuron

40 Cranial nerves Skull Anatomical terms of location Prognosis

41
A i s not correct. 9 % ch ose this .
42 Multiple sclerosis (MS) is considered the most common demyelinating process involving the central nervous system.
43 Presentation is usually with multiple focal neurologic events, separated by time, wh ich cannot be explained by a single lesion .
44 Although there is some overlap in the presentations of MS and amyotrophic lateral sclerosis (ALS), the prom inent mixture of
upper and lower motor neuron symptoms indicates ALS in this patient.
45
Amyotrophic lateral sclerosis Multiple sclerosis Central nervous system Neuron lower motor neuron Motor neuron lesion Nervous system Neurology Demyelinating disease Myelin
46
47
B is not correct. J Ofo chose this .
This description is that of adrenoleukodystrophy (ALD) . Patients with ALD are typically men, and the disease begins its
48
expression around ages 5-10 years, unli ke the patient in our case who did not exhibit any symptoms until much later in life.
49 Adrenoleukodystrophy
so
D i s not correct. SOfo chose this.
51
Myasthenia gravis (MG) is an acquired autoimmune disorder characterized clinically by weakness of skeletal muscles and
52 fatigability on exertion. The antibodies in MG are directed toward the acetylcholine receptor at the neuromuscular junction of
53 y skeletal muscles . This patient's symptoms are not consistent with MG. y

Lock
s
Suspend
0
End Block
Item: 53 of 99 ~ 1 • Mark <:] (:>- Jill ~· ~J
QID: 23387 ..1 Previous Next Lab'V!I!ues Notes Calculator
I
B is not correct. ] Ofo chose this .
31
This description is that of adrenoleukodystrophy (ALD) . Patients with ALD are typically men, and the disease begins its
32 expression around ages 5-10 years, unli ke the patient in our case who did not exhibit any symptoms until much later in life.
33 Adrenoleukodystrophy

34
D is not correct. 5 % chos e this .
35 Myasthenia gravis (MG) is an acquired autoimmune disorder characterized clinically by weakness of skeletal muscles and
36 fatigability on exertion. The antibodies in MG are directed toward the acetylcholine receptor at the neuromuscular junction of
37 skeletal muscles . This patient's symptoms are not consistent with MG.
Myasthenia gravis Neuromuscular junction Autoimmune disease Acetylcholine Antibody Autoimmunity Acetylcholine receptor Fatigue (medical) Skeletal muscle Muscle weakness
38
39 E is not correct. ] Ofo c hose th is.
40 The spinal muscular atrophies (SMAs) comprise a group of disorders that begin in infancy and generally result in early death,
41
although the age at death is variable . SMA is anatom ically characterized by the loss of lower motor neurons in the entire
spinal cord and in select bra instem motor nuclei (nuclei of cranial nerves V, VII, IX, and XII).
42 Spinal muscular atrophy Brainstem Cranial nerves Spinal cord Lower motor neuron Motor neuron Neuron Atrophy Nucleus (neuroanatomy) Spinal muscular atrophies
43 Cranial nerve nucleus Facial motor nucleus
44
45
46
Bottom line :
47 ALS primarily involves anterior horn cells in the spinal cord and cranial motor nerves, and involves both upper and lower
motor neurons.
48 Spinal cord Amyotrophic lateral sclerosis Lower motor neuron Anterior grey column Motor neuron Neuron Horn (anatomy) Cranial nerves Anatomical terms of location
49
so
51 References:
52 FA Step 2 CK 9th ed pp 276-277
FA Step 2 CK 8th ed p 264
53

Lock
s
Suspend
0
End Block
Item: 54 of 99 ~ 1 • Mark <:] (:>- Jill ~· ~J
QID: 23508 ..1 Previous Next Lab'V!I!ues Notes Calculator

32 A 28-year-old woman presents to her physician complain ing of painless double vision and a feeling of heaviness in her
eyes. Most recently she is having increased difficulty comlbing her hair and climbing stairs due to "feeling weak." Her
33
symptoms are generally worse toward the end of the day. She presents to the clinic as the first patient of the morning,
34 and on physical exam ination you do not find any neurologic deficits . However, her husband insists that in the evenings she is so
35 exhausted that she can barely keep her eyes open or get up from the couch .
36
37 Which of the following is the most likely site of disease?
38
39 A. Anterior horn cell
40
B. Dopaminergic neuron
41
C. Myelin sheath
42
43 D. Neuromuscular junction
44 E. Sensory ganglia
45

46
47
48
49
50
51
52
53
• 54

Lock
s
Suspend
0
End Block
Item: 54 of 99 ~ 1 • Mark <:] (:>- Jill ~· ~J
QID: 23508 ..1 Previous Next Lab'V!I!ues Notes Calculator

32
33
34
The correct answer is D. 910/o chose this.
Myasthenia gravis (MG) is an autoimmune disease of the· neuromuscu lar junction in which autoantibodies target and bind the
35
acetylcholine receptor, and interfere with neuromuscular transmission . Typically, MG causes weakness and fatigue of proximal
36 muscles, as well as the extraocular and eyelid muscles. Severe cases can lead to fatigue of the respiratory muscles and
37 necessitate mechanical ventilatory support. Treatment of MG is with anticholinesterases such as pyridostigmine.
Myasthenia gravis Autoimmune disease Neuromuscular junction Pyridostigmine Acetylcholine Acetylcholinesterase inhibitor Autoantibody Autoimmunity Acetylcholine receptor
38
Mechanical ventilation Fatigue (medical) Eyelid Muscle weakness Receptor (biochemistry) Thoracic diaphragm ~1uscles of respiration Muscle Anatomical terms of location
39
40 A is not correct. 1 Ofo chose this.
41 The anterior horn cel l is the site of degeneration in amyotrophic lateral sclerosis (ALS) . This results in progressive weakness,
42 muscle wasting, and fasciculations. There would not be any diurnal variation in symptoms with ALS.
Amyotrophic lateral sclerosis Anterior grey column Anatomical terms of location Horn (anatomy) Fasciculation Muscle Muscle atrophy
43
44 B is not correct. 10/o chose this.
45 Symptoms of Parkinson's disease are attributed to the degeneration of dopaminergic neurons in the substantia nigra and their
46
projections into the striatum . Th is leads to progressive weakness and tremor, as well as dementia .
Parkinson's disease Substantia nigra Striatum Dopaminergic Tremor Dementia Neuron
47
48 C is not correct. 6% chose this.
49 MG is not a demyelinating disease, and as such does not affect the myelin sheath . Mu ltipl e sclerosis is the most common
demyelinating process, which often does present with vision changes if there is involvement of the optic nerves . Symptoms
50
can be intermittent, but are generally progressive and do not improve with rest in a consistent manner such as this .
51 Multiple sclerosis Demyelinating disease Myelin Optic nerve

52
E is not correct. 10/o chose this.
53
The sensory ganglia are not a primary site of involvement in the pathogenesis of MG, and there are no sensory deficits in this
54 y oatient.

Lock
s
Suspend
0
End Block
Item: 54 of 99 ~ 1 • Mark <:] (:>- Jill ~· ~J
QID: 23508 ..1 Previous Next Lab'V!I!ues Notes Calculator

32
B is not correct. 10/o chose this.
33
Symptoms of Parkinson's disease are attributed to the degeneration of dopam inergic neurons in the substantia nigra and their
projections into the striatum . This leads to progressive w eakness and tremor, as well as dementia .
34 Parkinson's disease Substantia nigra Striatum Dopaminergic Tremor Dementia Neuron
35
C is not correct. 60/o chose this.
36
MG is not a demyelinating disease, and as such does not affect the myelin sheath . Multiple sclerosis is the most common
37
demyelinating process, which often does present with vision changes if there is involvement of the optic nerves . Symptoms
38 can be intermittent, but are generally progressive and do not improve with rest in a consistent manner such as this .
39 Multiple sclerosis Demyelinating disease Myelin Optic nerve

40 E is not correct. 10/o chose this.


41 The sensory ganglia are not a primary site of involvement in the pathogenesis of MG, and there are no sensory deficits in this
42 patient.
Dorsal root ganglion Ganglion Pathogenesis
43
44
45 Bottom line:
46 Myasthenia gravis is an autoimmune disease of the neuromuscular junction in which autoantibodies target and bind the
47 acetylcholine receptor and interfere with neuromuscular transmission . It typically causes weakness and fatigue of proximal
48 muscles as well as the extraocular and eyelid muscles .
Myasthenia gravis Autoimmune disease Neuromuscular junction Acetylcholine Autoantibody Autoimmunity Acetylcholine receptor Fatigue (medical) Eyelid Muscle weakness
49
Anatomical terms of location Receptor (biochemistry)
50
51
52 References:
53 FA Step 2 CK 9th ed p 274
FA Step 2 CK 8th ed p 261
54

Lock
s
Suspend
0
End Block
Item: 55 of 99 ~ 1 • Mark <:] (:>- Jill ~· ~J
QID: 23510 ..1 Previous Next Lab'V!I!ues Notes Calculator

33 A 9-year-old girl is brought to the outpatient clinic by her mother, w ho r eports the girl has had 5 days of low-grade fevers, ~~AI
headache, and mild neck stiffness. One month ago, she developed reddish discoloration of her leg at the site of a tick bite .
34
35
Damage to what cran ial nerve is responsible for the most common cranial neuropathy associated w ith Lyme disease?
36
37
A. II
38
39 B. VII
40 C. VIII
41
X
42
43 E. XII
44
45
46
47
48
49
so
51
52
53
54
• 55

Lock
s
Suspend
0
End Block
Item: 55 of 99 ~ 1 • Mark <:] (:>- Jill ~· ~J
QID: 23510 ..1 Previous Next Lab'V!I!ues Notes Calculator

The correct answer IS B. 890/o chose th 1s.


33
Crania l neuropathy is a pot ential manifestation of early disseminated Lyme disease. Although any cranial nerve {CN) can
34 become involved, the facial nerve (Bell's pa lsy) is most f requently affect ed . Treatment for early neurologic Lyme disease
35 includes intravenous antibiot ics. Oral antibiotic therapy may be in it iated if the cerebrospinal flu id analysis is negative for
inflammatory changes .
36
Bell's palsy Lyme disease Cerebrospinal fluid Facial nerve Antibiotics Cranial nerves Peripheral neuropathy Intravenous therapy Neurology Skull
37
38 A is not correct. 2 % chose this.
39
There is document ation of optic nerve involvement in up to 3% of cases of Lyme disease with neurologic involvement . Optic
neuritis is a potential sequela . The most commonly involv ed CN, however, is CN VII .
40 Sequela Optic neuritis Lyme disease Optic nerve Facial nerve Neuritis Neurology
41
C is not correct. 6 % chose this.
42
The most commonly affect ed CN in ea rly disseminated Lyme disease is the facial nerve. Although all CNs may be affect ed, CN
43
VII is the most frequ ently involved .
44 Lyme disease Facial nerve

45
D is not correct. 2 % chose this.
46
Crania l neu ropathy is a potential neurolog ic manifestation of early disseminated Lyme disease . Any CN can be involved, but
47 the facial nerve is the most commonly affected .
48 Lyme disease Facial nerve Peripheral neuropathy Neurology Skull

49 E i s not correct. 1 0/o c hose th is.


so The most commonly affected CN in ea rly disseminated Lyme disease is the facial nerve. Although all CNs may be affected, CN
51 VII is the most frequently involved .
Lyme disease Facial nerve
52
53
54 Botto m line :
55 y

a
Lock
s
Suspend
0
End Block
Item: 55 of 99 ~ 1 • Mark <:] (:>- Jill ~· ~J
QID: 23510 ..1 Previous Next Lab'V!I!ues Notes Calculator

• There IS documentation of opt1c nerve mvolvement m up to 3% of cases of Lyme d1sease w1th neurolog1c mvolvement. Opt1c
33 neurit is is a potential sequela. The most commonly involved CN, however, is CN VII .
Sequela Optic neuritis Lyme disease Optic nerve Facial nerve Neuritis Neurology
34
35 C is not correct. 6% ch ose this .
36 The most commonly affected CN in early disseminated Lyme disease is the facial nerve. Although all CNs may be affected, CN
37 VII is the most frequently involved.
Lyme disease Facial nerve
38
39 D is not correct. 20/o chose this .
40 Cranial neuropathy is a potential neurologic manifestation of early disseminated Lyme disease . Any CN can be involved, but
41
the facial nerve is the most commonly affected .
Lyme disease Facial nerve Peripheral neuropathy Neurology Skull
42
43 E is n ot correct. 1 0/o chose t his.

44
The most commonly affected CN in early disseminated Lyme disease is the facial nerve. Although all CNs may be affected, CN
VII is the most frequently involved.
45 Lyme disease Facial nerve
46
47
48
Bottom li ne:
49 The facial nerve {CN VII) is most frequently affected in early dissem inated Lyme disease . Treatment for early neurologic
Lyme disease includes intravenous antibiotics.
so Lyme disease Facial nerve Intravenous therapy Antibiotics Neurology
51
52
53 Refere n ces:
54 FA Step 2 CK 9th ed pp 226-227
FA Step 2 CK 8th ed p 243
55

Lock
s
Suspend
0
End Block
Item: 56 of 99 ~ 1 • Mark <:] (:>- Jill ~· ~J
QID: 24607 ..1 Previous Next Lab'V!I!ues Notes Calculator

34 A previously healthy 26-year-old man presents to the clin ic complain ing of headache, rhinorrhea, nonproductive cough,
fevers, and myalgias for the past 3 days. Mu lt iple members of his family have had a similar illness that resolved without
35
treatment. He is diagnosed with a vira l upper r espiratory infection and sent home with supportive care. The patient returns
36 to the clinic 10 days later with lower extremity weakness that has been progressive for 3 days . He also reports that he feels
37 unusua lly short of breath when conversing with his coworkers. On physical exam, his temperature is 37 .2oc (99 .00F),
38 respiratory rate is 22/min, heart rate is 108/min, and blood pressure is 120/72 mm Hg . Physical exam revea ls 3/5 strength in
the bi lateral lower extremities and 4/5 strength in the upper extremities. He has absent Ach illes and patellar reflexes. Biceps
39
and triceps are dim inished. Routine labs, x-ray of the chest, and CT scan of the head are normal.
40
41 Which of the following is the most appropriate treatment of this patient?
42
43
A. Glucocorticoids
44
B. Interferon-~
45
46 C. Nonsteroidal anti-inflammatory drugs
47
D. Plasma exchange
48
E. Supportive therapy only
49
50
51
52
53
54
55
• 56

Lock
s
Suspend
0
End Block
Item: 56 of 99 ~ 1 • Mark <:] (:>- Jill ~· ~J
QID: 24607 ..1 Previous Next Lab'V!I!ues Notes Calculator

34
The correct answer i s D. 670/o chose this .
35
This patient's presentation is consistent with Guillain-Barre syndr ome {GBS ). GBS is an autoimmune demyeli nating disease of
36 unclear etiology. It is often preceded by a viral upper respiratory infection or diarrheal illness several weeks before. GBS
37 presents w ith ascending paralysis and sensory changes that can involve cranial nerves and respiratory muscles as well as
38 autonomic dysfunction. Neurologic exam typically reveals absent or near-absent reflexes. Support ive care, including
intubation and mechanical ventilation, are sometimes r equired . The main moda lities of therapy include plasma exchange and
39
intravenous immune globulin (IVIG) . The use of both modalit ies combined is not beneficial. The use of glucocorticoids alone
40 or w ith plasma exchange or IVIG is also not beneficial. Patients reco ver sooner when t reated early. Diagnosis is clinical , but a
41 cerebrospinal flu id {CSF) sample wil l often show an elevated protein level wit h normal WBC count
42 (album inocytologic dissociation ). Testing of t he CS F may also reveal presence of the GMl antibody.
Cerebrospinal fluid Demyelinating disease Mechanical ventilation Plasmapheresis Immunoglobulin therapy Cranial nerves Dysautonomia Intravenous therapy Blood plasma
43
Autonomic nervous system Paralysis Glucocorticoid Protein Upper respiratory tract infection Autoimmunity Etiology Autoimmune disease Antibody Respiratory tract infection Intubation
44
Neurology Myelin Infection GMl Virus
45
46 A i s not correct. 1 30/o chose t h is.
47 Glucocorticoids ar e potent anti-inflammatory and immunosuppressive agents used to treat autoimmune diseases, chron ic
inflammation or transplant rejection. In the central nervous system, glucoco rt icoids ar e used to minimize inflammation
48
follow ing trauma , stroke, infection , and seizur es . However, t hey have not been shown to be beneficial in t he treatment of this
49 patient's condition .
50 Central nervous system Transplant rejection Glucocorticoid Anti-inflammatory Inflammation Immunosuppression Autoimmune disease Nervous system Autoimmunity Stroke

51 Epileptic seizure Immunosuppressive drug

52
B is not correct. 60/o chose t h is.
53 Interferon - ~ is a cytokine used to t r eat m ult iple sclerosis {MS) . MS is the most common immune- mediated inflammatory
54 demyelinating disease of the centra l nervous system {CNS) . MS typica lly presents in young female patients and is
55 characterized by r elapsing and remitting symptoms of CNS dysfunction such as unilateral visual loss, diplopia,
motor weakness, and ataxia. This patient's presentation is not consistent with a diagnosis of MS . Treatment wit h i nterferon-~
56

Lock
s
Suspend
0
End Block
Item: 56 of 99 ~ 1 • Mark <:] (:>- Jill ~· ~J
QID: 24607 ..1 Previous Next Lab'V!I!ues Notes Calculator

• Ep1lept1c se1zure Immunosuppressive drug


34
B is not correct. 60/o chose this .
35
Interferon - ~
is a cytokine used to treat mult iple sclerosis ( MS) . MS is the most common immune- mediated inflammatory
36 demyelinating disease of the centra l nervous system (CNS) . MS typica lly presents in young female patients and is
37 characterized by relapsing and remitting symptoms of CNS dysfunction such as unilateral visual loss, diplopia,
38 motor weakness, and ataxia. This patient's presentation is not consistent with a diagnosis of MS . Treatment wit h i nterferon-~
has been reported to be effective in individual cases of this patient's condit ion, but no benefit has been demonstrated in
39
controlled studies.
40 ~1ultiple sclerosis Diplopia Cytokine Central nervous system Demyelinating disease Ataxia Nervous system Immune system Autoimmunity Inflammation Myelin
41 Immune-mediated inflammatory diseases Visual impairment
42
C is not correct. 2% ch ose this .
43
Nonsteroidal anti-inflammatory drugs ( NSAIDs) are used as analgesic and antipyretic agents. However, there is no role for
44 NSAIDs in the treatment of t his patient's condit ion .
45 Antipyretic Analgesic Anti-inflammatory Nonsteroidal anti-inflammatory drug

46
E is n ot correct. 1 2% chose this .
47 Supportive care is extremely important in this patient's condit ion, due to the potential for rapid development of respiratory
48 failure and autonomic dysfunction in advanced stages of the disease . However, these patients also require addit ional therapy
49 in order to accelerate recovery. Supportive treatment alone is inappropriate in this patient.
Dysautonomia Respiratory failure Autonomic nervous system
50
51
52 Bottom li ne:
53 Guillain-Barre syndrome is an autoimmune polyradiculopathy, typica lly of an ascending nature, that can be severe and often
54 requ ires intensive care unit and ventilatory care . Treatment of choice is with plasma exchange or IVIG .
Plasmapheresis Autoimmunity Autoimmune disease Radiculopathy Intensive care u nit Blood plasma Intensive care medicine
55
56

Lock
s
Suspend
0
End Block
Item: 56 of 99 ~ 1 • Mark <:] (:>- Jill ~· ~J
QID: 24607 ..1 Previous Next Lab'V!I!ues Notes Calculator

B is not correct. 6 0/o chose t his.


34
Interferon - ~ is a cytokine used to treat mult iple sclerosis (MS) . MS is the most common immune- mediated inflammatory
35 demyelinating disease of the centra l nervous system (CNS) . MS typica lly presents in young female patients and is
36 characterized by relapsing and remitting symptoms of CNS dysfunction such as unilateral visual loss, diplopia,
37 motor weakness, and ataxia . This patient's presentation is not consistent with a diagnosis of MS . Treatment with i nterferon-~
has been reported to be effective in ind ividua l cases of this patient's condition, but no benefit has been demonstrated in
38
controlled studies.
39 Multiple sclerosis Diplopia Cytokine Central nervous system Demyelinating disease Ataxia Nervous system Immune system Autoimmunity Inflammation ~1yelin

40 Immune-mediated inflammatory diseases Visual impairment


41
C i s not correct. 2% ch ose this.
42
Nonsteroidal anti-inflammatory drugs (NSAIDs) are used as analgesic and antipyretic agents. However, there is no role for
43 NSAIDs in the treatment of this patient's condition.
44 Antipyretic Analgesic Anti-inflammatory Nonsteroidal anti-inflammatory drug

45
E i s n ot correct. 1 2% ch ose this.
46
Supportive care is extremely important in this patient's condition, due to the potential for rapid development of respiratory
47 failure and autonomic dysfunction in advanced stages of the disease . However, these patients also require add it ional therapy
48 in order to accelerate recovery. Supportive treatment alone is inappropriate in this patient.
Dysautonomia Respiratory failure Autonomic nervous system
49
50
51 Bottom Li ne:
52 Guillain-Barre syndrome is an autoimmune polyradiculopathy, typica lly of an ascending nature, that can be severe and often
53 requ ires intensive care unit and ventilatory care . Treatment of choice is with plasma exchange or IVIG .
Plasmapheresis Autoimmunity Autoimmune disease Radiculopathy Intensive care unit Blood plasma Intensive care medicine
54
55
56

Lock
s
Suspend
0
End Block
Item: 57 of 99 ~ 1 • Mark <:] (:>- Jill ~· ~J
QID: 23295 ..1 Previous Next Lab'V!I!ues Notes Calculator
.
35 A 16-year-old boy is brought to the emergency department ( ED) by paramed ics w hile seizing . The neighbors who called 9- 1•AI
1-1 report that he has a long-standing seizure disorder and add that they believe he often abuses cocaine; they are A
36
unaware of how long he was seizing before they found him 20 minutes ago. On arrival at the ED, the patient's pulse is
37 125/ m in, blood pressure is 160/ 100 mm Hg, temperature is 38 .9°C (102°F), and respiratory rate is 22/ min . On examination ,
38 the patient's arms are extended rigidly at his side, and he is arching his back rhythm ically. He appears to be aspirating .
39
Nasopharyngeal intubation is successful on the second att empt by the ED resident on call; intravenous access is obtained and
venous blood is drawn for measurement of glucose and el ectrolytes .
40
41
Which of the following is the most appropriate next step in management?
42
43
A. Intravenous glucose, thiamine, and naloxone, and oxygen via face mask
44
45 B. Intravenous lorazepam and a loading dose of valproic acid
46 C. Intravenous phenobarbital
47
D. Neuromuscular blockade
48
49 E. Placement of electroencephalographic monitor
so
51
52
53
54
55
56
• 57

Lock
s
Suspend
0
End Block
Item: 57 of 99 ~ 1 • Mark <:] (:>- Jill ~· ~J
QID: 23295 ..1 Previous Next Lab'V!I!ues Notes Calculator
.
35
The correct a nswer i s B. 7 90/o chose t h is.
Status epilepticus (SE) is defined as one continuous seizure for >5 minutes or two seizures in a ro w without rega ining
36
consciousness or an incomplete regaining of consciousne·ss between both attacks. Practically speaking, anyone who is brought
37 to an ED who has been seizing for >5 minutes w ill be treated as having SE. SE is a medical emergency with a w ide range of
38 potential complications (eg, neurona l death, rhabdomyolysis, and lactic acidosis) and 20% mortality. Following a basic
39 primary survey (the "ABCs": Airway, Breathing, and Circulation) and venous blood draw for glucose and electrolytes, the
next step in management is the administration of a short-acting benzodiazepine like lorazepam and a loading dose of va lproic
40
acid . Of note, 2010 guidelines from the American Heart Association suggest that CABs replace ABCs to emphasize the
41 importance of early compressions in a resuscitation for cardiac arrest.
42 Benzodiazepine Rhabdomyolysis Lorazepam Valproate Status epilepticus Lactic acidosis American Heart Association Cardiac arrest Acidosis Medical emergency Epileptic seizure

43 Glucose Electrolyte Cardiopulmonary resuscitation

44 A i s not correct. 5% ch ose this.


45 At many centers, intra venous glucose, thiamine, and naloxone, and oxygen via face mask are given presumptively to all
46 unconscious patients who arrive in the ED. Although these therapies are associated with little risk and may be appropriate in
47 this patient given his high-risk behavior with il licit drugs, the next logical step in a patient with presumed status epilepticus
would be the administration of medication capable of ending seizure activity. Also, seeing that this patient is known to have
48
long-standing epilepsy, it is less likely that he is having alcohol withdrawal seizures .
49 Naloxone Thiamine Status epilepticus Alcohol withdrawal syndrome Epilepsy Delirium tremens Intravenous therapy Glucose Recreational drug use Epileptic seizure Alcohol Oxygen
so Alcoholic beverage
51
C i s not corr ect. 1 30/o chose this.
52
If intra venous benzodiazepines and phenytoin are ineffective and seizures continue for more than 10 m inutes, intravenous
53 phenobarbital should be started, but it is not a first-line agent.
54 Phenobarbital Phenytoin Benzodiazepine Intravenous therapy Epileptic seizure

55 D is not correct. 3% chose this.


56 Brief neu romuscular blockade may become necessary if rapid sequence induction intubation is requ ired to secure the patient's
57 y airway, However, in this patient w ith a secure airway, further blockade is unnecessary unless the patient develops respiratory y

Lock
s
Suspend
0
End Block
Item: 57 of 99 ~ 1 • Mark <:] (:>- Jill ~· ~J
QID: 23295 ..1 Previous Next Lab'V!I!ues Notes Calculator
:.o • I I it~T• .t"

35 C is not correct. 1 30/o chose this.


36 If intravenous benzodiazepines and phenytoin are ineffective and seizures continue for mor e than 10 m inutes, intravenous
37 phenobarbital should be started, but it is not a first-line agent.
Phenobarbital Phenytoin Benzodiazepine Intravenous therapy Epileptic seizure
38
39 D is not correct. 3% chose this.
40 Brief neuromuscular blockade may become necessary if r apid sequence induction intubation is required to secure the patient's
41 airway. However, in this patient w ith a secur e airway, further blockade is unnecessary unless the patient develops respiratory
distress and requires paralysis and mechanical ventilation or rhabdomyolysis.
42
Rhabdomyolysis Rapid seQuence induction Neuromuscular-blocking drug Mechanical ventilation Neuromuscular junction Respiratory tract Intubation Paralysis Dyspnea
43
Ventilation (physiology)
44
45 E is not correct.
46
Complex studies such as electroencephalographic monitoring and brain imag ing, although they may be useful in establishing
the diagnosis, are best deferr ed until the patient has been stabilized.
47 Electroencephalography Neuroimaging Human brain Brain Complex systems
48
49
so Bottom li ne:
51 Following a basic primary survey, the next step in management in a patient in status epilepticus is the administration of a
short-acting benzod iazepine and a loading dose of phenytoin.
52 Benzodiazepine Phenytoin Status epilepticus Loading dose
53
54
55 References:
56 FA Step 2 CK 9th ed p 271
FA Step 2 CK 8th ed p 257
57

Lock
s
Suspend
0
End Block
Item: 58 of 99 ~ 1 • Mark <:] (:>- Jill ~· ~J
QID: 23464 ..1 Previous Next Lab'V!I!ues Notes Calculator

36 A 65-year-old man is brought to the emergency department by ambulance after the acute loss of movement on the right ~~AI
side of his body. He has been unable to speak for the past 2 hours since the onset of his right-sided paralysis and does not
37
make appropriate responses to commands .
38
39
Which lesion best expla ins his aphasia?
40
41
A. Arcuate fasciculus infarction
42
43
B. Hypoperfusion of the left dorsolateral prefrontal cortex
44 C. Lesion of the left supplementary motor area
45
D. Occlusion of the left middle cerebral artery
46
47
E. Right superior cerebellar infarction
48
49
50
51
52
53
54
55
56
57
• 58

Lock
s
Suspend
0
End Block
Item: 58 of 99 ~ 1 • Mark <:] (:>- Jill ~· ~J
QID: 23464 ..1 Previous Next Lab'V!I!ues Notes Calculator

36 The correct answer i s D. 770/o chose this.


37 The patient suffers from a global aphasia, characterized by a global impairment in language comprehension as well
38 as fluency and word repetition . The magnetic resonance angiogram shows total occlusion of the left middle cerebral
artery (MCA), which supplies the lateral aspect of the bra in . Blockage of the left MCA can result in contralateral
39
right-sided hemiparesis and global aphasia since the left hemisphere controls language in most people . Both
40 Broca's and Wernicke's areas are supplied by the MCA.
41 Hemiparesis Aphasia Middle cerebral artery Angiography Global aphasia Lateralization of brain function Magnetic resonance angiography Human brain Sentence processing

42 Expressive aphasia Contralateral Brain Receptive aphasia Vascular occlusion

43
A i s not correct. 7 % ch ose this.
44
An infarct in the arcuate fascicu lus would resu lt in a conduction aphasia . Conduction aphasia is marked by difficulty with
45 repetition while fluency and comprehension are preserved . Both the patient's presentation and magnetic resonance
46 angiogram are consistent w ith a large lesion to the left hem isphere resulting in a global aphasia .
Conduction aphasia Arcuate fasciculus Aphasia Infarction Global aphasia Lateralization of brain function Lesion Angiography
47
48 B is not correct. 8 0/o chose this .
49 Broca's aphasia, which is often caused by ischemic damage to the left dorsolateral prefrontal cortex, is characterized by
50
difficulty with language production while comprehension is preserved. The lesion would most likely be a focal lesion in Broca's
area rather than the large affected area shown on the magnetic resonance angiogram .
51 Broca's area Prefrontal cortex Aphasia Dorsolateral prefrontal cortex Expressive aph asia Angiography Ischemia Lesion Anatomical terms of location Cerebral cortex Cortex (anatomy)
52
C i s not correct. 7 % chose this.
53
A lesion of the supplementary motor area (SMA) would result in temporary mutism, as the SMA is thought to be involved in
54
the voluntary control of respiration necessary for speech production . The lesion on the magnetic resonance angiogram in this
55 patient suggests a large insult to the left hem isphere, making a globa l aphasia more li kely than temporary mutism for his
56 inability to speak.
Supplementary motor area Aphasia Angiography Lesion Magnetic resonance angiography Respiration (physiology) Lateralization of brain function
57
58 E i s not correct. 1 0/o c hose th is.

Lock
s
Suspend
0
End Block
Item: 58 of 99 ~. 1 • Mark -<1 t.> Jil ~' ~~
QID: 23464 J. Previous Next Lab ~ues Notes Calculator

36 The correct answer is D.


37 The patient suffers from a gl anguage comprehension as well
38 as fluency and word repetiti lusion of the left middle cerebral
artery (MCA), which supplies A can result in contralateral
39
right-sided hemiparesis and age in most people. Both
40 Broca's and Wernicke's areas
41 Hemiparesis Aphasia Middle cerebral e angiography Human brain Sentence processing

42 Expressive aphasia Contralateral Brain

43
A is not correct. 70/o ch
44
An infarct in the arcuate aphasia is marked by difficulty with
45 repetition while fluency and ntation and magnetic resonance
46 angiogram are consistent obal aphasia .
Conduction aphasia Arcuate fasciculus y
47
48 B is not correct. SO/o •
49 Broca's aphasia, which is refrontal cortex, is characterized by
50
difficulty with language prod uld most likely be a focal lesion in Broca's
area rather than the large a
51 Broca's area Prefrontal cortex Aphasia atomical terms of location Cerebral cortex Cortex (anatomy)
52
C is not correct. 7% chose
53
A lesion of the supplementa Image courtesy of Watanabe et at , J Med Case Reports as the SMA is thought to be involved in
54
the voluntary control of 2009; 3. 8389.
he magnetic resonance angiogram in this
55 patient suggests a large ins e likely than temporary mutism for his
56 inability to speak.
Supplementary motor area Aphasia zation of brain function
57
58 ~
E is not correct. 1 Ofo chose

Lock
s
Suspend
0
End Block
Item: 58 of 99 ~ 1 • Mark <:] (:>- Jill ~· ~J
QID: 23464 ..1 Previous Next Lab'V!I!ues Notes Calculator

36
B is not correct. 80/o chose this .
37
Broca's aphasia, w hich is often caused by ischemic damage to the left dorsolateral prefrontal cortex, is characterized by
difficulty with language production while comprehension is preserved. The lesion would most likely be a focal lesion in Broca's
38 area rather than the large affected area show n on the magnetic resonance angiogram .
39 Broca's area Prefrontal cortex Aphasia Dorsolateral prefrontal cortex Expressive aph asia Angiography Ischemia Lesion Anatomical terms of location Cerebral cortex Cortex (anatomy)

40
C is not correct. 7 % chos e this .
41
A lesion of the supplementary motor area (SMA) would result in temporary mutism, as the SMA is thought to be involved in
42 the voluntary control of r espiration necessary for speech production . The lesion on the magnetic resonance angiogram in this
43 patient suggests a lar ge insult to the left hem ispher e, making a globa l aphasia more li kely than temporary mutism for his
44
inability to speak.
Supplementary motor area Aphasia Angiography Lesion Magnetic resonance angiography Respiration (physiology) Lateralization of brain function
45
46 E is not correct. 1 0/o c hose th is.
47
Infarction of the superior cerebellum, particularl y if near the vermis, wou ld result in dysarthria, not the global aphasia with
w hich the patient is presenting. The magnetic r esonance angiogram in this case confirms a lesion of the MCA .
48 Dysarthria Cerebellum Aphasia Angiography Magnetic resonance angiography Lesion Infarction Global aphasia
49
50
51
Bottom line :
52 Blockage of the left MCA can result in contralateral right-sided hemiparesis and global aphasia, because the left hemisphere
controls language in most people.
53 Hemiparesis Aphasia Global aphasia Contralateral Lateralization of brain function
54
55
56 References:
57 FA Step 2 CK 9th ed pp 288-289
FA Step 2 CK 8th ed p 274
58

Lock
s
Suspend
0
End Block
Item: 59 of 99 ~ 1 • Mark <:] (:>- Jill ~· ~J
QID: 23776 ..1 Previous Next Lab'V!I!ues Notes Calculator
.
37 A 16-year-old girl presents to the emergency department after hitting her head on the side of a pool during a failed fli p
turn . Immediately after the accident, the patient was able to exit the pool on her own. The lifeguard on duty noted a
38
pulsating, bleeding wound in the scalp. He applied direct pressure to the area and proceeded to call for help.
39 Approximately 2 m inutes after the accident the patient began a progressive loss of consciousness .
40
41 Which of the following vascular structures was most likely damaged?
42
43 A. Basilar artery
44
B. Bridging vein
45
46 C. Circle of Wil lis
47 D. Middle meningeal artery
48
E. Posterior cerebral artery
49
so
51
52
53
54
55
56
57
58
• 59

Lock
s
Suspend
0
End Block
Item: 59 of 99 ~ 1 • Mark <:] (:>- Jill ~· ~J
QID: 23776 ..1 Previous Next Lab'V!I!ues Notes Calculator
.
37
38 The correct answer i s D. 850/o chose this .
39 The patient's history of trauma to the sku ll with bleeding and a period of lucency prior to loss of consciousness are strongly
predictive of an epidural hematoma. The most common site of vascular disruption in this condition is a branch of the middle
40
meningeal artery that courses between the dura mater and squamous portion of the temporal bone.
41 Dura mater Epidural hematoma Middle meningeal artery Temporal bone Hematoma Squamous part of temporal bone Human skull Bone Epidural administration Skull
42 Squamous epithelial cell Blood vessel Major trauma Meninges
43
A i s not correct. 2% ch ose this .
44
The basilar artery is a very large vessel buried relatively deeply in the brain. Its position makes it relatively unlikely to be
45
damaged in such a trauma. Should it be damaged, the patient li kely wou ld have failed to experience the extended lucid
46 interval with gradual decl ine, and may have vomited secondary to a rapid increase in intracranial pressure .
47 Basilar artery Intracranial pressure Lucid interval Brain

48 B is not correct. 1 00/o chose t h is.


49 The bridging veins are delicate structures that drain blood into the superior sagittal sinus . These vessels are ruptured in the
so context of subdura l bleeding, which is generally secondar y to head trauma. Despite commonly arising from trauma, the
51
presence of a pulsating bleed and period of lucency followed by gradual loss of consciousness indicate an epidural bleed as
the most likely occurrence in th is case. Subdural hemorrhage may present with loss of consciousness at the time of the
52 trauma, but the subsequent course is slower, with progressive headache, paresis, and changes in personality occurring over
53 days, not hours.
Superior sagittal sinus Subdural hematoma Bridging veins Paresis Headache Head injury Bleeding Syncope (medicine) Sagittal plane Unconsciousness
54
55 C i s not corr ect. 20/o ch ose t his.
56 The circle of Willis is formed at the convergence of the blood supplied by the internal carotid artery and the basilar artery. The
57 posterior commun icating artery, which physically joins t he two circulations, originates at the trifurcation of the anterior
cerebra l artery, middle cerebral artery, and internal carotid . The circle of Willis is a common site for cerebral aneurysms .
58
Rupture of an aneurysm would result in a subarachnoid hemorrhage, which classically presents as sudden onset of the "worst
59 y hQ::orl::orhQ ,..f ~" lif"' 11

Lock
s
Suspend
0
End Block
Item: 59 of 99 ~ 1 • Mark <:] (:>- Jill ~· ~J
QID: 23776 ..1 Previous Next Lab'V!I!ues Notes Calculator
.
37
38 The correct answer i s D. 850/o chose this .
39 The patient's history of trauma to the sku ll with bleeding and a period of lucency prior to loss of consciousness are strongly
predictive of an epidural hematoma. The most common site of vascular disruption in this condition is a branch of the middle
40
meningeal artery that courses between the dura mater and squamous portion of the temporal bone.
41 Dura mater Epidural hematoma Middle meningeal artery Temporal bone Hematoma Squamous part of temporal bone Human skull Bone Epidural administration Skull
42 Squamous epithelial cell Blood vessel Major trauma Meninges
43
A i s not correct. 2% ch ose this .
44
The basilar artery is a very large vessel buried relatively deeply in the brain. Its position makes it relatively unlikely to be
45
damaged in such a trauma. Should it be damaged, the patient li kely wou ld have failed to experience the extended lucid
46 interval with gradual decl ine, and may have vomited secondary to a rapid increase in intracranial pressure .
47 Basilar artery Intracranial pressure Lucid interval Brain

48 B is not correct. 1 00/o chose t h is.


49 The bridging veins are delicate structures that drain blood into the superior sagittal sinus . These vessels are ruptured in the
so context of subdura l bleeding, which is generally secondar y to head trauma. Despite commonly arising from trauma, the
51
presence of a pulsating bleed and period of lucency followed by gradual loss of consciousness indicate an epidural bleed as
the most likely occurrence in th is case. Subdural hemorrhage may present with loss of consciousness at the time of the
52 trauma, but the subsequent course is slower, with progressive headache, paresis, and changes in personality occurring over
53 days, not hours.
Superior sagittal sinus Subdural hematoma Bridging veins Paresis Headache Head injury Bleeding Syncope (medicine) Sagittal plane Unconsciousness
54
55 C i s not corr ect. 20/o ch ose t his.
56 The circle of Willis is formed at the convergence of the blood supplied by the internal carotid artery and the basilar artery. The
57 posterior commun icating artery, which physically joins t he two circulations, originates at the trifurcation of the anterior
cerebra l artery, middle cerebral artery, and internal carotid . The circle of Willis is a common site for cerebral aneurysms .
58
Rupture of an aneurysm would result in a subarachnoid hemorrhage, which classically presents as sudden onset of the "worst
59 y hQ::orl::orhQ ,..f ~" lif"' 11

Lock
s
Suspend
0
End Block
Item: 59 of 99 ~ 1 • Mark <:] (:>- Jill ~· ~J

- .. .. . . ..
..1
.. . .. .. ... . . .. . .. . .
QID: 23776 Previous Next Lab'V!I!ues Notes Calculator
. .
' . .. . . .. . .. . .
37
days, not hours.
Superior sagittal sinus Subdural hematoma Bridging veins Paresis Headache Head injury Bleeding Syncope (medicine) Sagittal plane Unconsciousness
38
39
C is not correct. 20/o chose this.
40
The circle of Willis is formed at the convergence of the blood supplied by the internal carotid artery and the basilar artery. The
posterior commun icating artery, which physically joins t he two circulations, originates at the trifurcation of the anterior
41
cerebra l artery, middle cerebral artery, and internal carotid . The circle of Willis is a common site for cerebral aneurysms .
42 Rupture of an aneurysm would result in a subarachnoid hemorrhage, which classically presents as sudden onset of the "worst
43 headache of my life."
Subarachnoid hemorrhage Circle of Willis Basilar artery Anterior cerebral artery Posterior communicating artery Middle cerebral artery Intemal carotid artery Aneurysm Carotid artery
44
Bleeding Headache Common carotid artery
45
46 E is n ot correct. 1 0/o chose th is.
47 The posterior cerebral artery is a very large vessel buried relatively deeply in the brain. Its position makes it relatively
48
unlikely to be damaged in such a trauma. If it were damaged, the patient li kely would have failed to experience the extended
lucid interval with gradual decline, and may have vomited secondary to a rapid increase in intracranial pressure.
49 Posterior cerebral artery Intracranial pressure Lucid interval Brain
so
51
52
Bottom li ne:
53 Trauma to the skull w ith bleeding and a period of lucency prior to loss of consciousness are strongly predictive of an epidural
hematoma, generally caused by disruption of the middle meningeal artery.
54 Epidural hematoma Middle meningeal artery Hematoma Epidural administration Human skull Unconsciousness Meninges Skull Syncope (medicine)
55
56
57 References:
58 FA Step 2 CK 9th ed p 263
FA Step 2 CK 8th ed pp 247-249
59

Lock
s
Suspend
0
End Block
Item: 60 of 99 ~ 1 • Mark <:] (:>- Jill ~· ~J
QID: 24443 ..1 Previous Next Lab'V!I!ues Notes Calculator

38 A 23-year-old female presents to her primary care physic ~ an with complaints of recurrent facial pains. The facial pains
have been occurring for one month and are described as severe with an electric shock-like and stabbing quality. Most
39
episodes last several seconds, with the longest attack lasting approximately 1 minute. The pain is bilateral and rad iates to
40 the right and left sides of the jaw. The patient denies a specific trigger for painful episodes. She has not noticed any rashes or
41 facial weakness but recalls an episode of sudden, painful vision loss in her right eye that occurred one year ago and resolved
42 over the course of two weeks .
43
44
Which of the following is this patient's most likely underlying problem?
45
46 A. Cluster headache
47 B. Giant cell arteritis
48
C. Herpes zoster
49
50
D. Multiple sclerosis
51 E. Temporomandibular disorder
52
53
54
55
56
57
58
59
• 60

Lock
s
Suspend
0
End Block
Item: 60 of 99 ~ 1 • Mark <:] (:>- Jill ~· ~J
QID: 24443 ..1 Previous Next Lab'V!I!ues Notes Calculator

38
39
40
The correct answer is D. 520/o chose this.
41
This patient is presenting with trigeminal neuralgia. Mult iple sclerosis is associated with trigeminal neuralgia, in which case it
42 is termed symptomatic trigem inal neuralgia as it is a symptom of another illness . Of note, patients with mult iple sclerosis
43 tend to have bilateral trigem inal neuralgia rather than unilateral neuralgia. Mult iple sclerosis can lead to demyelination of one
44 or more of the trigeminal nerve nuclei or cause other structura l lesions of the bra instem. I n mu lt iple sclerosis, the plaque of
demyelination typically occurs in the root entry zone of the trigemina l nerve . This patient also had an episode of unilatera l,
45
painful vision loss, likely due to optic neurit is. Other causes of symptomatic trigem inal neuralgia include compression by
46 tortuous arteries, acoustic schwannomas, and posterior fossa men ingiomas . First-li ne med ical treatment is with
47 carbamazepine.
Optic neuritis Carbamazepine Multiple sclerosis Trigeminal neuralgia Trigeminal nerve Brainstem Demyelinating disease Neuralgia Posterior cranial fossa Meningioma Neuritis Symptom
48
Trigeminal nerve nuclei Visual impairment Schwannoma Nucleus (neuroanatomy) Artery
49
50 A i s not correct. 7% ch ose this.
51 Cluster headaches present with severe unilatera l orbita l or temporal pa in that is commonly accompan ied by autonomic
52 symptoms, such as lacrimation, m iosis, conjunctival injection, congestion, or rh inorrhea . Episodes are recurrent and may
switch laterality. Cluster headaches are most commonly seen in men . Cluster headaches may present as part of the cluster-tic
53
syndrome, which also manifests symptoms of trigemina l neuralgia. However, the sequence of symptoms seen in our patient,
54 her sex, and the lack of recurrence of orbital pain makes th is diagnosis less likely.
55 Miosis Trigeminal neuralgia Rhinorrhea Tears Cluster headache Conjunctiva Autonomic nervous system Conjunctivitis Neuralgia Trigeminal nerve

56 B is not correct. 1 60/o chose t h is.


57 The pain associated with temporal arterit is is usua lly persistent, bilateral, and temporal. Patients also typica lly complain of
58 jaw claud ication when chewing and may also have system ic signs including fever, weight loss, and fatigue. Patients are
59 almost always >50 years old, wh ich makes this diagnosis less likely.
Giant-cell arteritis Claudication Fatigue (medical) Weight loss Jaw Fever
60 y

Lock
s
Suspend
0
End Block
Item: 60 of 99 ~ 1 • Mark <:] (:>- Jill ~· ~J
QID: 24443 ..1 Previous Next Lab'V!I!ues Notes Calculator
g g • J y g g

38
B is not correct. 160/o chose this.
39
The pain associated with temporal arteritis is usua lly persistent, bilateral, and temporal. Patients also typica lly complain of
40 jaw claudication when chewing and may also have systemic signs including fever, weight loss, and fatigue. Patients are
41 almost always >50 years old, wh ich makes this diagnosis less likely.
Giant-cell arteritis Claudication Fatigue (medical) Weight loss Jaw Fever
42
43 Cis not correct. 110/o chose this.
44 Herpes zoster may induce a trigem inal neuropathy during both acute infection and post-active infection. In active herpetic
45 zoster, patient experiences sharp, shock-like pain in the trigeminal dermatome, which precedes a vesicular rash . I n post-
herpetic neuralgia, the patient experiences continued shock- like sensations months after the in it ial flare-up without the
46
development of a dermatomal rash . Affected patients are typically older in age or immunocompromised, making this
47 diagnosis less likely for our patient.
48 Dermatome (anatomy) Shingles Immunodeficiency Postherpetic neuralgia Peripheral neuropathy Herpes simplex Neuralgia Somite Trigeminal nerve Rash

49 Vesicle (biology and chemistry) Infection

50 E is not correct. 14% chose this.


51 Temporomandibular disorder {TMD) is commonly triggered by chewing or jaw clenching . Patients may experience a dull/sore
52 facial pain, headaches, ear fu llness, orbital pain, shoulder pain, and dizziness. Given that our patient has no specific trigger
53
for her facia l pain, the diagnosis of TMD is less likely.
Temporomandibular joint dysfunction Trismus Jaw
54
55
56 Bottom Line:
57 Trigeminal neuralg ia typica lly presents as brief episodes of unilateral, lancinating pain lasting a few seconds to minutes .
58 Trigeminal neuralgia is associated with mu ltiple sclerosis, in which case trigeminal neuralgia is typically bilateral.
Multiple sclerosis Trigeminal neuralgia Neuralgia Trigeminal nerve
59
60

Lock
s
Suspend
0
End Block
Item: 60 of 99 ~ 1 • Mark <:] (:>- Jill ~· ~J
QID: 24443 ..1
-.. . .. . . .. ..
Previous
..
Next
.-. .
.
Lab'V!I!ues
. -. Notes
.
Calculator
.. . . .. . . .. . .. . .
almost always >50 years old, which makes this diagnosis less likely.
38
Giant-cell arteritis Claudication Fatigue (medical) Weight loss Jaw Fever
39
40 C is not correct. 110/o chose this.
41
Herpes zoster may induce a trigeminal neuropathy during both acute infection and post-active infection. In active herpetic
zoster, patient experiences sharp, shock-li ke pain in the trigeminal dermatome, which precedes a vesicu lar rash . In post-
42
herpetic neuralgia, the patient experiences continued shock- like sensations months after the in itial flare-up without the
43 development of a dermatomal rash . Affected patients are typically older in age or immunocompromised, making th is
44 diagnosis less likely for our patient.
Dermatome (anatomy) Shingles Immunodeficiency Postherpetic neuralgia Peripheral neuropathy Herpes simplex Neuralgia Somite Trigeminal nerve Rash
45
Vesicle (biology and chemistry) Infection
46
47 E is not correct. 14% chose this.
48 Temporomandibular disorder {TMD) is commonly triggered by chewing or jaw clenching . Patients may experience a dull/sore
49
facial pain, headaches, ear fullness, orbital pain, shoulder pain, and dizziness. Given that our patient has no specific trigger
for her facial pain, the diagnosis of TMD is less likely.
50 Temporomandibular joint dysfunction Trismus Jaw
51
52
53
Bottom li ne:
54 Trigeminal neuralgia typically presents as brief episodes of unilateral, lancinating pain lasting a few seconds to m inutes .
Trigeminal neuralgia is associated with multiple sclerosis, in which case trigeminal neuralgia is typically bilateral.
55 Multiple sclerosis Trigeminal neuralgia Neuralgia Trigeminal nerve
56
57
58 References:
59 FA Step 2 CK 9th ed p 275
FA Step 2 CK 8th ed pp 263-263
60

Lock
s
Suspend
0
End Block
Item: 61 of 99 ~ 1 • Mark <:] (:>- Jill ~· ~J
QID: 23265 ..1 Previous Next Lab'V!I!ues Notes Calculator
.
39 A 50-year-old woman presents to her neurologist for a folllow- up visit. Since 6 months ago, she noticed a gradual onset of ~~AI
weakness in her arms and difficulties with walking . Amyot rophic latera l sclerosis was diagnosed subsequently. She works
40
in the hospitality industry. Recently she has noticed drooling, wh ich is beginning to interfere with her job . She is still
41 searching for a suitable long-term career in view of the recent diagnosis. In the meantime, she wishes to have the drooling
42 controlled as her job requires her to engage with customers.
43
44 Which of the following is the best management?
45
46 A. Administer glycopyrrolate, an anticholinergic, 0 .1-0.2 mg orally two to three times a day
47
B. Baclofen, a y-aminobutyric acid analogue muscle !relaxant, 5-10 mg two to three t imes a day
48
C. Carbamazepine, an antiepileptic, 200 mg twice daily
49
so D. Consult psychiatry to rule out depression
51 E. Diphenhydramine, an antihistam ine, 50-100 mg at night
52
53
54
55
56
57
58
59
60
• 61

Lock
s
Suspend
0
End Block
Item: 61 of 99 ~ 1 • Mark <:] (:>- Jill ~· ~J
QID: 23265 ..1 Previous Next Lab'V!I!ues Notes Calculator
.
39
The correct answer is A. 750/o chose this.
40
Amyotrophic lateral sclerosis is a slowly progressive, generalized motor muscle paralysis involving both lower and upper
neurons that typical ly progresses to death in 3-5 years. Drooling is a common symptom and is due to facial muscle weakness
41
and reduced swallowing ability rather than increased sec1retions . However, anticholinerg ic drugs such as glycopyrrolate can be
42 useful in decreasing secretions in ord er to manage these symptoms.
43 Anticholinergic Amyotrophic lateral sclerosis Glycopyrronium bromide Neuron Paralysis Muscle weakness Symptom Drooling

44 B is not correct. 100/o chose this.


45 Baclofen is a GABA agonist used for treating muscle spasm in different disorders, including amyotrophic lateral sclerosis,
46 multiple sclerosis, and cerebral palsy. It is not used to treat drooling .
Amyotrophic lateral sclerosis Multiple sclerosis Cerebral palsy Baclofen Gamma-Aminobutyric acid Spasm Agonist GABA receptor agonist Muscle
47
48 C is not correct. 3% chose this.
49 Carbamazepine is an antiepileptic but it is also used to treat muscle spasms and cramps in amyotrophic lateral sclerosis.
Carbamazepine Amyotrophic lateral sclerosis Anticonvulsant Spasm Muscle
so
51 D is not correct. 2% chose this.
52 Although depression, pseudobu lbar symptoms (outbursts of laughter or tearfu lness), and cognitive loss are symptoms in
53
amyotrophic lateral sclerosis, this patient's current problem is medical and does not requ ire psychiatric evaluation .
Amyotrophic lateral sclerosis Major depressive disorder Depression (mood) Pseudobulbar palsy
54
55 E is not correct. 10% chose this.
56 Diphenhydramine is a sedative and is potentia lly a usefu l treatment for insomnia, which is very common in patients with
amyotrophic lateral sclerosis . Sedatives should be used sparingly in these patients. Although diphenhydramine has
57
anticholinerg ic effects, it is not used for treating sia lorrhea . Other agents with anticholinergic properties that can be used are
58 atropine and hyoscyam ine .
59 Anticholinergic Hyoscyamine Diphenhydramine Amyotrophic lateral sclerosis Atropine Sedative Hypersalivation Insomnia

60
61 a-••-_... 1 •--·

Lock
s
Suspend
0
End Block
Item: 61 of 99 ~ 1 • Mark <:] (:>- Jill ~· ~J
QID: 23265 ..1 Previous Next Lab'V!I!ues Notes Calculator
. • AmyotrophiC lateral scleros1s Multiple scleros1s Cerebral palsy Baclofen Gamma-Ammobutync ac1d Spasm Agomst GABA receptor agomst Muscle
39
C is not correct. 3% chose this.
40
Carbamazepine is an antiepileptic but it is also used to treat muscle spasms and cramps in amyotrophic latera l sclerosis.
41 Carbamazepine Amyotrophic lateral sclerosis Anticonvulsant Spasm Muscle
42
D is not correct. 2% chose this.
43
Although depression, pseudobu lbar symptoms (outbursts of laughter or tearfulness), and cognit ive loss are symptoms in
44
amyotroph ic lateral sclerosis, this patient's current problem is medical and does not requ ire psychiatric evaluation .
45 Amyotrophic lateral sclerosis Major depressive disorder Depression (mood) Pseudobulbar palsy

46
E is not correct. 10% chose this.
47
Diphenhydramine is a sedative and is potentia lly a usefu l treatment for insomnia, which is very common in patients with
48 amyotrophic lateral sclerosis . Sedatives should be used sparingly in these patients. Although diphenhydramine has
49 antichol inergic effects, it is not used for treating sia lorrhea . Other agents w ith anticholinergic properties that can be used are
so atropine and hyoscyam ine .
Anticholinergic Hyoscyamine Diphenhydramine Amyotrophic lateral sclerosis Atropine Sedative Hypersalivation Insomnia
51
52
53 Bottom Line:
54 Amyotrophic latera l sclerosis is a slowly progressive, generalized motor muscle paralysis involv ing both lower and upper
55 neurons that typical ly progresses to death 3·5 years after diagnosis. Anticholinergic drugs such as glycopyrrolate can be
useful to decrease secretions and manage symptoms .
56 Anticholinergic Amyotrophic lateral sclerosis Glycopyrronium bromide Paralysis Neuron Muscle
57
58
59 References:
60 FA Step 2 CK 9th ed pp 276· 277
FA Step 2 CK 8th ed p 264
61

Lock
s
Suspend
0
End Block
Item: 62 of 99 ~ 1 • Mark <:] (:>- Jill ~· ~J
QID: 22425 ..1 Previous Next Lab'V!I!ues Notes Calculator

40 A 15-year-old boy suffers a blow to the right side of his head during a football game and loses consciousness . He quickly ~~AI
regains consciousness and is apparently normal, but after a few hours he is brought to the emergency department (ED) by
41
his parents because he progressively becomes more confused . While in the ED, he has an episode of vomit ing . On physical
42 exam, he has a large ecchymosis in the right temporal area and papilledema is noted on fundoscopic exam of the right eye . A
43 stat head CT reveals an intracranial bleed .
44
45 Which of the following structures was most likely injured?
46
47 A. Basilar artery
48
B. Dural venous sinus
49
C. Middle meningeal artery
50
51 D. Superior sagittal sinus
52 E. Vertebral artery
53
54
55
56
57
58
59
60
61
• 62

Lock
s
Suspend
0
End Block
Item: 62 of 99 ~ 1 • Mark <:] (:>- Jill ~· ~J
QID: 22425 ..1 Previous Next Lab'V!I!ues Notes Calculator

40
41
42
The correct answer is C. 890/o chose this.
The patient has suffered an epidural hematoma, which is characteristically caused by damage to the middle meningeal artery
43
due to fracture of the temporal bone. This patient has an ecchymosis in the temporal area, ind icating damage to this
44 structure. Patients with epidural hematomas typically suffer a traumatic blow to the head, followed by a lucid interval in which
45 the patient regains consciousness and is asymptomatic, and finally the patient quickly deterioriates and presents w ith nausea,
46 vom it ing, and lethargy. These patients require urgent ne,u rosurgical consultation and evacuation of the hematoma as a rapid
high pressure arterial bleed can quickly lead to increased intracranial pressure, mass effect, brain hern iation, and ult imately
47
death . This patient shows signs of increased intracranial pressure, as evidenced by his papilledema on fundoscopic exam.
48 Following his lucid interval, his confusion and vomiting in the ED indicate that he is rapidly deteriorating and requires
49 intervention.
Ecchymosis Brain herniation Middle meningeal artery Epidural hematoma Papilledema Intracranial pressure Temporal bone Hematoma Nausea Epidural administration Ophthalmoscopy
50
Neurosurgery Vomiting Ludd interval ~1eninges Mass effect {medidne) Fracture Asymptomatic Bone Human brain Lethargy Brain
51
52 A is not correct. 2% chose this.
53 Damage to the basilar artery would likely produce intraparenchymal bleeding . The basilar artery arises from the confluence of
54 the two vertebra l arteries at the level of the medulla oblongata . It ascends in the central gutter (sulcus basilaris) inferior to
the pons and divides into the two posterior cerebral arteries. Patients with damage to the basilar artery may progress into a
55
coma .
56 Basilar artery Medulla oblongata Vertebral artery Pons Cerebral arteries Sulcus (neuroanatomy) Posterior cerebral artery Coma Artery Sulcus (morphology)
57 lntraparenchymal hemorrhage
58
B is not correct. ] Ofo chose this.
59
The dural venous sinus would not be injured in this type of injury. Damage to the venous sinus typically presents with
60 thrombosis with in the sinus. Dura l sinus thrombosis is associated with symptoms such as headache, disturbances of vision,
61 focal neurologic deficits, and seizures.
Dural venous sinuses Cerebral venous sinus thrombosis Dura mater Focal neurologic signs Headache Epileptic seizure Thrombosis Vein Neurology
62 y

Lock
s
Suspend
0
End Block
Item: 62 of 99 ~ 1 • Mark <:] (:>- Jill ~· ~J
QID: 22425 ..1 Previous Next Lab'V!I!ues Notes Calculator

Intraparenchymal hemorrhage
40
B is not correct. ]Ofo chose this.
41
The dural venous sinus would not be injur ed in this type of injury. Damage to the venous sinus typica lly presents with
42
thr ombosis with in the sinus. Dural sinus thrombosis is associated with symptoms such as headache, disturbances of v ision,
43 focal neurologic deficits, and seizures.
44 Dural venous sinuses Cerebral venous sinus thrombosis Dura mater Focal neurologic signs Headache Epileptic seizure Thrombosis Vein Neurology

45
D is not correct. 2% chose this.
46 The superior sagittal sinus is not typically affected by this mechanism . Damage to the venous sinus typically presents w ith
47 thrombosis. Dura l sinus throm bosis is associated with symptoms such as headache, disturbances of vision, focal neurologic
48 deficits, and seizur es.
Cerebral venous sinus thrombosis Superior sagittal sinus Dural venous sinuses Headache Epileptic seizure Focal neurologic signs Neurology Thrombosis Vein
49
50 E is not correct. OOfo chose this.
51 Damage to the vertebral artery would likely produce intrapar enchyma l bleeding, not an epidural hematoma . Patients typically
present with neck pain and focal neurolog ic deficits.
52
Epidural hematoma Vertebral artery Hematoma Focal neurologic signs Epidural administration Neurology Intraparenchymal hemorrhage
53
54
55 Bottom Line:
56 Epidural hematomas are characteristica lly caused by damage to the middle meningeal artery, usually due to fracture of the
57
tempora l bone .
Middle meningeal artery Temporal bone Bone Hematoma Meninges
58
59
60 References:
61 FA Step 2 CK 9th ed p 265
62
FA Step 2 CK 8th ed p 251

Lock
s
Suspend
0
End Block
Item: 63 of 99 ~ 1 • Mark <:] (:>- Jill ~· ~J
QID: 23466 ..1 Previous Next Lab'V!I!ues Notes Calculator
• I

41 Accompan ied by her son, a 63-year-old female presents to the emergency department for recent head trauma . He reports
that his mother slipped in the bathroom earlier today and hit her head . She lost consciousness for about five minutes but
42
did not have any convulsions or any bowel or bladder incontinence. However, since gain ing consciousness, her speech has
43 been incomprehensible, and the son reports it sounds mostly like gibberish . He is concerned because he has never seen her like
44 this and says she is a retired college professor. On examination, she has a large ecchymosis over the frontal sca lp but no
45 evidence of an open wound or skull fracture . Her speech is fluent; however, she makes many paraphasic errors and uses
frequent neologisms, and does not appear able to follow the train of conversation well.
46
47
What additional find ing on examination wou ld be most consistent w ith th is patient's lesion?
48
49
A. Ability to follow a three-step command
so
51 B. Intact repetition
52 C. Oral apraxia
53
D. Preserved ora l reading
54
55 E. Superior quadrantanopia
56
57
58
59
60
61
62
• 63

Lock
s
Suspend
0
End Block
Item: 63 of 99 ~ 1 • Mark <:] (:>- Jill ~· ~J
QID: 23466 ..1 Previous Next Lab'V!I!ues Notes Calculator
• I

41
The correct answer is E. 28% chose this.
42 ~---/ ParieiM lObe
Reading
comprehension area
43
44
Sensory speech
45 area of Wernicke long!Wdinal
fis3ure
46 Fronlat lobe
Premotof Jrea
47 Preoonb'• gyrus
Poetcentral gyrua
48 Parietal lObe
~~~ Qocooooal -
49
so This patient's history and location of lesion are most consistent with Wernicke aphasia, which usually presents w ith fluent but
51
nongrammatical speech with impaired comprehension . Optic radiat ions traveling f rom the inferior r etina to the visual cortex
pass throu gh the temporal lobe and are called Meyer loops . A lesion resulting in damage to Wernicke area may also damage
52
these neurons, resulting in a superior quadrantanopia . Patients may exhibit reduced ability to respond to "threats" during
53 physical examination f rom the right side. The above image shows the locations of Broca and Wernicke areas and their
54 relationship to su rroundin g neuroanatom ica l structures.
Receptive aphasia Aphasia Temporal lobe Retina Visual cortex Wernicke's area Lesi on Quadrantanopia Neuroanatomy Neuron Physical examination Cortex (anatomy)
55
56 A is not correct. 150/o chose this.
57 Comprehension is usua lly im paired in Wernicke aphasia, thus it is un li kely that this patient would be able to follow a three-
58
step command. Limb apraxias ar e common neighborhood signs in Wernicke aphasia and may also contribute to the inability
to perform the task.
59 Receptive aphasia Apraxia Aphasia
60
B is not correct. 210/o chose this.
61
Although speech is fluent in Wernicke aphasia, comprehension is usual ly impaired as well as repetition.
62 Receptive aphasia Aphasia
63

Lock
s
Suspend
0
End Block
Item: 63 of 99 ~ 1 • Mark <:] (:>- Jill ~· ~J
QID: 23466 ..1 Previous Next Lab'V!I!ues Notes Calculator
• I
• to perform the task.
41 Receptive aphasia Apraxia Aphasia

42
B is not correct. 210/o chose this.
43
Although speech is fluent in Wernicke aphasia, comprehension is usually impaired as well as repetition .
44 Receptive aphasia Aphasia

45
C is not correct. 200/o chose this.
46
Oral ( ie, buccofacial) apraxia is most common ly associated with Broca aphasia (intact comprehension with loss of the abil ity
47 to produce speech) or damage to the dorsolateral prefrontal cortex, opercu lum, or anterior parietal cortex . Limb apraxia is
48 more common ly associated with Wern icke aphasia, w hich is t he speech deficit in this case.
Receptive aphasia Dorsolateral prefrontal cortex Aphasia Prefrontal cortex Parietal l obe Apraxia Posterior parietal cortex Cortex (anatomy) Parietal bone
49
so D is not correct. 160/o chose this.
51 Although patients w ith Wernicke aphasia may have preserved oral reading, impairment in r eading most typically accompanies
52
the compr ehension deficits in Wernicke aphasia . Patients' writing may include paraphasic errors, although grammar is usually
better preserved than in Broca aphasia .
53 Receptive aphasia Aphasia Paraphasia
54
55
56
Bottom line:
57 Wern icke aphasia presents with fluent but nongrammatkal speech with impaired comprehension . A lesion resulting in
damage to Wern icke area may also damage Meyer loops, resulting in a superior quadranta nopia .
58 Receptive aphasia Aphasia Wernicke's area Quadrantanopia Lesion
59
60
61 References:
62 FA Step 2 CK 9th ed pp 288-289
FA Step 2 CK 8th ed p 274
63

Lock
s
Suspend
0
End Block
Item: 64 of 99 ~ 1 • Mark <:] (:>- Jill ~· ~J
QID: 23269 ..1 Previous Next Lab'V!I!ues Notes Calculator
.
42 Three weeks ago, a pr eviously hea lthy 25-year-old man presented to his physician complaining of 2 days of upper
respiratory congestion and mild myalgias . The physician diagnosed him with a viral infection and recommended plenty of
43
liquids and bedrest. The patient returns complaining of progr essive lower extremity weakness and par esthesias of the
44 finger s. He claims that his face began to feel numb while in the wait ing room, and he is unable to sm ile on either side du ring the
45 physical examination .
46
47 Which of the following is the most appropriate next step"
48
49 A. Advise the patient to get to an emergency department for emer gency thrombolytic therapy
50 B. Apologize to the patient for m isdiagnosing him
51
C. Arrange for immediate admission to an intensive care unit setting
52
53 D. Recommend HIV screen and start trimethoprim/ su lfamethoxazole immediately
54 E. Tell the patient that his condition will most likely r esolve with t ime
55
56
57
58
59
60
61
62
63
• 64

Lock
s
Suspend
0
End Block
Item: 64 of 99 ~ 1 • Mark <:] (:>- Jill ~· ~J
QID: 23269 ..1 Previous Next Lab'V!I!ues Notes Calculator
.
42 The correct answer i s C. 690/o chose this.
43
Guillain- Barre syndrome is an inflammatory demyelinating polyneuropathy characterized by the acute onset of muscle
weakness, areflexia, and eventually flaccid para lysis commonly occurring after a viral infection of the upper respiratory or
44
gastrointestinal tract. The ascending paralysis of Guillain-Barre syndrome is somewhat unpredictable and can progress rapidly
45 (over hou rs) to compromise respiratory function . Patients w ith bilateral facia l involvement, rapid progression (<7 days),
46 inability to ra ise their head against gravity, bu lba r dysfunction, significant autonom ic dysfunction, or obvious aspiration merit
intensive care unit monitoring .
47
Flaccid paralysis Paralysis Human gastrointestinal tract Gastrointestinal tract Autonomic nervous system Musde weakness Polyneuropathy Dysautonomia Reflex Intensive care unit
48
Hyporeflexia Intensive care medicine Myelin Muscle Demyelinating disease Virus Viral disease Inflammation Infection Medulla oblongata
49
50 A i s not correct. SOfo chose this .
51
Thrombolytic therapy m ight be indicated in an acute vascular process such as a myocardial infarction or stroke . However, in a
25-year-old man with these symptoms, neither is very likely.
52 Thrombolytic drug Thrombolysis Myocardial infarction Stroke Blood vessel
53
B is not correct. 6 0/o chose this .
54
There is no need to apologize to this man because there is currently no way of predicting which patients will suffer from
55
Guillain- Barre syndrome, which is a rare idiopath ic disorder that affects on ly about 1000th of a percent of the population .
56 Idiopathy

57
D i s not correct. 3% chose this.
58
Guillain- Barre syndrome is more common in patients with HIV. However, there is no reason to suspect that this patient has
59 HI V. Furthermore, trimethoprim/sulfamethoxazole is recommended for patients with CD4+ cell counts <200/mm 3 , and
60 Guillain- Barre syndrome is more often seen in those with CD4+ cell counts >300/mm 3 •
Trimethoprim/sulfamethoxazole CD4
61
62 E i s not corr ect. 17% chose this.
63 It is true that most patients with Guillain- Ba rre synd rome recover completely or with only mild deficits . However, 5%-10%
64
are severely disabled and 3%-8% die w ithout care. I n this patient with a rapidly progressive course, it wou ld be inappropriate
y

Lock
s
Suspend
0
End Block
Item: 64 of 99 ~ 1 • Mark <:] (:>- Jill ~· ~J
QID: 23269 ..1 Previous Next Lab'V!I!ues Notes Calculator
.
42 B is not correct. 60/o chose this.
43
There is no need to apologize to this man because there is currently no way of pred icting which patients will suffer from
Guillain- Barre syndrome, which is a rare idiopath ic disorder that affects on ly about l OOOth of a percent of t he population .
44 Idiopathy
45
D is not correct. 3% chose this.
46
Guillain- Barre syndrome is more common in patients with HIV. However, there is no reason t o suspect that t his patient has
47
HI V. Furthermore, trimethoprim/sulfamethoxazole is recommended for pati ents with CD4 + cell counts <200/mm 3 , and
48 Guillain- Barre syndrome is more oft en seen in those with CD4+ cell counts >300/mm 3 •
49 Trimethoprim/sulfamethoxazole CD4

50 E i s not correct. 17% chose this.


51 It is true that most patients with Guillain-Barre syndrome recover completely or with only mild deficits . However, 5%-10%
52 are severely disabled and 3%-8% die w ithout care. I n this patient with a rapidly progressive course, it wou ld be inappropriate
53
to offer premature reassurance and more appropriate to admit him to an intensive care unit .
Intensive care unit Intensive care medicine
54
55
56 Bottom Line :
57 Guillain- Barre syndrome is an inflammatory demyelinating polyneuropathy characterized by the acute onset of muscle
58 weakness, areflexia, and eventually flaccid para lysis . It commonly occurs after a vira l infection . Patients should be observed
in the ICU to mon itor respiratory function .
59 Flaccid paralysis Polyneuropathy Muscle weakness Reflex Hyporeflexia Paralysis Intensive care unit Virus Muscle Demyelinating disease Inflammation Viral disease Myelin
60
61
62 References:
63 FA Step 2 CK 9th ed p 276
FA Step 2 CK 8th ed pp 263-264
64

Lock
s
Suspend
0
End Block
Item: 65 of 99 ~ 1 • Mark <:] (:>- Jill ~· ~J
QID: 23300 ..1 Previous Next Lab'V!I!ues Notes Calculator
.
43 A physician gets paged to the emergency department (ED) to evaluate a young, muscu lar man weigh ing 102 kg (225 lb)
who came to the ED complaining of weakness initially involving the legs, but now progressing to facial and oropharyngeal
44
weakness. While his history is taken, the physician notice.s that he is using accessory neck muscles to breathe. The patient
45 is a nurse who said he's been mostly healthy. On further questioning, he mentions that a few weeks ago he took a few days off
46 from work due to upper respiratory symptoms and severe diarrhea. The doctor places him on oxygen by nasal cannula and
47 orders pulmonary function tests. Pulmonary function tests reveal a vital capacity of 1.5 L. The physician decides to intubate due
to impending respiratory failure. Cerebrospinal fluid analysis shows an elevated protein level and normal WBC count. Resu lts of
48
an electromyographic study are pending .
49
so First-line treatment of this disorder consists of wh ich of the following?
51
52
A. Antithymic immunoglobulin
53
B. Corticosteroids
54
55 C. Cyclosporine and tacrolimus
56 D. Plasmapheresis or intravenous immunog lobulin
57
E. Tumor necrosis factor-a receptor blockers
58
59
60
61
62
63
64
• 65

Lock
s
Suspend
0
End Block
Item: 65 of 99 ~ 1 • Mark <:] (:>- Jill ~· ~J
QID: 23300 ..1 Previous Next Lab'V!I!ues Notes Calculator
.
43
The correct answer is D. 830/o chose t his.
44
The diagnosis is Guillain-Barre syndrome, suggested by the patient's weakness, recent viral illness, and CSF analysis
45 sign ificant for albuminocytological dissociation (elevation in CSF protein without elevation in WBC count) . Campy/obacter
46 jejuni, a possible etiology for this patient's diarrheal illness, is, in fact, the most commonly identified precipitant of Guillain-
47 Barre syndrome . This syndrome is an inflammatory demyelinating polyneuropathy characterized by the acute onset of muscle
weakness, areflexia, and eventually flaccid para lysis, commonly occurring after a vira l infection of the upper respiratory or
48
gastrointestinal tract. The ascending paralysis of Guillain-Barre syndrome is somewhat unpredictable and can progress rapidly
49 (over hours) to compromise respiratory function . After addressing the breath ing difficulty, first-line treatment consists of
so plasmapheresis or intravenous immunoglobu lin . The presumed mechanism of action of both is inhibition of antibody-mediated
51 autoimmune demyelination, although the details remain obscure .
Campylobacter jejuni Plasmapheresis Immunoglobulin therapy Demyelinating disease Flaccid paralysis Autoimmune disease Campylobacter Human gastrointestinal tract
52
Polyneuropathy Antibody Protein Virus Intravenous therapy Autoimmunity Reflex Gastrointestinal tract Paralysis Hyporeflexia Etiology Viral disease Muscle weakness Inflammation
53
54 A is not correct. 2% chose this.
55 Antithymic immunoglobulin is made by infusing human t hymocytes into rabbits then harvesting the rabbit serum, wh ich
contains antibodies against human T cells . It is an immunosuppressive therapy given before organ transplantation to prevent
56
acute rejection of the transplanted organ .
57 Organ transplantation Thymocyte Antibody Immunosuppression T cell Transplant rejection Blood plasma Rabbit Serum (blood)
58
B is not correct. 1 20/o chose t his.
59
Corticosteroids are immunosuppressive agents useful for treating asthma, COPD, anaphylaxis, and a number of autoimmune
60 condit ions, including rheumatoid arthritis, IBD, multiple sclerosis, and vasculitides. Steroid therapy has many undesirable side
61 effects such as central weight gain, moon facies, hyperglycemia, insulin resistance, osteoporosis, hypertension, and increased
62 susceptibility to infection . Because of th is, steroids are used as sparingly as possible. Additionally, patients w ith adrenal
insufficiency are treated with corticosteroids to replace what is normally produced by the adrenal glands, but these patients
63
can usua lly avoid the adverse effects .
64 Multiple sclerosis Osteoporosis Anaphylaxis Rheumatoid arthritis Asthma Adrenal insufficiency Hyperglycemia Insulin resistance Hypertension Steroid Immunosuppression

65 y Autoimmune disease Chronic obstructive oulmonarv disease Adrenal oland Vasculitis Insulin Autoimmunitv Corticosteroid Arthritis Weioht oain Immunosuooressive druo

Lock
s
Suspend
0
End Block
Item: 65 of 99 ~ 1 • Mark <:] (:>- Jill ~· ~J
QID: 23300 ..1 Previous Next Lab'V!I!ues Notes Calculator
.
43
44
45
The correct answ e r is D. 8 30/o chose t his.
46 The diagnosis is Guillain-Barre syndrome, suggested by the patient's weakness, recent viral illness, and CSF analysis
47 sign ificant for albuminocytological dissociation (elevation in CSF protein without elevation in WBC count). Campy/obacter
48 jejuni, a possible etiology for this patient's diarrheal illness, is, in fact, the most commonly identified precipitant of Guillain-
Barre syndrome . This syndrome is an inflammatory demyelinating polyneu ropathy characterized by the acute onset of muscle
49
weakness, areflexia, and eventually flaccid para lysis, commonly occurring after a viral infection of the upper respiratory or
so gastrointestinal tract. The ascending paralysis of Guillain-Barre syndrome is somewhat unpredictable and can progress rapidly
51 (over hours) to compromise respiratory function . After addressing the breath ing difficulty, first-line treatment consists of
52 plasmapheresis or intravenous immunoglobu lin . The presumed mechanism of action of both is inhibition of antibody- mediated
autoimmune demyelination, although the details remain obscure .
53 Campylobacter jejuni Plasmapheresis Immunoglobulin therapy Demyelinating disease Flaccid paralysis Autoimmune disease Campylobacter Human gastrointestinal tract
54
Polyneuropathy Antibody Protein Virus Intravenous therapy Autoimmunity Reflex Gastrointestinal tract Paralysis Hyporeflexia Etiology Viral disease Muscle weakness Inflammation
55
A is not correct. 2% chose this.
56
Antithymic immunoglobulin is made by infusing human t hymocytes into rabbits then harvesting the rabbit serum, which
57
contains antibodies against human T cells . It is an immunosuppressive therapy given before organ transplantation to prevent
58 acute rejection of the transplanted organ .
59 Organ transplantation Thymocyte Antibody Immunosuppression T cell Transplant rejection Blood plasma Rabbit Serum (blood)

60 B is not correct. 1 20/o chose t his.


61 Corticosteroids are immunosuppressive agents useful for treating asthma, COPD, anaphylaxis, and a number of autoimmune
62 condit ions, includ ing rheumatoid arthritis, IBD, multiple sclerosis, and vasculitides. Steroid therapy has many undesirable side
63 effects such as central weight gain, moon facies, hyperglycemia, insulin resistance, osteoporosis, hypertension, and increased
susceptibility to infection . Because of th is, steroids are used as sparingly as possible. Additionally, patients with adrenal
64
insufficiency are treated with corticosteroids to replace what is normally produced by the adrenal glands, but these patients
65 y l':i'ln ll~IJi'l l lv i'lVOir! thA i'lOVAr~A AffAr:t~ .

Lock
s
Suspend
0
End Block
Item: 65 of 99 ~ 1 • Mark <:] (:>- Jill ~· ~J
QID: 23300 ..1 Previous Next Lab'V!I!ues Notes Calculator
.
B IS not correct. 120/o chose th1s.
43
Corticosteroids are immunosuppr essive agents useful for treating asthma, COPD, anaphylaxis, and a number of autoimmune
44 conditions, including rheumatoid arthritis, IBD, multiple sclerosis, and vasculitides. Steroid therapy has many undesirable side
45 effects such as central weight gain, moon facies, hyperglycemia, insulin resistance, osteoporosis, hypertension, and increased
46
susceptibility to infection . Because of this, steroids are used as sparingly as possible. Additionally, patients w ith adrenal
insufficiency are treated with corticosteroids to r eplace what is normally produced by the adrenal glands, but these patients
47
can usually avoid the adverse effects .
48 Multiple sclerosis Osteoporosis Anaphylaxis Rheumatoid arthritis Asthma Adrenal insufficiency Hyperglycemia Insulin resistance Hypertension Steroid Immunosuppression

49 Autoimmune disease Chronic obstructive pulmonary disease Adrenal gland Vasculitis Insulin Autoimmunity Corticosteroid Arthritis Weight gain Immunosuppressive drug

so Adverse drug reaction Side effect


51
C is not correct. 2% chose this.
52
Cyclosporine and tacrolimus are immunosu ppressive agents used to prevent graft-versus-host disease and rejection after
53 solid organ transplantation . Both agents act by in hibiting transcription of IL-2, a cytokine needed forT-cell maturation.
54 Tacrolimus Graft-versus-host disease Cytokine Ciclosporin Organ transplantation lnterleukin 2 Immunosuppression T cell Transcription (genetics} Immunosuppressive drug

55 E is not correct. 10/o chose this.


56 Tumor necrosis factor-a (TN Fa) is a pro-inflammatory cytokine that is dysregulated in many forms of autoimmune disease,
57 such as rheumatoid arthritis, psoriasis, and IBD . TNFa receptor blockers are used to treat these diseases. They are known to
58 increase susceptibility to opportunistic pathogens, such as tuberculosis and fungi.
Rheumatoid arthritis Cytokine Autoimmune disease Psoriasis Fungus Tuberculosis Necrosis Autoimmunity Neoplasm Pathogen Opportunistic infection Arthritis
59
Proinflammatory cytokine Receptor (biochemistry)
60
61
62 Bottom Line:
63 First-line treatment of Guillain-Barre syndrome consists of plasmapheresis or intravenous immunoglobu lin .
64 Plasmapheresis Immunoglobulin therapy Intravenous therapy Antibody

65

Lock
s
Suspend
0
End Block
Item: 65 of 99 ~ 1 • Mark <:] (:>- Jill ~· ~J
QID: 23300 ..1 Previous Next Lab'V!I!ues Notes Calculator

:.0
p y•
I p gy•
p •
y, p • • • •

43 insufficiency are treated with corticosteroids to replace what is normally produced by the adrenal glands, but these patients
44
can usua lly avoid the adverse effects .
Multiple sclerosis Osteoporosis Anaphylaxis Rheumatoid arthritis Asthma Adrenal insufficiency Hyperglycemia Insulin resistance Hypertension Steroid Immunosuppression
45
Autoimmune disease Chronic obstructive pulmonary disease Adrenal gland Vasculitis Insulin Autoimmunity Corticosteroid Arthritis Weight gain Immunosuppressive drug
46
Adverse drug reaction Side effect
47
48 C is not correct. 20/o chose this.
49 Cyclosporine and tacrolimus are immunosuppressive agents used to prevent graft-versus-host disease and rejection after
solid organ transplantation. Both agents act by inhibiting transcription of IL-2, a cytokine needed forT-cell maturation.
so Tacrolimus Graft-versus-host disease Cytokine Ciclosporin Organ transplantation Interleukin 2 Immunosuppression T cell Transcription (genetics} Immunosuppressive drug
51
52
E is n ot correct. 1 0/o chose t his.
Tumor necrosis factor-a (TN Fa) is a pro-inflammatory cytokine that is dysregulated in many forms of autoimmune disease,
53
such as rheumatoid arthrit is, psoriasis, and IBD . TNFa receptor blockers are used to treat these diseases. They are known to
54 increase susceptibility to opportunistic pathogens, such as tuberculosis and fungi.
55 Rheumatoid arthritis Cytokine Autoimmune disease Psoriasis Fungus Tuberculosis Necrosis Autoimmunity Neoplasm Pathogen Opportunistic infection Arthritis

56 Proinflammatory cytokine Receptor (biochemistry}

57
58
Bottom li ne:
59
First-line treatment of Gu il lain-Barre syndrome consists of plasmapheresis or intravenous immunoglobu li n.
60 Plasmapheresis Immunoglobulin therapy Intravenous therapy Antibody
61
62
63 References:
64 FA Step 2 CK 9th ed p 276
FA Step 2 CK 8th ed pp 263-264
65

Lock
s
Suspend
0
End Block
Item: 66 of 99 ~ 1 • Mark <:] (:>- Jill ~· ~J
QID: 23504 ..1 Previous Next Lab'V!I!ues Notes Calculator
.
44 A 32-year-old man presents to the emergency departmen t complaining of acute loss of vision in his left eye. Two years ~~AI
ago he had a 1-week episode of right upper and lower extremity numbness and weakness that resolved spontaneously. He
45
has no other significant past medical history. Physical examination reveals a left afferent pupillary defect. MRI of the head
46 shows an en larged and enhancing left optic nerve.
47
48 Which of the following is the most appropriate treatment for this patient?
49
50 A. Baclofen
51
B. Intravenous gamma-globulin
52
53 C. Intravenous methylprednisolone
54 D. Neostigmine bromide
55
E. Oral prednisone
56
57
58
59
60
61
62
63
64
65
• 66

Lock
s
Suspend
0
End Block
Item: 66 of 99 ~ 1 • Mark <:] (:>- Jill ~· ~J
QID: 23504 ..1 Previous Next Lab'V!I!ues Notes Calculator
.
44
45 The correct answer is C. 640/o chose this.
46 Young adults who experience various neurological events separated by time should raise suspicion of multiple sclerosis (MS) .
47
MS is an immune-mediated disease that targets the white matter of the central nervous system and causes focal areas of
demyelination . Sequelae include weakness, paralysis, sensory loss, and changes in mood and mentation. Imag ing can show
48 involvement of the optic nerve, manifested by acute pai nful vision loss and an afferent pupillary defect. This is characteristic
49 of optic neuritis, for which intravenous high-dose methylprednisolone is indicated .
Optic neuritis Multiple sclerosis ~1ethylprednisolone Central nervous system Demyel inating disease Optic nerve White matter Nervous system Intravenous therapy SeQuela Neurology
50
Paralysis Afferent nerve fiber Neuritis Visual impairment Immune disorder Autoimmunity
51
52 A is not correct. 3% chose this.
53 This patient is experiencing optic neuritis from an acute MS attack, as evidenced by the painful loss of vision in one eye and
54 the presence of an afferent pupillary defect. His history of right-sided weakness that resolved spontaneously also suggests a
diagnosis of MS. 6aclofen, an antispastic agent and muscle relaxant, is indicated for symptomatic relief from spasticity. It is
55
not appropriate therapy for optic neuritis.
56 Optic neuritis Baclofen Spasticity Afferent nerve fiber Hemiparesis Neuritis Muscle relaxant Muscle
57
B is not correct. 130/o chose this.
58
Intravenous imm unoglobulin is a solution of globulins containing antibodies that are normally present in adult human blood .
59 It may be used to treat Guillain-Barre syndrome; however, it is not ind icated for the t reatment of optic neuritis in patients
60 w ith MS.
Optic neuritis Immunoglobulin therapy Antibody Neuritis Intravenous therapy Globulin
61
62 D is not correct. 3% chose this.
63 Neostigmine bromide is a cholinesterase inhibitor used to treat myasthenia gravis . This patient's presentation is most
64 consistent w ith optic neuritis in a patient with MS. Neosti'gmine is not indicated for the treatment of optic neuritis.
Neostigmine Optic neuritis Myasthenia gravis Acetylcholinesterase inhibitor Cholinesterase Neuritis
65
66 E is not correct. 17% chose this.

Lock
s
Suspend
0
End Block
Item: 66 of 99 ~ 1 • Mark <:] (:>- Jill ~· ~J
QID: 23504 ..1 Previous Next Lab'V!I!ues Notes Calculator
. • Optic neunt1s Baclofen Spasticity Afferent nerve fiber Hem1pares1s Neunt1s Muscle relaxant Muscle
44
B is not correct. 1 30/o chos e th is.
45
Intravenous immunoglobuli n is a solution of globuli ns contain ing antibodies that are normally present in adult human blood .
46
It may be used to treat Gu illain-Barre syndrome; however, it is not ind icated for the treatment of optic neuritis in patients
47 w ith MS.
48 Optic neuritis Immunoglobulin therapy Antibody Neuritis Intravenous therapy Globulin

49 D is not correct. 3% chose this .


50 Neostigmine bromide is a cholinesterase in hibitor used to treat myasthenia gravis . This patient's presentation is most
51 consistent w ith optic neuritis in a patient with MS . Neostigmine is not ind icated for the treatment of optic neuritis.
Neostigmine Optic neuritis Myasthenia gravis Acetylcholinesterase inhibitor Cholinesterase Neuritis
52
53 E is n ot correct. 17% ch ose this .
54 An oral prednisone 2-week taper is often prescribed following intravenous methylprednisolone in the treatment of optic
55
neuritis in patients with MS . However, oral prednisone alone is not the standard of care for optic neuritis during the acute
phase.
56 Optic neuritis Prednisone Methylprednisolone Intravenous therapy Neuritis
57
58
59
Botto m li ne:
60 Optic neuritis manifests as acute painful vision loss and an afferent pupillary defect. Treat w ith high-dose IV
methylprednisolone.
61 Optic neuritis Methylprednisolone Afferent nerve fiber Neuritis Visual impairment Marcus Gunn pupil
62
63
64 References:
65 FA Step 2 CK 9th ed pp 275-276
FA Step 2 CK 8th ed pp 263-263
66

Lock
s
Suspend
0
End Block
Item: 67 of 99 ~ 1 • Mark <:] (:>- Jill ~· ~J
QID: 22404 ..1 Previous Next Lab'V!I!ues Notes Calculator
..
45 1•AI
A 40-year-old man presents to his internist at the insistence of his wife, who says he has been behaving strangely and that
"something's wrong ." She reports that he has become increasing ly restless and fidgety over the past 2 years. Lately he A
46
has been making bizarre jerky movements with his arms and legs that almost seem dance-like. He reports that he cannot
47 control himself when this happens . She also says he has become more forgetful and has been having trouble completing tasks.
48 He has otherwise been healthy and has no past medical history. He was adopted as a baby and does not know his family history.
49
Physical examination is unremarkable . Basic laboratory tests are unrevealing . He passes away 8 years later.
so
Patholog ic examination of the brain during autopsy would most likely reveal which of the following?
51
52
53 A. Atrophy of the caudate nucleus
54 B. Lewy bodies
55
C. Neurofibrillary tangles
56
57
D. Pick bodies
58 E. Senile plaques
59
60
61
62
63
64
65
66
• 67

Lock
s
Suspend
0
End Block
Item: 67 of 99 ~ 1 • Mark <:] (:>- Jill ~· ~J
QID: 22404 ..1 Previous Next Lab'V!I!ues Notes Calculator
..
45
46
47
The correct answer i s A. 890/o chose this.
48
This patient's presentation is consistent with Huntington's disease (HD) . HD is an incurable inherited progr essive
49 neur odegenerative disease . It is a trinucleotide r epeat disor der and typically Huntingtin genes (located on chromosome 4 )
so w ith greater than 40 repeats (CAG) are pathologic. The pathologic hallmark of this disease is atrophy of the caudate nucleus.
51 Clinically, these patients pr esent with chorea and dementia. Chor ea is involuntary movements involving the face, trunk, and
li mbs, and ar e named for their dance-like character. Unfortunately no cure or disease-modifying treatments curr ently exist
52
and treatment is supportive .
53 Trinucleotide repeat disorder Huntington's disease Caudate nucleus Neurodegeneration Huntingtin Chromosome 4 (human) Chorea Dementia Chromosome Atrophy Cell nucleus
54
B is not correct. 40/o chose t h is.
55
Lewy bodies (intranuclear inclusion bod ies) and depigmentation of the substantia nigra ar e typically seen in Parkinson's
56 disease . Parkinson's disease manifests as rigidity w ith t remors in extremities, akinesia, and shuffling gait. Patients may also
57 exhibit lack of facial expression, dr ooling, dysarthria, and dystonia. Patients pr esent in m iddle to late life. Treatment options
58 include levodopa, anticholinergic agents, and dopamine agonists. Deep brain stimulation can also be pursued .
Anticholinergic Substantia nigra Dysarthria Parkinson's disease Deep brain stimulation Hypokinesia Dopamine L-DOPA Dystonia Lewy body Dopamine agonist Gait (human) Brain
59
Agonist Gait Inclusion bodies
60
61 C i s not correct. 3% ch ose this.
62 Neur ofibrillary tangles and senile plaques ar e associated with Alzheimer's disease . Cerebrocortical atrophy may also be seen.
Alzheimer's disease is the most common cause of dementia. The parietal cortex is most often affected . The onset is usually
63
later in life ( beyond the sixth decade) and patients prese·nt w ith insidious pr ogr essive memory loss over a period of years.
64 Alzheimer's is a diagnosis of exclusion and is only definit i vely diagnosed at autopsy. Medica l treatment includes psychotropic
65 medications and behavioral interventions.
Alzheimer's disease Senile plaques Dementia Neurofibrillary tangle Diagnosis of exclusion Parietal lobe Atrophy Psychoactive drug Autopsy Amnesia
66
67 D is not correct. 3% chose this.

Lock
s
Suspend
0
End Block
Item: 67 of 99 ~ 1 • Mark <:] (:>- Jill ~· ~J
QID: 22404 ..1 Previous Next Lab'V!I!ues Notes Calculator
.. Alzhe1mer s d1sease Senile plaques Dement1a Neurofibnllary tangle D1agnos1s of exclusiOn Panetallobe Atrophy Psychoact1ve drug Autopsy Amnes1a
45
D is not correct. 3% chose this.
46
Pick's disease manifests as progressive dementia . In contrast to Alzheimer's disease, in wh ich the parietal cortex is most
47
often affected, Pick's mainly affects the frontal lobes. Histopathology shows severe atrophy, neuronal loss, and gliosis. Pick's
48 cells (swol len, ba llooned neurons) and Pick's bodies (intracytoplasmic neuronal inclusion bod ies) are associated with Pick's
49 disease . Pick's disease occurs in a younger group of patients than Alzheimer's (the peak is around 55-65 years old). Patients
so typically present with aggressive behavior and are socially inappropriate, or apathetic. Speech and language deficits and
incontinence also occur early, in contrast to Alzheimer's d isease. Patients are treated symptomatically w ith nonsteroidal anti-
51
inflammatory drugs and antidepressants . Choli nergic agents and dopam inergic agents have also been t ried, but no clinical
52 studies have been done in patients with Pick's disease.
53 Pick's disease Gliosis Alzheimer's disease Cholinergic Histopathology Parietal lobe Dementia Atrophy Neuron Frontal lobe Dopaminergic Anti-inflammatory Urinary incontinence

54 Antidepressant Cerebral cortex

55 E is not correct. 10/o chose this.


56 Neurofibrillary tangles and sen ile plaques are associated with Alzheimer's disease . Cerebrocortica l atrophy may also be seen.
57 Alzheimer's disease is the most common cause of dementia . The parieta l cortex is most often affected . The onset is usual ly
58
later in life (beyond the sixth decade) and patients prese·nt w ith insidious progressive memory loss over a period of years .
Alzheimer's disease is a diagnosis of exclusion and can be definitively diagnosed only at autopsy. Medical treatment includes
59
psychotropic med ications and behavioral interventions.
60 Alzheimer's disease Senile plaques Dementia Neurofibrillary tangle Diagnosis of exclusion Parietal lobe Atrophy Psychoactive drug Autopsy Amnesia

61
62
Bottom line:
63
Huntington's disease is an incu rable inherited progressive neu rodegenerative disease, transmitted by trinucleotide repeats
64
(CAG) on chromosome 4 . The pathologic hallmark of this disease is atrophy of the caudate nucleus . Clin ically, these patients
65 present with chorea and dementia.
66 Huntington's disease Caudate nucleus Neurodegeneration Chorea Dementia Chromosome 4 (human) Trinucleotide repeat disorder Chromosome Atrophy Cell nucleus

67

Lock
s
Suspend
0
End Block
Item: 67 of 99 ~ 1 • Mark <:] (:>- Jill ~· ~J
QID: 22404 ..1 Previous Next Lab'V!I!ues Notes Calculator
"'"' - - - -- - - - - - - - - -- - --- - -- - - -- -- -- - - -- - -- --- - -
cells (swollen, ballooned neurons) and Pick's bodies (intracytoplasm ic neuronal inclusion bodies) are associated with Pick's
45
disease . Pick's disease occurs in a younger group of patients than Alzheimer's (the peak is around 55-65 years old). Patients
46 typically present with aggressive behavior and are socially inappropriate, or apathetic. Speech and language deficits and
47 incontinence also occur early, in contrast to Alzheimer's disease. Patients are treated symptomatically w ith nonsteroidal anti-
48
inflammatory drugs and antidepressants . Cholinergic agents and dopaminergic agents have also been tried, but no clinical
studies have been done in patients with Pick's disease.
49 Pick's disease Gliosis Alzheimer's disease Cholinergic Histopathology Parietal lobe Dementia Atrophy Neuron Frontal lobe Dopaminergic Anti-inflammatory Urinary incontinence
so Antidepressant Cerebral cortex
51
E i s n ot correct. 1 0/o c hose t his.
52
Neurofibrillary tangles and senile plaques are associated with Alzheimer's disease . Cerebrocortical atrophy may also be seen.
53
Alzheimer's disease is the most common cause of dementia . The parietal cortex is most often affected . The onset is usua lly
54 later in life (beyond the sixth decade) and patients prese·nt w ith insidious progressive memory loss over a period of years .
55 Alzheimer's disease is a diagnosis of exclusion and can be defin it ively diagnosed only at autopsy. Medical treatment includes
56
psychotropic medications and behavioral interventions.
Alzheimer's disease Senile plaques Dementia Neurofibrillary tangle Diagnosis of exclusion Parietal lobe Atrophy Psychoactive drug Autopsy Amnesia
57
58
59 Bottom Li ne:
60 Huntington's disease is an incu rable inherited progressive neurodegenerative disease, transmitted by trinucleotide repeats
61 (CAG) on chromosome 4 . The pathologic hall mark of th is disease is atrophy of the caudate nucleus . Clinically, these patients
present with chorea and dementia.
62 Huntington's disease Caudate nucleus Neurodegeneration Chorea Dementia Chromosome 4 (human) Trinucleotide repeat disorder Chromosome Atrophy Cell nucleus
63
64
65 References:
66 FA Step 2 CK 9th ed p 282
FA Step 2 CK 8th ed pp 268-269
67

Lock
s
Suspend
0
End Block
Item: 68 of 99 ~ 1 • Mark <:] (:>- Jill ~· ~J
QID: 22207 ..1 Previous Next Lab'V!I!ues Notes Calculator
.
46 A 22-year-old woman suffers from migraine headaches. These headaches last >24 hours, occur 2-3 times per week, and
impair her ability to study effectively or attend her college courses . Her headaches are responsive to sumatriptan once
47
they begin. Her physical examination is with in normal range.
48
49
What prophylactic regimens are recommended to prevent and/ or reduce the occu rrence of her migraine headaches?
so
51
A. Acetaminophen
52
53
B. Benzodiazepines
54 C. Ergotamine
55
D. Nonsteroidal anti-inflammatory drugs
56
57
E. Propranolol
58 F. Triptans
59
60
61
62
63
64
65
66
67
• 68

Lock
s
Suspend
0
End Block
Item: 68 of 99 ~ 1 • Mark <:] (:>- Jill ~· ~J
QID: 22207 ..1 Previous Next Lab'V!I!ues Notes Calculator
.
46
47
The correct answer is E. 680/o chose this.
48
Prophylactic therapy for migraine headaches is indicated when the patient suffers from migraines more than four times per
49 month and when the episodes last > 12 hours or interfere with functioning . When a migra ine headache occurs, the first line of
so therapy is abortive therapy, ranging from nonsteroida l anti-inflammatory drugs to triptans. To prevent and/or reduce the
51 occurrence of these episodes, prophylactic therapy is r ecommended . Prophylactic migraine medications that have
documented high efficacy in clinical trials include propranolol, metoprolol, timolol, amitriptyline, topiramate, and valproate.
52
Valproate is contraindicated in pregnancy and shou ld be avoided in women of ch ildbearing age.
53 Propranolol Topiramate Amitriptyline ~1etoprolol Timolol Valproate Triptan Preventive healthcare Migraine Nonsteroidal anti-inflammatory drug Headache Anti-inflammatory
54 Contraindication Pregnancy
55
A is not correct. 2% chose this.
56
Acetaminophen is used as primary abortive therapy for t hese headaches.
57 Paracetamol
58
B is not correct. 20/o chose this.
59
Benzodiazepines are used as primary abortive therapy for migraine headaches w hen other abortive choices are not
60 efficacious.
61 Migraine Benzodiazepine

62
Cis not correct. 110/o chose this.
63 Ergotamine is used as primary abortive therapy for migra ine headaches .
64 Ergotamine Migraine

65
D is not correct. 60/o chose this.
66
Nonsteroidal anti-inflammatory drugs are used as primary abortive therapy for migraine headaches.
67 Migraine Anti-inflammatory Nonsteroidal anti-inflammatory drug

68 ..- I"":---.&. ------.1. • • "''- - L - - - .a.L:-

Lock
s
Suspend
0
End Block
Item: 68 of 99 ~ 1 • Mark <:] (:>- Jill ~· ~J
QID: 22207 ..1 Previous Next Lab'V!I!ues Notes Calculator
.
46 B is not correct. 20/o chose this.
47 Benzodiazepines are used as primary abortive therapy for migraine headaches when other abortive choices are not
efficacious.
48
Migraine Benzodiazepine
49
Cis not correct. 110/o chose this.
so
Ergotamine is used as primary abortive therapy for migraine headaches .
51
Ergotamine Migraine
52
53
D is not correct. 6% chose this.
54
Nonsteroidal anti-inflammatory drugs are used as primary abortive therapy for m igraine headaches.
Migraine Anti-inflammatory Nonsteroidal anti-inflammatory drug
55
56
F is not correct. 110/o chose this.
57
Triptans are used as primary abortive therapy for mig rai ne headaches .
Migraine Triptan
58
59
60 Bottom Line:
61 Prophylactic therapy for m igraine headaches is ind icated when the patient suffers from m igraines more than four times per
62 month and when the episodes last >12 hours or interfer e with functioning . Prophylactic migraine medications with
documented high efficacy in clinical trials include propranolol, metoprolol, timolol, amitriptyline, topiramate, and valproate.
63 Propranolol Topiramate Amitriptyline ~1etoprolol Timolol Migraine Valproate Preventive healthcare
64
65
66 References:
67 FA Step 2 CK 9th ed p 266
FA Step 2 CK 8th ed p 253
68

Lock
s
Suspend
0
End Block
Item: 69 of 99 ~ 1 • Mark <:] (:>- Jill ~· ~J
QID: 22221 ..1 Previous Next Lab'V!I!ues Notes Calculator
..
47 A 47-year-old man involved in a motor veh icle accident is unresponsive and unarousable . His respiratory rate is 8/min,
heart rate is 120/min, blood pressure is 90/65 mm Hg, and temperature is 37 .1 °C (98.8°F) . He does not open his eyes,
48
nor does he vocalize spontaneously or in response to painful stimuli such as a sternal rub or nai l bed stimulation . He
49 makes no spontaneous movements, but his muscles do flex in r esponse to painfu l stimuli. Toxicology results were negative and
so glucose levels are within the standard range .
51
52 Which of the following is the most appropriate next step?
53
54 A. Administer intravenous mann itol
55 B. CT of the head
56
C. Intubate
57
58 D. Measure blood glucose
59 E. Push tissue plasminogen activator
60
61
62
63
64
65
66
67
68
• 69

Lock
s
Suspend
0
End Block
Item: 69 of 99 ~ 1 • Mark <:] (:>- Jill ~· ~J
QID: 22221 ..1 Previous Next Lab'V!I!ues Notes Calculator
..
47
48
49
The correct answer is C. 850/o chose this.
The patient is comatose, characterized by unresponsiveness to painful stimuli, including a sterna l rub or na il bed stimu lation.
so
He has a Glasgow Coma Scale (GCS) score of 5 (no eye or verba l response, and decorticate posturing to pain). Furthermore,
51 the toxicology screen and glucose test ru le out intoxication or hyperg lycem ic coma . Anyone with a GCS score <8 requires
52 immediate intubation due to the li kelihood of increased intracranial pressure and consequent r espiratory depression.
Glasgow Coma Scale Hypoventilation Intracranial pressure Intubation Glucose Coma Toxicology Hyperglycemia Depression (mood) Major depressive disorder
53
54 A is not correct. 2% chose this.
55 Mann itol can be used to temporarily reduce brain swelling secondary to trauma, and it may be useful in this patient if signs of
56 increased intracrania l pressure (ICP) appear. However, to stabilize the patient's airway and breathing in the event of increased
ICP, intubation is a higher priority.
57
Mannitol Intracranial pressure Respiratory tract Cerebral edema Intubation Human brain Brain
58
59
B is not correct. 120/o chose this.
CT of the head is an important step in this patient's work-up because a bra in injury is likely, but the patient's breath ing must
60
be secured first.
61 Human brain Traumatic brain injury CT scan Acquired brain injury
62
Dis not correct. 1% chose this.
63
An important cause of coma that must be ruled out is hyperglycem ia. However, a GCS <8 in this trauma patient indicates
64 severe brain damage, and measuring blood glucose is secondary to stabilizing the patient's respiration .
65 Hyperglycemia Blood sugar Coma Brain damage Respiration (physiology) Cellular respiration Glucose Human brain Brain

66
E is not correct. OOfo chose this.
67
It is possible that this patient has had a severe embolic stroke . However, given that the internal bleeding of this trauma
68 patient has not yet been assessed, t issue plasminogen activator is not appropriate at this time . After stabi lizing the patient
69 y
and imaging studies, this option may be r econsidered . y

Lock
s
Suspend
0
End Block
Item: 69 of 99 ~ 1 • Mark <:] (:>- Jill ~· ~J
QID: 22221 ..1 Previous Next Lab'V!I!ues Notes Calculator
.. ICP, mtubat1on IS a h1gher pnonty.
47 Mannitol Intracranial pressure Respiratory tract Cerebral edema Intubation Human brain Brain

48
B is not correct. 120/o chose this.
49 CT of the head is an important step in this patient's work-up because a brain injury is likely, but the patient's breathing must
so be secured first.
Human brain Traumatic brain injury CT scan AcQuired brain injury
51
52 Dis not correct. 1% chose this.
53 An important cause of coma that must be ruled out is hyperglycem ia. However, a GCS <8 in this trauma patient indicates
54 severe brain damage, and measuring blood glucose is secondary to stabilizing the patient's respiration .
Hyperglycemia Blood sugar Coma Brain damage Respiration (physiology) Cellular respiration Glucose Human brain Brain
55
56 E is not correct. OO/o chose this.
57 It is possible that this patient has had a severe embolic stroke . However, given that the internal bleed ing of this trauma
58
patient has not yet been assessed, t issue plasminogen activator is not appropriate at this time . After stabi lizing t he patient
and imaging studies, this option may be r econsidered .
59 Tissue plasminogen activator Stroke Plasmin Embolism
60
61
62
Bottom line:
63 Anyone with a Glasgow Coma Scale score <8 requ ires immediate intubation due to the likeli hood of increased intracranial
pressure and consequent respiratory depression .
64 Glasgow Coma Scale Hypoventilation Intracranial pressure Intubation Coma Major depressive disorder Depression (mood)
65
66
67 References:
68 FA Step 2 CK 9th ed pp 289-290
FA Step 2 CK 8th ed pp 274-276
69

Lock
s
Suspend
0
End Block
Item: 70 of 99 ~ 1 • Mark <:] (:>- Jill ~· ~J
QID: 23502 ..1 Previous Next Lab'V!I!ues Notes Calculator
.
48 A 39-year-old woman presents w ith depression, facial twitching, and writhing movements of her distal extremities.
Molecular genetic testing reveals expansion of the CAG repeats on chromosome 4 .
49
so
What will CT of the head most li kely demonstrate?
51
52
A. Atrophy of the caudate with narrowing of the lateral ventricle
53
54 B. Atrophy of the caudate w ith preservation of the lateral ventricle
55 C. Atrophy of the caudate with widening of the lateral ventricle
56
D. Atrophy of the thalamus w ith narrowing of the lateral ventricle
57
58 E. Atrophy of the thalamus w ith widening of the lateral ventricle
59
60
61
62
63
64
65
66
67
68
69
• 70

Lock
s
Suspend
0
End Block
Item: 70 of 99 ~ 1 • Mark <:] (:>- Jill ~· ~J
QID: 23502 ..1 Previous Next Lab'V!I!ues Notes Calculator

• •
48
49
so
51 The correct answer i s C. 660/o chose t h is.
52 Huntington's disease is an autosomal dominant neurodegenerative disease that typically presents in the 30s or 40s . It is
characterized by dementia, psychiatric changes, and characteristic movements (chorea, athetosis, tremor). The affected gene
53
is on the short arm of chromosome 4. The classic neuroimaging finding on CT or MRI is atrophy of the head of the caudate
54 with widening of the lateral ventricles .
55 Huntington's disease Neurodegeneration Dominance (genetics) Gene Athetosis Chorea lateral ventricles Dementia Neuroimaging Ventricular system Caudate nucleus Autosome

56 Chromosome Tremor Magnetic resonance imaging Atrophy Chromosome 4 (human) Ventricle (heart)

57
A i s not correct. 8% ch ose this.
58
Caudate atrophy results in increase in the size of the frontal tip of the lateral ventricle; thus, the lateral ventricle becomes
59 wider, not narrower. Atrophy of the caudate generally occurs early in the course of Huntington's disease.
60 Huntington's disease lateral ventricles Caudate nucleus Ventricular system Atrophy Ventricle (heart)

61 B is not correct. 240/o chose t h is.


62 The major site of neurodegeneration in Huntington's disease is in the striatum, wh ich is comprised of the caudate nucleus and
63 putamen . Typically, neuronal loss is more prom inent in the caudate than in the putamen . On CT of the head, the
64
characteristic finding is atrophy of the caudate with resultant widening of the latera l ventricles .
Huntington's disease Caudate nucleus Putamen Striatum lateral ventricles Neurodegeneration Ventricular system Atrophy Ventricle (heart) Cell nucleus
65
66 Dis not correct. 1% chose this.
67
The classic finding in Huntington's disease is marked atrophy of the striatum (caudate > putamen) w ith the loss of neurons in
the caudate resulting in an "ex vacuo" dilation of the anterior horns of the lateral ventricles . Generalized bra in atrophy can
68
also occur, but atrophy of the thalamus is not a prominent feature.
69 Huntington's disease Putamen Thalamus Striatum lateral ventricles Neuron Ventricular system Caudate nucleus Atrophy Cerebral atrophy Human brain Ventricle (heart)

70 Anatomical terms of location Vasodilation

Lock
s
Suspend
0
End Block
Item: 70 of 99 ~ 1 • Mark <:] (:>- Jill ~· ~J
QID: 23502
"'
..1
. . . . .. .
Previous Next
.. . . .. .
Lab'V!I!ues Notes Calculator
. .. .' . . .. . .. . . . .. .
48
putamen . Ty pically, neuronal loss is more prominent in the caudate than in the putamen . On CT of the head, the
characteristic finding is atrophy of the caudate with resultant widening of the latera l ventricles .
49 Huntington's disease Caudate nucleus Putamen Striatum Lateral ventricles Neurodegeneration Ventricular system Atrophy Ventricle {heart) Cell nucleus
so
D is not correct. 1 Ofo chose this.
51
The classic finding in Huntington's disease is marked atrophy of the striatum (caudate > putamen) with the loss of neurons in
52 the caudate result ing in an "ex vacuo" dilation of the anterior horns of the lateral ventricles . Generalized brain atrophy can
53 also occur, but atrophy of the thalamus is not a prom inent feature.
54 Huntington's disease Putamen Thalamus Striatum Lateral ventricles Neuron Ventricular system Caudate nucleus Atrophy Cerebral atrophy Human brain Ventricle (heart)

55 Anatomical terms of location Vasodilation

56 E is not correct. 1 0/o chose t his.


57 The putamen and the caudate nucleus comprise the striatum . Pathologically, the characteristic change in adults with
58 Huntington's disease is sign ificant atrophy of the striatum that is typically more marked in the caudate than in the putamen,
59
with widening of the lateral ventricle. Thalamic atrophy is not a characteristic feature.
Huntington's disease Caudate nucleus Putamen lateral ventricles Striatum Ventricular system Thalamus Atrophy Cell nucleus Ventricle (heart)
60
61
62 Bottom Li ne:
63 Huntington's disease is an autosomal dominant neurodegenerative disease characterized by atrophy of the head of the
64 caudate with widening of the lateral ventricles . It presents with dementia, psychiatric changes, and characteristic
movements.
65 Huntington's disease Neurodegeneration Dominance (genetics) Lateral ventricles Dementia Autosome Ventricular system Caudate nucleus Atrophy
66
67
68 References:
69 FA Step 2 CK 9th ed p 282
FA Step 2 CK 8th ed pp 268- 269
70

Lock
s
Suspend
0
End Block
Item: 71 of 99 ~ 1 • Mark <:] (:>- Jill ~· ~J
QID: 22220 ..1 Previous Next Lab'V!I!ues Notes Calculator
.:
49 A 55-year-old man with a history of diabetes, hypertension, and atrial fibrillation presents w ith a newly diagnosed aphasia. ~~AI
His word repetition ability and language comprehension are both impaired, but his fluency is preserved .
so
51
Where is the lesion responsible for this kind of aphasia located?
52
53
A. Infarct in the territory of the left posterior cerebra l artery.
54
55 B. Stroke in the arcuate fasciculus
56 C. Stroke in the lenticulostriate branches of the m iddle cerebral artery
57
D. Stroke involving the inferior division of the middle cerebral artery
58
59 E. Stroke involv ing the superior division of the middle cerebral artery
60
61
62
63
64
65
66
67
68
69
70
• 71

Lock
s
Suspend
0
End Block
Item: 71 of 99 ~ 1 • Mark <:] (:>- Jill ~· ~J
QID: 22220 ..1 Previous Next Lab'V!I!ues Notes Calculator
:.0 : • •• •
49
so
51
52 The correct answer is D. 560/o chose this.
53 This patient presents with Wern icke's aphasia, featurin g problems w ith language comprehension and impairment of both
language input and output. Usua lly the speech is fluent but does not make sense. Sometimes Wern icke's aphasia is
54
accompanied by a contralateral superior homonymous quadrantanopia. The lesion is located in Wernicke's area, and is most
55 likely caused by a stroke in the inferior division of the middle cerebral artery.
56 Wernicke's area Aphasia Middle cerebral artery Receptive aphasia Quadrantanopia Contralateral Sentence processing Lesion Stroke Cerebral arteries Anatomical terms of location

57 Primary progressive aphasia

58
A is not correct. 4% chose this.
59
An infarct in the territory of the left posterior cer ebral artery would render a transcortical sensory aphasia .
60 Transcortical sensory aphasia Posterior cerebral artery Aphasia Receptive aphasia Infarction Cerebral arteries

61
B is not correct. 130/o chose this.
62
A stroke in the arcuate fasciculus would li kely result in conduction aphasia, wh ich is characterized by problems with repeating
63 what is said, but preserved fluency and comprehension.
64 Arcuate fasciculus Aphasia Conduction aphasia Stroke

65 C is not correct. 7% chose this.


66 A stroke in the lenticulostriate branches of the m iddle cerebral artery would affect the subcortical regions of the brain, such
67 as the internal capsule, pons, and basal gang lia, and thus wou ld more likely present with motor and sensory symptoms
68
instead of with aphasia, which is a man ifestation of cortical damage .
Basal ganglia ~1iddle cerebral artery Internal capsule Aphasia Pons Anterolateral central arteries Stroke Ganglion Cerebral arteries Human brain Cortex (anatomy) Cerebral cortex
69
Brain
70
71 E is not correct. 20% chose this.

Lock
s
Suspend
0
End Block
Item: 71 of 99 ~ 1 • Mark <:] (:>- Jill ~· ~J
QID: 22220 ..1 Previous Next Lab'V!I!ues Notes Calculator
:.o : .• _ . • ,._. __ ~.- ..• __ .,. •• __ ...• , . . • . . . • -··--··

49 B is not correct. 130/o chose this.


so A stroke in the arcuate fasciculus would li kely result in conduction aphasia, wh ich is characterized by problems with repeating
51 what is said, but preserved fluency and comprehension .
Arcuate fasciculus Aphasia Conduction aphasia Stroke
52
53 C is not correct. 7% chose this.
54 A stroke in the lenticulostriate branches of the middle cer ebral artery would affect the subcortical regions of the brain, such
55 as the internal capsule, pons, and basa l gang lia, and thus wou ld more likely present with motor and sensory symptoms
instead of with aphasia, which is a man ifestation of cortical damage .
56
Basal ganglia ~1iddle cerebral artery Internal capsule Aphasia Pons Anterolateral central arteries Stroke Ganglion Cerebral arteries Human brain Cortex (anatomy) Cerebral cortex
57
Brain
58
59
E is not correct. 20% chose this.

60
Broca's aphasia is characterized by preserved comprehension, but problems with language production . The cause is most
likely a lesion in Broca's area, usually caused by a large stroke in the territory of the superior division of the midd le cerebral
61 artery, and can pre sent w ith associated right-sided hemiparesis.
62 Broca's area Hemiparesis Aphasia Middle cerebral artery Expressive aphasia Lesion Stroke Cerebral arteries Primary progressive aphasia

63
64
Bottom line:
65
Wernicke's aphasia is most likely caused by a stroke involving the inferior division of the midd le cerebral artery.
66 Aphasia Middle cerebral artery Receptive aphasia Stroke Cerebral arteries Wernicke's area
67
68
69 References:
70 FA Step 2 CK 9th ed p 289
FA Step 2 CK 8th ed p 274
71

Lock
s
Suspend
0
End Block
Item: 72 of 99 ~ 1 • Mark <:] (:>- Jill ~· ~J
QID: 21073 ..1 Previous Next Lab'V!I!ues Notes Calculator
..
so A 65-year-old woman with a history of myoca rdial infarction, hypertension, and asthma presents with new-onset ~~AI
hallucinations. She can no longer sleep at night because she sees small children and cats in her apartment. She thinks she
51
must be going crazy and is too frightened to explain the symptoms to her husband. She has no prior psychiatric history.
52 Her blood pressure is 115/80 mm Hg supine and 90/60 mm Hg stand ing . Physical examination reveals an alert, oriented elderly
53 woman with a slight r esting tremor and mild rigidity in her upper and lower extrem ities, but no cogwheeling. Mini-Mental State
54 Examination reveals deficits in long-term reca ll.
55
What abnorma l neuronal finding is expected?
56
57
58
A. Birefringent crysta ls
59 B. Dark pigmentation in the substantia nigra
60
C. Eosinophilic cytoplasmic inclusions
61
62
D. Hypersegmented nuclei
63 E. Neurofibrillary tangles and webs
64
65
66
67
68
69
70
71
• 72

Lock
s
Suspend
0
End Block
Item: 72 of 99 ~ 1 • Mark <:] (:>- Jill ~· ~J
QID: 21073 ..1 Previous Next Lab'V!I!ues Notes Calculator
..
so
51
52
The correct answer is C. 550/o chose this.
53
Eosinophilic cytoplasmic inclusions are also known as Lewy bodies and ar e prevalent neuronal findings in patients with this
54 form of dementia. Lewy body dementia (LBD) is characterized by new onset hallucinations, delusions, extrapyramidal signs,
55 and/ or parkinsonism symptoms. Patients may experience periods of lucidity and may undergo extensive delirium evaluations.
56 It is im portant to distinguish LBD from Parkinson disease with Lewy Bodies (PDLB) . One of the easiest ways to distinguish the
diseases is by their temporal courses. I f dementia occurs within 12 months of parkinsonism symptoms, then it is LBD. If,
57
however, dementia occurs more than 12 months after the onset of parkinsonism symptoms, then it is PDLB. Other
58 distinguishing characteristics are that those with LBD have a less prominent tremor compared to those w ith PDLB, and only
59 "'25%-50% of patients with LBD have parkinsonism symptoms at time of dementia diagnosis, wh ile all of patients with PDLB
60
have parkinsonism at time of dementia diagnosis.
Dementia with lewy bodies Extrapyramidal symptoms Parkinsonism lewy body Tremor Parkinson's disease Hallucination Extrapyramidal system Dementia Delirium Cytoplasm
61
62 A is not correct. 2% chose this.
63 Birefringent crystals are found in joint aspirate of patients with gout.
Birefringence Gout
64
65 B is not correct. 220/o chose this.
66 Lewy body disease causes neuronal loss in the frontal lobes, locus ceruleus, and substantia nigra . However, darkly pigmented
neurons are normal neuronal findings in the substantia nigra, so this answer does not reflect expected abnormal findings.
67
locus coeruleus Dementia with lewy bodies lewy body Substantia nigra Frontal lobe locus (genetics) Neuron
68
D is not correct. 5% chose this.
69
Hypersegmented nuclei are features of polymorphonuclear neutrophils, not neurons in Lewy body disease .
70
lewy body Dementia with lewy bodies Cell nucleus Neuron Neutrophil
71
72
E is not correct. 16% chose this.

Lock
s
Suspend
0
End Block
Item: 72 of 99 ~ 1 • Mark <:] (:>- Jill ~· ~J
QID: 21073 ..1 Previous Next Lab'V!I!ues Notes Calculator
.. • B1refnngence Gout
so
B is not correct. 220/o chose this.
51
Lewy body disease causes neuronal loss in the frontal lobes, locus ceruleus, and substantia nigra . However, darkly pigmented
52 neurons are normal neuronal findings in the substantia nigra, so this answer does not reflect expected abnormal findings.
53 locus coeruleus Dementia with lewy bodies lewy body Substantia nigra Frontal lobe locus (genetics) Neuron

54
D i s not correct. 5 % chose this.
55
Hypersegmented nuclei are features of polymorphonuclear neutrophils, not neurons in Lewy body disease .
56 lewy body Dementia with lewy bodies Cell nucleus Neuron Neutrophil

57
E i s not cor rect. 1 6% ch ose this.
58
Neurofibrillary tangles and webs can be seen in other forms of dementia but are not specific for Lewy body disease.
59 Neurofibrillary tangles may also be found in Alzheimer disease and in the Lewy bodies of Parkinson disease. Alzheimer
60 disease would not present with hall ucinations, but with classic presentation of memory decline . It is only after sign ificant
61
memory decline has occurred that other symptoms such as behavioral changes and motor symptoms present themselves.
Dementia with lewy bodies lewy body Neurofibrillary tangle Alzheimer's disease Parkinson's disease Dementia Hallucination
62
63
64 Botto m Line :
65 Symptoms of Lewy body dementia include new onset hallucinations, delusions, extrapyram ida l signs, and/or repeated loss
66 of balance. Lewy bodies, or eosinophilic cytoplasmic inclusions, are prevalent neuronal findings in patients with this form of
dementia .
67 Dementia with lewy bodies Extrapyramidal symptoms Dementia Extrapyramidal system Hallucination lewy body Delusion Eosinophilic
68
69
70 References:
71 FA Step 2 CK 9th ed p 281
FA Step 2 CK 8th ed p 269
72

Lock
s
Suspend
0
End Block
Item: 73 of 99 ~ 1 • Mark <:] (:>- Jill ~· ~J
QID: 23273 ..1 Previous Next Lab'V!I!ues Notes Calculator
I

51 A 32-year-old man presents to the emergency department complaining of difficulty br eathing . On examination , he has
marked ptosis, is dyspneic, and is having difficulty speaki ng and swallowing . Serial measurements of the patient's vital
52
capacity are 700, 620, and 580 ml. X-ray of the chest r evea ls a right middle lobe infiltrate .
53
54
Which of the following is the best next step in management?
55
56
A. Albuterol nebulizer treatments
57
58
B. High-dose edrophonium

59 C. Immediate endotracheal intubation and plasmapheresis


60
D. Inhaled racemic epinephrine
61
62
E. Intravenous antibiotics
63
64
65
66
67
68
69
70
71
72
• 73

Lock
s
Suspend
0
End Block
Item: 73 of 99 ~ 1 • Mark <:] (:>- Jill ~· ~J
QID: 23273 ..1 Previous Next Lab'V!I!ues Notes Calculator
I

51 The correct answer is C. 700/o chose this.


52 Myasthenia gravis is an autoimmune disease caused by antibodies aga inst postsynaptic acetylcholine receptors . Treatment is
53 typically aimed at down regulating the immune response or increasing the concentration of acetylcholine with in the synapse.
In emergency situations such as during myasthenia crisis with impend ing respiratory failure, plasmapheresis and intravenous
54
immunoglobulin after intubation provide a means of temporarily reliev ing symptoms . Indicators of respiratory compromise
55 include loss of upper airway integrity and decreasing vital capacity or negative inspiratory force . The arterial blood gas is not
56 a good indicator of the need for intubation in these patients. The lifetime risk of a myasthen ic crisis is 15%-20% and is often
57 precipitated by infection, metabolic stress, or nondepolarizing anesthetic agents .
Plasmapheresis Myasthenia gravis Autoimmune disease Arterial blood gas Immunoglobulin therapy Negative inspiratory force Vital capacity Acetylcholine Intravenous therapy
58
Respiratory failure Immune system Autoimmunity Synapse Antibody Respiratory tract Anesthetic Chemical synapse Intubation Anesthesia Acetylcholine receptor Metabolism
59
Muscle weakness Receptor (biochemistry) Infection
60
61 A is not correct. 5% chose this.
62 Albuterol is helpful as a treatment in asthmatic and chronic obstructive pulmonary disease treatments with obstructive
63
defects. This patient has a neuromuscu lar disorder leading to restrictive defect.
Chronic obstructive pulmonary disease Salbutamol Asthma Neuromuscular disease Neuromuscular junction Respiratory disease Pulmonology
64
65 B is not correct. 150/o chose this.
66
Edrophonium is useful in diagnosing myasthenia, although it is rarely used as treatment.
Edrophonium
67
68 D is not correct. 5% chose this.
69 Inhaled racemic epinephrine can be a useful bronchodilator in pediatric patients w ith croup . However, it is not indicated in
adults with restrictive defects .
70 Bronchodilator Epinephrine Croup Racemic mixture Pediatrics
71
E is not correct. 50/o chose this.
72
Intravenous (IV) antibiotics wou ld be usefu l in a patient whose shortness of breath was caused by pneumonia but not a
73 Y - - · · - - - · · - - · · · - - .1-C--.t.. 1\l.t..t-- , , - t - .t..L.- -:-t-.a.. -:..J.JI- 1-t- - : .... .C:I.r.. .... - " ' - : - -··----'-=••- -C - - - · · - - - : - ""'1-- --&.:--.1.'- ------.a..-.a..:-- ---1

Lock
s
Suspend
0
End Block
Item: 73 of 99 ~ 1 • Mark <:] (:>- Jill ~· ~J
QID: 23273 ..1 Previous Next Lab'V!I!ues Notes Calculator
I

51
B is not correct. 150/o chose th is.

52
Edrophonium is useful in diagnosing myasthenia, although it is rarely used as treatment.
Edrophonium
53
54 D is not correct. 50/o chos e this .
55
Inhaled racemic epinephrine can be a usefu l bronchodilator in pediatric patients w ith croup . However, it is not indicated in
adults with restrictive defects .
56 Bronchodilator Epinephrine Croup Racemic mixture Pediatrics
57
E is not correct. 50/o c hose th is.
58
Intravenous (IV) antibiotics wou ld be useful in a patient whose shortness of breath was caused by pneumonia but not a
59
neuromuscular defect. Although the right m iddle lobe infiltrate is suggestive of pneumon ia, the patient's presentation and
60 declining vital capacity is compatible with a myasthenic crisis that has been precipitated by the infection . Clearly, I V
61 antibiotics would be ind icated but only after emergent int ubation .
Pneumonia Myasthenia gravis Vital capacity Antibiotics Intravenous therapy Dyspnea Neuromuscular junction Intubation
62
63
64 Bottom line :
65 Myasthenia gravis is an autoimmune disease caused by antibodies against postsynaptic acetylcholine receptors . I n
66 emergency situations such as myasthenic crisis with impending respiratory failure, plasmapheresis and intravenous
67 immunoglobulin after intubation provide a means of temporarily relieving symptoms .
Plasmapheresis Myasthenia gravis Autoimmune disease Immunoglobulin therapy Acetylcholine Antibody Intravenous therapy Respiratory failure Autoimmunity Chemical synapse
68 Intubation Muscle weakness Acetylcholine receptor
69
70
71 References:
72 FA Step 2 CK 9th ed p 274
FA Step 2 CK 8th ed p 261
73

Lock
s
Suspend
0
End Block
Item: 74 of 99 ~ 1 • Mark <:] (:>- Jill ~· ~J
QID: 22067 ..1 Previous Next Lab'V!I!ues Notes Calculator

52 A 58-year-old African-American man w ith hypertension and diabetes presents to the ophthalmology clinic for a routine
examination . The patient has no visual complaints. Visual acuity is 20/20 in both eyes . Funduscopic examination reveals
53
enlarged optic nerve head cupping with significant rim pa Ilor in both eyes. Gon ioscopy r eveals open angles, and
54 applanation tonometry reveals borderline intra ocular pressu res.
55
56 Which of the following agents could be used to treat this patient's condition?
57
58 A. Corticosteroids
59
B. Intravenous mannitol
60
61 C. Isoproterenol
62 D. Phenoxybenzamine
63
E. Pilocarpine
64
65
66
67
68
69
70
71
72
73
• 74

Lock
s
Suspend
0
End Block
Item: 74 of 99 ~ 1 • Mark <:] (:>- Jill ~· ~J
QID: 22067 ..1 Previous Next Lab'V!I!ues Notes Calculator

52
53 The correct answer is E. 64% chose this.
54 This patient has open-angle glaucoma (OAG) . This condition is diagnosed by an increased intraocular pressure, abnormal
optic disk findings, and typical v isual field loss. The eye is not red or painful, the pupil and cornea appea r normal, and it
55
commonly affects both eyes . OAG can be treated with a variety of medications. Pilocarpine is a muscarinic agonist that
56 produces rapid miosis and contraction of the ciliary muscles. This can be used in the acute treatment of OAG or closed-angle
57 glaucoma to open the trabecular meshwork around Schlemm's canal, increasing drainage of aqueous humor and decreasing
58 intra ocular pressu re. Other agents that increase dra inage of aqueous humor secretion include prostaglandins (latanoprost).
OAG can also be treated w ith agents that decrease aqueous humor synthesis which include ~-blockers (timolol), ca rbon ic
59
anhydrase inhibitors, aragonists (brimonidine), and epinephrine .
60 Pilocarpine Miosis Trabecular meshwork Epinephrine Glaucoma Muscarinic agonist Comea Schlemm's canal Brimonidine Aqueous humour Latanoprost Intraocular pressure
61 Carbonic anhydrase inhibitor Prostaglandin Carbonic anhydrase Ciliary muscle Agonist ~1uscarinic acetylcholine receptor Optic disc Visual field Visual field loss
62
A is not correct. 5% chose this.
63
Corticosteroids are the treatment of choice for a number of ophthalm ic conditions including uveitis, retrobulbar neuritis, optic
64 neuritis, and allergic conjunctivitis . However, steroids are not indicated for the treatment of glaucoma .
65 Optic neuritis Glaucoma Uveitis Conjunctivitis Allergic conjunctivitis Corticosteroid Neuritis Ophthalmology Allergy Medulla oblongata

66
B is not correct. 120/o chose this.
67 Unlike open angle glaucoma (OAG), narrow-angle glaucoma is caused by the blockage of the aqueous outflow passageway.
68 Dilation of the pupils may precipitate an attack since the iris bunches up and narrows the angle. This condition presents with
69 intense pain and conjunctival injection. The patient may see halos around lights, may have decreased vision, and may
present with nausea and vomiting . Treatment includes pilocarpine to constrict the pupil, topical ~-blockers, ca rbon ic
70
anhydrase inhibitors, and a 2 -agonists . Patients should also be treated with intravenous mannitol, an osmotic agent that draws
71 water out of the eye and reduces pressure . Intravenous mannitol is not used for the treatment of OAG.
72 Pilocarpine Glaucoma Mannitol Carbonic anhydrase inhibitor Carbonic anhydrase Nausea Intravenous therapy Vomiting Mydriasis Conjunctivitis Vasodilation

73 C is not correct. 130/o chose this.


74

Lock
s
Suspend
0
End Block
Item: 74 of 99 ~ 1 • Mark <:] (:>- Jill ~· ~J
QID: 22067 ..1 Previous Next Lab'V!I!ues Notes Calculator

I e• • • •
52 Di lation of the pupils may precipitate an attack since the iris bunches up and narrows the angle . This condition presents with
53
intense pain and conjunctival injection. The patient may see halos around lights, may have decreased vision, and may
present with nausea and vomiting . Treatment includes pilocarpine to constrict the pupil, topical 13-blockers, ca rbon ic
54
anhydrase inhibitors, and a 2 -agonists . Patients should also be treated with intravenous mann itol, an osmotic agent that draws
55 water out of the eye and reduces pressure . Intravenous mann itol is not used for the treatment of OAG.
Pilocarpine Glaucoma Mannitol Carbonic anhydrase inhibitor Carbonic anhydrase Nausea Intravenous therapy Vomiting Mydriasis Conjunctivitis Vasodilation
56
57 C is not correct. 130/o chose this.
58 Isoproterenol is a 13-agonist. It has no use in the ophtha lmic t reatment of glaucoma. Glaucoma is treated w ith topica l 13-
59 blockers.
Glaucoma Isoprenaline Ophthalmology
60
61 D is not correct. 6% chose this.
62 Phenoxybenzamine is a nonselective a-blocker. It has no use in the ophthalmic t reatment of glaucoma. Glaucoma is treated
63
w ith aragonists.
Glaucoma Phenoxybenzamine Functional selectivity Ophthalmology
64
65
66 Bottom Line:
67 Open-angle glaucoma is diagnosed by an increased intraocular pressure, abnorma l optic disk f indings, and typica l v isual
68 field loss . Treatment options include pilocarpine, prostaglandins (latanoprost), epineph rine, 13-blockers (timolol), ca rbon ic
anhydrase inhibitors, and a 2 -agonists (brimonidine) .
69 Pilocarpine Epinephrine Glaucoma Carbonic anhydrase Brimonidine Intraocular pressure latanoprost Carbonic anhydrase inhibitor Prostaglandin Visual field Visual field loss
70
71
72 References:
73 FA Step 2 CK 9th ed p 293
FA Step 2 CK 8th ed p 278
74

Lock
s
Suspend
0
End Block
Item: 75 of 99 ~ 1 • Mark <:] (:>- Jill ~· ~J
QID: 28003 ..1 Previous Next Lab'V!I!ues Notes Calculator

The following vignette applies to the next 2 items. The items in the set must be answered in sequential order. Once you click Proceed to Next Item, you will not be
53
able to add or change an answer.
54
55
A 28-year-old woman with a seizure disorder is brought to the emergency department by her boyfriend because of ~~Aj
repetitive shaking w ithout intervening periods of consciou1sness that began 15 minutes prior to arrival. The boyfriend
56 states that she has been feeling sad and stressed lately, and has not been compliant with her medications . Hner
57 temperature is 37 .0°C (98 .6°F), blood pressure is 124/76 mm Hg, pulse is 72/min, and respiratory rate is 14/m in . She is
58 unconscious and has repetitive rig id extension of her extr,em ities followed by jerking movements . She is intubated and
mechanically ventilated.
59
60
What is the most appropriate next step in management?
61
62
A. Administer intravenous lorazepam
63
64 B. Administer intravenous phenobarbital
65
C. Administer thiamine, glucose, and naloxone
66
D. CT of the head with and without contrast
67
68 E. Perform a urine toxicology screen
69
70 PROCEED TO NEXT ITEM
71
72
73
74
54
55 The correct answer is A. 770/o chose this.
56 The patient is in status epilepticus, which occurs when a patient has been seizing for > 10 minutes or has repetit ive seizures
57 in a short period without a return to baseline. Given her history, she is most likely in status epilepticus due to discontinuation
58
of her anticonvulsant medication . Because the patient is currently hemodynamica lly stable and has a secure airway, the initial
therapy is to administer a short-acting benzod iazepine such as intravenous lorazepam .
59 Benzodiazepine Anticonvulsant lorazepam Status epilepticus Intravenous therapy Respiratory tract Epileptic seizure Pharmaceutical drug
60
B is not correct. 90/o chose this.
61
Intravenous phenobarbital is used to treat refractory status epilepticus. I f the patient is still seizing after therapy with
62
lorazepam followed by a loading dose of fosphenytoin and intubation , then phenobarbital can be given .
63 lorazepam Phenobarbital Fosphenytoin Status epilepticus Intravenous therapy Intubation

64
C is not correct. 6% chose this.
65
Although thiamine, glucose, and naloxone may be warranted, these medications are not the most appropriate next step for
66 treating status epilepticus in a patient with a history suggestive of recent anticonvulsant discontinuation .
67 Naloxone Anticonvulsant Thiamine Status epilepticus Glucose

68 D is not correct. 3% chose this.


69 Given the patient's history, it is unlikely that the seizure is secondary to an intracran ial hemorrhage or tumor. Thus CT would
70 not be the most appropriate next step in management.
Intracranial hemorrhage Bleeding Neoplasm Epileptic seizure
71
72 E is not correct. SO/o chose this.
73 Although a urine toxicology screen may be warran ted, it is not the most appropriate next step in management for this
74 patient.
Urine Toxicology
Item: 75 of 99 ~ 1 • Mark <:] (:>- Jill ~· ~J
QID: 28 003 ..1 Previous Next Lab'V!I!ues Notes Calculator

•• ••
53 Intravenous phenobarbital is used to treat refractory status epilepticus. If the patient is still seizing after therapy with
54 lorazepam followed by a loading dose of fosphenytoin and intubation, then phenobarbita l can be given .
lorazepam Phenobarbital Fosphenytoin Status epilepticus Intravenous therapy Intubation
55
56 C is not correct. 60/o chose this.
57 Although thiam ine, glucose, and naloxone may be warranted, these medications are not the most appropriate next step for
treating status epilepticus in a patient with a history suggestive of recent anticonvulsant discontinuation.
58
Naloxone Anticonvulsant Thiamine Status epilepticus Glucose
59
60
D is not correct. 3% chose this.
Given the patient's history, it is unlikely that the seizure is secondary to an intracran ial hemorrhage or tumor. Thus CT would
61
not be the most appropriate next step in management.
62 Intracranial hemorrhage Bleeding Neoplasm Epileptic seizure
63
E is not correct. SO/o chose this.
64
Although a urine toxicology screen may be warran ted, it is not the most appropriate next step in management for this
65 patient.
66 Urine Toxicology

67
68
Bottom line:
69
The in itia l treatment for status epilepticus is to adm inister a short-acting benzodiazepine such as lorazepam .
70 Benzodiazepine lorazepam Status epilepticus
71
72
73 References:
74 FA Step 2 CK 9th ed pp 270-271
FA Step 2 CK 8th ed p 257
Item: 76 of 99 ~ 1 • Mark <:] (:>- Jill ~· ~J
QID: 28 004 ..1 Previous Next Lab'V!I!ues Notes Calculator

54 A 32-year-old man with a seizure disorder is brought to the emerg ency department by his girlfriend because of repetitive
shaking w ithout intervening periods of consciousness that began 15 minutes prior to arrival. The girlfriend states that he
55
has been feeling sad and stressed lately, and has not been compliant w ith his medications. His temperature is 37 .0°C
56 (98 .6°F), blood pressure is 124/76 mm Hg, pulse is 72/min, and respiratory rate is 14/m in . He is unconscious and has repetitive
57 rigid extension of his extrem ities followed by jerking movements . After giving him two doses of intravenous lorazepam and a
58
loading dose of fosphenytoin, followed by a half loading dose of fosphenytoin, the patient is still seizing.
59
What is the most appropriate next step in management?
60
61
62
A. Administration of clonazepam
63 B. Administration of topiramate
64
C. CT scan of the head with and without contrast
65
66
D. Obtain an electroencephalog ram
67 E. Rapid sequence intubation
68
69
70
71
72
73
74

.~
Lock
s
Suspend
0
End Block
Item: 76 of 99 ~ 1 • Mark <:] (:>- Jill ~· ~J
QID: 28 004 ..1 Previous Next Lab'V!I!ues Notes Calculator

54
55
56
The correct answer is E. 74% chose this.
57
Remember CAB : Circulation, Airway, and Breath ing . For any patient in status epilepticus, a rapid general assessment w ith
58 attention to respiratory and circulatory status is needed . Given the length of time of the patient's seizure, he is at risk of
59 losing his airway. His airway needs to be protected while he is being prepared for intubation . Thus airway management
60 becomes paramount if the patient fails to respond to initi al medical management. Rapid sequence intu bation (RSI) involves a
rapidly active sedative agent and a neuromuscular blocking agent in order to render the patient unconscious and paralyzed in
61
order to facilitate control of the airway in emergent situations, such as status epilepticus. Common sedatives used for RSI in
62 status epilepticus patients include midazolam and thiopenta l. Concomitant with intubation, third-line medical therapy with
63 phenoba rbital should be in itiated .
Phenobarbital Midazolam Status epilepticus Sedative Intubation Airway management Rapid sequence induction Sodium thiopental Respiratory tract Neuromuscular junction
64
Epileptic seizure Paralysis
65
66 A is not correct. 7% chose this.
67 Clonazepam is a benzodiazepine, which works by decreasing abnormal electrical activity in the brain . Outside of the United
68 States, IV clonazepam is commonly used as first-line therapy for status epilepticus. Since the patient is still seizing after the
administration of lora zepam and fosphenytoin, it would not be effective in this scenario .
69
Benzodiazepine Lorazepam Clonazepam Fosphenytoin Status epilepticus Brain
70
B is not correct. 100/o chose this.
71
Small case reports have demonstrated some efficacy in using topiramate in refractory status epilepticus. However, because
72
more studies are needed, its use is not currently recommended .
73 Topiramate Status epilepticus Refractory
74
C is not correct. 4% chose this.

~6 y
Given the patient's history, it is unlikely that the seizure 'is secondary to an intracran ial hemorrhage or tumor. Thus CT would

Lock
s
Suspend
0
End Block
Item: 76 of 99 ~ 1 • Mark <:] (:>- Jill ~· ~J
QID: 28 004 ..1 Previous Next Lab'V!I!ues Notes Calculator

• States, IV clonazepam IS commonly used as f1rst-lme therapy for status ep1lept1cus. Smce the pat1ent IS st1ll se1zmg after the
54 administration of lorazepam and fosphenytoin, it would not be effective in this scenario.
Benzodiazepine Lorazepam Clonazepam Fosphenytoin Status epilepticus Brain
55
56 B is not correct. 100/o chose this.
57 Small case reports have demonstrated some efficacy in using topiramate in refractory status epilepticus. However, because
58 more studies ar e needed, its use is not currently recommended .
Topiramate Status epilepticus Refractory
59
60 C is not correct. 4% chose this.
61 Given the patient's history, it is unlikely that the seizure is secondary to an intracran ial hemorrhage or tumor. Thus CT would
not be the most appropriate next step in management.
62
Intracranial hemorrhage Bleeding Neoplasm Epileptic seizure
63
64 D is not correct. 5% chose this.
65
Many cases of status epilepticus, such as this one, can be diagnosed on neurologic exam alone . Although an
electroencephalogram may be warranted to diagnose or m onitor complex cases, it is not the most appr opriate next step in
66
management.
67 Electroencephalography Status epilepticus Neurological examination Neurology

68
69
Bottom line:
70
Remember CAB {Circulation, Airway, Breathing ) when t r eating status epilepticus .
71 Status epilepticus
72
73
74 References:
FA Step 2 CK 9th ed pp 270-271

~ FA Step 2 CK 8th ed p 257

Lock
s
Suspend
0
End Block
Item: 77 of 99 ~ 1 • Mark <:] (:>- Jill ~· ~J
QID: 22062 ..1 Previous Next Lab'V!I!ues Notes Calculator
.
55 A 65-year-old African-American woman presents to the clinic because of chronic recurrent episodes of headache, unilateral ~~AI
pain, and blurry vision in her left eye. These attacks often occur at night and resolve spontaneously overnight. In the
56
clin ic, she reports no current symptoms. Intraocular pressures are 15 mm Hg in the right eye and 20 mm Hg in the left
57 eye . Gonioscopy examination shows bilateral na rrow ante·rior chamber angles .
58
59 Which of the following is the most appropriate cu rative t reatment for th is patient?
60
61 A. Acetazolamide
62
B. Atropine
63
64 C. Laser iridotomy
65 D. Piloca rpine
66
E. Timolol
67
68
69
70
71
72
73
74

Lock
s
Suspend
0
End Block
Item: 77 of 99 ~ 1 • Mark <:] (:>- Jill ~· ~J
QID: 22062 ..1 Previous Next Lab'V!I!ues Notes Calculator
.
55
56 The correct answer is C. 590/o chose this.
57 This patient has symptoms of subacute closed-angle glaucoma . Although medical treatment can provide symptomatic relief,
only laser iridotomy provides cu rative t reatment. Laser iridotomy forms a permanent connection between the anterior and the
58
posterior chambers, and prevents recu rrence. In some cases, the fellow eye shou ld undergo prophylactic laser iridotomy.
59 Another important concept is that 20% of cases of glaucoma can occu r with intraocular pressures <2 1 mm Hg .
60 Glaucoma Iridectomy Posterior chamber of eyeball Laser

61
A is not correct. 110/o chose this.
62 Acetazolam ide is carbonic anhydrase inhibitor that can decrease aqueous humor secretion and reduce intraocular pressure .
63 However, it on ly provides symptomatic treatment and is not cu rative .
Acetazolamide Carbonic anhydrase inhibitor Carbonic anhydrase Aqueous humour Intraocular pressure Enzyme inhibitor
64
65 B is not correct. 40/o chose this.
66 Atropine is an anti muscarinic agent that results in mydriasis, which w ill further exacerbate the patient's symptoms of closed-
67 angle glaucoma by its action on the t rabecu lar meshwork.
Muscarinic antagonist Mydriasis Atropine Trabecular meshwork Glaucoma
68
69 D is not correct. 140/o chose this.
70 Piloca rpine is a muscarinic agonist that produces rapid miosis and contraction of the ciliary muscles. This can be used in the
71
acute t reatment of closed- or open-angle glaucoma to open the trabecular meshwork around Schlemm's canal, increasing
drainage of aqueous humor and decreasing intraocular pr essure. However, it only provides symptomatic treatment and is not
72 cu rative .
73 Pilocarpine Miosis Trabecular meshwork Glaucoma Muscarinic agonist Schlemm's canal Aqueous humour Intraocular pressure Muscarinic acetylcholine receptor Agonist Ciliary muscle

74
E is not correct. 12% chose this.

~ Timolol is a nonselective ~-antagon i st that reduces the p roduction of aqueous humor in the eye . It can be used topically in
6 the t reatment of chronic open-ang le glaucoma, but it does not provide curative treatment for ch ron ic closed-angle glaucoma .
77 Timolol Glaucoma Aqueous humour Binding selectivity
y

Lock
s
Suspend
0
End Block
Item: 77 of 99 ~ 1 • Mark <:] (:>- Jill ~· ~J
QID: 22062 ..1 Previous Next Lab'V!I!ues Notes Calculator
. • Acetazolamide carbOniC anhydrase InhibitOr carbOniC anhydrase AQueous humour Intraocular pressure Enzyme InhibitOr
55
B is not correct. 40/o chose this.
56
Atropine is an anti muscarinic agent that results in mydriasis, w hich w ill further exacerbate the patient's symptoms of closed-
57 angle glaucoma by its action on the trabecular meshwork.
58 ~1uscarinic antagonist Mydriasis Atropine Trabecular meshwork Glaucoma

59
D is not correct. 140/o chose this.
60
Pilocarpine is a muscarinic agonist that produces rapid m iosis and contraction of the ciliary muscles. This can be used in the
61 acute treatment of closed- or open-angle glaucoma to open the trabecular meshwork around Schlemm's canal, increasing
62 drainage of aqueous humor and decreasing intraocular pr essure. However, it only provides symptomatic treatment and is not
curative.
63
Pilocarpine Miosis Trabecular meshwork Glaucoma Muscarinic agonist Schlemm's canal Aqueous humour Intraocular pressure Muscarinic acetvlcholine receotor Agonist Ciliary muscle
64
65
E is not correct. 12% chose this.
Timolol is a nonselective ~-antagonist that reduces the production of aqueous humor in the eye . It can be used topically in
66
the treatment of chronic open-angle glaucoma, but it does not provide curative treatment for chronic closed-angle glaucoma .
67 Timolol Glaucoma Aqueous humour Binding selectivity
68
69
70
Bottom line:
71 Laser iridotomy forms a permanent connection betweern the anterior and the posteri or chambers, and prevents recurrence
of subacute closed-angle glaucoma .
72 Glaucoma Iridectomy Posterior chamber of eyeball Laser
73
74

~
References:
FA Step 2 CK 9th ed p 293
FA Step 2 CK 8th ed pp 277-278
77

Lock
s
Suspend
0
End Block
Item: 78 of 99 ~ 1 • Mark <:] (:>- Jill ~· ~J
QID: 22225 ..1 Previous Next Lab'V!I!ues Notes Calculator

56 A 59-year-old man presents to the clinic with a broad-based unsteady gait. He is unable to walk in a straight li ne and ~~AI
moves from side to side as he walks. His upper limb coordination is intact. He has no history of vertigo, and no nystagmus
57
is noted on physical examination .
58
59
Which of the following is the most likely cause of this patient's symptoms?
60
61
A. Cerebellar hemisphere infarction
62
63
B. Cerebellar verm is lesion
64 C. Hydrocephalus
65
D. Meniere's disease
66
67
E. Vestibular neuronitis
68
69
70
71
72
73
74

~
77
• 78

Lock
s
Suspend
0
End Block
Item: 78 of 99 ~ 1 • Mark <:] (:>- Jill ~· ~J
QID: 22225 ..1 Previous Next Lab'V!I!ues Notes Calculator

56 The correct answer is B. 630/o chose this.


57 The cerebellum integrates and coordinates sensory perception and motor output. Therefore, cerebellar lesions do not cause
58 paralysis, but typically cause fine motor movement disorders, as well as deficits in equilibrium, posture and motor learn ing .
The patient presents w ith gait ataxia, wh ich is typically of a vermis lesion .
59
Cerebellum Ataxia Gait abnormality Motor learning Gait (human) Lesion Cerebellar vermis Paralysis Gait Perception Movement disorder
60
61
A is not correct. 160/o chose this.
62
A cerebellar hemisphere lesion would present with ipsilateral lim b ataxia. Hemispheric lesions are characterized by
decomposition of movement, dysmetria and rebound, intention tremor, or kinetic tremors (presenting in motion).
63 Hemispheric cerebellar dysfunction is best ascertained clinically via the finger-to-nose and heel-to-knee tests.
64 Dysmetria Intention tremor Cerebellar ataxia Ataxia Lesion Tremor Cerebellum Anatomical terms of location

65
C is not correct. 120/o chose this.
66
The classic triad of clinical features in normal pressure hydrocephalus includes gait disturbance, urinary incontinence, and
67 cognitive disturbance. These features are believed to be derived from dysfunction of the periventricular white matter tracts,
68 especially of the frontal lobe connections. A patient with hydrocephalus would more likely present with a frontal ataxia, also
known as a magnetic gait or gait apraxia w ith impaired gait in itiation . There is the appearance of the patientls feet being
69
"stuck" to the floor, with short steps, decreased stride length and height, a broadened base, and outwardly rotated feet.
70 Normal pressure hydrocephalus Hydrocephalus Bruns apraxia Urinary incontinence Frontal lobe Gait abnormality White matter Ataxia Magnetic gait Apraxia Gait (human) Gait
71 Cognition
72
D is not correct. 3% chose this.
73
Meniere's disease is associated w ith episodic vertigo, sensorineural hearing loss, an aura of fullness or pressu re in the ear or
74
side of the head, and tinnitus . The episodic vertigo can be followed by a period of nausea and unsteadiness for several hours.

~
Meniere's disease Tinnitus Sensorineural hearing loss Vertigo Nausea Hearing loss

6
E is not correct. 60/o chose this.
77
Patients with vestibular neuronitis present with rapid onset of a severe vertigo along with nausea, vomiting, and gait
78 y instabilitv. The ohvsica l examination is sianificant for a soontaneous vestibular nvstaamus that is unilateral. horizontal. or

Lock
s
Suspend
0
End Block
Item: 78 of 99 ~ 1 • Mark <:] (:>- Jill ~· ~J
QID: 22225 ..1 Previous Next Lab'V!I!ues Notes Calculator

C i s not correct. 1 20/o chose this.


56
The classic triad of cl inical features in normal pressure hydrocephalus includes gait disturbance, urinary incontinence, and
57 cogn it ive disturbance. These featur es ar e believed to be derived from dysfunction of the peri ventricular w hite matter t racts,
58 especially of the fr ontal lobe connections. A patient with hydrocephalus would more li kely present with a fr ontal ataxia, also
59 know n as a magnetic gait or gait apraxia w ith impaired gait initiation. There is the appearance of the patientls feet being
"stuck" to the floor, with short steps, decreased stride length and height, a broadened base, and outwar dly rotated feet.
60
Normal pressure hydrocephalus Hydrocephalus Bruns apraxia Urinary incontinence Frontal lobe Gait abnormality White matter Ataxia Magnetic gait Apraxia Gait (human) Gait
61
Cognition
62
63
D i s not correct. 3% chose this.
64
Meniere's disease is associated w ith episodic vertigo, sensorineural hearing loss, an aura of full ness or pressure in the ear or
side of the head, and t innitus . The episod ic vertigo can be followed by a period of nausea and unsteadiness for several hour s.
65 Meniere's disease Tinnitus Sensorineural hearing loss Vertigo Nausea Hearing loss
66
E i s not correct. 60/o chose th is.
67
Patients with vestibu lar neuronit is present with rapid onset of a sever e vert igo along with nausea, vom it ing, and gait
68 instability. The physical examination is sign ificant for a spontaneous vestibular nystagmus that is unilateral, horizontal, or
69 horizontal-torsional , and that is suppressed with visual fixation and does not change direction with gaze . The beating of the
70 fast phase is away from the affected side. This patient does not exhibit nystagmus.
Labyrinthitis Nystagmus Vertigo Vestibular system Nausea Vomiting Physical examination Gait
71
72
73 Botto m Li ne:
74 Cerebellar vermis lesions pr esent w ith fine motor movement disor ders as well as deficits in equilibrium, posture, and motor

~
learning .
Cerebellar vermis Motor learning Cerebellum

77
78

Lock
s
Suspend
0
End Block
Item: 79 of 99 ~ 1 • Mark <:] (:>- Jill ~· ~J
QID: 23485 ..1 Previous Next 'V!I!ues
Lab Notes Calculator
.
57 A 78-year-old woman presents to the emergency department complaining of episodes of severe dizziness over the past
two weeks. She reports intermittently feeling like "the room is spinning;" this sensation is exacerbated when she gets up
58
from her bed, and it lasts about 1 minute after she stops moving. She denies any confusion, weakness, loss of sensation,
59 visual changes, or speech difficulties. She reports no tinnitus or loss of hearing . She has a past medical history of asthma and
60 endometrial hyperplasia. Her vital signs are normal, as are her mental status examination and her motor and sensory
61
examinations. She exhibits horizontal nystagmus during the cranial nerve examination, and her dizziness and nystagmus are
exacerbated with movement of the head. She is able to p~erform heel-knee-shin and finger-nose-finger tests w ithout difficulty.
62
She is able to stand, but she cannot walk w ithout assistance . Her Romberg test is negative, and CT of the head shows no
63 abnormalities.
64
65 Which of the following is the most likely mechanism causing this patient's symptoms?
66
67 A. Atherosclerosis
68
B. Cerebellar tumor
69
70
C. Dislodged otolith in the semicircular canals
71 D. Increased endolymph volume
72
E. Viral infection of the vestibular nerve
73
74

~
77
78
• 79

Lock
s
Suspend
0
End Block
Item: 79 of 99 ~ 1 • Mark <:] (:>- Jill ~· ~J
QID: 23485 ..1 Previous Next Lab 'V!I!ues Notes Calculator
.
57
58 The correct answer is C. 800/o chose this.
59 When dealing with vertigo, the first step is to distinguish between central and peripheral causes . Clinical signs that indicate
60 peripheral causes (which are more common) include persistent horizontal nystagmus, hearing loss, and the lack of other
61 neurologic signs . Benign paroxysmal positiona l vertigo (BPPV) is the most common cause of peripheral disequilibrium, and it
accounts for >50% of cases . Patients complain that the "room is spinning;" they have short, intermittent dizziness spells that
62
last a few minutes, and they do not have any auditory symptoms or other neurologic signs . The dizziness causes nausea, but
63 patients usually do not vomit. Nystagmus is usually present, and it is exacerbated by head rotation or the Dix-Hallpike
64 maneuver. BPPV is usually caused by the dislodgement of calcium otoliths in the semicircular canals .
Nystagmus Benign paroxysmal positional vertigo Vertigo Otolith Semicircular canals Nausea Dizziness Hearing loss Dix-Hallpike Vomiting Dix-Hallpike test Calcium Sense of balance
65
Neurology Hearing Paroxysmal attack Benignity
66
67 A is not correct. 2% chose this.
68 Vertebrobasilar insufficiency as a result of stenosis or plaque ruptu re in the vertebral or basilar arteries can present with
69 dizziness, but it usually also presents with the dysfunction of other cranial nerves, which this patient does not exhibit.
Vertebrobasilar insufficiency Cranial nerves Stenosis Dizziness Artery
70
71 B is not correct. 30/o chose this.
72 Patients with cerebellar tumors often present with dizziness, but cerebellar examination and head CT were normal in this
patient.
73
Dizziness Neoplasm Cerebellum
74
D is not correct. 100/o chose this.

~
77
M€miere disease is caused by increased endolymph volume in the inner ear. These patients present not on ly with vertigo but
also with tinnitus, hearing loss, and aural fullness . The episodes of vertigo generally last for minutes to hours and are
associated w ith nausea and vomiting .
78 Endolymph Tinnitus Vertigo Hearing loss Inner ear Nausea Vomiting

79

Lock
s
Suspend
0
End Block
Item: 79 of 99 ~ 1 • Mark <:] (:>- Jill ~· ~J
QID: 23485 ..1 Previous Next Lab 'V!I!ues Notes Calculator

• • • p • I • • • •
57 patient.
Dizziness Neoplasm Cerebellum
58
59 D is not correct. 100/o chose this.
60 M€miere disease is caused by increased endolymph volume in the inner ear. These patients present not on ly with vertigo but
61
also with tinnitus, hearing loss, and aural fullness . The episodes of vertigo generally last for minutes to hours and are
associated w ith nausea and vomiting .
62 Endolymph Tinnitus Vertigo Hearing loss Inner ear Nausea Vomiting
63
E is not correct. SOlo chose this.
64
Vestibular neuronitis is an acute, benign, and self-limited disorder that usually presents with severe vertigo, nausea, and
65
vomiting . Patients are still able to walk, but may sway or fall toward the affected side . Nystagmus is usually present and
66 doesn't change direction . If there is hearing loss in the affected side, the disorder is called labyrinthitis; otherwise, hearing is
67 unaffected . Other neurologic signs are usually absent. It is a clinical diagnosis . Symptoms can last up to several days and
68
gradually fade. This patient is unable to walk, and her symptoms appear to be more chronic than those of vestibular
neuronitis--this is not the correct diagnosis.
69 labyrinthitis Nystagmus Vertigo Nausea Hearing loss Vestibular system Vomiting Benign tumor Benignity Neurology
70
71
72
Bottom line:
73 Benign paroxysmal positiona l vertigo is usually caused by the dislodgement of calcium otoliths in the semicircular canals. I t
is the most common cause of peripheral disequilibrium.
74 Benign paroxysmal positional vertigo Vertigo Semicircular canals Otolith Paroxysmal attack Benignity Calcium Sense of balance

~
77 References:
78 FA Step 2 CK 9th ed pp 271-272
FA Step 2 CK 8th ed p 258
79

Lock
s
Suspend
0
End Block
Item: 80 of 99 ~ 1 • Mark <:] (:>- Jill ~· ~J
QID: 222 12 ..1 Previous Next Lab'V!I!ues Notes Calculator

58 A 66-year-old woman with a newly diagnosed brain tumor presents with aphasia . Her ability to repeat words is preserved
and she is fluent but does not make sense . Also, her language comprehension is impaired . She has no other symptoms
59
and the remainder of her neurologic examination is unremarkable.
60
61
From wh ich kind of aphasia is she suffering?
62
63
A. Broca's aphasia
64
65
B. Conduction aphasia
66 C. Transcortical motor aphasia
67
D. Transcortical sensory aphasia
68
E. Wernicke's aphasia
69
70
71
72
73
74

~
77
78
79
• 80

Lock
s
Suspend
0
End Block
Item: 80 of 99 ~ 1 • Mark <:] (:>- Jill ~· ~J
QID: 222 12 ..1 Previous Next Lab'V!I!ues Notes Calculator

58 The correct answer is D. 320/o chose this.


59 The patient suffers from a transcortical sensory aphasia, characterized by impaired comprehension of language, but
60 preserved repetition and fluency. Transcortical sensory aphasia is caused by a lesion in the region known as the temporal-
occipital-parietal junction, located behind Wernicke's area .
61
Transcortical sensory aphasia Wernicke's area Aphasia Lesion Receptive aphasia
62
63
A is not correct. 8% chose this.
64
Broca's aphasia is characterized by preserved comprehension, but problems with language production . Both fluency and
repetition are impaired . It is caused by a lesion in the frontal lobe, in the inferior frontal gyrus.
65 Frontal lobe Inferior frontal gyrus Aphasia Expressive aphasia Lesion Broca's area
66
B is not correct. 100/o chose this.
67
Conduction aphasia is characterized by problems w ith repetition, but preserved fluency and comprehension. Conduction
68
aphasia is caused by a lesion in the arcuate fasciculus . The arcuate fasciculus is the neural pathway that connects Wernicke's
69 and Broca's areas via the parietal lobe.
70 Conduction aphasia Arcuate fasciculus Parietal lobe Aphasia Neural pathway Lesion Expressive aphasia Receptive aphasia

71 C is not correct. 90/o chose this.


72 This type of aphasia is characterized by impaired fluency, but preserved repetition and comprehension . Transcortical motor
73 aphasia is caused by a lesion in the anterior superior frontal lobe. The area of insult is a watershed region surrounding
74
Broca's area.
Broca's area Transcortical motor aphasia Frontal lobe Aphasia Expressive aphasia Lesion watershed stroke

~6
77
E is not correct. 41% chose this.
Wern icke's aphasia represents problems with language comprehension, which affects both language output and input.
Fluency is preserved but repetition is impaired . It is caused by a lesion in the posterior temporal lobe, in the superior
78
tempora l gyrus .
79 Superior temporal gyrus Aphasia Temporal lobe Receptive aphasia Sentence processing Lesion Wernicke's area Primary progressive aphasia

80 y

Lock
s
Suspend
0
End Block
Item: 80 of 99 ~ 1 • Mark <:] (:>- Jill ~· ~J
QID: 22212 ..1 Previous Next Lab'V!I!ues Notes Calculator
• I I • W • I I ..

58 Conduction aphasia is characterized by problems with repetition, but preserved fluency and comprehension. Conduction
59
aphasia is caused by a lesion in the arcuate fasciculus . The arcuate fasciculus is the neural pathway that connects Wernicke's
and Broca's areas v ia the parieta l lobe.
60 Conduction aphasia Arcuate fasciculus Parietal lobe Aphasia Neural pathway Lesion Expressive aphasia Receptive aphasia
61
C is not correct. 90/o chose this.
62
This type of aphasia is characterized by impa ired fluency, but preserved repetition and comprehension . Transcortical motor
63
aphasia is caused by a lesion in the anterior superior fronta l lobe. The area of insult is a watershed reg ion surrounding
64 Broca's area.
65 Broca's area Transcortical motor aphasia Frontal lobe Aphasia Expressive aphasia Lesion Watershed stroke

66 E is not correct. 41% chose this.


67 Wern icke's aphasia represents problems with language comprehension, which affects both language output and input.
68 Fluency is preserved but repetition is impaired . It is caus.ed by a lesion in the posterior temporal lobe, in the superior
69
temporal gyrus .
Superior temporal gyrus Aphasia Temporal lobe Receptive aphasia Sentence processing Lesion Wernicke's area Primary progressive aphasia
70
71
72 Bottom Line:
73 Transcortical sensory aphasia is characterized by impaired comprehension of la nguage with preserved repetition and fluency.
74 Transcortical sensory aphasia is caused by a lesion in the region known as the temporal-occipital-parietal junction, located
behind Wernicke's area .

~
Transcortical sensory aphasia Wernicke's area Aphasia Lesion Receptive aphasia

77
78 References:
79 FA Step 2 CK 9th ed pp 288-289
FA Step 2 CK 8th ed p 274
80

Lock
s
Suspend
0
End Block
Item: 81 of 99 ~ 1 • Mark <:] (:>- Jill ~· ~J
QID: 23777 ..1 Previous Next Lab'V!I!ues Notes Calculator
.
59 A child is brought to the emergency department (ED) by her parents after falling down the sta irs at home. The child is
conscious but appears drowsy and is unable to focus long enough to answer detailed questions. She complains of a
60
headache and feels nauseated . Th is is the child's third ED visit in the past month; her last v isit was because of a bone
61 fracture after falling off her swing set in the backya rd. Physica l exam ination r eveals several lacerations and bruises in different
62 stages of healing scattere d over the child 's body. The child is sent for imaging to evaluate the cause for her altered level of
63
consciousness; MRI of the brain is shown in the image.
64
65
66
67
68
69
70
71
72
73
74

~
77
Which vascular structu re is most likely damaged in this patient?

A. Basilar artery
78
79 B. Bridging vein
80 C. Middle meningeal artery
• 81

Lock
s
Suspend
0
End Block
Item: 81 of 99 ~ 1 • Mark <:] (:>- Jill ~· ~J
QID: 23777 ..1 Previous Next Lab'V!I!ues Notes Calculator
. • stages of healing scattered over the childs body. The ch1ld IS sent for 1magmg to evaluate the cause for her altered level of
59 consciousness; MRI of the brain is shown in the image.
60
61
62
63
64
65
66
67
68
69
70
71
72 Which vascular structure is most likely damaged in this patient?
73
74 A. Basilar artery

~
B. Bridging vein
C. Middle meningeal artery
77
78 D. Posterior cerebral artery
79 E. Superior sagittal sinus
80
• 81

Lock
s
Suspend
0
End Block
Item: 81 of 99 ~ 1 • Mark <:] (:>- Jill ~· ~J
QID: 23777 ..1 Previous Next Lab'V!I!ues Notes Calculator
.
59 The correct answer i s B. 570/o chose th is.
60 The image and clinica l scenari o (be it falling down the stairs or child abuse) are consistent with a subdura l hematoma {SDH ) .
61 Note the diffuse and even distribution of homogenous signal surrounding compacted brain t issue that retains its gyri on the
62 Tl - weighted MRI image. SDH occurs w hen there is bleed ing from the low- pressur e bridging veins that t raverse the subdural
space to empty blood into the dural venous sinuses. Th is results in blood accumulation between the dura and arachnoid
63
layers . Although this is an MRI scan, subdural hematomas are also visible on CT as a crescentic accumulation along the inner
64 table of the skull. SDHs are usually secondary to external physica l trauma ( eg, a fall, direct blow from a hard object,
65 shaking) . It presents 1- 14 days after the injury with pr ogr essively worsening symptoms of headache, increased intracranial
66
pressure, personality changes, or focal neurolog ic signs as the hematoma slowly enlarges .
Subdural space Subdural hematoma Dural venous sinuses Intracranial pressure Bridging veins Hematoma Magnetic resonance imaging Arachnoid mater Focal neurologic signs
67
Dura mater Headache Gyrus Human skull Major trauma Neurology Brain Child abuse Uterus Sinus (anatomy) Paranasal sinuses Skull
68
69 A i s not correct. 4 % ch ose this.
70 The basilar artery is a very large vessel buried r elatively deeply in the brain. Its posit ion makes it r elatively unlikely to be
damaged in such a t rauma and shou ld damage to it have occurred, the image would have displayed more pervasive bleeding
71
of a different distribution.
72 Basilar artery Brain Human brain

73
C i s not corr ect. 2 10/o chose this.
74
Damage to the middle meningeal artery is characteri stic of an epidural hematoma, another common complication of head

~
trauma . Epidural hemorrhage occurs when the skull is fractured due to blunt physica l trauma to the head and subsequent
6 tearing of blood vessels, usually arterial , near the fractur e. It results in a hematoma between the dura and the skull, leading
77
to rapidly progressive neurologic symptoms as the high- pressure arterial bleed cr eates an en larging hematoma . An epidural
hematoma appears as a convex mass on head images because it cannot progress past the sku ll 's sutures, where the dura
78
mater is attached to the sku ll, and instead expands against the bra in. The image is inconsistent with this diagnosis,
79 secondary to a diffuse versus focal ar ea of blood collection , and the appreciation of dura persisting in its attachment to the
80 skull.
Dura mater Middle meningeal artery Epidural hematoma Basilar skull fracture Major trauma Hematoma Bleeding Head injury Meninges Epidural administration Human skull
81 y

Lock
s
Suspend
0
End Block
Item: 81 of 99 ~ 1 • Mark <:] (:>- Jill ~· ~J
QID: 23777 ..1 Previous Next Lab'V!I!ues Notes Calculator
. -- . - -. - - --
59 Cis not correct. 210/o chose this.
60 Damage to the m iddle men ingea l artery is characteri stic of an epidural hematoma, another common complication of head
61 trauma . Epidural hemorrhage occurs when the sku ll is fractured due to blunt physical trauma to the head and subsequent
tearing of blood vessels, usually arteria l, near the fractur e. It resu lts in a hematoma between the dura and the sku ll, leading
62
to rapidly progressive neurolog ic symptoms as the high-pressure arteria l bleed cr eates an enla rging hematoma . An epidural
63 hematoma appea rs as a convex mass on head images because it cannot prog ress past the skull's sutu res, where the dura
64 mater is attached to the skul l, and instead expands aga inst the brain . The image is inconsistent with this diagnosis,
65 secondary to a diffuse versus focal ar ea of blood collection , and the appreciation of dura persisting in its attachment to the
skull.
66
Dura mater Middle meningeal artery Epidural hematoma Basilar skull fracture Major trauma Hematoma Bleeding Head injury Meninges Epidural administration Human skull
67
Human brain Bone fracture Neurology Surgical suture Blood vessel Brain
68
69
D is not correct. 5% chose this.
The posterior cer ebral artery is a very large vessel buried relatively deeply in the brain . Its posit ion makes it relatively
70
un li kely to be damaged in such a trauma . Shou ld it be damaged, the image would have revea led mor e pervasive bleeding of
71 a different distribution .
72 Posterior cerebral artery Human brain Brain Cerebral arteries

73
E is not correct. 13% chose this.
74 The superior sagittal sinus, though superficial, is not typi cally damaged in head trauma . It is the relatively frail bri dging veins

~
that provide the sinus with blood supply that are ruptu red in SDH .
Superior sagittal sinus Bridging veins Sagittal plane Head injury

77
78 Bottom line:
79
Subdural hematomas occu r when there is bleeding f rom the low-pressure bridging veins that traverse the subdural space to
80 empty blood into the du ral venous sinuses. They are usually secondary to externa l physica l trauma .
81 Subdural space Dural venous sinuses Bridging veins Dura mater Subdural hematoma Sinus (anatomy) Hematoma ~1ajor trauma Uterus Vein

Lock
s
Suspend
0
End Block
Item: 81 of 99 ~ 1 • Mark <:] (:>- Jill ~· ~J
QID: 23777
:
..1
..... . . -· Previous
. . . ..
Next
. ..
Lab'V!I!ues
.
Notes
. . .
Calculator
.. . . . - . . -. . . . . ... .
hematoma appears as a convex mass on head images because it cannot progress past the sku ll 's sutures, where the dura
59
mater is attached to the sku ll, and instead expands against the bra in. The image is inconsistent with this diagnosis,
60 secondary to a diffuse versus focal ar ea of blood collection , and the appreciation of dura persisting in its attachment to the
61 skull.
Dura mater Middle meningeal artery Epidural hematoma Basilar skull fracture Major trauma Hematoma Bleeding Head injury Meninges Epidural administration Human skull
62
Human brain Bone fracture Neurology Surgical suture Blood vessel Brain
63
64 D is not correct. 5% chose this.
65 The posterior cer ebral artery is a very large vessel buried relatively deeply in the brain. Its posit ion makes it relatively
66 unlikely to be damaged in such a trauma . Should it be damaged, the image would have revealed mor e pervasive bleeding of
a different distribution .
67
Posterior cerebral artery Human brain Brain Cerebral arteries
68
69
E is not correct. 1 3% chose this.
70
The superior sagittal sinus, though superficial, is not typically damaged in head trauma . It is the relatively frail bridging veins
that provide the sinus with blood supply that are ruptured in SDH .
71 Superior sagittal sinus Bridging veins Sagittal plane Head injury
72
73
74
Bottom li ne:
Subdural hematomas occu r when there is bleeding from the low-pr essure bridging veins that traverse the subdural space to

~
77
empty blood into the dural venous sinuses. They are usually secondary to external physical trauma .
Subdural space Dural venous sinuses Bridging veins Dura mater Subdural hematoma Sinus (anatomy) Hematoma ~1ajor trauma Uterus Vein

78
79 References:
80 FA Step 2 CK 9th ed p 265
FA Step 2 CK 8th ed p 251
81

Lock
s
Suspend
0
End Block
Item: 82 of 99 ~ 1 • Mark <:] (:>- Jill ~· ~J
QID: 23291 ..1 Previous Next Lab'V!I!ues Notes Calculator

60 A 65-year-old man is brought to the emergency department in a confused state following a seizure in public. Witnesses
report the seizure as occurring sudden ly and lasting at least 2-3 m inutes . A call to his fam ily reveals that he has no major
61
medical problems other than hypertension and high anxiety, and that both are well controlled by his medications. On
62 examination the patient's tongue is unharmed, although his pants are covered in fresh urine and feces. He is oriented to person,
63 but not place or time, and has no recollection of what occurred . Physical examination is otherwise unremarkable .
64
65 Withdrawa l from which drug is most likely responsible for this patient's seizure'
66
67 A. Captopril
68
B. Cocaine
69
C. Diazepam
70
71 D. Lithium
72 E. Verapamil
73
74

~
77
78
79
80
81
• 82

Lock
s
Suspend
0
End Block
Item: 82 of 99 ~ 1 • Mark <:] (:>- Jill ~· ~J
QID: 23291 ..1 Previous Next Lab'V!I!ues Notes Calculator

60
61
62
The correct answer is C. 760/o chose this.
Benzodiazepines are sedatives that bind at specific receptors and act to enhance the inhibitory tone of GABA receptors in the
63
centra l nervous system . Some common indications include induction of anesthesia, alcohol withd rawal, insomnia, epilepsy,
64 and generalized anxiety. Patients who have developed benzodiazepine dependency are at risk for potentia lly fata l
65 complications from withdrawa l, includ ing seizures. Benzodiazepine withdrawal shou ld be suspected in any patient taking a
66 drug from this class who suffers a seizure .
Benzodiazepine Central nervous system Epilepsy Gamma-Aminobutyric acid Alcohol withdrawal syndrome Insomnia GABA receptor Anesthesia Anxiety
67
Benzodiazepine withdrawal syndrome Epileptic seizure Nervous system Alcoholic beverage Alcohol Sedative Receptor (biochemistry)
68
69 A is not correct. 3% chose this.
70 Angiotensin-converting enzyme inhibitors would be a logical choice for treating th is patient's hypertension, although this class
is not known to cause severe withdrawal symptoms or seizures .
71
Angiotensin-converting enzyme Enzyme ACE inhibitor Hypertension Epileptic seizure Enzyme inhibitor
72
73
B is not correct. 90/o chose this.
74
Coca ine is among many common illegal drugs known to cause seizures. However, there is no reason to suspect that this man
uses cocaine. Furthermore, withdrawal from cocaine is far less likely to cause seizures than its use or abuse .

~
Cocaine Illegal drug trade Prohibition of drugs Epileptic seizure
6
D is not correct. 7% chose this.
77
Lithium is known to cause seizures; however, there is no evidence of the patient taking this substance. Furthermore,
78
w ithdrawa l from lithium is far less likely to cause seizures than its use or abuse .
79 lithium Epileptic seizure

80
E is not correct. SOlo chose this.
81
Calcium channel blockers would be a logical choice for treating th is patient's hypertension, although this class is not known to
82 y cause severe withdrawal svmotoms or seizures.

Lock
s
Suspend
0
End Block
Item: 82 of 99 ~ 1 • Mark <:] (:>- Jill ~· ~J
QID: 23291 ..1 Previous Next Lab'V!I!ues Notes Calculator
• • •
60 Ang iotensin-converting enzyme inhibitors would be a logical choice for treating this patient's hypertension, although this class
61 is not known to cause severe withdrawal symptoms or seizures .
Angiotensin-converting enzyme Enzyme ACE inhibitor Hypertension Epileptic seizure Enzyme inhibitor
62
63 B is not correct. 9 0/o chose this .
64 Cocaine is among many common illega l drugs known to cause seizures. However, there is no reason to suspect that this man
uses cocaine. Furthermore, withdrawal from cocaine is far less likely to cause seizures than its use or abuse .
65
Cocaine Illegal drug trade Prohibition of drugs Epileptic seizure
66
67
D is not correct. 7 % chos e this .
Lithium is known to cause seizures; however, there is no evidence of the patient taking this substance. Furthermore,
68
w ithdrawa l from lithium is far less likely to cause seizures than its use or abuse .
69 lithium Epileptic seizure
70
E is not correct. SO/o chose th is.
71
Calcium channel blockers would be a logical choice for treating this patient's hypertension, although this class is not known to
72 cause severe w ithdrawa l symptoms or seizures.
73 Calcium channel blocker Hypertension Calcium channel Epileptic seizure Calcium Drug withdrawal

74

~
77
Bottom line :
Benzodiazepine withdrawal should be suspected in any [Patient taking a drug from th is class who suffers a seizure .
Benzodiazepine Benzodiazepine withdrawal syndrome Epileptic seizure
78
79
80 References:
81 FA Step 2 CK 9th ed pp 269-270
FA Step 2 CK 8th ed pp 256-257
82

Lock
s
Suspend
0
End Block
Item: 83 of 99 ~ 1 • Mark <:] (:>- Jill ~· ~J
QID: 23457 ..1 Previous Next Lab'V!I!ues Notes Calculator

••
61 An 80-year-old woman with a 30-pack-year smoking history presents to her physician with declining vision in her right eye ~~AI
over the past month. She has trouble seeing her needlew ork or read ing because there appears to be a dark circle in the
62
center of her v isual field. On funduscopic examination her right eye shows subretina l fluid and a localized exudative retina l
63 detachment.
64
65 Which of the following is a factor in the disease pathogenesis that contributes to her visual loss?
66
67 A. Abnorma l growth of choroidal vessels
68
B. Decreased scleral elasticity
69
70 C. Ischem ic damage to photoreceptors
71 D. Optic nerve atrophy
72
E. Thr omboembolism to the retina l artery
73
74

~
77
78
79
80
81
82
• 83

Lock
s
Suspend
0
End Block
Item: 83 of 99 ~ 1 • Mark <:] (:>- Jill ~· ~J
QID: 23457 ..1 Previous Next Lab'V!I!ues Notes Calculator

••
61
62 The correct answer i s A. 4 30/o chose this.
63 This patient likely has wet-type age-related macular degeneration (ARMD) . She has a loss of central vision over a period of
weeks to months, as well as subretinal flu id and retinal detachment. Neovascularization is the underlying mechanism of wet
64
ARMD and can be treated with thermal laser photocoagullation, photodynamic therapy, or intravit reous inj ection of a vascu lar
65 endothelial growth factor inhibitor.
66 Retinal detachment Macular degeneration Vascular endothelial growth factor Photodynamic therapy laser coagulation Growth factor Neovascularization Endothelium laser Retinal

67 Blood vessel

68
B is not correct. 1 30/o chose th is.
69
Decreased scleral elasticity is a proposed mechanism for dry ARMD. This patient's presentation with un ilateral visual loss over
70 weeks to months and findings of subretinal flu id are more consistent with wet ARMD, wh ich is due to neovascularization.
71 Macular degeneration Neovascularization Visual impairment

72 C i s not correct. 2 0 0/o chose this.


73 Ischem ic damage to photoreceptors is a proposed mechanism for dry ARMD. A patient with dry ARMD would have areas of
74 retinal atrophy, depigmentation, and drusen on funduscopic examination, and not subretinal fluid as in this patient w ith wet
ARMD.

~6
77
Macular degeneration Ophthalmoscopy Photoreceptor cell Ischemia Drusen Atrophy

D i s not correct. 9 % chose this.


78
This patient has wet age-related macular degeneration secondary to neovascularization . Th is may lead to subretinal
hemorrhages or fluid collections. Optic nerve atrophy is not involved in the acute presentation of this disease but is seen in
79
glaucoma .
80 Glaucoma Macular degeneration Optic nerve Neovascularization Atrophy Bleeding Wet age-related macular degeneration

81
E i s n ot corr ect. 15% chose this .
82
Thromboembolism is unlikely to be the cause of this patient's visual loss. Her presentation of worsening central vision over a
83 y month and funduscooic examination find inas are more consistent w ith wet ARMD. Thromboembolism oresents acutelv.

Lock
s
Suspend
0
End Block
Item: 83 of 99 ~ 1 • Mark <:] (:>- Jill ~· ~J
QID: 23457 ..1 Previous Next Lab'V!I!ues Notes Calculator

•• • Macular degeneratton Neovasculanzat1on V1sual1mpatrment


61
C is not correct. 2 0 0/o chose this.
62
Ischem ic damage to photoreceptors is a proposed mechanism for dry ARMD. A patient with dry ARMD would have areas of
63 retinal atrophy, depigmentation, and drusen on funduscopic examination, and not subretinal fluid as in this patient w ith wet
64 ARMD.
Macular degeneration Ophthalmoscopy Photoreceptor cell Ischemia Drusen Atrophy
65
66 D is not correct. 9 % chose this.
67 This patient has wet age-related macular degeneration secondary to neovascularization. Th is may lead to subretinal
68 hemorrhages or fluid collections. Optic nerve atrophy is not involved in the acute presentation of this disease but is seen in
69
glaucoma .
Glaucoma Macular degeneration Dptic nerve Neovascularization Atrophy Bleeding Wet age-related macular degeneration
70
71
E is not correct. 15% chose this.
72
Thromboembolism is unlikely to be the cause of this patient's visual loss. Her presentation of worsening central vision over a
month and funduscopic examination findings are more consistent w ith wet ARMD. Thromboembolism presents acutely.
73 Macular degeneration Ophthalmoscopy Thrombosis Visual impairment
74

~
77
Bottom line :
Neovascularization is a component in the pathogenesis of wet-type age- related macular degeneration, which may present
w ith loss of central vision over a period of weeks to months, subretinal fluid, and retinal detachment.
78 Retinal detachment Macular degeneration Neovascularization Pathogenesis Retinal
79
80
81 References:
82 FA Step 2 CK 9th ed pp 293-294
FA Step 2 CK 8th ed pp 278-279
83

Lock
s
Suspend
0
End Block
Item: 84 of 99 ~ 1 • Mark <:] (:>- Jill ~· ~J
QID: 22075 ..1 Previous Next Lab'V!I!ues Notes Calculator
.
62 A 63-year-old man with diabetes and hypertension presents to the ophthalmology clinic compla ining of a bli nd spot in his
right eye. He reports that it is painless, occurred suddenly, and is on ly in his right eye . He has a visual field loss in the
63
right superior hemifield . Examination of the retina shows periphera l hemorrhages in one quadrant and disc congestion .
64
65
Which of the following is the most likely diagnosis?
66
67
A. Acute ophthalmic artery occlusion
68
69
B. Branch retinal vein occlusion
70 C. Central retinal vein occlusion
71
D. Diabetic retinopathy
72
E. Hypertensive retinopathy
73
74

~
77

78
79
80
81
82
83
• 84

Lock
s
Suspend
0
End Block
Item: 84 of 99 ~ 1 • Mark <:] (:>- Jill ~· ~J
QID: 22075 ..1 Previous Next Lab'V!I!ues Notes Calculator
.
62 The correct answer is B. 430/o chose this.
63 The differentia l diagnosis of pain less unilateral vision loss can include several possibilities such as central and peripheral vein
64 occlusion, diabetic retinopathy, and hypertensive retinopathy. The patient r eports a history of hypertension and diabetes, both
of wh ich ar e risk factors for branch retinal vein occlusion. Given the location of the occlusion, on ly part of the visual field will
65
be affected as opposed to the entire field if the occlusion were to affect the centra l vein . Additionally, funduscopic
66 exam ination would show superficial hemorrhages in a particula r sector of the retina along a retinal vein in the periphery as
67 well as disc congestion . Such branch vein hemorrhages usual ly do not cross the horizontal raphe (m idline) .
Hypertensive retinopathy Retina Diabetic retinopathy Ophthalmoscopy Hypertension Diabetes mellitus Retinopathy Differential diagnosis Central retinal vein occlusion
68
Visual impairment Visual field Bleeding
69
70 A is not correct. 120/o chose this.
71 In acute ophtha lmic artery occlusion, the entire retina typica lly appea rs wh itened . Such disease occurs in the setting of
72 carotid artery occlusive disease or other ischemic disease such as embolic disease and coagulopathy.
Ophthalmic artery Retina Carotid artery Embolism Ophthalmology Coagulopathy Ischemia
73
74 C is not correct. 190/o chose this.

~
Central retina l vein occlusion can also present with pain less unilateral v ision loss. It occurs in patients who are elderly and is
often idiopathic. The exam ination typically shows diffuse retina l hemorrhages in all four quad rants with dilated, tortuous
retinal veins. There may also be cotton- wool spots, disc edema, and neovascula rization of the disc or r etina .
77 Central retinal vein occlusion Retina Central retinal vein Edema Neovascularization Idiopathy Visual impairment Bleeding Vascular occlusion Retinal
78
D is not correct. 130/o chose this.
79
Diabetic ret inopathy usually presents w ith bilateral visua I defects that can lead to blindness because the retina is very
80
sensitive to hyperglycemic effects .It can lead to loss of both ret inal pericytes and m icrovascu lar endothel ial cells as well as
81 thickening of retinal basement membrane . Funduscopic examination typica lly shows dot-and-blot hemorrhages and
82 microaneurysms that extend across the horizontal raphe .
Diabetic retinopathy Retina Basement membrane Retinopathy Pericyte Ophthalmoscopy Endothelium Diabetes mellitus Hyperglycemia Bleeding Visual impairment
83
84 E is not correct. 13% chose this.

Lock
s
Suspend
0
End Block
Item: 84 of 99 ~ 1 • Mark <:] (:>- Jill ~· ~J
QID: 22075 ..1 Previous Next Lab'V!I!ues Notes Calculator
• I I - .. • I I '"

62 Central retinal vein occlusion can also present with painless unilateral v ision loss. It occurs in patients who are elderly and is
63
often idiopathic. The examination typically shows diffuse retinal hemorrhages in all four quadrants with dilated, tortuous
retinal veins. There may also be cotton- wool spots, disc edema, and neovascu larization of the disc or retina .
64 Central retinal vein occlusion Retina Central retinal vein Edema Neovascularization Idiopathy Visual impairment Bleeding Vascular occlusion Retinal
65
D is not correct. 130/o chose this .
66
Diabetic retinopathy usually presents w ith bilateral visua I defects that can lead to blindness because the retina is very
67
sensitive to hyperglycemic effects .It can lead to loss of both retinal pericytes and microvascular endothelial cells as well as
68 thickening of retinal basement membrane . Funduscopic exam ination typically shows dot-and-blot hemorrhages and
69 microaneurysms that extend across the horizontal raphe.
Diabetic retinopathy Retina Basement membrane Retinopathy Pericyte Ophthalmoscopy Endothelium Diabetes mellitus Hyperglycemia Bleeding Visual impairment
70
71 E is n ot correct. 13% chos e this .
72 Hypertensive retinopathy usua lly presents with bilateral v isual defects. Hemorrhages are not confined to a sector of the retina
73
and usually cross the horizontal raphe. There are two contributing pathologies to this disease, including arteriolar thickening
and acute vascu lar inj ury.
74 Hypertensive retinopathy Retina Retinopathy Diabetic retinopathy Bleeding Arteriole Hypertension Blood vessel

~
77
Bottom line :
78 Hypertension and diabetes are risk factors for branch retinal vein occlusion . Funduscopic exam would show superficial
hemorrhages in a sector of the retina along a retinal ve'in branch .
79 Retina Diabetes mellitus Hypertension Central retinal vein occlusion Ophthalmoscopy Bleeding Retinal Vascular occlusion
80
81
82 References:
83 FA Step 2 CK 9th ed p 294
FA Step 2 CK 8th ed p 279
84

Lock
s
Suspend
0
End Block
Item: 85 of 99 ~ 1 • Mark <:] (:>- Jill ~· ~J
QID: 23779 ..1 Previous Next Lab'V!I!ues Notes Calculator
.
63 A 22-year-old woman presents to the emergency department w ith a 2-day history of headache, photophobia,
phonophobia, nausea, vomiting, and malaise. The patient complains of a sudden-onset, constant, throbbing right-sided
64
headache as well as lethargy and fluid retention . She says she has had similar headaches in the past, but none as severe
65 as th is. She denies any fever or recent trauma . She is not using birth control and is on the third day of her period . She is
66 accompanied by a friend who reports dining with the patient and sharing a bottle of red wine prior to the onset of her headache.
67
The patient reports that she is otherwise healthy and exer cises regularly. She has a noncontributory family history and denies
the use of drugs. On examination her temperature is 36 .8°C (98 .2°F), heart rate is 70/min, blood pressure is 120/78 mm Hg,
68
respiratory rate is 12/min. She is alert and oriented to time, person, and place; has no neck stiffness; cardiovascular,
69 respiratory, and neurologic examination are noncontributory.
70
71 Which of the following symptoms or signs would support the most li kely diagnosis?
72
73 A. Aphasia
74
B. Ipsilateral tearing of the eye

~
77
C. Jaw claudication
D. Meningismus
78
E. Pericranial band-like pain
79
80
F. Scintillating scotomas
81
82
83
84
• 85

Lock
s
Suspend
0
End Block
Item: 85 of 99 ~ 1 • Mark <:] (:>- Jill ~· ~J
QID: 23779 ..1 Previous Next Lab'V!I!ues Notes Calculator
. . - -- .
63
64
65
The correct answer is F. 470/o chose this.
66
Scintillating scotomas (bright light or flashing lights) are a common form of the v isual auras that often precede classic
67
migraine headaches . Twelve percent of ind ividua ls in the United States are affected by migraine; women are affected more
68 often than men, and 60% of patients have a family history. Migraine without aura is more common than migraine with aura .
69 Both types exhibit symptoms including lethargy, craving of food, depression, and fluid retention . The clinical features of the
70
condition include a throbbing, unilateral head pain along with photophobia, phonophobia, anorexia, nausea, vomiting, and
general malaise. Triggers of migraine include red w ine, chocolate, monosodium glutamate, lack of sleep, menses, and stress.
71 The patient's signs and symptoms in conjunction w ith risk factors of gender, red wine, and menstruation make migraine
72 headache the most likely diagnosis. Management of migraine includes avoiding triggers, analgesia with NSAIDs or triptans,
73 and antiemetics for acute attacks . If more than 2 episodes per month, prophylactic treatment is indicated with ~-blockers or
antidepressants .
74
Monosodium glutamate Migraine Photophobia Phonophobia Nausea Anorexia (symptom) Glutamic acid Analgesic Triptan Headache Vomiting Aura (symptom) ~1alaise Antiemetic

~6
77
Anorexia nervosa Nonsteroidal anti-inflammatory drug Menstruation Antidepressant Scotoma

United States
~1ajor depressive disorder Edema Depression (mood) lethargy Preventive healthcare

78 A is not correct. 7% chose this.


79
Aphasia might support a diagnosis of stroke. Stroke is extremely unlikely in this patient because of her age and lack of risk
factors for thrombosis, includ ing birth control .
80 Aphasia Thrombosis Stroke Birth control
81
B is not correct. 130/o chose this.
82
Ipsilateral eye tearing is associated with cluster headache . Cluster headache is a headache syndrome that is characterized by
83
headache episodes lasting 15 minutes to 3 hours with ipsi lateral periorbital or temporal pain, eye tearing, and rhinorrhea.
84 These episodes often occur in clusters lasting 6-12 weeks and are more common in males . The patient reports no history of
85 y
this phenomenon and her pain does not wax and wane, making this diagnosis unlikely. y

Lock
s
Suspend
0
End Block
Item: 85 of 99 ~ 1 • Mark <:] (:>- Jill ~· ~J
QID: 23779 ..1 Previous Next Lab'V!I!ues Notes Calculator
. Aphasia Thrombosis Stroke Birth control
63
64
B is not correct. 1 30/o chose th is.
65
Ipsilateral eye tearing is associated with cluster headache . Cluster headache is a headache syndrome that is characterized by
headache episodes lasting 15 minutes to 3 hours with ipsilateral periorbital or tempora l pain, eye tearing, and rhinorrhea.
66 These episodes often occur in clusters lasting 6·12 weeks and are more common in males . The patient reports no history of
67 this phenomenon and her pa in does not wax and wane, making this diagnosis unlikely.
Cluster headache Rhinorrhea Headache Ipsilateral Anatomical terms of location
68
69 C is not correct. 6 0/o chos e this .
70 The headache of temporal (giant cell) arteritis tends to present in indiv iduals over the age of 60, with signs and symptoms of
71 a un ilateral headache in the distribution of the temporal artery, ipsilateral visua l loss (secondary to vasculitis of the
ophthalmic artery), and jaw claud ication . Fifty percent of patients w ith temporal arteritis have polymya lgia rheumatica . This
72
patient's age and lack of v ision loss, fever, or myalgias/arthralgias argue against this diagnosis.
73 Polymyalgia rheumatica Ophthalmic artery Giant-cell arteritis Superficial temporal artery Claudication Headache Vasculitis Giant cell Visual impairment Arteritis Fever Temporal artery
74 Jaw

~
77
D is not correct. 100/o chose this .
Meningismus wou ld support the diagnosis of subarachnoi d hemorrhage or meningitis. Given her long duration of symptoms,
lack of fever, and no history of trauma, these are less li kely than migraine .
78 Subarachnoid hemorrhage Migraine Meningitis Meningism Bleeding Fever

79
E is n ot correct. 17% chose this .
80
Band·like pain is typical of tension-type headaches. Tension-type headaches are accompanied by muscular contractions of the
81 head and neck, neck stiffness, a lack of a prodrome, and are generally bilateral. All of these findings are inconsistent w ith the
82 patient's presentation .
Meningism Prodrome
83
84
85 Botto m li ne: I.
Lock
s
Suspend
0
End Block
Item: 85 of 99 ~ 1 • Mark <:] (:>- Jill ~· ~J
QID: 23779
. ..1
.. .. . .. . . .
Previous Next Lab'V!I!ues
. Notes
. .. Calculator
. . . . .. . . .. . .. .' . .. . .. .
a un ilateral headache in the distribution of the temporal artery, ipsilateral visua l loss (secondary to vasculitis of the
63
ophthalmic artery), and jaw claudication . Fifty percent of patients w ith temporal arteritis have polymya lgia rheumatica . This
64 patient's age and lack of vision loss, fever, or myalgias/arthralgias argue against this diagnosis.
65 Polymyalgia rheumatica Ophthalmic artery Giant-cell arteritis Superficial temporal artery Claudication Headache Vasculitis Giant cell Visual impairment Arteritis Fever Temporal artery

66 Jaw

67
D is not correct. 100/o chose this.
68
Meningismus wou ld support the diagnosis of subarachnoi d hemorrhage or meningitis. Given her long duration of symptoms,
69 lack of fever, and no history of trauma, these are less li kely than migraine .
70 Subarachnoid hemorrhage Migraine Meningitis Meningism Bleeding Fever

71 E is not correct. 17% chose this.


72 Band-like pain is typical of tension-type headaches. Tension-type headaches are accompanied by muscular contractions of the
73 head and neck, neck stiffness, a lack of a prodrome, and are generally bilateral. All of these findings are inconsistent w ith the
74
patient's presentation ,
Meningism Prodrome

~
77 Bottom Line :
78 Clinical features of migraines include a throbbing, unilateral head pain along with photophobia, phonophobia, anorexia,
79 nausea, vomiting, and general malaise . Triggers may include red wine, chocolate, monosodium glutamate, lack of sleep,
menses, and stress.
80 Monosodium glutamate Photophobia Phonophobia Migraine Glutamic acid Nausea Anorexia (symptom) Malaise Vomiting Anorexia nervosa Menstruation
81
82
83 References:
84 FA Step 2 CK 9th ed p 267
FA Step 2 CK 8th ed p 253
85

Lock
s
Suspend
0
End Block
Item: 86 of 99 ~ 1 • Mark <:] (:>- Jill ~· ~J
QID: 2207 2 ..1 Previous Next Lab'V!I!ues Notes Calculator
.
64 A 63-year-old African-American woman with osteoarthrit is is seen in the ophthalmology clinic for routine exam ination. Her
last ocular examination was 5 years ago. She reports no problems with her vision since her last examination . Her visual
65
acuity is 20/ 40 in both eyes; intraocular pressure is 29 m m Hg in the right eye and 31 mm Hg in the left eye. Gonioscopic
66 evaluation shows open anterior-chamber ang le and no peripheral anterior synechiae. Results of a funduscopic examination are
67 shown in the image.
68
69
70
71
72
73
74

~
77
78
79
80
81
82
83
84
85
• 86

Lock
s
Suspend
0
End Block
64
65
66
67
68
69
70
71
72
73
74

~
77 Which of the following would be the most appropriate next step in management of the patient?
78
79 A. Atropine
80 B . Latanoprost
81
C. Observation with follow-up in 6 weeks
82
83 D . Timolol
84 E. Trabeculectomy
85
• 86 •

a
Lock
s
Suspend
8
End Diode
Item: 86 of 99 ~ 1 • Mark <:] (:>- Jill ~· ~J
QID: 2207 2 ..1 Previous Next Lab'V!I!ues Notes Calculator
.
64 The correct answer i s B. 3 7 0/o chose th is.
65 The patient has primary open-angle glaucoma (OAG), the most common type of glaucoma. The eyes undergo a progressive
66 loss of peripheral vision and then central v ision . On funduscopic examination, the optic disc's hollowed-out appearance is
67 referred to as "cupping ." There are no other symptoms, wh ich high lights the importance of measuring intraocular pressure to
screen for this disease . Topical prostag landins are the first line treatment for OAG. They decrease intraocular pressure (lOP)
68
by increasing uveoscleral outflow of aqueous humor. They have been shown in clin ical tria ls to be superior to beta- blockers
69 such as timolol in decreasing lOP. Add it ionally, latanoprost has been shown to have less side effects such as hyperemia than
70 other prostaglandins.
Timolol Glaucoma Aqueous humour latanoprost Ophthalmoscopy Hyperaemia Intraocular pressure Prostaglandin Beta blocker Optic disc Peripheral vision
71
72 A i s not correct. SOfo ch ose this .
73 Atropine is an anti muscarinic agent that results in mydriasis and is not an acceptable form of treatment for glaucoma.
Muscarinic antagonist Mydriasis Atropine Glaucoma
74

~
Cis not correct. 11% chose this.
6 This patient has primary open-angle glaucoma with visib le cupping of the optic nerve, retinal nerve fiber layer defects, and
77 increased intraocu lar pressu re. It is necessary to provide· the patient with a form of treatment because the pathology will
li kely progress without appropriate treatment. In addition, the vision loss is irreversible.
78 Glaucoma Optic nerve Intraocular pressure Nerve fiber Nerve fiber layer Visual impairment Retinal
79
D i s not correct. 3 7 0/o chose t his .
80
Ti molol is a nonselective ~-antagonist that reduces the p roduction of aqueous humor in the eye . It can be used topically to
81
treat open-angle glaucoma (OAG) . Historically considered first-li ne for OAG, recent studies have shown there are better
82 options. It is important to check for a history of card iac and pulmonary disease prior to prescribing .
83 Timolol Glaucoma Aqueous humour Binding selectivity

84 E i s n ot correct. 10% ch ose this.


85 Although trabecu lectomy can provide curative treatment of closed-angle glaucoma, it is not first-li ne treatment for open-
86 y
angle glaucoma . A patient with closed-ang le glaucoma usually presents w ith headache, malaise, and general distress, with y

Lock
s
Suspend
0
End Block
Item: 86 of 99 ~ 1 • Mark <:] (:>- Jill ~· ~J
QID: 2207 2 ..1 Previous Next Lab'V!I!ues Notes Calculator
.
64 C is not correct. 110/o chose this .
65
This patient has primary open·angle glaucoma with visib le cupping of the optic nerve, retinal nerve fiber layer defects, and
increased intraocu lar pressure. It is necessary to provide· the patient with a form of treatment because the pathology will
66 li kely progress without appropriate treatment. In addition, the vision loss is irreversible.
67 Glaucoma Optic nerve Intraocular pressure Nerve fiber Nerve fiber layer Visual impairment Retinal

68
D is not correct. 3 7 0/o chose t his .
69
Ti molol is a nonselective ~-antagonist that reduces the production of aqueous humor in the eye . It can be used topically to
70 treat open-angle glaucoma (OAG) . Historically considered first-li ne for OAG, recent studies have shown there are better
71 options. It is important to check for a history of cardiac and pulmonary disease prior to prescribing .
Timolol Glaucoma Aqueous humour Binding selectivity
72
73 E is n ot correct. 10% ch ose this .
74 Although trabecu lectomy can provide curative treatment of closed-angle glaucoma, it is not first-li ne treatment for open-
angle glaucoma . A patient with closed-angle glaucoma usually presents with headache, malaise, and general distress, with

~
77
loss of visual acuity as intraocular pressure increases .
Trabeculectomy Intraocular pressure Glaucoma Visual acuity Malaise Headache

78
79
Botto m li ne:
80 Latanoprost is a prostaglandin analog that increases outflow of aqueous humor in the eye and is used first-line to treat
open-angle glaucoma.
81 Glaucoma Prostaglandin Prostaglandin analogue Latanoprost Aqueous humour
82
83
84 References:
85 FA Step 2 CK 9th ed p 293
FA Step 2 CK 8th ed p 278
86

Lock
s
Suspend
0
End Block
Item: 87 of 99 ~ 1 • Mark <:] (:>- Jill ~· ~J
QID: 22066 ..1 Previous Next Lab'V!I!ues Notes Calculator
..
65 A 68-year-old woman with a central retinal vein occlusion presents to the ophthalmology cli nic for an annual examination . ~~AI
She had a cerebrovascular accident 2 years ago . She notes that for the past 3 months, she has been experiencing
66
periorbital pain around her right eye, associated with an ipsilateral headache. She also complains of intermittent blurry
67 v ision, and "seeing halos around objects ."
68
69 Which of the following is the most appropriate curative treatment for this patient?
70
71 A. Acetazolamide
72
B. Atropine
73
74 C. Laser iridotomy

~
D. Pilocarpine

E. Ti molol
77
78
79
80
81
82
83
84
85
86
• 87

Lock
s
Suspend
0
End Block
Item: 87 of 99 ~ 1 • Mark <:] (:>- Jill ~· ~J
QID: 22066 ..1 Previous Next Lab'V!I!ues Notes Calculator
..
65
66 The correct answer is C. 560/o chose this.
67 This patient has symptoms of subacute closed-angle glaucoma . Wh ile medical treatment can provide symptomatic relief, on ly
laser iridotomy provides curative treatment. Laser iridotomy forms a permanent connection between the anterior and
68
posterior chambers and prevents recurrence. In some cases, the fellow eye should undergo prophylactic laser iridotomy.
69 Glaucoma Iridectomy Posterior chamber of eyeball laser
70
A is not correct. 130/o chose this.
71
Acetazolamide is carbonic anhydrase in hibitor that can decrease aqueous humor secretion and reduce intraocular pressure .
72 However, it on ly provides symptomatic treatment and is not curative .
73 Acetazolamide Carbonic anhydrase inhibitor Carbonic anhydrase AQueous humour Intraocular pressure Enzyme inhibitor

74
B is not correct. 50/o chose this.

~
Atropine is an antimuscarinic agent that results in mydriasis, w hich will further exacerbate her symptoms of closed-a ngle
6 glaucoma .
77 Muscarinic antagonist Mydriasis Atropine Glaucoma

78 D is not correct. 120/o chose this.


79 Pilocarpine is a muscarinic agonist that produces rapid m iosis and contraction of t he ciliary muscles. This ca n be used in the
80 acute t reatment of closed- or open-angle glaucoma to open t he trabecular meshwork around Schlemm's canal, increasing
81
drainage of aqueous humor and decreasing intraocular pr essure. However, it only provides symptomatic treatment and is not
curative .
82 Pilocarpine Miosis Trabecular meshwork Glaucoma Muscarinic agonist Schlemm's canal AQueous humour Intraocular pressure Muscarinic acetylcholine receptor Agonist Ciliary muscle
83
E is not correct. 14% chose this.
84
Timolol is a nonselective ~-antagon ist that reduces the p roduction of aqueous humor in the eye . It can be used topically in
85
treatment of chronic open-angle glaucoma, but does not provide curative treatment for chronic closed-a ngle glaucoma .
86 Timolol Glaucoma AQueous humour Binding selectivity

87 y

Lock
s
Suspend
0
End Block
Item: 87 of 99 ~ 1 • Mark <:] (:>- Jill ~· ~J
QID: 22066 ..1 Previous Next Lab'V!I!ues Notes Calculator
.. • However, 1t only prov1des symptomatiC t reatment and IS not curat1ve .
65 Acetazolamide Carbonic anhydrase inhibitor Carbonic anhydrase Aqueous humour Intraocular pressure Enzyme inhibitor

66
B is not correct. 5 0/o chose this.
67 Atropine is an antimuscarinic agent that results in mydriasis, w hich w ill further exacerbate her symptoms of closed-angle
68 glaucoma .
~1uscarinic antagonist Mydriasis Atropine Glaucoma
69
70 D is not correct. 1 20/o chose this.
71 Pilocarpine is a muscarinic agonist that produces rapid m iosis and contraction of the ciliary muscles. Th is can be used in the
72 acute treatment of closed- or open-angle glaucoma to open the trabecular meshwork around Sch lemm's canal, increasing
drainage of aqueous humor and decreasing intraocular pr essure. However, it only provides symptomatic treatment and is not
73
curative.
74 Pilocarpine Miosis Trabecular meshwork Glaucoma Muscarinic agonist Schlemm's canal Aqueous humour Intraocular pressure Muscarinic acetylcholine receptor Agonist Ciliary muscle

~
77
E is not correct. 14% chose this.
Ti molol is a nonselective ~-antagonist that reduces the production of aqueous humor in the eye . It can be used topically in
treatment of chronic open-angle glaucoma, but does not provide curative t reatment for chronic closed-angle glaucoma.
78 Timolol Glaucoma Aqueous humour Binding selectivity

79
80
Bottom line :
81
Laser iridotomy provides curative t r eatment for subacute closed-angle glaucoma.
82 Glaucoma Iridectomy Laser
83
84
85 References:
86 FA Step 2 CK 9th ed p 293
FA Step 2 CK 8th ed pp 277-278
87

Lock
s
Suspend
0
End Block
Item: 88 of 99 ~ 1 • Mark <:] (:>- Jill ~· ~J
QID: 22189 ..1 Previous Next Lab'V!I!ues Notes Calculator
.
66 A 77-year-old white man is brought to the clinic by his daughter, who says that her father is having trouble remembering
important family occasions and difficulty managing his finances. These behaviors have progressively worsened over the
67
past three years, but her father denies that anything is wrong . He has no history of depression, anxiety, or head trauma . A
68 neurologic examination is performed, and wh ile tests for motor, sensory, reflex, and gait function are normal, he shows
69 impairments on mental status exam . His immediate recall is intact, yet he recalls zero of three words after a five- minute delay.
70
His medical history is sign ificant for type 2 diabetes mellitus diagnosed 20 years ago and atria l fibrillation . An uncle was
diagnosed with Parkinson disease at the age of 60 years.
71
72
Which of the following is the most likely diagnosis?
73
74
A. Delirium

~
77
B. Neurocognitive disorder due to Alzheimer disease

C. Neurocognitive disorder due to Lewy body disease


78
D. Neurocognitive disorder due to vascu lar disease
79
80 E. Progressive supranuclear palsy
81
82
83
84
85
86
87
• 88

Lock
s
Suspend
0
End Block
Item: 88 of 99 ~ 1 • Mark <:] (:>- Jill ~· ~J
QID: 22189 ..1 Previous Next Lab'V!I!ues Notes Calculator
. E. Progressive supranuclear palsy
66
67
68
69 The correct answer i s B. 770/o chose this .
70 Neurocognitive disorder due to Alzheimer disease is manifested by a progressively worsen ing dementia characterized by
71 declines in memory, learning, and at least one other cognitive domain ( eg, complex attention, executive function, language,
72 perceptual-motor, or social cognition). Per the Diagnostic and Statistical Manual of Mental Disorders, Fifth Edition (DSM-5)
diagnostic criteria, there must be no evidence for another etiology. This diagnosis is also categorized as mild or major based
73
upon the extent to which the cognitive deficits interfere w ith one's capacity for independence in everyday activit ies . Although
74 patients w ith Alzheimer disease can have imag ing findings suggestive of increased cerebral atrophy, it is not uncommon for

~
CT stud ies to be normal and age appropriate.
DSM-5 Diagnostic and Statistical Manual of Mental Disorders Alzheimer's disease Dementia Executive functions Cerebral atrophy Cognition Neurocognitive Social cognition Etiology
6
Atrophy Medical diagnosis
77
78 A i s not correct. 2% chose this.
79 Delirium is a disturbance in attention (ie, reduced ability to direct, focus, sustain, and shift attention) and awareness
80 (reduced orientation to the environment) that develops over hours to days and may fluctuate in severity during the course of
a day. It is accompan ied by an addit ional disturbance in cognition ( eg, memory deficit, disorientation, language, visuospatial
81
ability, or perception) . This patient's symptoms began and evolved over the course of years, making delirium an incorrect
82 answer choice.
83 Delirium Amnesia Cognition

84 C i s not correct. 4 % chose this.


85 Lewy body disease is characterized by insidious onset and gradual symptom progression. Per the Diagnostic and Statistical
86 Manual of Mental Disorders, Fifth Edition (DSM- 5) criteria, diagnosis is contingent upon having a combination of core and
87 suggestive diagnostic features for either probable or possible neurocognitive disorder with Lewy bod ies . Core diagnostic
features include : 1) fluctuating cognit ion with pronounced va riations in attention and alertness; 2) recurrent v isual
88 y

Lock
s
Suspend
0
End Block
Item: 88 of 99 ~ 1 • Mark <:] (:>- Jill ~· ~J
QID: 22189 ..1 Previous Next Lab'V!I!ues Notes Calculator
. g

66
C is not correct. 4 % chose this.
67
Lewy body disease is characterized by insidious onset and gradual symptom progression. Per the Diagnostic and Statistical
68 Manual of Mental Disorders, Fifth Edition (DSM- 5) criteria, diagnosis is contingent upon having a combination of core and
69 suggestive diagnostic features for either probable or possible neurocognitive disorder with Lewy bod ies . Core diagnostic
70
features include : 1) fluctuating cognit ion with pronounced va riations in attention and alertness; 2) recurrent v isual
hallucinations that are well formed and detailed; 3) spontaneous features of parkinson ism, with onset subsequent to the
71 development of cognitive decline. Suggestive diagnostic features include : 1) meeting criteria for rapid eye movement sleep
72 behavior disorder; 2) severe neuroleptic sensitivity. This patient's presentation is not consistent with a diagnosis of
73 neurocognit ive disorder due to Lewy body disease.
Rapid eye movement sleep behavior disorder Diagnostic and Statistical Manual of Mental Disorders DSM-5 Lewy body Rapid eye movement sleep Parkinsonism
74
Dementia with Lewy bodies Symptom Antipsychotic Cognition Hallucination Dementia

~6
77
D is not correct. 160/o chose this.
Neurocognitive disorder due t o vascu lar disease is differentiat ed from ot her types of dement ia by t he observed onset of
78
cogn it ive symptoms immediately following a stroke, with infa rcts seen on imaging and find ings on physical examination
consistent w ith a prior stroke. The onset of cognitive dysfunction is very abrupt rather than progressive, and is then followed
79 by a continued deterioration . Although atrial fibrillation increases this man's risk for a stroke, there is no history of a stroke
80 and no foca l neurologic deficits .
Atrial fibrillation Dementia Blood vessel Vascular disease Stroke Neurology Physical examination Infarction Cognition
81
82 E is not correct. 1 0/o chose t his.
83 Progressi ve supranuclear palsy (PSP) is a dementia syndrome characterized by vertica l supranuclear palsy with downward
84 gaze disturbance. Other characteristics include postural instability and personality changes such as apathy, anxiety, and
disinhibition . The diagnosis of PSP is currently based upon the clinica l features, as no laboratory or imaging studies are
85
diagnostic, and there are no consensus guidelines for the cl inical diagnosis of the PSP va riants . Neuropathologic examination
86 is the gold standard for its definitive diagnosis . The patholog ic diagnosis of PSP is based upon the identification of
87 neurofibrillary tangles in a distribution considered typical for PSP (the most consistent sites of pathology are in the basal
88 y
ganglia) .

Lock
s
Suspend
0
End Block
Item: 88 of 99 ~ 1 • Mark <:] (:>- Jill ~· ~J
QID: 22189 ..1 Previous Next Lab'V!I!ues Notes Calculator
• ~-1 • II J:- I • •- el- I :f..,. I • I -. - I ... - • -e •. I - :'f:•- • e- . - ... •" - •• - 1':-1 I - •

66 cogn it ive symptoms immediately following a stroke, with infa rcts seen on imaging and findings on physical examination
consistent w ith a prior stroke. The onset of cognitive dysfunction is very abrupt rather than progressive, and is then followed
67
by a continued deterioration . Although atrial fibrillation increases this man's risk for a stroke, there is no history of a stroke
68 and no focal neurologic deficits .
69 Atrial fibrillation Dementia Blood vessel Vascular disease Stroke Neurology Physical examination Infarction Cognition

70 E i s not cor r ect. 1 0/o chose th is.


71 Progressi ve supranuclear palsy (PSP) is a dementia syndrome characterized by vertica l supranuclear palsy with downward
72 gaze disturbance. Other characteristics include postural instability and personality changes such as apathy, anxiety, and
73 disinhibition . The diagnosis of PSP is currently based upon the clinica l features, as no laboratory or imaging studies are
diagnostic, and there are no consensus guidelines for the cl inical diagnosis of the PSP va riants . Neuropathologic exam ination
74
is the gold standard for its definitive diagnosis . The pathologic diagnosis of PSP is based upon the identification of

~
neurofibrillary tangles in a distribution considered typical for PSP (the most consistent sites of pathology are in the basal
ganglia) .
Progressive supranuclear palsy Basal ganglia Dementia Neurofibrillary tangle Anxiety
77
78
79 Bottom line :
80 Neurocognitive disorder due to Alzheimer disease is characterized by declines in memory, learning, and at least one other
81 cogn it ive doma in (eg, complex attention, executive function, language, perceptual-motor, or social cognition) and functional
82 impairment. Patients can have imaging find ings suggestive of increased cerebral atrophy, but it is not uncommon for CT
studies to be normal and age appropriate.
83 Alzheimer's disease Executive functions Cerebral atrophy Cognition Neurocognitive Social cognition Atrophy
84
85
86 References:
87 FA Step 2 CK 9th ed pp 277-278
FA Step 2 CK 8th ed pp 265-266
88

Lock
s
Suspend
0
End Block
Item: 89 of 99 ~ 1 • Mark <:] (:>- Jill ~· ~J
QID: 23705 ..1 Previous Next Lab'V!I!ues Notes Calculator
..
67 A 46-year-old woman compla ins of recurrent episodes of "feeli ng like I'm in a boat" for hours at a t ime for the past few ~~AI
days . This rocking feeling makes her unsteady, and she often vomits . She complains that her right ear "feels fu ll " and has
68
lately had difficu lty hearing from that ear. She also has noticed some low- pitched "ringing" in her ear. Results of MRI of the
69 head are normal. Hemog lobin is 11 .8 g/dL and WBC count is SOOO/mm 3 .
70
71 Which of the following is the pathogenesis of th is disease?
72
73 A. Demyelinating disease
74
B. Dislodged otolith

~
77
C. Excess endolymphatic fluid
D. Severe anemia
78
E. Tumor surrounding the eighth cran ial nerve
79
80
81
82
83
84
85
86
87
88
• 89

Lock
s
Suspend
0
End Block
Item: 89 of 99 ~ 1 • Mark <:] (:>- Jill ~· ~J
QID: 23705 ..1 Previous Next Lab'V!I!ues Notes Calculator
..
67
68 The correct answer i s C. 770/o chose this.
69 This recurrent nature and duration of the patient's symptoms (vertigo, hearing loss, aura of "fullness/ and t innitus) are
suggestive of Menier e's disease. These symptoms are cau sed by excess endolymphatic fluid, with a low-sodium diet and
70
diur etic therapy being the first-line treatment.
71 Tinnitus Diuretic Endolymph Hearing loss
72
A i s not correct. 3% chose this.
73
Mult iple sclerosis (MS ) is a demyelinating disease that can pr esent w ith symptoms similar to Meniere's disease . However,
74 nystagmus is usually the predominant symptom seen during attacks. White matter lesions are often seen on MRI in patients

~
w ith MS, making th is choice less likely.
~1ultiple sclerosis Nystagmus Demyelinating disease White matter Magnetic resonance imaging Symptom Myelin
6
77 B is not correct. 150/o chose th is.
78 Benign paroxysmal posit ional vertigo is a common cause of recurr ent peripheral vertigo r esulting from a dislodged otolith in
79 the semicircular canals. Patients usually present with transient, episodic vert igo (lasting < 1 m inute) and torsiona l nystagmus
triggered by changes in head posit ion (tradit ionally when getting out of bed ). The duration of the sy mptoms as well as the
80
presence of hearing loss and t innitus make this answer less li kely.
81 Otolith Tinnitus Nystagmus Benign paroxysmal positional vertigo Vertigo Semicircular canals Hearing loss Paroxysmal attack Benignity Benign tumor
82
Dis not correct. 1% chose this.
83
Severe anemia rarely can cause vertigo and t innitus. While this patient's hemoglobin is low, it is not severe enough to cause
84 such symptoms. Furthermore, a patient with severe anem ia would likely complain of fatigue and weakness .
85 Tinnitus Vertigo Hemoglobin Anemia Fatigue (medical)

86
E i s n ot cor rect. 4 0/o c hose th is.
87
A normal MRI makes the possibility of an acoustic neuroma lower on the list of differential diagnoses . Furthermore, hearing
88 loss due to a tumor is usually not this acute.
89 Vestibular schwannoma Magnetic resonance imaging Hearing loss Neoplasm Neuroma
y

Lock
s
Suspend
0
End Block
Item: 89 of 99 ~ 1 • Mark <:] (:>- Jill ~· ~J
QID: 23705 ..1 Previous Next Lab'V!I!ues Notes Calculator
.. • Multiple scleros1s Nystagmus Demyelinating d1sease Whote matter Magnetoc resonance omagong Symptom Myelin
67
B is not correct. 150/o chose th is.
68
Benign paroxysmal positional vertigo is a common cause of recurrent peripheral vertigo resu lting from a dislodged otolith in
69 the semicircular canals. Patients usually present with transient, episodic vertigo (lasting < 1 m inute) and torsiona l nystagmus
70 triggered by changes in head position (traditionally when getting out of bed). The duration of the symptoms as well as the
71 presence of hearing loss and tinnitus make this answer less li kely.
Otolith Tinnitus Nystagmus Benign paroxysmal positional vertigo Vertigo Semicircular canals Hearing loss Paroxysmal attack Benignity Benign tumor
72
73 D is not correct. 1 Ofo chose this.
74 Severe anemia ra rely can cause vertigo and tinnitus. While this patient's hemoglobin is low, it is not severe enough to cause
such symptoms. Furthermore, a patient with severe anemia would likely complain of fatigue and weakness .

~
77
Tinnitus Vertigo Hemoglobin Anemia Fatigue (medical)

E is not correct. 4 0/o chose th is.


78
A normal MRI makes the possibility of an acoustic neuroma lower on the list of differential diagnoses . Furthermore, hearing
loss due to a tumor is usually not this acute.
79 Vestibular schwannoma Magnetic resonance imaging Hearing loss Neoplasm Neuroma
80
81
82
Bottom li ne:
83 Vertigo, hearing loss, aura of "fu llness," and tinnitus are symptoms suggestive of Meniere's disease. These symptoms are
often recurrent, lasting hours to days at a time, and are caused by excess endolymphatic fluid .
84 Tinnitus Endolymph Vertigo (film) Hearing loss
85
86
87 References:
88 FA Step 2 CK 9th ed pp 272-273
FA Step 2 CK 8th ed pp 259-260
89

Lock
s
Suspend
0
End Block
Item: 90 of 99 ~ 1 • Mark <:] (:>- Jill ~· ~J
QID: 22411 ..1 Previous Next Lab'V!I!ues Notes Calculator
.
68 During routine prenatal ultrasound testing, a 24-week-old fetus is found to have a cardiac mass, raising suspicion of a
congenital syndrome. Following birth, MRI of the body reveals the ca rdiac mass to be a rhabdomyoma, along w ith
69
hamartomas in the brain and eye . When 2 years old, the patient begins to exhibit seizure activity.
70
71
What condition is associated with this patient's diagnosis?
72
73
A. Acute lymphoblastic leukem ia
74
B. Astrocytoma

~
77
C. Early-onset Alzheimer's disease
D. Malignant melanoma
78
79
80
81
82
83
84
85
86
87
88
89
• 90

Lock
s
Suspend
0
End Block
Item: 90 of 99 ~ 1 • Mark <:] (:>- Jill ~· ~J
QID: 22411 ..1 Previous Next Lab'V!I!ues Notes Calculator
.
68
69
70
The correct answer i s B. 760/o chose th is.
71
Tuberous sclerosis is an autosomal dominant condit ion characterized by the triad of mental reta rdation, seizures, and
72 adenoma sebaceum , as well as hamartomas in mult iple organs, includ ing the heart, brain, eyes, skin, and other organs .
73 Symptoms may be present at birth, but diagnosis is often delayed, as the full spectrum of disease may take some t ime to
74
develop . It is caused by de novo mutations. Patients with tuber ous sclerosis are predisposed to astr ocytomas and cardiac
rhabdomyomas.

~
Tuberous sclerosis Dominance (genetics} Adenoma ~1utation Autosome Epileptic seizure Astrocytoma Intellectual disability Rhabdomyoma Hamartoma Brain Human brain

A i s not correct. 100/o chose this.


77
Tuberous sclerosis is not associated with an incr eased incidence of acute lymphoblastic leukemia { ALL) . Patients with Down's
78
syndrome ar e at increased risk for ALL.
79 Down syndrome Acute lymphoblastic leukemia Tuberous sclerosis Leukemia Tuber

80
C i s not correct. 6 % chose this.
81
Tuberous sclerosis is not associated with an early-onset Alzheimer's disease. Patients with Dow n's syndrome are at increased
82 risk for early-onset Alzheimer's disease .
83 Down syndrome Alzheimer's disease Tuberous sclerosis Early-onset Alzheimer's disease

84 D i s not correct. SO/o chose this.


85 Tuberous sclerosis is not associated with an incr eased incidence of malignant melanoma . Patients with xeroderma
86 pigmentosum ( XP) are at increased risk for malignant melanoma . XP is an autosomal recessive genetic disorder of DNA
87
repair, in w hich the ability to repair damage caused by ultraviolet light is deficient.
Xeroderma pigmentosum Tuberous sclerosis Genetic disorder Melanoma DNA repair Autosomal recessive Dominance (genetics} Ultraviolet Autosome Malignancy Recessive Cancer
88
DNA
89
90

Lock
s
Suspend
0
End Block
Item: 90 of 99 ~ 1 • Mark <:] (:>- Jill ~· ~J
QID: 22411 ..1 Previous Next Lab'V!I!ues Notes Calculator
. A i s not correct. 100/o chose this.
68
Tuberous sclerosis is not associated with an increased incidence of acute lymphoblastic leukemia (ALL) . Patients with Down 's
69 syndrome are at increased risk for ALL.
70 Down syndrome Acute lymphoblastic leukemia Tuberous sclerosis Leukemia Tuber

71
C i s not co rr ect. 6 % chose this.
72 Tuberous sclerosis is not associated with an early-onset Alzheimer's disease. Patients with Down's syndrome are at increased
73 risk for early-onset Alzheimer's disease .
74 Down syndrome Alzheimer's disease Tuberous sclerosis Early-onset Alzheimer's disease

~
D is not correct. 8 % chose this.
Tuberous sclerosis is not associated with an increased incidence of malignant melanoma . Patients with xeroderma
77 pigmentosum (XP) are at increased risk for malignant melanoma . XP is an autosomal recessive genetic disorder of DNA
repair, in which the ability to repair damage caused by ultraviolet light is deficient.
78
Xeroderma pigmentosum Tuberous sclerosis Genetic disorder Melanoma DNA repair Autosomal recessive Dominance (genetics) Ultraviolet Autosome Malignancy Recessive Cancer
79
DNA
80
81
82 Bottom Line :
83 Tuberous sclerosis is an autosomal dominant condition characterized by mental retardation, seizures, and hamartomas in
84 mu lt iple organs, including the heart, brain, eyes, skin, and other organs . Patients w ith tuberous sclerosis are pred isposed to
astrocytomas and cardiac rhabdomyomas .
85 Tuberous sclerosis Dominance (genetics) Epileptic seizure Astrocytoma Rhabdomyoma Autosome Intellectual disability Hamartoma Human brain Brain
86
87
88 References:
89 FA Step 2 CK 9th ed pp 287-288
FA Step 2 CK 8th ed p 273
90

Lock
s
Suspend
0
End Block
Item: 91 of 99 ~ 1 • Mark <:] (:>- Jill ~· ~J
QID: 24447 ..1 Previous Next Lab'V!I!ues Notes Calculator
..
69 A 76-year-old man presents to the clin ic for a follow- up vi sit. Physical examination findings are shown in the video; these
findings have been consistent for the past few visits . He has no complaints during the visit and is otherwise healthy. He
70
has a remote history of drug abuse 15 years ago and suffered a concussion during a motor vehicle accident two years ago.
71 He has a strong family history of cerebrovascular accidents. He shows no cognitive neurologic defects but his wife reports
72 periods of outstanding anger and seems depressed frequently. The remainder of his physical examination is within normal lim its
73
and unchanged from previous visits.
74 OPEN MEDIA

~
77
An abnormal level of which neurotransmitter results in the movement disorder associated with this medical condition?

78
A. Acetylcholine
79
80 B. Dopamine
81 C. Glutamate
82
D. Norepinephrine
83
84
E. Serotonin
85
86
87
88
89
90
• 91

Lock
s
Suspend
0
End Block
Item: 91 of 99 ~ 1 • Mark <:] (:>- Jill ~· ~J
QID: 24447 ..1 Previous Next Lab'V!I!ues Notes Calculator
..
69 A 76-year-old man presents to the clin ic for a follow- up vi sit. Physica l exam ination findings are shown in the video; these
findings have been consistent for the past few visits . He has no complaints during the visit and is otherwise healthy. He
70
has a remote history of drug abuse 15 years ago and suffered a concussion during a motor vehicle accident two years ago.
71 He has a strong family history of cerebrovascular accidents. He shows no cogn it ive neurologic defects but his wife reports
72 periods of outstanding anger and seems depressed frequently. The remainder of his physica l exam ination is within normal lim its
73
and unchanged from previous visits.
74 I~
~
77
78
79
80
81
82
83
84
85
86
87
88
89
90
• 91

Lock
s
Suspend
0
End Block
69
70
71
72
73
74

~
77
78
79
80
81
82 An abnormal level of which neurotransmitter results in the movement disorder associated with this medical condition?
83
84 A. Acetylcholine
85 B. Dopamine
86
C. Glutamate
87
88 D. Norepinephrine
89 E. Serotonin
90
• 91

Lock
s
Suspend
0
End Block
Item: 91 of 99 ~ 1 • Mark <:] (:>- Jill ~· ~J
QID: 24447 ..1 Previous Next Lab'V!I!ues Notes Calculator
..
69 The correct answer is B. 750/o chose this.
70 The patient demonstrates the classic tremor of Pa rkinson disease. The v ideo shows a unilatera l 4-Hz resting t remor that is
71 decreased with movement . This can be classified as a "pill rolling" tremor. Parkinson disease is caused by loss of dopamine-
72 secreting neurons found in the substantia nigra pars compacta in the ventra l midbrain . Th is results in difficulty with initiation
of movement, slowed movements (bradykinesia), and rig idity. The Parkinson tremor likely reflects a deficit in feedback of
73
motor movements at the level of the tha lamus due to the dopamine deficit. Mu lt iple treatment options exist, but first-line
74 treatment involves dopamine replacement with carbidopa-levodopa .

~
Substantia nigra Bradykinesia Midbrain Dopamine Thalamus Parkinson's disease Tremor Neuron Carbidopa/levodopa Pars compacta Anatomical terms of location Ventral

6 A is not correct. 120/o chose this.


77 Acetylcholine is an excitatory neu rotransmitter involved in both the central and peripheral nervous systems. It is involved in
78 triggering muscle contractions and the release of specific hormones ( ie, prolactin, growth hormone, etc) . I n t he central
79
nervous system, acetylcholine plays a role in anger, th irst, attentiveness, and sexuality. I t is not believed to be involved in the
pathophysiology of Parkinson disease, but it known to be decreased in certain regions of the brain in Alzheimer disease.
80 Prolactin Central nervous system Neurotransmitter Acetylcholine Parkinson's disease Growth hormone Hormone Nervous system Alzheimer's disease Excitatory postsynaptic potential
81 Pathophysiology Excitatory synapse Human brain Muscle Brain
82
C is not correct. 7% chose this.
83
Glutamate functions as an excitatory neu rotransmitter and plays a role in learn ing and memory. I t is not believed to be
84
involved in the pathophysiology of Parkinson disease, but is believed to be involved in Alzheimer disease and other types of
85 dementia where one of the key symptoms includes memory problems.
86 Neurotransmitter Glutamic acid Parkinson's disease Alzheimer's disease Dementia Pathophysiology Excitatory postsynaptic potential Excitatory synapse

87 D is not correct. 3% chose this.


88 Norepineph rine functions as both a hormone and a neurotransmitter directly involved in the sympathetic nervous system . It
89 plays a key role in the fight-or-flight response, attentiveness, sleeping, dream ing, emotions, and learning . Norepinephrine is
90 not believed to be involved in the pathophysiology of Pa rkinson disease, but is believed to involved in depression, attention-
deficit hyperactivity disorder (ADHD), and mania .
91 y ,. ..........~: ...... ,.~,..c;,..: .. "-•• .................: ••: ..... A: .........-~ ...... r: ... r..• ..... 4:1: ... k• .................... ,.., ..................: ......... r..~ ...... r ..................r.. .....: .............,.. . . ... ,................................. _.. .............. :........ t'\.,. ..,,: ............ •... A: ........ ,.. .......................... u ....... : ... ,. ............................. .......

Lock Suspend
s 0
End Block
Item: 91 of 99 ~ 1 • Mark <:] (:>- Jill ~· ~J
QID: 24447
.: -.
..1
... - . . Previous
. ..
Next
... .. ..
Lab'V!I!ues
...
Notes Calculator
. . . .. ... . - . .. .
involved in the pathophysiology of Parkinson disease, but is believed to be involved in Alzheimer disease and other types of
. . .. . ...
69
dementia where one of the key symptoms includes memory problems.
70 Neurotransmitter Glutamic add Parkinson's disease Alzheimer's disease Dementia Pathophysiology Excitatory postsynaptic potential Excitatory synapse
71
D i s not correct. 3% chose this.
72
Norepinephrine functions as both a hormone and a neurotransmitter directly involved in the sympathetic nervous system . It
73 plays a key role in the fight-or- flight response, attentiveness, sleeping, dreaming, emotions, and learning. Norepinephrine is
74 not believed to be involved in the pathophysiology of Parkinson disease, but is beli eved to involved in depression, attention-
deficit hyperactivity disorder (ADHD), and mania .

~
77
Attention deficit hyperactivity disorder Fight-or-flight response Norepinephrine Sympathetic nervous system Neurotransmitter Parkinson's disease Hormone Mania Nervous system

Pathophysiology Major depressive disorder Depression (mood)

78 E i s n ot cor rect. 30/o c hose th is.


79 Serotonin is a neurotransmitter involved in mood, body temperature, sleep, appetite, and pain regu lation . It is not believed to
80
be involved in the pathophysiology of Parkinson disease, but imbalance of this neurotransmitter is known to be present in
depression, anxiety disorders, impu lsive behavior, aggressive, and suicide.
81 Serotonin Neurotransmitter Parkinson 's disease Anxiety Pathophysiology Anxiety disorder Major depressive disorder Thermoregulation Depression (mood) Suicide
82
83
84
Bottom line :
85 Parkinson disease is characterized by a resting tremor, r igidity, movement disorders (bradykinesia), postural instability, and
depleted dopamine-secreting neurons.
86 Bradykinesia Balance disorder Neuron Parkinson's disease Movement disorder Tremor
87
88
89 References:
90 FA Step 2 CK 9th ed p 283
FA Step 2 CK 8th ed pp 269-270
91

Lock
s
Suspend
0
End Block
Item: 92 of 99 ~ 1 • Mark <:] (:>- Jill ~· ~J
QID: 23706 ..1 Previous Next Lab'V!I!ues Notes Calculator


70 A 25-year-old woman comes to her primary care doctor oompla ining of episodes of vertigo that make her vom it . These ~~AI
episodes are transient, seem to happen when she is moving her head, and last roughly 1 minute . Upon lying supine with
71
20 degrees of neck extension, when her head is rotated by 45 degrees nystagmus is observed . This patient's sitting blood
72 pressure is 120/80 mm Hg and pulse is 77 /min . While standing her blood pressure is 127/83 mm Hg and pulse is 85/min.
73
74 Which of the following is the most likely diagnosis?

~
77
A. Acoustic neuroma
B. Benign paroxysmal positional vertigo
78
79 C. Meniere's disease
80 D. Migraine
81
E. Postural hypotension
82
83 F. Psychiatric disorder
84
85
86
87
88
89
90
91
• 92

Lock
s
Suspend
0
End Block
Item: 92 of 99 ~ 1 • Mark <:] (:>- Jill ~· ~J
QID: 23706 ..1 Previous Next Lab'V!I!ues Notes Calculator


70
71
72 The correct answer is B. 780/o chose this.
73 This patient is most likely suffering from benign paroxysmal positional vertigo ( BPPV) . BPPV is the most common form of
74 positional vertigo. It is often a result of calcium deposits in the posterior semicircular canal. If nystagmus is seen during a
Dix-Hallpike maneuver (described in the stem, with neck extension and head rotat ion), the diagnosis of BPPV in a pat ient

~
77
w ith a typical history is confirmed . BPPV can usually be treated using the Epley maneuver. This is essential ly a modification of
the Dix-Hallpike maneuver that uses gravity to move the calcium deposits, or canal iths, to a less symptomatic area . This can
be repeated until the symptoms resolve.
78 Semicircular canals Benign paroxysmal positional vertigo Nystagmus Epley maneuver Vertigo Dix-Hallpike Paroxysmal attack Dix-Hallpike maneuver Calcium Benignity

79 Dix-Hallpike test Gravity

80 A is not correct. 1% chose this.


81 The median age for acoustic neuroma is 50 years, and this patient is much younger. Acoustic neuromas present w ith hearing
82 loss and t innitus, unsteadiness while walking (true spinning vertigo is uncommon), and facial numbness/paresis, hypesthesia,
83 taste changes, and facial pain .
Vestibular schwannoma Tinnitus Hypoesthesia Vertigo Neuroma Hearing loss
84
85 C is not correct. 6% chose this.
86 Meniere's disease is idiopathic and usually presents with the t riad of nausea, vertigo, and tinnitus. There is no association
w ith position, and a Dix-Hallpike maneuver would not show nystagmus. Treatment for Meniere disease is usually watchful
87
waiting with treatment of symptoms ( eg, scopolamine for vertigo).
88 Meniere's disease Hyoscine hydrobromide Tinnitus Nystagmus Vertigo Nausea Watchful waiting Idiopathy
89
D is not correct. OOfo chose this.
90
A migraine can cause vertigo. However, this patient is not complaining of a headache, making this choice less likely.
91 Migraine Vertigo Headache
92

Lock
s
Suspend
0
End Block
Item: 92 of 99 ~ 1 • Mark <:] (:>- Jill ~· ~J
QID: 23706 ..1 Previous Next Lab'V!I!ues Notes Calculator

.· • w1th pos1t1on, and a D1x-Hallp1ke maneuver would not show nystagmus. Treatment for Men1ere d1sease IS usually watchful
70 wait ing with treatment of symptoms ( eg, scopolam ine for vertigo ).
Meniere's disease Hyoscine hydrobromide Tinnitus Nystagmus Vertigo Nausea Watchful waiting Idiopathy
71
72 D is not correct. OOfo chose this.
73 A migraine can cause vertigo. However, this patient is not complaining of a headache, making this choice less likely.
74 Migraine Vertigo Headache

~
E is not correct. 10/o chose this.
This patient does not have orthostatic hypotension, which presents as lightheadedness when moving from a lying/ sitting
77 posit ion to standing . To be diagnosed with orthostatic hy[potension, there must be a difference of >20 mm Hg systolic or >10
mm Hg diastolic between the sitting and stand ing blood pressures within 3 minutes of standing up.
78
Orthostatic hypotension Hypotension Ughtheadedness Systole Diastole
79
80
F is not correct. 140/o chose this.
81
Patients can have drop attacks (fainting spells) and conversion disorders as a resu lt of a psychiatric illness. However, given
the reproducible nystagmus, this diagnosis is less likely.
82 Nystagmus Syncope (medicine) Conversion disorder Mental disorder
83
84
85
Bottom line:
86 Benign paroxysma l posit iona l vertigo is the most common form of posit iona l vertigo. It often resu lts from calcium deposits
in the posterior sem icircular canal. Diagnosis is confirmed by the presence of nystagmus during a Dix-Hallpike maneuver.
87 Semicircular canals Nystagmus Benign paroxysmal positional vertigo Vertigo Dix-Hallpike Dix-Hallpike maneuver Paroxysmal attack Benignity Calcium Dix-Hallpike test
88
89
90 References:
91 FA Step 2 CK 9th ed pp 271-272
FA Step 2 CK 8th ed pp 258-259
92

Lock
s
Suspend
0
End Block
Item: 93 of 99 ~ 1 • Mark <:] (:>- Jill ~· ~J
QID: 22192 ..1 Previous Next Lab'V!I!ues Notes Calculator
I

71 A 55-year-old woman with a medical history of myocardial infarction and hypertension is suffering from un ilateral
throbbing headaches preceded by a 10-minute visual aura . The headaches last between 8 and 12 hours and are
72
accompanied by photophobia. She has tried nonsteroidal anti-inflammatory drugs to stop the headaches once they occur,
73 but they are not effective .
74

~
Which medication is the next logical abortive therapy to try in this patient?

77 A. ~ Blockers
78
B. Acetaminophen
79
80 C. Ergotamine
81 D. Triptans
82
E. Valproate
83
84
85
86
87
88
89
90
91
92
• 93

Lock
s
Suspend
0
End Block
Item: 93 of 99 ~ 1 • Mark <:] (:>- Jill ~· ~J
QID: 22192 ..1 Previous Next Lab'V!I!ues Notes Calculator
I

71
72 The correct answer is B. 230/o chose this.
73 For mild to moderate migra ine attacks not associated with severe nausea or vomiting, simple analgesics (NSAIDs,
acetaminophen) or combination of analgesics are first choice agents. If nonsteroidal anti- inflammatory drugs (NSAIDs) are
74
not effective, other analgesics such as acetaminophen, and acetaminophen/ caffeine combinat ion med ications (such as

~
Excedrin and other over -the-counter migraine analgesics) can be used to abort a migra ine attack. These agents ar e effective,
6 less expensive, and less likely to adverse effects than m igraine-specific agents such as triptans or ergots . Acetaminophen
77 alleviates pain associated w ith an attack, and the caffeine constricts the arteries of the cerebrovascular system and breaks
the headache .
78
Paracetamol Caffeine Migraine Aspirinj paracetamoljcaffeine Nausea Over-the-counter drug Triptan Headache Vomiting Nonsteroidal anti-inflammatory drug Analgesic
79
Anti-inflammatory Artery Cerebrovascular disease Ergoline
80
81
A is not correct. 130/o chose this.
~ Blockers ar e used as prophylactic therapy for m igraine headaches, preventing migraines from occurring. They do not help
82
end a headache once it has occurred .
83 Migraine Preventive healthcare Headache
84
C is not correct. 160/o chose this.
85
Erg otamines should be avoided in patients with coronary artery disease and hypertension because they cause coronary artery
86 vasoconstriction and hypertension . Erg otam ine is contraindicated in women who are or may become pregnant, since the
87 drugs may cause fetal harm . Ergotamine is also contraindicated in patients with peripheral vascular disease, coronary heart
88 disease, uncontrolled hypertension, stroke, impaired hepatic or renal function, and sepsis .
Ergotamine Coronary artery disease Vasoconstriction Hypertension Sepsis Peripheral artery disease Cardiovascular disease Liver Stroke Renal function Contraindication
89
Coronary circulation Kidney Vascular disease Blood vessel
90
91 D is not correct. 400/o chose this.
92 Triptans can be helpful, but for mild to moderate attacks not associated with vomiting or severe nausea (as with this patient),
93
triptans are not the next best choice of agents . For moderate to severe migraine attacks, oral migraine-specific agents, such
y ... • •• • ' .. . • .. 0. • • • • . • •

Lock
s
Suspend
0
End Block
Item: 93 of 99 ~ 1 • Mark <:] (:>- Jill ~· ~J
QID: 22192 ..1 Previous Next Lab'V!I!ues Notes Calculator
I ••• • . •• .. • ••• • • • • • • • •• ,, • • • • • •• • • ••• • •

71
disease, uncontrolled hypertension, stroke, impaired hepatic or renal function, and sepsis .
Ergotamine Coronary artery disease Vasoconstriction Hypertension Sepsis Peripheral artery disease Cardiovascular disease Liver Stroke Renal function Contraindication
72
Coronary circulation Kidney Vascular disease Blood vessel
73
74 D is not correct. 400/o chose this.
Triptans can be helpful, but for mild to moderate attacks not associated with vomiting or severe nausea (as with this patient),

~
77
triptans are not the next best choice of agents . For moderate to severe migraine attacks, oral migraine-specific agents, such
as triptans are first line.Triptans have proven to be safe and effective for most patients w ith migraine. A systematic r ev iew of
observational studies found no association between triptan use and the risk of cardiovascu lar events, though only four
78 relevant studies were identified . Simi larly, a large cohort study found no association between triptan prescript ion and stroke,
other cardiovascular events, or death.
79
Triptan Migraine Nausea Systematic review Vomiting Stroke Circulatory system
80
81
E is not correct. SO/o chose this.
82
Valproate is an anticonvulsive medication that is most effective as prophylactic treatment of migra ine headaches because it
helps prevent migraines, but, is not helpfu l in stopping an attack once it has occurred .
83 Migraine Valproate Anticonvulsant Preventive healthcare Pharmaceutical drug
84
85
86
Bottom line:
87 If NSAIDs are not effective in stopping a migraine headache (abortive therapy), other analgesics (eg, acetaminophen) and
acetaminophen/ caffeine combination medications can be used for m ild to moderate attacks.
88 Migraine Paracetamol Analgesic Nonsteroidal anti-inflammatory drug Headache
89
90
91 References:
92 FA Step 2 CK 9th ed p 267
FA Step 2 CK 8th ed p 253
93

Lock
s
Suspend
0
End Block
Item: 94 of 99 ~ 1 • Mark <:] (:>- Jill ~· ~J
QID: 23281 ..1 Previous Next Lab'V!I!ues Notes Calculator

72 A 75-year-old man with a history of carotid stenosis presents with acute left-sided hemiparesis and aphasia . He was
brought to the emergency department from his nursing home by an ambulance, which was held up in traffic for at least
73
half an hour. He remains somewhat responsive with stable vita l signs but his neurolog ic deficits are unchanged . The
74 physician also notes that his cloth ing is soiled w ith urine and feces, although he is known to be continent at baseline.

~
77
Which of the following is the most appropriate initial imag ing study?

78 A. Carotid ultrasound
79
B. Cerebral angiography
80
81 C. CT of the head with contrast
82 D. CT of the head without contrast
83
E. MRI of the brain
84
85
86
87
88
89
90
91
92
93
• 94

Lock
s
Suspend
0
End Block
Item: 94 of 99 ~ 1 • Mark <:] (:>- Jill ~· ~J
QID: 23281 ..1 Previous Next Lab'V!I!ues Notes Calculator

72
73 The correct answer is D. 790/o chose this.
74 CT w ithout contrast is the imaging study of choice in this patient, chiefly because of its speed, safety, and efficacy in
distinguishing between the most common causes of neurologic injury. When stroke is suspected, the scan is performed

~6
77
without contrast in order to aid in distinguishing between hemorrhagic and ischemic stroke .
Ischemic stroke Stroke Ischemia Neurology CT scan Bleeding

A is not correct. 4% chose this.


78
If this patient's stroke is in fact related to his carotid disease, then a duplex ultrasound may indeed help identify the
79 underlying cause and direct subsequent therapy ( eg, emergent carotid endarterectomy) . While this would be an appropriate
80 test following a CT scan of the head showing a stroke in the anterior circulation, it would be inappropriate as an initial
81 diagnostic study.
Carotid endarterectomy Medical ultrasound CT scan Ultrasound Endarterectomy Duplex ultrasonography Stroke Common carotid artery Carotid artery
82
83 B is not correct. 3% chose this.
84 Cerebral angiography is traditional ly r egarded as the gold standard for identifying vascular pathology ( eg, aneurysms or
occlusion). However, because it is invasive and carries the risk of substantial complications, it is rarely used without prior
85
imaging studies such as CT, MRI, and/or ultrasound.
86 Cerebral angiography Angiography Gold standard (test) Ultrasound Medical ultrasound Magnetic resonance imaging Aneurysm Pathology Vascular occlusion
87
C is not correct. 100/o chose this.
88
CT with contrast increases the density of structures with high amount of blood flow. It can be used to evaluate possible
89 tumors or infections, though it is not as useful as MRI for these indications. An infused CT is contraindicated as an initial
90 study because of the risk of further damage to the brain parenchyma from extravasated contrast dye if the stroke is
91 hemorrhagic.
Parenchyma Magnetic resonance imaging Radiocontrast agent Blood flow Stroke Bleeding Human brain Contraindication Neoplasm Brain CT scan
92
93 E is not correct. 40/o chose this.
94 y
A properly performed MRI of the brain has anatomic resolution that is superior to CT of the head in many ways. MRI is more y

Lock
s
Suspend
0
End Block
Item: 94 of 99 ~ 1 • Mark <:] (:>- Jill ~· ~J
QID: 23281 ..1 Previous Next Lab'V!I!ues Notes Calculator

• • • • • • • •
72 occlusion). However, because it is invasive and carries the risk of substantial complications, it is rarely used without prior
73
imaging studies such as CT, MRI, and/or ultrasound.
Cerebral angiography Angiography Gold standard (test) Ultrasound Medical ultrasound Magnetic resonance imaging Aneurysm Pathology Vascular occlusion
74
C is not correct. 100/o chose this.

~
77
CT w ith contrast increases the density of structures with high amount of blood flow. It can be used to evaluate possible
tumors or infections, though it is not as usefu l as MRI for these indications. An infused CT is contraindicated as an initia l
study because of the risk of further damage to the brain pa renchyma from extravasated contrast dye if the str oke is
78 hemorrhagic.
79 Parenchyma Magnetic resonance imaging Radiocontrast agent Blood flow Stroke Bleeding Human brain Contraindication Neoplasm Brain CT scan

80 E is not correct. 40/o chose this.


81 A properly performed MRI of the brain has anatomic resolution that is superior to CT of the head in many ways. MRI is mor e
82 sensitive in detecting ea rly ischemic changes, imag ing t he brain stem/ posteri or fossa, and determining the date of injury.
83
However, because the image acquisit ion times are considerably longer for MRI, obtaining high-quality images is dependent on
the patient's ability to sit still. Due to this current li mitati on, MRI is typically r eserved for the subacute setting and is often
84
used to follow-up CT findings in stroke victims.
85 Magnetic resonance imaging Neuroimaging Human brain Ischemia Stroke Brain CT scan

86
87
Bottom line:
88
When stroke is suspected, perform CT without contrast to help distingu ish between hemorrhagic and ischem ic stroke.
89 Stroke Ischemic stroke Ischemia Bleeding
90
91
92 References:
93 FA Step 2 CK 9th ed p 258
FA Step 2 CK 8th ed pp 245-246
94

Lock
s
Suspend
0
End Block
Item: 95 of 99 ~ 1 • Mark <:] (:>- Jill ~· ~J
QID: 22181 ..1 Previous Next Lab'V!I!ues Notes Calculator

73 A 30-year-old African-American woman with no significant past medical history presents w ith episodes of dizziness induced 1•AI
by rolling over to one side wh ile lying down in bed . The di zziness lasts about 1 minute and is associated with intense A
74
nausea. The episodes have occurred periodically for the past 2.5 months. Nothing alleviates the symptoms, and there are

~
no other associated symptoms. There is no history of head trauma . On physical examination, changing of the position of her
head is associated with the upper poles of the eye beating toward the floor for 30 seconds. On repeat examinations the intensity
77
and duration of the nystagmus dim inished . The rest of the neurologic examination was within normal lim its.
78
What is the pathophysiology of this condit ion?
79
80
81
A. Excess fluid in endolymphatic space of inner ear
82 B. Free- floating canalith within the posterior semicircular canal
83
C. Migrainous vertigo
84
85
D. Postural hypotension
86 E. Viral infection involving vestibular nerve
87
88
89
90
91
92
93
94
• 95

Lock
s
Suspend
0
End Block
Item: 95 of 99 ~ 1 • Mark <:] (:>- Jill ~· ~J
QID: 22181 ..1 Previous Next Lab'V!I!ues Notes Calculator

73
74

~6
77
The correct answer is B. 790/o chose this.
The history given is classic for ben ign paroxysmal positional vertigo (BPPV), the most common form of posit ional vertigo,
which is attributed to calcium debris within the posterior semicircular canal. The vertigo lasts for less than 1 minute, and is
78 induced by specific head movements and by the Dix-Ha ll pike maneuver (a series of manipulations of the patient's head). The
79 nystagmus is upward beating and torsional, lasting 30 seconds or less w ith noticed latency, transiency, and fatigability, in the
absence of other neurologic findings. Particle r eposition ing can be accomplished by the Epley maneuver, an effective
80
treatment for patients with BPPV.
81 Semicircular canals Benign paroxysmal positional vertigo Nystagmus Vertigo Epley maneuver Paroxysmal attack Calcium Benignity Dix-Hallpike Dix-Hallpike test Neurology
82
A is not correct. 150/o chose this.
83
Meniere's disease is characterized by intermittent spells of vertigo lasting minutes to hours . It is also associated w ith
84 un ilateral sensorineura l hearing loss and tinnitus, sometimes with a sense of fullness in the ear. It is caused by the buildup of
85 excess endolymphatic fluid in the inner ear. It is best diagnosed with audiometry to check for hearing loss . Treatment is
86 complicated and requires specia lty consultation, involves a low-salt diet, diuretics for vertigo, and there have been some
reports of caffeine and tobacco restriction being helpfu l. This patient's episodes are far too brief and there are no associated
87
symptoms.
88 Tinnitus Meniere's disease Vertigo Sensorineural hearing loss Endolymph Caffeine Inner ear Hearing loss Low sodium diet Diuretic Audiometry

89
C is not correct. 20/o chose this.
90
Although migra ines can cause episodic vertigo, the dizziness is transient and occurs over hours to days, as opposed to weeks
91 or months observed with BPPV. Often there are other migraine symptoms accompanying the vertigo. Treatment includes
92 triptans, acetazolam ide, ergotamine, and vestibular suppressants (promethazine, meclizine).
Meclizine Acetazolamide Ergotamine Promethazine Migraine Vertigo Dizziness Triptan Vestibular system
93
94 D is not correct. 2% chose this.
95 y Although postural orthostatic hypotension can cause dizziness induced by a change in position, it is generally not caused by y

Lock
s
Suspend
0
End Block
Item: 95 of 99 ~ 1 • Mark <:] (:>- Jill ~· ~J
QID: 22181 ..1 Previous Next Lab'V!I!ues Notes Calculator
- .... '"'"'. .. . .. ... .. .. . ...
73 D is not correct. 2% chose this.
74 Although postural orthostatic hypotension can cause dizziness induced by a change in position, it is generally not caused by

~
rolling over in bed or lying dow n. Tr eatment includes add ressing the underlying cause of the orthostatic hypotension .
Orthostatic hypotension Hypotension Dizziness

77 E is not correct. 20/o chose this.


78 Acute vestibular neurit is is another relatively common cause of vertigo in otherwise hea lthy patients. It usually pr esents
79
abruptly with vertigo that does not dissipate along with nausea that is made worse by head turning for days . Patients may
compla in of occasional vertigo brought on by head movem ent after the acute period resolves . It is usua lly caused by a v iral
80
infection that involves the vestibular nerve . Nystagmus is often spontaneous and can sometimes be suppressed by visual
81 fixation . Hearing may be impaired in the acute phase. It can present simi larly to a cerebellar infarction and patients shou ld
82 have an MRI or magnetic resonance angiography to rule th is out. Treatment is usually to control the symptoms of nausea,
while steroids may help minim ize inflammation. This patrent does not have any init ial intense period of dizziness consistent
83
w ith this diagnosis.
84 Nystagmus Vertigo Magnetic resonance angiography Nausea Vestibular system Labyrinthitis Angiography Vestibular nerve Neuritis Dizziness Magnetic resonance imaging Cerebellum
85 Inflammation Viral disease
86
87
88
Bottom line:
89 Benign paroxysma l posit iona l vertigo is the most common form of posit iona l vertigo, wh ich is attributed to calcium debris
w ithin the posterior sem icircular cana l.
90 Semicircular canals Benign paroxysmal positional vertigo Vertigo Paroxysmal attack Benignity Calcium
91
92
93 References:
94 FA Step 2 CK 9th ed pp 271-272
FA Step 2 CK 8th ed pp 258-259
95

Lock
s
Suspend
0
End Block
Item: 96 of 99 ~ 1 • Mark <:] (:>- Jill ~· ~J
QID: 24091 ..1 Previous Next Lab'V!I!ues Notes Calculator

74 A 42-year-old man is referred to a neurologist after 3 months of bilateral hand weakness and difficulty swallowing. ~~AI
Recently he has begun slurring his words. He has no medical history, and further review of systems is negative . Physical

~
77
examination reveals visible tongue fasciculations, bilateral atrophy of the interosseous muscles, 3/5 weakness in the upper
extremities and 3+ reflexes in the right lower extremity.

78 Where is this patient's lesion most li kely located?


79
80 A. Anterior horn cells
81
B. Muscle
82
83 C. Neuromuscular junction
84 D. Peripheral nerve
85
E. Substantia nigra
86
87
88
89
90
91
92
93
94
95
• 96

Lock
s
Suspend
0
End Block
Item: 96 of 99 ~ 1 • Mark <:] (:>- Jill ~· ~J
QID: 24091 ..1 Previous Next Lab'V!I!ues Notes Calculator

74

~
The correct answer is A. 730/o chose this.
This patient has amyotrophic lateral sclerosis, a disease of t he upper and lower motor neurons in the brain (motor cortex)
and spinal cord (anterior horn cells) . Patients present with m ixed upper mot or and lower motor neuron signs . Upper motor
77
neuron signs include spasticity, hyperrefl exia, and other central nervous system findings. Lower motor neuron signs incl ude
78 flaccidity, hyporefl exia, and fascicu lat ions.
79 Amyotrophic lateral sderosis Hyporeflexia Lower motor neuron Hyperreflexia Central nervous system Spasticity Neuron Upper motor neuron Motor cortex Motor neuron Spinal cord

80 Anterior grey column Fasciculation Flaccid paralysis Nervous system Upper motor neuron syndrome Brain Anatomical tenns of location Cortex (anatomy)

81
B is not correct. 30/o chose this.
82
Patients with myopathy typica lly present w it h proximal w eakness, such as having difficulty combing their hair or cli mbing
83 stairs. These patients present with lower motor neuron signs. Examples are muscula r dyst rophy and dermatomyositis.
84 Neuron Dermatomyositis Myopathy Lower motor neuron Muscular dystrophy Motor neuron Anatomical terms of location

85 C is not correct. 90/o chose this.


86 Myasthenia gravis is a disease of t he neu romuscular junction . Pati ents experience diffuse, fatigable weakness and often
87 present with ptosis or double vision .
Myasthenia gravis Neuromuscular junction Ptosis (eyelid) Diplopia Musde weakness
88
89 D is not correct. 9% chose this.
90 Patients with peripheral neuropat hy display lower mot or neuron signs and usually sensory alteration in a glove and stocking
91
distribution . Causes include metabolic and inflammatory syndromes; an exam ples is Guillain-Barr e synd rome.
Neuron Lower motor neuron Peripheral neuropathy Motor neuron Metabolism
92
93 E is not correct. 60/o chose th is.
94 Loss of dopaminergic neurons in the substantia nigra would be consistent w it h Pa rkinson's disease.
Parkinson's disease Substantia nigra Dopaminergic Neuron
95
96

Lock
s
Suspend
0
End Block
Item: 96 of 99 ~ 1 • Mark <:] (:>- Jill ~· ~J
QID: 24091 ..1 Previous Next Lab'V!I!ues Notes Calculator

• Pat1ents w1th myopathy typ1ca lly present w1th prox1mal weakness, such as havmg d1ff1culty combmg the1r ha 1r or cl1mbmg
74 stairs. These patients present with lower motor neuron signs. Examples are muscular dystrophy and dermatomyositis.

~
Neuron Dermatomyositis Myopathy lower motor neuron Muscular dystrophy Motor neuron Anatomical terms of location

C is not correct. 9% chose this.


77 Myasthenia gravis is a disease of the neuromuscular junction . Patients experience diffuse, fatigable weakness and often
78 present with ptosis or double vision .
Myasthenia gravis Neuromuscular junction Ptosis (eyelid) Diplopia Muscle weakness
79
80 D is not correct. 90/o chose this.
81 Patients with periphera l neuropathy display lower motor neuron signs and usua lly sensory alteration in a glove and stocking
distribution . Causes include metabolic and inflammatory syndromes; an examples is Guillain-Barre syndrome.
82
Neuron lower motor neuron Peripheral neuropathy Motor neuron Metabolism
83
84 E is not correct. 60/o chose this.

85
Loss of dopaminergic neurons in the substantia nigra would be consistent with Parkinson's disease.
Parkinson's disease Substantia nigra Dopaminergic Neuron
86
87
88 Bottom Line:
89 Amyotrophic latera l sclerosis is a disease of the upper a.nd lower motor neurons in the brain (motor cortex) and spinal cord
90 (anterior horn cells) . Patients present with both upper motor and lower motor neuron signs.
Amyotrophic lateral sclerosis lower motor neuron Neuron Motor cortex Motor neuron Spinal cord Anterior grey column Anatomical terms of location Hom (anatomy) Cortex (anatomy)
91 Human brain
92
93
94 Refer en ces :
95 FA Step 2 CK 9th ed pp 276- 277
FA Step 2 CK 8th ed p 264
96

Lock
s
Suspend
0
End Block
Item: 97 of 99 ~ 1 • Mark <:] (:>- Jill ~· ~J
QID: 2448 0 ..1 Previous Next Lab'V!I!ues Notes Calculator
.
A 32-year-old man with a seizure disorder is brought to the emergency department by his girlfriend because of repetitive

~
77
shaking without intervening periods of consciousness that began 15 minutes prior to arrival. The girlfriend states that he
has been feeli ng sad and stressed lately, and has not been compliant w ith his medications. His temperature is 37 .0°C
(98 .6°F), blood pressure is 124/76 mm Hg, pulse is 72/min, and respiratory rate is 14/m in. He is unconscious and has repetitive
78 rigid extension of his extremities followed by jerking movements . After giving him two doses of intravenous lorazepam and a
79
load ing dose of fosphenytoin, followed by a half loading dose of fosphenytoin, the patient is still seizing .
80
What is the most appropriate next step in management?
81
82
83
A. Adm inistration of clonazepam
84 B. Administration of topiramate
85
C. CT scan of the head with and without contrast
86
87
D. Obtain an electroencephalogram
88 E. Rapid sequence intubation
89
90
91
92
93
94
95
96
• 97

Lock
s
Suspend
0
End Block
Item: 97 of 99 ~ 1 • Mark <:] (:>- Jill ~· ~J
QID: 2448 0 ..1 Previous Next Lab'V!I!ues Notes Calculator
.

~6
77
The correct answer is E. 81% chose this.
78
Remember the ABC's : A irway, Breathing, and Circulation . For any patient in status epilepticus, a rapid general assessment
79 with attention to respiratory and circulatory status is needed . Given the length of t ime of the patient's seizure, he is at risk of
80 losing his airway. His airway needs to be protected while he is being prepared for intubation . Thus airway management
81 becomes paramount if the patient fa ils to respond to initi al medical management. Rapid sequence intubation (RSI) involves a
rapidly active sedative agent and a neuromuscular blocking agent in order to render the patient unconscious and paralyzed in
82
order to facilitate control of the airway in emergent situtations, such as status epilepticus. Common sedatives used for RSI in
83 status epilepticus patients include midazolam and thiopen ta l. Concomitant with intubation, th ird-li ne med ica l therapy with
84 phenobarbital shou ld be init iated.
Phenobarbital Midazolam Status epilepticus Sedative Intubation Airway management Sodium thiopental Respiratory tract Neuromuscular junction Epileptic seizure
85
Rapid sequence induction Paralysis
86
87 A is not correct. 7% chose this.
88 Clonazepam is a benzodiazepine that works by decreasing abnormal electrical activity in the brain . Outside of the United
89 States, IV clonazepam is commonly used as first-li ne therapy for status epilepticus. Since the patient is still seizing after the
administration of lorazepam and fosphenytoin, it would not be effective in this scenario.
90
Benzodiazepine Lorazepam Clonazepam Fosphenytoin Status epilepticus Brain
91
B is not correct. 70/o chose this.
92
Small case reports have demonstrated some efficacy in using topiramate in refractory status epilepticus. However, because
93
more studies are needed, its use is not currently recommended .
94 Topiramate Status epilepticus Refractory
95
C is not correct. 2% chose this.
96
Given the patient's history, it is unlikely that the seizure 'is secondary to an intracran ial hemorrhage or tumor. Thus CT would
97 y

Lock
s
Suspend
0
End Block
Item: 97 of 99 ~ 1 • Mark <:] (:>- Jill ~· ~J
QID: 2448 0 ..1 Previous Next Lab'V!I!ues Notes Calculator
.
A is not correct. 7% chose this.
~
77
Clonazepam is a benzodiazepine that works by decreasing abnormal electrical activity in the bra in. Outside of the United
States, IV clonazepam is commonly used as first-li ne therapy for status epilepticus. Since the patient is still seizing after the
admin istration of lorazepam and fosphenytoin, it would not be effective in this scenario .
78 Benzodiazepine lorazepam donazepam Fosphenytoin Status epilepticus Brain
79
B is not correct. 70/o chose this.
80
Small case r eports have demonstrated some efficacy in using topiramate in refractory status epilepticus. However, because
81 more studies are needed, its use is not currently recommended .
82 Topiramate Status epilepticus Refractory

83 C is not correct. 2% chose this.


84 Given the patient's history, it is unlikely that the seizure is secondary to an intracran ial hemorrhage or tumor. Thus CT would
85 not be the most appropriate next step in management.
Intracranial hemorrhage Bleeding Neoplasm Epileptic seizure
86
87 D is not correct. 3% chose this.
88 Many cases of status epilepticus, such as this one, can be diagnosed on neurologic exam alone . Although an
89 electroencephalogram may be warranted to diagnose or monitor complex cases, it is not the most appropriate next step in
management.
90
Electroencephalography Status epilepticus Neurological examination Neurology
91
92
93 Bottom line:
94 Remember ABC { A irway, Breath ing, Circulation) when treating status epilepticus.
Status epilepticus American Broadcasting Company
95
96
97

Lock
s
Suspend
0
End Block
A 45-year-old man visits his family practitioner complaining of severe periorbital headaches. The patient states that for the 1•AI
past month, he has been awoken in the middle of the night by boring pain around his right eye that is 9 of 10 in intensity. A
77
This happens about twice a night, and the pa in usually lasts about half an hour. It is associated with lacrimation and nasal
78 discharge. He says he has had this type of pain before, about 1 year ago. At that t ime the episodes lasted for about 1 month
79 and then went away.
80
81 What nerve is most likely involved in creating the pain in ti:his disease?
82
83 A. Facial
84
B. Ocu lomotor
85
C. Optic
86
87 D. Trigeminal
88 E. Vagus
89
90
91
92
93
94
95
96
97
• 98

Lock
s
Suspend
0
End Block
77
78
The correct answer is D. 680/o chose this.
The patient has a classic presentation of a cluster headache. Cluster headaches usually appear in brief episodes lasting
79
minutes to hou rs and begin with a sudden onset. These episodes can occu r several t imes a day for many months at a t ime,
80 and are known as "clusters." Patients typica lly have "cluster-free" periods lasting a few weeks to yea rs. Pain is usually
81 described as intense and lancinating . Pain is usua lly unilateral in the distribution of cranial nerves V 1 and V2 and is associated
82 w ith lacrimation and nasal stuffiness .
Cluster headache Tears Cranial nerves Headache
83
84 A is not correct. 150/o chose this.
85 Although the facia l nerve innervates the lacrima l gland, it does not likely have a role in the pa in produced in cluster
headaches. A palsy of the facial nerve is known as a Bell' s palsy.
86
Bell's palsy Lacrimal gland Facial nerve Cluster headache Lacrimal bone Gland
87
88
B is not correct. 90/o chose this.
The oculomotor nerve along with the abducens and trochlear nerve are responsible for movements of the eye . They are not
89
thought to have a role in cluster headaches . Ocu lomotor palsy, usually caused by trauma, aneurysm, or mass lesion, results
90 in ptosis and myd riasis.
91 Trochlear nerve Mydriasis Oculomotor nerve Dtosis (eyelid) Oculomotor nerve palsy Abducens nerve Cluster headache Aneurysm Neoplasm Lesion Mass effect (medicine)

92 Eye movement

93
C is not correct. 6% chose this.
94
The optic nerve is responsible for vision and does not likely have a role in the pa in of cluster headaches. Optic nerve disorders
95 such as aplasia, tumors, and cysts can cause optic distu rbances.
96 Optic nerve Cluster headache Cyst Neoplasm

97 E is not correct. 20/o chose this.


98 y The vaous nerve or ovides oarasvmoathetic innervation from the neck down to the second seament of the transverse colon . It

Lock
s
Suspend
0
End Block
B is not correct. 90/o chose this.
The oculomotor nerve along with the abducens and trochlear nerve are responsible for movements of the eye . They are not
77 thought to have a role in cluster headaches . Ocu lomotor palsy, usually caused by trauma, aneurysm, or mass lesion, results
78 in ptosis and mydriasis.
Trochlear nerve Mydriasis Oculomotor nerve ptosis (eyelid) Oculomotor nerve palsy Abducens nerve Cluster headache Aneurysm Neoplasm Lesion Mass effect (medicine)
79
Eye movement
80
81 C is not correct. 6% chose this.
82 The optic nerve is responsible for vision and does not likely have a role in the pa in of cluster headaches. Optic nerve disorders
83 such as aplasia, tumors, and cysts can cause optic disturbances.
Optic nerve Cluster headache Cyst Neoplasm
84
85 E is not correct. 20/o chose this.
86 The vagus nerve provides parasympathetic innervation from the neck down to the second segment of the transverse colon . It
87
is not thought to have a role in cluster headaches. Damage to the recurrent laryngeal branch of the vagus nerve can cause
partial paralysis of the larynx, known as laryngeal palsy.
88 Transverse colon Larynx Vagus nerve Parasympathetic nervous system Recurrent laryngeal nerve Colon (anatomy) Paralysis Cluster headache Nerve
89
90
91
Bottom line:
92 Cluster headaches present with unilatera l pain in the distribution of crania l nerves Vl and V2 . They are associated with
lacrimation and nasal stuffiness.
93 Tears Cranial nerves Cluster headache Visual cortex
94
95
96 References:
97 FA Step 2 CK 9th ed pp 267-268
FA Step 2 CK 8th ed p 253
98

Lock
s
Suspend
0
End Block
Item: 99 of 99 ~ 1 • Mark <:] (:>- Jill ~· ~J
QID: 23389 ..1 Previous Next Lab'V!I!ues Notes Calculator

~
77 Over the course of a year, a 45-year-old woman has experienced several episodes of a variety of neurologic symptoms, ~~AI
including double vision, lower extremity weakness, and loss of balance. Each acute episode lasts several days, after wh ich
78
her symptoms resolve . In between episodes she is symptom-free, with norma l neurologic function. She is seen by a
79 neurologist who orders a FLAIR MRI of the brain, shown in the image .
80
81
82
83
84
85
86
87
88
89
90
91
92
93
94
95
Image courtesy of Dooley et at, Optic Neuritis, J Ophthalmic Vis Res. 2010 Jut; 5 (3): 182-187.
96
97
Which of the following therapies is the most appropriate treatment to give during episodes of recurrent neurologic dysfunction?
98
• 99

Lock
s
Suspend
0
End Block
77
78
79
80
81
82
83
84
85
86
87
88 Image courtesy of Dooley et at, Optic Neuritis, J Ophthalmic Vis Res. 2010 Ju/; 5(3): 182-187.

89
90 Which of the following therapies is the most appropriate treatment to give during episodes of recurrent neurologic dysfunction?
91
92 A. Azathioprine
93 B. Cyclophosphamide
94
C. Interferon - ~
95
96 D. Methylprednisolone
97 E. Mitoxantrone
98
• 99

Lock
s
Suspend
0
End Block
Item: 99 of 99 ~ 1 • Mark <:] (:>- Jill ~· ~J
QID: 23389 ..1 Previous Next Lab'V!I!ues Notes Calculator

~
77 The correct answer is D. 620/o chose this.
78 Multiple sclerosis (MS) is a progressive, autoimmune inflammatory disease that attacks myelinated axons in the centra l
79 nervous system . Its pathophysiology is believed to involve both genetic susceptibi lity and environmental triggers. The image
80 shows active MS plaques with in both hemispheres. The initial phase of the disease is characterized by acute onset, recurrent
neurologic dysfunction, followed by partial or complete neurolog ic recovery. The course of disease in MS is characterized by
81
the relapsing and rem itting of neurologic dysfunction . Also characteristic of MS, is that the lesion burden increases w ith each
82 successive r elapse, and as such, 75%-85% of MS patients enter a phase of illness in which relapses become less common,
83 and gradual, progressive, and sustained loss of neurologfic function is seen . Corticosteroids are the treatment administered
84
during episodes of recurrent neurologic dysfunction in ord er to shorten the episode's duration . Severe symptoms may warrant
a course of high-dose steroids ( eg, methylprednisolone), given intravenously, followed by an outpatient-based, ora l steroid
85 taper.
86 ~1ultiple sclerosis Methylprednisolone Central nervous system Corticosteroid Steroid Autoimmune disease Axon Nervous system Autoimmunity Intravenous therapy Myelin

87 Pathophysiology Lesion Neurology Cerebral hemisphere Relapse Inflammation

88
A is not correct. 3% chose this.
89
Azathioprine is an immunosuppressive antimetabolite drug . It is an imidazolyl derivative of 6-mercaptopurine that has been
90 used off-label to slow the progression of mu ltiple sclerosi s. However, it is not used acutely.
91 Azathioprine Antimetabolite ~1ultiple sclerosis Mercaptopurine Immunosuppression

92 B is not correct. 40/o chose this.


93 Cyclophosphamide is believed to suppress immune cell production by inducing DNA cross-linking . It has been used off-label
94 for secondary progressive multiple sclerosis, especially for patients with dramatic, rapid progression . It is not used acutely.
Multiple sclerosis Cyclophosphamide White blood cell Crosslinking of DNA DNA Off-label use
95
96 C is not correct. 270/o chose this.
97 Interferon-~ is indicated for treatment of patients w ith r elapsing forms of mu lt iple sclerosis (MS) to slow the accumulation of
98 physical disability and to decrease the frequency of clin ical exacerbations . Interferon-~ may decrease expression of B7-1 (a
proinflammatory molecule) on the surface of immune cells and increase levels of tumor growth factor-~ (anti-inflammatory)
99 •

Lock
s
Suspend
0
End Block
Item: 99 of 99 ~ 1 • Mark <:] (:>- Jill ~· ~J
QID: 23389 ..1 Previous Next Lab'V!I!ues Notes Calculator
~ • I I • " • I I ...

77 Cyclophosphamide is bel ieved to suppress immune cell p roduction by inducing DNA cross-linking . I t has been used off-label
78
for secondary prog ressive multiple sclerosis, especially for patients with dramatic, rapid prog ression . It is not used acutely.
Multiple sclerosis Cyclophosphamide White blood cell Crosslinking of DNA DNA Off-label use
79
80 C is not correct. 270/o chose this.
Interferon-~ is indicated for treatment of patients w ith relapsing forms of mu lt iple sclerosis (MS) to slow the accumulation of
81
physical disability and to decrease the frequency of clinical exacerbations . In terferon-~ may decrease expression of B7-1 (a
82
proinflammatory molecule) on the surface of immune cells and increase levels of tumor growth factor-~ (anti- inflammatory)
83 in the circulation of patients w ith MS. It does not help shorten acute attacks.
84 Multiple sclerosis Immune system Neoplasm Anti-inflammatory White blood cell COSO Molecule

85 E is not correct. 40/o chose this.


86 Mitoxantrone induces DNA cross-linking and strand breaks and leads to apoptosis, as well as interfering with RNA and
87 inh ibiting DNA repair. It is indicated for reducing neu rologic disability and/or the frequency of clinica l relapses in several types
88
of mult iple sclerosis (MS), although not in primary progressive MS . It does not help shorten acute attacks. It is associated
w ith a risk of severe myelosuppression and heart dysfunction .
89 Multiple sclerosis Mitoxantrone Bone marrow suppression Apoptosis DNA repair DIIIA RNA Crosslinking of DNA
90
91
92
Bottom line:
93 Mult iple sclerosis is a progressive, immune-mediated inflammatory disease that attacks myelinated axons in the CNS . Treat
w ith corticosteroids during an attack to shorten the du ration of an acute episode and lessen the lesion burden .
94 Multiple sclerosis Corticosteroid lesion Axon Myelin Immune system Autoimmunity Inflammation Immune disorder
95
96
97 Refere n ces:
98 FA Step 2 CK 9th ed pp 275-276
FA Step 2 CK 8th ed pp 263-263
99

Lock
s
Suspend
0
End Block

S-ar putea să vă placă și